Download Rhinology Maintenance of Certification Review

Document related concepts

Adherence (medicine) wikipedia , lookup

Hygiene hypothesis wikipedia , lookup

Management of multiple sclerosis wikipedia , lookup

Transcript
SUPPLEMENT 1 MAY–JUNE 2014
Rhinology Maintenance of Certification Review
Includes Examination Review Questions
Edited by
Douglas D. Reh, M.D., Bradford A. Woodworth, M.D. and David Poetker, M.D.
www.ajra.com
Copyright © Oceanside Publications, Inc. All rights reserved. DO NOT COPY
For permission to copy go to https://www.oceansidepubl.com/permission.htm
Copyright © Oceanside Publications, Inc. All rights reserved. DO NOT COPY
For permission to copy go to https://www.oceansidepubl.com/permission.htm
American Journal of
Rhinology & Allergy
May–June 2014
Volume 28
Number 3
Supplement 1
EDITORIAL
S1
Editorial: Preparing for the Maintenance of Certification (MOC)
Examination in Rhinology
D. D. Reh and D. Poetker
S3
Sinonasal anatomy and function
D. M. Dalgorf and R. J. Harvey
S7
Nasal obstruction
J. L. Osborn and R. Sacks
S9
Epistaxis
R. Sacks, P.-L. Sacks, and R. Chandra
S11
Chronic rhinosinusitis
R. A. Settipane, A. T. Peters, and R. Chandra
S16
Nasal polyps
R. A. Settipane, A. T. Peters, and A. G. Chiu
S22
Allergic fungal rhinosinusitis
A. M. Laury and S. K. Wise
S24
Invasive fungal rhinosinusitis
P. Duggal and S. K. Wise
S27
Benign sinonasal neoplasms
P. T. Hennessey and D. D. Reh
S31
Sinonasal malignancies
R. J. Harvey and D. M. Dalgorf
S35
Granulomatous diseases and chronic sinusitis
M. A. Kohanski and D. D. Reh
S38
Cystic fibrosis chronic rhinosinusitis: A comprehensive review
M. R. Chaaban, A. Kejner, S. M. Rowe, and B. A. Woodworth
S47
Pediatric rhinosinusitis: Definitions, diagnosis and management—
An overview
S. K. Chandran and T. S. Higgins
S51
Surgery for sinonasal disease
T. S. Higgins and A. P. Lane
S54
Augmenting the nasal airway: Beyond septoplasty
P. Simon and D. Sidle
S60
The role of the nose in sleep-disordered breathing
E. K. Meen and R. K. Chandra
American Journal of Rhinology & Allergy
Copyright © Oceanside Publications, Inc. All rights reserved. DO NOT COPY
For permission to copy go to https://www.oceansidepubl.com/permission.htm
S68
Olfactory disorders
A. Gaines
S71
Nonallergic rhinitis
R. A. Settipane and M. A. Kaliner
S75
Allergic rhinitis
R. A. Settipane and C. Schwindt
S79
Determining the role of allergy in sinonasal disease
R. A. Settipane, L. Borish, and A. T. Peters
S82
The united allergic airway: Connections between allergic rhinitis,
asthma, and chronic sinusitis
C. H. Feng, M. D. Miller, and R. A. Simon
S86
Immunomodulation of allergic sinonasal disease
R. A. Settipane, A. T. Peters, and L. Borish
S90
Subcutaneous and sublingual immunotherapy for allergic rhinitis:
What is the evidence?
S. K. Wise and R. J. Schlosser
S95
The risk and management of anaphylaxis in the setting of
immunotherapy
P. Lieberman
S101 Pathophysiology of hereditary angioedema
B. L. Zuraw and S. C. Christiansen
S107 Rhinology MOC Review - Questions
About the cover: With Permission from the American Journal of Rhinology and Allergy: Singh U, Bernstein JA, Haar L, et al.
Azelastine desensitization of transient receptor potential vanilloid 1: A potential mechanism explaining its therapeutic effect in
nonallergic rhinitis. Am J Rhinol Allergy 28:215-224, 2014.
American Journal of Rhinology & Allergy posts select in-press articles online, in advance of their appearance in the print edition. These
articles, referred to as ‘‘Fast Track’’ articles are available at the American Journal of Rhinology & Allergy web site, www.AJRA.com
by clicking on the ‘‘on line access’’ link, which connects to the journal’s Ingenta web site and the Fast Track link. Each print
article will acknowledge the e-publication date (the date when the article first appeared online). As soon as an article is
published online, it is fully citable through use of its Digital Object Identifier (DOI).
A2
May–June 2014, Vol. 28, No. 3
Copyright © Oceanside Publications, Inc. All rights reserved. DO NOT COPY
For permission to copy go to https://www.oceansidepubl.com/permission.htm
Contributing Authors
Larry Borish, MD
Michael Kaliner, MD
University of Virginia Health System
Charlottesville, Virginia, MD
George Washington School of Medicine
Washington, DC
Mohamad Chaaban, MD
Alexandra Kejner, MD
University of Alabama at Birmingham
Birmingham, Alabama
University of Alabama at Birmingham
Birmingham, Alabama
Rakesh Chandra, MD
Michael Kohanski, MD
Vanderbilt University
Nashville, Tennessee
John Hopkins Sinus Center
Baltimore, Maryland
Swapna Chandran, MD
University of Louisville School of Medicine
Louisville, Kentucky
Alexander G. Chiu, MD
University of Arizona
Tucson, Arizona
Sandra Christiansen, MD
Kaiser Permanente Allergy
La Jolla, CA
Dustin Dalgorf, MD
Andrew Lane, MD
John Hopkins School of Medicine
Baltimore, Maryland
Adrienne Laury, MD
Emory University Sinus, Nasal and Allergy Center
Atlanta, Georgia
Phil Lieberman, MD
University of Tennessee College of Medicine
Germantown, Tennessee
Applied Medical Research Center
University of New South Wales and
Macquarie University
Darlinghurst, Sidney, Australia
Eric Meen, MD
Praveen Duggal, MD
Scripps Green Hospital
La Jolla, California
Emory University Sinus, Nasal and Allergy Center
Atlanta, Georgia
CH Feng, MD
Scripps Green Hospital
La Jolla, California
Alan Gaines, MD
Warren Alpert Medical School
Brown University
Providence, Rhode Island
Richard Harvey, MD
Applied Medical Research Center
University of New South Wales and
Macquarie University
Darlinghurst, Sidney, Australia
University of Manitoba
Winnipeg, Manitoba, Canada
Michaela Miller, MD
Jodi Osborn, MD
Sydney Adventist Hospital
Hornsby, Australia
Anju Peters, MD
Northwestern University
Chicago, Illinois
David Poetker, MD
Medical College of Wisconsin
Milwaukee, Wisconsin
Douglas D. Reh, MD
John Hopkins Sinus Center
Baltimore, Maryland
Patrick Hennessey, MD
Steven Rowe, MD
John Hopkins Sinus Center
Baltimore, Maryland
University of Alabama at Birmingham
Birmingham, Alabama
Thomas Higgins, MD
Peta-Lee Sacks, MBBS VI
Kentuckiana ENT
Louisville, Kentucky
University of New South Wales
New South Wales, Australia
American Journal of Rhinology & Allergy
Copyright © Oceanside Publications, Inc. All rights reserved. DO NOT COPY
For permission to copy go to https://www.oceansidepubl.com/permission.htm
Raymond Sacks, MD
Patrick Simon, MD
Australian School of Advanced Medicine
Macquarie University and
University of Sydney Medical School
Sydney, Australia
Feinberg School of Medicine
Northwestern University
Chicago, Illinois
Rodney Schlosser, MD
Scripps Green Hospital
La Jolla, California
Medical University of South Carolina
Charleston, South Carolina
Christina Schwindt, MD
Allergy & Asthma Associates
Mission Viejo, California
Russell Settipane, MD
Warren Alpert Medical School
Brown University
Providence, Rhode Island
Douglas Sidle, MD
Feinberg School of Medicine
Northwestern University
Chicago, Illinois
Ronald Simon, MD
Sarah Wise, MD
Emory University Sinus, Nasal and Allergy Center
Atlanta, Georgia
Bradford A. Woodworth, MD
University of Alabama at Birmingham
Birmingham, Alabama
Bruce Zuraw, MD
University of California, San Di ego
San Diego, California
May–June 2014, Vol. 28, No. 3
Copyright © Oceanside Publications, Inc. All rights reserved. DO NOT COPY
For permission to copy go to https://www.oceansidepubl.com/permission.htm
Editorial: Preparing for the Maintenance of Certification
(MOC) Examination in Rhinology
In 2000, the American Board of Medical Specialties (ABMS) adopted
Maintenance of Certification (MOC), as a change in physician selfregulation.1 Specifically MOC is designed to encourage physician
self-assessment, lifelong learning and continuous performance improvement. Multiple factors brought about this change, including an
increase in the complexity of health care delivery that parallels improvements in development of new methods of diagnosis and treatment.2 Consumers became more interested in the delivery of appropriate health care in the setting of unsustainable cost increases, and
heightened scrutiny over the use of limited funds. Additionally,
technological improvements allowed for more careful monitoring of
health care delivery, leading to increased accountability. The demand
for increased value, quality and accountability is what effectively led
to MOC.2
The idea of MOC was met with a fair amount of controversy. While
most physicians support the need to demonstrate their ongoing competence through formal MOC programs, more extensive debates have
focused on how to develop MOC methods, how to demonstrate
competency, and how to pay for these initiatives.2 The cost-effectiveness of the MOC is also a point of debate. Many physicians were not
interested in the time and financial burdens associated with the MOC,
given the uncertain benefits and questionable importance. However,
the risk of government agencies bypassing the ABMS to institute their
own regulations exists, particularly if they are not satisfied with the
stringency of the current MOC. There is also a conflict between the
desire for change and the transparency of cost effectiveness data on
various MOC components.
Substantial evidence exists for the need of MOC; studies have
shown that up to 12% of physicians fail to maintain standards and
patients receive approximately 50% of the care that is indicated for
their specific conditions.1 The data consistently shows that physician
knowledge and skills, evidence based medicine and outcomes decline, while adverse actions by state licensing boards increase as the
time from medical training increases.
While various specialty boards are approaching the MOC differently, all MOC programs consist of 4 parts:3
1.
2.
3.
4.
Professional standing
Continuing education and self-assessment
Cognitive experience
Performance in practice
Each part is meant to enhance physician competence with a goal
towards improvement in quality of care and patient outcomes. For
MOC part 1, data shows that physician communication skills impact
patient satisfaction, quality of life, and outcomes, and have been
linked to malpractice as well as state licensing board actions.1 Patient
feedback has also been shown to motivate providers to improve
communication skills. The feedback from peers and other medical
personnel provide reliable, valid information for professional development. Although the data is skewed toward higher ratings, there
exists enough variability to distinguish performance levels. Physicians have responded favorably to feedback from peers and coworkers. Both peer and patient assessments may be implemented
with time to enhance and maintain professional standing.1 Currently
the ABOto requires that all of its diplomats possess a valid ABOto
certificate and hold an unrestricted medical license in each state that
they practice medicine. Adverse actions taken by state licensing
boards, hospitals or others against ABOto diplomats can be communicated to the ABOto via the Disciplinary Alert Notification System
(DANS).1 This process allows the ABOto to maintain and enforce
standards on its board certified diplomats.
American Journal of Rhinology & Allergy
Table 13: MOC Part III Subsecialty Areas
General Otolaryngology
Head and Neck surgery
Otology
Allergy
Pediatrics/bronchoesophagology
Laryngology
Rhinology
Facial plastic surgery
Neurotology*
Sleep Medicine*
*For individuals subcertified in these areas
Part II of the MOC process focuses on education and learning. There
is increasing data that lifelong learning and continuing medical education (CME) improves physician performance and outcomes.1 Live sessions with multiple media types provide the greatest positive results.
Currently the ABOto requires its diplomats to earn as many category 1
CME credits as are required by their state licensing boards. In those
states in which there is no CME credit requirement, the ABOto requires
a minimum of 15 hours of category 1 CME credits.3 60% of these CME
credits must pertain to the specialty of Otolaryngology – Head and Neck
surgery. In the future the ABOto may implement patient simulation
on-line modules that will allow certified Otolaryngologists to participate
in interactive patient cases. Their clinical decisions in these cases can be
assessed and areas of deficiencies can be identified in order to provide
feedback to assist with ongoing learning.3
The MOC Part III involves medical knowledge and clinical diagnostic reasoning. It has been shown that physicians need strong
knowledge and clinical skills in order to appropriately synthesize
data, a skill required while making differential diagnoses. Research
shows that failure to acquire new knowledge, declining cognitive
skills, and inaccuracies in self-assessment support the need for periodic assessment.1 Exam scores correlate with other measures of clinical performance, as well as peer assessment. Board certification has
been shown to be associated with better quality of care and outcomes.
Currently at the end of a 10 year cycle following initial board certification, diplomats must pass a computer based multiple choice examination. This examination may be made available to the MOC
participants three years prior to the expiration of their certificate so
that the individual has three opportunities to pass the examination.
There are two components to the exam. The first part is a fundamental
module that consists of questions that all otolaryngologists should
know such as fluid management, ethics, antibiotics, anesthesia and
patient safety. The second module is a specialty module that the
examinee can choose based on the focus of their practice. These
subspecialty modules are outlined in Table 1.
The MOC part IV is specifically designed to help physicians assess
and improve the quality of safety of their practice and health care in
general. Physicians can meet these requirements either through assessment of their own practices using performance based methods or
involvement in group, institutional or national QI projects. With the
growing changes in health care and initiatives by the Federal government to regulate health care delivery, quality and costs, this component of the MOC will certainly grow in importance and new initiatives by ABMS and ABOto will likely be ongoing.1
The purpose of this supplement is to provide the reader with
review articles4–27 and associated board style questions. This MOC
Review (available at www.AJRA.com) should help Otolaryngologists
Copyright © Oceanside Publications, Inc. All rights reserved. DO NOT COPY
For permission to copy go to https://www.oceansidepubl.com/permission.htm
S1
to study and enhance their medical knowledge in order to prepare
them for the MOC part III 10 year examination. Specifically this MOC
Review is meant to focus on topics relating to the subspecialties of
Allergy and Rhinology.
Douglas D. Reh, M.D.
David Poetker, M.D.
REFERENCES
1.
2.
3.
4.
5.
6.
7.
8.
9.
10.
11.
12.
S2
Hawkins RE, Lipner RS, Ham HP, et al. American board of medical
specialties maintenance of certification: Theory and evidence regarding the current framework. J Contin Educ Health Prof 33(S1):S7–S19,
2013.
Van Harrison R, and Olson CA. Evolving health care systems and
approaches to maintenance of certification. J Contin Educ Health Prof
33(S1):S1–S4, 2013.
Miller RH. Certification and maintenance of certification in otolaryngology – head and neck surgery. Otolaryngol Clin North Am 40:
1347–1357, 2007.
Dalgorf DM, Harvey RJ. Chapter 1: Sinonasal anatomy and function.
Am J Rhinol Allergy. 27 Suppl 1:S3–6, 2013.
Osborn JL, Sacks R. Chapter 2: Nasal obstruction. Am J Rhinol
Allergy. 27 Suppl 1:S7–8, 2013.
Sacks R, Sacks PL, Chandra R. Chapter 3: Epistaxis. Am J Rhinol
Allergy. 27 Suppl 1:S9–10, 2013.
Settipane RA, Peters AT, Chandra R. Chapter 4: Chronic rhinosinusitis. Am J Rhinol Allergy. 27 Suppl 1:S11–5, 2013.
Settipane RA, Peters AT, Chiu AG. Chapter 6: Nasal polyps. Am J
Rhinol Allergy. 27 Suppl 1:S20–5, 2013.
Laury AM, Wise SK. Chapter 7: Allergic fungal rhinosinusitis. Am J
Rhinol Allergy. 27 Suppl 1:S26–7, 2013.
Duggal P, Wise SK. Chapter 8: Invasive fungal rhinosinusitis. Am J
Rhinol Allergy. 27 Suppl 1:S28–30, 2013.
Hennessey PT, Reh DD. Chapter 9: Benign sinonasal neoplasms.
Am J Rhinol Allergy. 27 Suppl 1:S31–4, 2013.
Harvey RJ, Dalgorf DM. Chapter 10: Sinonasal malignancies. Am J
Rhinol Allergy. 27 Suppl 1:S35–8, 2013,
13.
14.
15.
16.
17.
18.
19.
20.
21.
22.
23.
24.
25.
26.
27.
Kohanski MA, Reh DD. Chapter 11: Granulomatous diseases and
chronic sinusitis. Am J Rhinol Allergy. 27 Suppl 1:S39–41, 2013.
Chaaban MR, Kejner A, Rowe SM, Woodworth BA. Cystic fibrosis
chronic rhinosinusitis: A comprehensive review. Am J Rhinol Allergy. 27(5):387–95, 2013.
Chandran SK, Higgins TS. Chapter 5: Pediatric rhinosinusitis: definitions, diagnosis and management–an overview. Am J Rhinol Allergy. 27 Suppl 1:S16–9, 2013.
Higgins TS, Lane AP. Chapter 12: Surgery for sinonasal disease. Am J
Rhinol Allergy. 27 Suppl 1:S42–4, 2013.
Simon P, Sidle D. Augmenting the nasal airway: beyond septoplasty.
Am J Rhinol Allergy. 26(4):326–31, 2012.
Meen EK, Chandra RK. The role of the nose in sleep-disordered
breathing. Am J Rhinol Allergy. 27(3):213–20, 2013.
Gaines A. Chapter 13: Olfactory disorders. Am J Rhinol Allergy. 27
Suppl 1:S45–7, 2013.
Settipane RA, Kaliner MA. Chapter 14: Nonallergic rhinitis. Am J
Rhinol Allergy. 27 Suppl 1:S48–51, 2013.
Settipane RA, Schwindt C. Chapter 15: Allergic rhinitis. Am J Rhinol
Allergy. 27 Suppl 1:S52–5, 2013.
Settipane RA, Borish L, Peters AT. Chapter 16: Determining the role
of allergy in sinonasal disease. Am J Rhinol Allergy. 27 Suppl 1:S56–8,
2013.
Feng CH, Miller MD, Simon RA. The united allergic airway: connections between allergic rhinitis, asthma, and chronic sinusitis. Am J
Rhinol Allergy. 26(3):187–90, 2012.
Settipane RA, Peters AT, Borish L. Chapter 17: Immunomodulation
of allergic sinonasal disease. Am J Rhinol Allergy. 27 Suppl 1:S59–62,
2013.
Wise SK, Schlosser RJ. Subcutaneous and sublingual immunotherapy
for allergic rhinitis: what is the evidence? Am J Rhinol Allergy.
26(1):18–22, 2012.
Lieberman P. The risk and management of anaphylaxis in the setting
of immunotherapy. Am J Rhinol Allergy. 26(6):469–74, 2012.
Zuraw BL, Christiansen SC. Pathophysiology of hereditary angioedema. Am J Rhinol Allergy. 25(6):373–8, 2011.
May–June 2014, Vol. 28, No. 3
Copyright © Oceanside Publications, Inc. All rights reserved. DO NOT COPY
For permission to copy go to https://www.oceansidepubl.com/permission.htm
Sinonasal anatomy and function
Dustin M. Dalgorf, M.D., and Richard J. Harvey, M.D.
ABSTRACT
An understanding of paranasal sinus anatomy based on important fixed landmarks rather than variable anatomy is critical to ensure safe and complete
surgery. The concept of the paranasal surgical box defines the anatomic limits of dissection. The boundaries of the surgical box include the middle turbinate
medially, orbital wall laterally, and skull base superiorly. The “vertical component” of the surgical box defines the boundaries of the frontal recess and includes
the middle turbinate and intersinus septum medially, medial orbital wall and orbital roof laterally, nasofrontal beak anteriorly, and skull base and posterior
table of frontal sinus posteriorly. The paranasal sinuses are divided into anterior, posterior, and sphenoidal functional cavities based on their distinct drainage
pathways into the nose. The ultimate goal of surgery is to create a functional sinus cavity. Application of the paranasal surgical box and its vertical component
enables the surgeon to view the limits of dissection with a single position of the endoscope. This will ensure complete dissection of the functional sinonasal
compartments and effectively avoid leaving behind disconnected cells from the surgical cavity, mucocele formation, mucous recirculation, overcome obstructive
phenomenon and enable maximal delivery of topical therapy in the post-operative setting. This article reviews the structure and function of the nasal cartilages
and turbinates. It also describes the concept of the paranasal surgical box, key anatomical landmarks and limits of dissection. Normal anatomy and common
variants of normal anatomy are discussed.
A
thorough understanding of sinus anatomy is critical to ensure
safe and complete endoscopic sinus surgery. Surgery on the
paranasal sinuses is an exercise of anatomic dissection. The sinus
surgeon must successfully identify certain key anatomic landmarks to
delineate the limits of dissection. These defined anatomic limits establish the boundaries to which paranasal sinus surgery is confined.
The concept of the paranasal surgical box forms the basic framework
of functional endoscopic sinus surgery. The ultimate goal of surgery
is to be able to visualize these limits with a single position of the
endoscope. This will ensure removal of obstructive phenomenon,
postsurgical mucociliary function that is free of recirculation, and
maximal delivery of topical therapy to the sinus cavity in the postoperative setting.
The boundaries of the paranasal sinus box include the middle
turbinate medially, orbital wall laterally (lamina papyracea), and
skull base superiorly. Within the confines of this box, a series of
pneumatized air cells and variants of this normal anatomy must be
dissected. The boundaries of the vertical component of this box define
the frontal sinus recess and include the middle turbinate and intersinus septum medially, orbital wall laterally, nasofrontal beak anteriorly, and skull base and posterior table of the frontal sinus posteriorly.
Although much focus is placed on the internal turbinosinus anatomy, the anterior third of the nasal passage has a critical functional
role and can greatly influence nasal airflow. The anterior third of the
nasal passage is comprised of the nasal cartilages.
NASAL CARTILAGES
The nasal cartilages are structures consisting of hyaline cartilage
that attach to the bones of the anterior nasal aperture to form the
From the Applied Medical Research Centre, St. Vincent’s Hospital, University of New
South Wales, and Macquarie University, Darlinghurst, Sydney, Australia
The authors have no conflicts of interest to declare pertaining to this article
Address correspondence and reprint requests to Richard J. Harvey, M.D., 354 Victoria
Street, Darlinghurst, New South Wales, Australia, 2010
E-mail address: [email protected]; alternative: [email protected]
Originally published in Am J Rhinol Allergy 27, S3–S6, 2013
Copyright © 2014, OceanSide Publications, Inc., U.S.A.
American Journal of Rhinology & Allergy
skeletal framework of the external nose. These cartilages include the
upper and lower lateral cartilages, septum, and sesamoid complex.
The nasal septum has a bony portion comprised of the perpendicular
plate of the ethmoid bone, vomer, and maxillary crest and the quadrangular cartilage forms the cartilaginous portion of the nasal septum.
The lower lateral cartilage is divided into medial, intermediate, and
lateral crus that form the natural arch of the nasal ala. The upper
lateral cartilages are trapezoid-shaped cartilages that attach to the
dorsal septum in the midline, nasal bones cranially, and the lower
lateral cartilages caudally via the “scroll” area. All of this anatomy is
located anterior to the piriform aperture (Fig. 1).
The relationship and architecture of these cartilages form the external and internal nasal valves, which are critical to nasal airflow
(Fig. 1). The external nasal valve is comprised of the septum medially,
ala rim (lateral crus and fibrofatty/sesamoid complex) laterally, and
the nasal sill inferiorly. The internal nasal valve is bounded by the
septum medially, caudal edge of the upper lateral cartilage, and inferior
turbinate head laterally. Both cartilage integrity and structural and anatomic abnormalities in the nasal cartilages cause nasal airflow obstruction through a narrow space, valve stenosis, or a resulting dynamic valve
collapse that is exacerbated during inspiration.
NASAL TURBINATES
The inferior, middle, and superior nasal turbinates are internal
structures found along the lateral nasal wall. The middle and superior
turbinates arise from extensions of the ethmoid bones whereas the
inferior turbinate is an embryologically independent osseus structure.
The space between the lateral nasal wall and the inferior, middle, and
superior turbinates is called the inferior, middle, and superior meatus, respectively (Fig. 2). Turbinates are structures filled with vascular channels and venous sinusoids that serve to warm and humidify
air and modify nasal airflow resistance. The turbinates continuously
dilate and constrict under sympathetic control in response to environmental conditions. A process occurs every 0.5–3 hours in a normal
physiological phenomenon known as the “nasal cycle” resulting in
alternating congestion and decongestion of the nasal cavities. Turbinate hypertrophy is a common cause of nasal obstruction in which the
turbinates are either chronically congested or hypertrophied because
of allergic or nonallergic triggers as part of inflammatory rhinitis
conditions.
Copyright © Oceanside Publications, Inc. All rights reserved. DO NOT COPY
For permission to copy go to https://www.oceansidepubl.com/permission.htm
S3
Figure 1. The left internal nasal valve. (A) Sagittal CT scan
with green line indicating the level of the internal nasal valve.
(B) Endoscopic view of the internal nasal valve including (1)
septum, (2) upper lateral cartilage, (3) head of inferior turbinate, and (4) lateral crus as part of the external nasal valve.
Figure 2. The left middle meatus. (A) Coronal CT scan
including the maxillary sinus outflow tract (yellow arrow),
fontanelle (asterisk), medial orbital wall (arrow heads), and
Haller cell (white arrow). (B) Endoscopic view of the middle
meatus including (1) septum, (2) middle turbinate, (3) uncinate process, (4) ethmoid bulla, and (5) inferior turbinate.
PARANASAL SINUSES
The paranasal sinuses are paired structures lined by ciliated
pseudostratified columnar respiratory epithelium identical to that
in the lower airway. The cilia beat in a coordinated fashion to carry
the mucous blanket that traps particles from the sinus into the nose
through a series of well-defined pathways. The paranasal sinuses
are divided into anterior, posterior, and sphenoid functional cavities based on these drainage pathways. The anterior functional
cavity is comprised of the maxillary, anterior ethmoid, and frontal
sinuses. These sinuses drain into the nose through the ostiomeatal
complex in the middle meatus. The posterior ethmoid drains into
the nose through the superior meatus and the sphenoid sinus
drains through the sphenoethmoid recess located behind the superior turbinate and lateral to the septum. It is important to
understand that although the anterior and posterior ethmoid cavities share a common name, they are completely separate functional entities with different drainage pathways.
The Anterior Functional Cavity
Uncinate Process and Maxillary Sinus. The uncinate process is a
sickle-shaped bone that attaches inferiorly to the inferior turbinate
and palatine bone and anterosuperiorly to the lacrimal bone. The
posterosuperior attachment will be discussed later along with the
frontal sinus. The true maxillary ostial opening is covered by
the uncinate process and can not be viewed endoscopically in a
sinus cavity that has not been previously operated on. The uncinate together with a fold of mucosa called the fontanelle (anterior
and posterior) cover the opening to the maxillary sinus. Accessory
ostia may be present in the fontanelle that can be mistaken for the
true maxillary ostium. Failure to correctly identify the true ostia
during any part of sinus surgery will result in a phenomenon
known as mucous recirculation. During recirculation, mucous is
directed toward the natural opening along the mucociliary drain-
S4
age pathway and reenters the sinus through the accessory ostium.
On completing the maxillary antrostomy, the first landmark that
must be established is the roof of the maxillary sinus. This key
landmark defines the floor of the orbit, which is critical in surgical
orientation.1
Ethmoid Bulla. The ethmoid bulla is the largest and most consistent anterior ethmoid air cell. It attaches to the lamina papyracea
laterally and has variable attachments to the skull base and basal
lamella, creating a series of clefts and spaces within the middle
meatus that are well described but have little significance clinically. The reader is referred to an article by Kennedy and Stamberger on sinus anatomy nomenclature for a detailed description.2
A variant of normal anatomy in this region is called a Haller cell
(infraorbital ethmoid cell), which is an anterior ethmoid cell that
pneumatizes into the maxillary sinus causing obstruction (Fig. 3).
Removal of the ethmoid bulla is critical to define the medial orbital
wall.
Middle Turbinate. The basal lamella is formed by the second
segment of the middle turbinate. The complex shape of the middle
turbinate is divided into three segments according to the sagittal,
coronal, and axial planes to which it is oriented. The basal lamella
is the component that separates the anterior and posterior ethmoid
cavities. This partition is not smooth, because it represents posterior projections of anterior ethmoid air cells and anterior projections of posterior ethmoid air cells. The safe working distance from
the skull base established by the maxillary sinus roof is used as a
reference point to proceed through the basal lamella and enter the
posterior ethmoid cavity because there is no natural connection
between the two cavities.1 The middle turbinate is highly variable
in size, shape, and often absent secondary to disease or prior
surgery and should rarely be used to guide surgery.
Frontal Sinus. To dissect the frontal recess to its anatomic limits,
one must understand the concept of the vertical component of the
May–June 2014, Vol. 28, No. 3
Copyright © Oceanside Publications, Inc. All rights reserved. DO NOT COPY
For permission to copy go to https://www.oceansidepubl.com/permission.htm
Figure 3. Left sphenoethmoidal cavity and left frontal
recess (A) preoperative CT axial view with bony partitions intact; (B) postoperative CT axial view with removal
of bony partitions along the medial orbital wall and skull
base; (C) endoscopic view of the paranasal surgical box
with limits of anatomic dissection including middle turbinate (thin arrow), medial orbital wall (arrowhead), skull
base (asterisk), and sphenoid sinus (thick arrow); (D)
preoperative CT sagittal view with bony partitions intact;
(E) postoperative CT sagittal view with removal of bony
partitions from skull base and frontal recess; and (F)
endoscopic view of the frontal recess with demonstration
of the “vertical component” of the surgical box boundaries
including (1) medial orbital wall, (2) orbital roof, (3)
posterior table of frontal sinus, (4) skull base, (5) intersinus septum, and (6) middle turbinate.
surgical box that defines the boundaries of the frontal recess (Fig.
3). To define the limits of the frontal recess or vertical component
of the surgical box, the endoscopic surgeon must consider the
various cells that may encroach on this space from the anterior,
posterior, medial, and lateral directions.
Agger Nasi, Posterosuperior Uncinate Process, and Frontal Ethmoidal
Cells. Anterior structures encroaching on the frontal recess include the
agger nasi, lateral uncinate process, and frontal cells. The agger nasi
is the anterior most ethmoid air cell and its medial boarder is formed
by the uncinate process.3 The uncinate process can insert into the
medial orbit, skull base, or middle turbinate. Recent studies have
indicated that the uncinate has multiple attachments in ⬎50% of
cases.4,5 Classic teaching describing three distinct posterosuperior
attachments of the uncinate process, which determines the direction
of the frontal sinus drainage pathway, is neither surgically relevant
nor accurate. The frontal recess is medial to the uncinate in 85% of
cases as medial insertion of the uncinate to the orbita is present.4
Thus, an uncinate with only an attachment to the skull base or lateral
lamella (15% of cases) leads to a surgically obvious frontal drainage
lateral to the uncinate process.4 The degree of pneumatization of the
agger nasi influences the position of the superior uncinate process
attachment as well as the thickness of the bony nasofrontal beak.
Frontal cells are ethmoid cells that pneumatize above the agger
nasi toward the frontal sinus. According to the Bent and Kuhn
classification, a type 1 frontal cell is a single frontal ethmoidal cell
above the agger nasi, type 2 is a tier of cells above the agger nasi,
type 3 is a cell pneumatizing into the floor of the frontal sinus, and
type 4 is an isolated frontal ethmoidal cell within the frontal sinus.
Wormald further modified this classification using multiplanar
reconstructed imaging to more accurately describe type 3 cells as
frontal ethmoidal cells that fill ⬍50% of the frontal sinus whereas
type 4 cells filled ⬎50% of the frontal sinus.6 The nomenclature of
such anatomy is academic and designed to ensure that the surgeon
does not dissect into pneumatized air cells and mistake it for the
frontal sinus.
Supraorbital Ethmoid and Suprabulla Cells. Posterior structures encroaching on the frontal recess include supraorbital ethmoid cells,
suprabulla cells, and the ethmoid bulla. Supraorbital ethmoid cells
are anterior ethmoid air cells that extend superiorly and laterally
over the orbital roof. These cells are recognized on imaging giving
the appearance of a septated frontal sinus on coronal view and a
cell located posterior and lateral to the frontal sinus on axial view.
Supraorbital ethmoid cells have three clinically significant features: first, they can obstruct the frontal recess; second, they can be
falsely mistaken for the true frontal sinus leading to incomplete
surgery; and, finally, they are associated with a low-lying anterior
ethmoid artery because they pneumatize from the skull base be-
American Journal of Rhinology & Allergy
hind the artery. Suprabulla or frontal bulla cells are pneumatized
extensions above the ethmoid bulla up the skull base and on the
posterior table of the frontal sinus. These cells can become quite
large and can be mistaken for the skull base or frontal sinus.
Failure to recognize this preoperatively will also result in incomplete surgical dissection of the frontal recess.
Medial structures encroaching on the frontal recess include intersinus septal cells and a medially inserting uncinate process.
Intersinus septal cells represent pneumatization of the frontal sinus septum. Lateral encroaching structures include frontal cells,
agger nasi and a lateral uncinate process attachment.
The Posterior Functional Cavity
The posterior functional cavity includes the posterior ethmoid
air cells. On passing the basal lamella of the middle turbinate, the
posterior ethmoid sinus is entered. A variant of normal anatomy in
this region is a lateral pneumatization of a posterior ethmoid cell
called an Onodi cell. The clinical significance of this cell is that it
can pneumatize over the optic nerve exposing it to injury during
surgery. These cells can also be mistaken for the true sphenoid
sinus leading to incomplete surgery if not recognized.
The Sphenoid Functional Space
The second key anatomic landmark during surgery is identification of the sphenoid sinus. The sphenoid ostium is located
within the sphenoethmoid recess behind the superior turbinate at
the level of the maxillary sinus roof.1 Identification of the sphenoid
sinus assists the surgeon in determining the level of the skull base
posteriorly at its lowest position. The main contents of the sphenoid sinus include the optic nerve, carotid artery, and sella turcica
where the pituitary gland is located.
CLINICAL PEARLS
• A thorough analysis of preoperative imaging is important to recognize variants of normal anatomy and areas for potential complication to perform safe and complete surgery
• Identification of the anatomic limits of dissection applying the
concept of the paranasal box (with its vertical “frontal” component)
delineates the surgical boundaries and provides a basic framework
for endoscopic sinus surgery
• Early identification of key anatomic landmarks of the maxillary
sinus roof (orbital floor), medial orbital wall, and sphenoid sinus
(thus posterior skull base) helps to guide the surgeon throughout
the procedure and is critical during revision cases
Copyright © Oceanside Publications, Inc. All rights reserved. DO NOT COPY
For permission to copy go to https://www.oceansidepubl.com/permission.htm
S5
• The ultimate goal of surgery is to create a functional sinonasal
cavity. Partial posterior ethmoid dissection will fail to achieve
this end point. The ability to view the limits of dissection with a
single position of the endoscope will avoid leaving behind disconnected cells from the surgical cavity, mucocele formation,
mucus recirculation, overcome obstructive phenomenon, and enable maximal delivery of topical therapy in the postoperative
setting
REFERENCES
1.
S6
Harvey RJ, Shelton W, Timperley D, et al. Using fixed anatomical
landmarks in endoscopic skull base surgery. Am J Rhinol Allergy
24:301–305, 2010.
2.
Stammberger HR, Kennedy DW and Anatomic Terminology G. Paranasal sinuses: Anatomic terminology and nomenclature. Ann Otol
Rhinol Laryngol Suppl 167:7–16, 1995.
3. Wormald PJ. The agger nasi cell: The key to understanding the anatomy
of the frontal recess. Otolaryngol Head Neck Surg 129:497–507, 2003.
4. Zhang L, Han D, Ge W, et al. Anatomical and computed tomographic
analysis of the interaction between the uncinate process and the agger
nasi cell. Acta Otolaryngol 126:845–852, 2006.
5. Stamm A, Nogueira JF, Americo RR, and Solferini Silva ML. Frontal
sinus approach: The “vertical bar” concept. Clin Otolaryngol 34:407–
408, 2009.
6. Kew J, Rees GL, Close D, et al. Multiplanar reconstructed computed
tomography images improves depiction and understanding of the anatomy of the frontal sinus and recess. Am J Rhinol 16:119–123, 2002.
e
May–June 2014, Vol. 28, No. 3
Copyright © Oceanside Publications, Inc. All rights reserved. DO NOT COPY
For permission to copy go to https://www.oceansidepubl.com/permission.htm
Nasal obstruction
Jodi L. Osborn, B.Sc. (Med.), M.B.B.S.,1 and Raymond Sacks, F.C.S., F.R.A.C.S.2,3
ABSTRACT
Nasal obstruction is one of the most common presenting symptoms requiring medical attention at both the primary care physician and the otorhinolaryngologists’ level. Nasal obstruction may be caused by anatomic, physiological, or neurologic factors. Complexity is added to this situation because, often, the
causation may be multifactorial. Nasal obstruction is the primary symptom of persistent allergic rhinitis (AR) and affects up to 40% of the population. AR
must be treated throughout the year; thus, treatment choices and patient compliance must be considered. It is interesting to note that AR directly affects an
individual’s quality of life, quality of sleep, and workplace efficiency. Therefore, the magnitude of costs to both the individual and the society must be recognized.
Various medical therapies and surgical techniques will be described for the treatment of nasal obstruction.
N
asal obstruction is a common presenting symptom to both primary care physicians and otorhinolaryngologists. It may be
described as a discomfort, manifested as a sensation of insufficient
airflow through the nose.1 A range of anatomic, physiological, and
neurological/iatrogenic factors may cause nasal obstruction. It is
therefore pertinent to always complete a detailed history and thorough physical examination of each patient to appropriately diagnose
the cause of their nasal obstruction, remembering it may be multifactorial. Fraser and Kelly2 describe 11 points regarding nasal obstruction that should be covered on history: (1) bilateral, unilateral, or
alternating obstruction; (2) duration of symptoms; (3) seasonal or
diurnal variations; (4) associated nasal symptoms; (5) sense of smell;
(6) other medical problems; (7) previous nasal surgery or trauma; (8)
current and past medications; (9) illicit drug, alcohol, and tobacco use;
(10) pregnancy; and (11) work/occupation.2
Examination should include assessment of external features of the
nose for bony and cartilaginous deformities and evaluation for tip
ptosis.2 The nasal valve is the anatomic region first described by Mink
in 1903.3 It is the narrowest part of the nasal cavity and is a twodimensional opening between the caudal edge of the upper lateral
cartilage and the nasal septum, with a normal angle of 15–20°.4 The
internal nasal valve is what Mink originally described, whereas the
external nasal valve is the nasal ala and its relationship to the nasal
septum.4 Both must be reviewed for sites of obstruction, assessed
using Cottle’s maneuver, modified Cottle’s maneuver, and, later,
nasendoscopy.4 Anterior rhinoscopy is performed with a Thudichum
speculum and a head light before any decongestant therapy2; evaluating the septum, floor of nose, inferior and middle turbinate size and
mucosal surface; and identifying any masses, polyps or foreign bodies. Nasendoscopy requires decongestant spray and a flexible or rigid
nasendoscope2 performed to further assess mucosal condition. If topical decongestant objectively or subjectively relieves the obstruction,
this often correlates with an inflammatory condition as an underlying
cause. Endoscopy should note presence of any polyps, granulomaFrom the 1Sydney Adventist Hospital, Hornsby, Australia, 2Australian School of
Advanced Medicine at Macquarie University and 3Sydney Medical School at University of Sydney, Sydney, Australia
The authors have no conflicts of interest to declare pertaining to this article
Address correspondence and reprint requests to Raymond Sacks, F.C.S., F.R.A.C.S.,
Sydney Adventist Hospital, The ENT Centre, Suite 12, 25–29 Hunter Street, Hornsby,
NSW, Australia, 2077
E-mail address: [email protected]
Originally published in Am J Rhinol Allergy 27, S7–S8, 2013
Copyright © 2014, OceanSide Publications, Inc., U.S.A.
American Journal of Rhinology & Allergy
tous disease, abnormalities of middle turbinate or posterior inferior
turbinate, postnasal space, and postsurgical changes such as adhesion
formation or surgical scarring.5 The neck should be examined for
lymphadenopathy, particularly if malignancy is suspected.2 Objective
assessment of nasal obstruction includes acoustic rhinometry, nasal
peak flow, and rhinomanometry. Although commonly used by investigators during studies, these measures are not universally available
or accepted by clinicians; hence, they may be of minimal benefit in
medical decision making.6 Investigations may include blood tests for
systemic diseases or allergies (radioallergosorbent testing), skin-prick
tests, sinonasal swabs if pus is seen, CT of the paranasal sinuses and
magnetic resonance imaging if sinonasal neoplasm is suspected or if
the patient has signs of possible central nervous system disease.2
Anatomic causes of nasal obstruction include septal deviations,
either congenital or acquired, turbinate hypertrophy, or internal or
external nasal valve collapse or stenosis.5 When considering causes in
children, consideration should also be given to adenoid hypertrophy
obstructing the posterior choanae.7 Rare causes of nasal obstruction
include foreign bodies within the nose and benign or malignant
masses. Physiological causes of nasal obstruction include allergic
rhinitis (AR; IgE mediated) or nonallergic rhinitis, including inflammatory, infective (viral or bacterial), hormonal, autonomic, druginduced, systemic disease associated, or occupational.5 AR affects up
to 40% of the population and is characterized by nasal obstruction,
sneezing, clear/watery rhinorrhea, nasal itch, and ocular symptoms.8
In nonallergic rhinitis, both nasal itch and ocular symptoms, are rare.
Nonallergic triggers include perfumes, cold air, weather change,
smoke, and chemicals.9 Neurological causes of nasal obstruction include rhinitis medicamentosa, atrophic rhinitis, and empty nose syndrome (ENS). Rhinitis medicamentosa is a medication-induced rhinitis causing rebound nasal congestion due to overuse of topical
decongestants, specifically ␣-adrenergic agents.5 Atrophic rhinitis is a
chronic nasal pathology that can be classified as either primary or
secondary in origin.10 Symptoms include nasal crusting and obstruction, foetor, epistaxis, anosmia or cacosmia, secondary infection, and
nasal deformity.10 Classically, these symptoms are found in a roomy
nasal cavity resulting from progressive atrophy of nasal mucosa and
underlying bone.10 ENS is a rare complication of nasal or sinus
surgery, in particular of inferior turbinectomy,11 which was first
described by Kern and Moore in 1994.12 ENS has no consensual
definition but is distinguished from atrophic rhinitis and is associated
with loss of normal endonasal anatomy, especially the absence of one
or more turbinates.13 “Paradoxical” nasal obstruction is commonly
described by the patient, despite objective examination findings of
permeable cavities with no obstacle seen clinically or on CT imaging,
Copyright © Oceanside Publications, Inc. All rights reserved. DO NOT COPY
For permission to copy go to https://www.oceansidepubl.com/permission.htm
S7
acoustic rhinometry, or rhinomanometry.11 The pathophysiology of
ENS is currently unknown, although several hypotheses exist including loss of physiological nasal functions because of decreased mucosal area, causing a lack of humidification, warming, and filtering of
inhaled air.11 This in turn reduces sensory, tactile, and thermal receptors within the nose.11 Central involvement is currently under study,
and debate concerns the frequent association with psychiatric disorder and psychosomatic pathologies.13
Surgical treatments for anatomic causes of obstruction include septoplasty for septal deviations/spurs. Moore and Eccles concluded
that septal surgery does improve objective measures of nasal patency.14 Surgery on the inferior turbinates remains contentious and is
offered to patients with turbinate hypertrophy and severe nasal obstruction despite maximum medical management.2 Current surgery
remains more conservative than previous times, with a shift toward
mucosal sparing techniques, to maintain normal turbinate function,
such as powered submucosal turbinoplasty.2 No randomized control
trials of inferior turbinate surgery versus continued medical therapy
for AR or comparisons between two differing techniques after maximal medical treatments currently exist.15 Treatment options for nasal
valve dysfunction are primarily surgical and are designed to either
correct laxity of the lateral nasal wall or to enlarge the cross-sectional
area of the nasal valve.4 Currently, there is a trend toward minimally
invasive techniques, although open rhinoplastic approaches remain a
robust option.4 Surgical options include any combination of (1) autologous cartilage grafts—alar batten, alar strut, spreader, splay, and butterfly; (2) suspension suture techniques—to lateralize or strengthen the
lateral component of the nasal valve region; and (3) alloplastic implants—titanium, polyethylene implants. Autologous materials are
preferred wherever possible.
Treatment of inflammatory physiological causes of nasal obstruction are primarily medical therapies, although surgery may be needed
as an additional intervention.8 International guidelines recommend
the management of AR and include a combination of treating both the
upper and the lower airways, using patient education, allergy avoidance, pharmacologic treatment, and specific immunotherapy.8 Specific pharmacologic therapies include (1) antihistamines—oral or topical, (2) intranasal glucocorticosteroid, (3) leukotriene antagonists,
and (4) decongestants—oral or topical. Topical decongestants (␣1agonists) are very effective, although prolonged use risks rebound
vasodilatation and rhinitis medicamentosa. Intranasal glucocorticosteroids are first-line pharmacotherapy for obstructive symptoms of
rhinitis. Treatment of nonallergic rhinitis is very similar, involving
patient education, avoiding triggers, oral decongestants, and intranasal glucocorticosteroids. Notably, cotreatment with intranasal antihistamines displays synergistic effects compared with monotherapy.15
Patients with chronic rhinosinusitis, especially those with diffuse
sinonasal polyposis, may manifest nasal obstruction as a significant
symptom. This may require functional endoscopic sinus surgery if
there is failure of medical therapies.
Neurological causes of nasal obstruction are primarily treated with
patient reassurance. Treatment of rhinitis medicamentosa also requires weaning the patient off the causative agent over 7–10 days
while commencing an oral or intranasal glucocorticosteroids.8 Atrophic rhinitis requires conservative treatment including (1) saline irrigation and douches, (2) nose drops—glucose-glycerine and liquid
paraffin, (3) antibiotics and antimicrobials, (4) vasodilators, and (5)
prosthesis.10 If surgical treatments are required, they aim to reduce
the size of the nasal cavity, encourage regeneration of normal mucosa,
improve lubrication of dry nasal mucosa, and increase nasal cavity
vascularity.10 Treatment of ENS is primarily prevention, with current
surgical techniques now favoring a conservative approach to inferior
submucosal turbinoplasty.11 Medical therapies include (1) nasal la-
S8
vage, (2) nasal hydration ointment, (3) directed antibacterial therapy,
(4) aerosols and local corticosteroids, and (5) psychological support
for patients.11 Possible surgical treatments may include creating a
neoturbinate with a turbinal or septal cartilage graft to recreate turbinate volume with similar outcome aims to those with atrophic
rhinitis surgery.11
CLINICAL PEARLS
• Thorough history and examination of the patient is required to
correctly identify causative factors.
• Causes of nasal obstruction may be defined as anatomic, inflammatory, or neurological/iatrogenic/pseudoobstruction.
• Causes of nasal obstruction may be isolated, although are more
commonly multifactorial.
• If medical treatments are available, then maximum medical therapy
should be tried before considering surgical techniques.
• A variety of surgical techniques are available for each causative
factor.
• Because nasal obstruction is commonly multifactorial, multiple surgical techniques may be required for effective treatment of an
individual.
• Long-term treatment will often require a combination of medical
and surgical therapies.
REFERENCES
1.
2.
3.
4.
5.
6.
7.
8.
9.
10.
11.
12.
13.
14.
15.
Jessen M, and Malm L. Definition, prevalence and development of
nasal obstruction. Allergy 52(suppl 40):3–6, 1997.
Fraser L, and Kelly G. An evidence-based approach to the management of the adult with nasal obstruction. Clin Otolaryngol 34:151–
155, 2009.
Mink PJ. Le nez Comme Vioe Respiratorie [in French]. Belgium:
Presse Otolaryngol, 21:481–496, 1903.
Yarlagadda BB, and Dolan RW. Nasal valve dysfunction: Diagnosis
and treatment. Curr Opin Otolaryngol Head Neck Surg 19:25–29,
2011.
Moche JA, and Palmer O. Surgical management of nasal obstruction.
Oral Maxillofac Surg Clin North Am 24:229–237, 2012.
Rhee JS, Weaver EM, Park SS, et al. Clinical consensus statement:
Diagnosis and management of nasal valve compromise. Otolaryngol
Head Neck Surg 143:48–59, 2010.
van den Aardweg MTA, Schilder AGM, Herkert E, et al. Adenoidectomy for recurrent or chronic nasal symptoms in children. Cochrane Database Syst Rev 1:CD008282, 2010. (DOI: 10.1002/
14651858.CD008282.)
Wang DY, Tanveer Raza Md, and Gordon BR. Control of nasal
obstruction in perennial allergic rhinitis. Curr Opinn Allergy Clin
Immumol 4:165–170, 2004.
Kim YH, Oh YS, Kim KJ, and Jang TY. Use of cold dry air provocation
with acoustic rhinometry in detecting nonspecific nasal hyperreactivity. Am J Rhinol Allergy 24:260–262, 2010.
Mishra A, Kawarra R, and Gola M. Interventions for atrophic rhinitis.
Cochrane Database Syst Rev 2:CD008280, 2012. (DOI: 10.1002/
14651858.CD008280.pub2.)
Coste A, Dessi P, and Serrano E. Empty nose syndrome. Eur Ann
Otorhinol Head Neck Dis 129:93–97, 2012.
Moore EJ, and Kern EB. Atrophic rhinitis: A review of 242 cases. Am J
Rhinol 15:355–361, 2001.
Payne SC. Empty nose syndrome: What are we really talking about?
Otolaryngol Clin North Am 42:331–337:ix-x, 2009.
Moore M, and Eccles R. Objective evidence for the efficacy of surgical
management of the deviated septum as a treatment for chronic nasal
obstruction: A systematic review. Clin Otolaryngol 36:106–113, 2011.
Jose J, and Coatesworth AP. Inferior turbinate surgery for nasal
obstruction in allergic rhinitis after failed medical treatment. Cochrane Database Syst Rev 12:CD005235, 2010. (DOI: 10.1002/
14651858.CD005235.pub2.)
e
May–June 2014, Vol. 28, No. 3
Copyright © Oceanside Publications, Inc. All rights reserved. DO NOT COPY
For permission to copy go to https://www.oceansidepubl.com/permission.htm
Epistaxis
Raymond Sacks, F.C.S., F.R.A.C.S.,1,2 Peta-Lee Sacks, MBBS,3 and Rakesh Chandra, M.D.4
ABSTRACT
Epistaxis is a common problem that may range in severity from a minor nuisance to hemodynamically significant bleeding. Vascular anatomy allows for
predictable identification of suspicious bleeding sites. Historically, packing was the workhorse of management, but, currently, more directed interventions have
become available. These modalities may result in improvements in both cost-effectiveness and patient comfort.
T
he nasal mucosa is supplied by terminal branches of the external and internal carotid arteries. Various anastomoses exist between these two systems, the most important being Kiesselbach’s plexus in the anterior nasal septum. In this location,
branches of the anterior ethmoid, nasopalatine, and superior labial
arteries anastomose. There is also a contribution from the terminus
of the posterior septal artery. This the most common site of epistaxis, and ⬎90% of all epistaxis presentations occur in this area.1
These anastomoses are fundamental to an understanding of epistaxis and its management, underlying the significance of treating
the most distal site of bleeding.
Epistaxis is often characterized as either anterior or posterior. Historically, these distinctions have been arbitrary, respectively, based on
whether or not the practitioner is able to identify the site of bleeding
during anterior rhinoscopy. A more useful scheme stratifies the
source according to whether it originates anterior or posterior to the
maxillary sinus ostium. Anterior bleeding, which usually arises from
Kisselbach’s plexus, generally allows for easier visualization and
access. In contrast, posterior epistaxis, which usually arises from
branches of the sphenopalatine artery, is more difficult to visualize. In
this scenario, hemorrhage is often swallowed, resulting in difficulties
assessing the amount of blood lost.
ETIOLOGY
Epistaxis is often idiopathic but can be occasionally caused by
underlying pathology such as sinonasal tumor. As such, the following
differential should be considered (Table 1).
INITIAL ASSESSMENT
Initial assessment of epistaxis attempts to estimate the amount of
blood lost and the period over which the patient has been bleeding.
The patient’s vital signs should be assessed to exclude hypovolemic shock. It may be necessary to obtain i.v. access to check for any
From the 1Australian School of Advanced Medicine at Macquarie University, 2Sydney
Medical School at University of Sydney, Sydney, Australia, 3School of Medical Sciences, University of New South Wales, Sydney, Australia, and 4Department of Otolaryngology–Head and Neck Surgery, Northwestern University Feinberg School of
Medicine, Chicago, Illinois
R Chandra is a consultant/advisor for Intersect ENT, Gyrus/Olympus, and Sunovion
and received a research grant from Intersect ENT. P-L Sacks and R Sacks have no
conflicts of interest to declare pertaining to this article
Address correspondence and reprint request to Raymond Sacks, F.C.S., F.R.A.C.S.,
Sydney Adventist Hospital, The ENT Centre, Suite 12, 25–29 Hunter Street, Hornsby,
NSW, Australia, 2077
E-mail address: [email protected]
Originally published in Am J Rhinol Allergy 27, S9 –S10, 2013
Copyright © 2014, OceanSide Publications, Inc., U.S.A.
American Journal of Rhinology & Allergy
clotting abnormalities and to draw blood for type and screen. The
patient should be asked to apply constant pressure over the lower
cartilaginous part of the nose for ⬃20 minutes and to avoid swallowing blood in order that blood loss can be estimated.
FURTHER EVALUATION
Three steps underlie the management of epistaxis:
1. Establishing the site of bleeding
2. Stopping the bleeding
3. Treating the cause of bleeding2
The primary aims of the history are to assess the severity and
duration of the nosebleed and the circumstances in which it occurred.
The physician should also inquire about any other medical conditions, alcohol consumption, and any history of nosebleeds or bruising.
A family history of bleeding should also be considered. Examination
begins with anterior rhinoscopy. A significant amount of bleeding
may require suction, a headlamp, and a nasal speculum. Topical
vasoconstriction using 1% phenylephrine or 0.05% oxymetazoline
combined with a topical anesthetic may also be beneficial.3 If no
source of bleeding is revealed anteriorly, nasendoscopy should be
performed with particular attention to mucosal and submucosal lesions or masses within the middle meatus and nasopharynx. Laboratory investigations may be required according to the severity and
frequency of bleeding and may include a full blood count, coagulation studies, and hepatic and renal function tests. These tests may be
particularly relevant in patients taking warfarin and in those with
diseases that could result in coagulopathy. In the absence of a severe
bleed or a personal or family history suggestive of a bleeding disorder, laboratory evaluation for coagulopathy is typically not indicated.
Recurrent unilateral epistaxis that fails to respond to conservative
management should be investigated for neoplasm, particularly in
those who report symptoms of nasal obstruction, rhinorrhea, facial
pain, or an abnormal cranial nerve examination.4
MANAGEMENT
The approach to managing epistaxis tends to vary according to the
severity and location of the bleed as well as a variety of other factors.
As discussed previously, initial medical treatment aims to cease the
bleeding and is often used to improve visualization during the clinical
exam.
ANTERIOR EPISTAXIS
Should topical vasoconstriction be unsuccessful and an accessible
site of bleeding can be identified, cauterization should be implemented.5 This should be performed with caution to avoid damage to
Copyright © Oceanside Publications, Inc. All rights reserved. DO NOT COPY
For permission to copy go to https://www.oceansidepubl.com/permission.htm
S9
Table 1 Etiology of epistaxis
Local causes
Idiopathic
Trauma
Digit trauma
Foreign body
Nasal oxygen and CPAP
Nasal fracture
Postoperative
Anatomical
Septal deviation
Spurs
Inflammatory/infectious Viral/bacterial rhinosinusitis
Allergic rhinosinusitis
Granulomatous disease
Environmental irritants
Neoplastic
Hemangioma of septum or turbinates
Hemangiopericytoma
Nasal papilloma
Pyogenic granuloma
Angiofibroma
Carcinoma
Drugs
Topical intranasal corticosteroids
Cocaine abuse
Systemic causes
Inherited
Hemophilia
Von Willebrand’s disease
Hereditary hemorrhagic
telangiectasia
Platelet abnormalities
Thrombocytopenia
Platelet dysfunction
Malignancy
Leukemias
Drugs
Chemotherapy
Chronic alcoholism
Anticoagulants, e.g. Warfarin
Anti-platelet, e.g. aspirin
CPAP ⫽ continuous positive airway pressure.
zation,9 and surgical arterial ligation.3 Severe cases typically require
one of the two latter methods.10–11 Embolization is performed by a
neurointerventional radiologist. This method is ⬎85% effective but
carries risks of cerebrovascular accident if particles are released into
the internal carotid system iatrogenically or via collateral circulation.
Patients may also have postprocedural jaw discomfort and claudication. Presently, transnasal endoscopic sphenopalatine artery ligation
or cautery is the primary surgical alternative in these intractable
cases. The artery is controlled where it enters into the posterior nasal
cavity via the sphenopalatine foramen. This region is situated behind
the crista ethmoidalis, a landmark formed by the orbital process of the
palatine bone. Overall success is also in the range of 85%, and major
complications are rare. Minor complications include nasal crusting
and nasal/palatal paresthesias. There is significant debate as to which
of these methods, angiography with embolization or endoscopic
sphenopalatine artery ligation, is preferred. However, current perspectives indicate that early treatment with either of these modalities
is more successful and cost-effective than prolonged inpatient posterior packing, which is markedly uncomfortable and also carries risks
including staphylococcal infection, toxic shock syndrome, and bradydysrhythmia. Often, the algorithm for management of severe intractable posterior epistaxis will depend on the resources available at a
particular institution.
CLINICAL PEARLS
• The vast majority of epistaxis arises anteriorly, from Kiesselbach’s
plexus in the anterior nasal septum.
• Posterior epistaxis can be difficult to visualize but most often arises
from branches of the sphenopalatine artery.
• Although it may seem trivial to state, it is important to highlight
that finding the exact site of the bleeding is critical for safe and
efficient management.
• Maintain suspicion for underlying neoplastic conditions.
• Current trends favor early intervention for posterior epistaxis, with
either embolization or sphenopalatine artery ligation, rather than
prolonged packing.
REFERENCES
healthy surrounding mucosa. Cautery can be performed chemically
or electrically depending on the severity of the bleed. Silver nitrate
sticks, which release oxygen free radicals to coagulate tissue, are
useful in minor bleeding; however, it will likely be washed away by
severe bleeding before becoming effective. Electric cauterization can
be applied to anesthetized mucosa and is more useful in severe
bleeding. Laser cauterization has a limited role in acute epistaxis but
may be used in patients with hereditary hemorrhagic telangiectasia.3,6
It is important to note that cauterization by any means should be
applied only to one side of the septum to avoid perforation over a
period of 4–6 weeks.4
Failure of cauterization may indicate the need for nasal packing.5
There are various types of absorbable and nonabsorbable options.
Patients with nasal packing should commence topical antibiotics to
avoid toxic shock syndrome.3 Other complications of nasal packing
include septal hematomas, abscesses, and sinusitis. In the rare case
that anterior packing should fail, ethmoidal vessels may be ligated
through a Lynch incision. This approach reduces the risk of stroke
and blindness associated with embolization of anterior and posterior
ethmoid arteries.7
1.
2.
3.
4.
5.
6.
7.
8.
9.
POSTERIOR EPISTAXIS
10.
Posterior bleeding most commonly arises from branches of the
sphenopalatine artery,8 which is the medial (distal) termination of the
internal maxillary. The main modes of treatment in these patients
include endoscopic electric cauterization, posterior packing, emboli-
11.
S10
Douglas R, and Wormald PJ. Update on epistaxis. Curr Opin Otolaryngol Head Neck Surg 15:180–183, 2007.
Simmen D, and Jones N. Epistaxis. In Cummings Otolaryngology:
Head and Neck Surgery, 5th ed. Flint P, Haughey B, Lund V, et al.
(Ed). Philadelphia, PA: Mosby, Elsevier, 682–693, 2010.
Gifford TO, and Orlandi RR. Epistaxis. Otolaryngol Clin North Am
41:525–536, viii, 2008.
Schlosser RJ. Clinical practice. Epistaxis. N Engl J Med 360:784–789,
2009.
Kucik CJ, and Clenney T. Management of epistaxis. Am Fam Physician 71:305–311, 2005.
Harvey RJ, Kanagalingam J, and Lund VJ. The impact of septodermoplasty and potassium-titanyl-phosphate (KTP) laser therapy in the
treatment of hereditary hemorrhagic telangiectasia-related epistaxis.
Am J Rhinol 22:182–187, 2008.
Srinivasan V, Sherman IW, and O’Sullivan G. Surgical management
of intractable epistaxis: Audit of results. J Laryngol Otol 114:697–700,
2000.
Schwartzbauer HR, Shete M, and Tami TA. Endoscopic anatomy of
the sphenopalatine and posterior nasal arteries: Implications for the
endoscopic management of epistaxis. Am J Rhinol 17:63–66, 2003.
Gurney TA, Dowd CF, and Murr AH. Embolization for the treatment
of idiopathic posterior epistaxis. Am J Rhinol 18:335–339, 2004.
Christensen NP, Smith DS, Barnwell SL, and Wax MK. Arterial
embolization in the management of posterior epistaxis. Otolaryngol
Head Neck Surg 133:748–753, 2005.
Snyderman CH, Goldman SA, Carrau RL, et al. Endoscopic sphenopalatine artery ligation is an effective method of treatment for posterior epistaxis. Am J Rhinol 13:137–140, 1999.
e
May–June 2014, Vol. 28, No. 3
Copyright © Oceanside Publications, Inc. All rights reserved. DO NOT COPY
For permission to copy go to https://www.oceansidepubl.com/permission.htm
Chronic rhinosinusitis
Russell A. Settipane, M.D.,1 Anju T. Peters, M.D.,2 and Rakesh Chandra, M.D.3
ABSTRACT
Chronic rhinosinusitis (CRS) is the second most common chronic medical condition in the United States. It represents a group of disorders characterized
by inflammation of the nasal mucosa and paranasal sinuses of at least 12 weeks duration. CRS with or without nasal polyps is defined as inflammation of the
nose characterized by two or more symptoms, one of which should be either nasal blockage, obstruction, congestion, or nasal discharge (anterior/posterior nasal
drip); with or without facial pain/pressure; and/or with or without reduction or loss of smell. Symptomatology should be supported by obvious disease evident
in either nasal endoscopy or computed tomography imaging. Although CRS is not likely to be cured by either medical or surgical therapy, it can generally be
controlled. Best medical evidence supports maintenance therapy with intranasal corticosteroids and saline irrigation. For exacerbations, short to intermediate
courses of antibiotics (up to 4-weeks) with or without oral corticosteroids are recommended. For patients with difficult-to-treat CRS, functional endoscopic
sinus surgery provides an adjunctive therapeutic option.
C
hronic rhinosinusitis (CRS) represents a group of disorders characterized by inflammation of the nasal mucosa and paranasal
sinuses of at least 12 weeks duration.1 According to the European
Position Paper on Rhinosinusitis and Nasal Polyps 2012,2 rhinosinusitis with or without nasal polyps (NPs) is defined as inflammation of
the nose characterized by two or more symptoms, one of which
should be either nasal blockage, obstruction, congestion, or nasal
discharge (anterior/posterior nasal drip); with or without facial pain/
pressure; and/or with or without reduction or loss of smell. Symptomatology should be supported by obvious disease evident in either
nasal endoscopy or computed tomography (CT) imaging. Endoscopic
signs include NPs and/or purulent discharge from the middle meatus
and/or edema/mucosal obstruction of the middle meatus.3 CT findings include mucosal changes within the ostiomeatal complex and/or
sinuses.
Although multiple phenotypes exist,4,5 the literature most commonly
divides CRS into CRS without NPs (CRSsNPs) and CRS with NPs
(CRSwNPs).6 A third entity is known as allergic fungal rhinosinusitis.7
CRSsNPs is the most common form encountered in clinical practice,
accounting for ⬃60% of cases,8 and is the focus of this chapter.
ASSOCIATED OR PREDISPOSING CONDITIONS
OF CRS
Associated diseases, predisposing factors, and environmental conditions may contribute to the development of CRS14 (Table 1). In
contrast to acute rhinosinusitis, where anatomic pathology and associated obstruction of sinus ostia are of major etiologic contribution, in
CRS, anatomic pathology is often a secondary phenomenon, resulting
from a persistent mucosal inflammatory state. In CRSsNPs this inflammation may be mixed with a relatively greater proportion of
neutrophils than in cases of CRSwNPs, which more commonly present with eosinophilic predominance. As suggested by the united
allergic airway theory, eosinophilic inflammation is often present in
both the upper and the lower airway.15 Supporting this theory, the
incidence of rhinosinusitis is higher among individuals with asthma16; and increasing severity of asthma is associated with increasing
severity of CRS and degree of sinonasal tissue eosinophilia.2,15,17 In
addition to an association with asthma, increased risk of developing
rhinosinusitis is observed in certain systemic conditions such as granulomatosis with polyangiitis (Wegener’s),18 cystic fibrosis, cilia dysmotility syndromes, and various immunodeficiency diseases.19
BURDEN OF ILLNESS RELATED TO CRS
PATHOPHYSIOLOGY OF CRS
CRS affects all major race/ethnic groups,
ranks second in prevalence among all chronic conditions, afflicts ⬃16% of the U.S. population, and is associated with a significant impact on quality of life
(QOL).11 In the U.S., CRS direct health care costs are estimated at $8.6
billion annually.12 When indirect costs such as missed workdays and
decreased productivity at work are considered, rhinosinusitis ranks
among the top 10 most costly health conditions to U.S. employers.13
The pathogenesis of CRS is attributable to a multifactorial inflammatory disorder resulting from a dysfunctional host–environment interaction involving various exogenous agents and changes in the sinonasal
mucosa.2 The role of infection in the pathogenesis of CRS is limited and
appears to predominantly manifest as an immunologic response to
biofilm formation20,21 and/or acute infectious exacerbations. In the latter
setting, the organisms that are present are similar to those found in acute
rhinosinusitis (Streptococcus pneumoniae, Haemophilus influenzae, and
Moraxella catarrhalis), but with the additional presence of Pseudomonas
aeruginosa, Staphylococcus aureus, coagulase-negative staphylococci,
Gram-negative enteric bacteria, and various anaerobes.14
Two popular hypotheses for the pathogeneses of CRS, are the “staphylococcal superantigen hypothesis” and the “immune barrier hypothesis.”
The staphylococcal superantigen hypothesis has been more often used to
explain the Th2 mediated eosinophilic inflammation observed in CRSwNP.
It proposes that exposure to Staphylococcus aureus enterotoxins induces an
inflammatory mucosal response characterized by a Th2 lymphocytic response with inhibition of regulatory T cells, localized polyclonal IgE formation, and amplification of eosinophilic mucosal inflammation.22–24
The immune barrier hypothesis suggests that a multitude of potential
defects in mechanical (epithelial) and immunologic (innate and adaptive) barriers contribute to CRS.25 Among these defects is dysfunction of
9,10
From 1Department of Medicine, Warren Alpert Medical School of Brown University,
Providence, Rhode Island, 2Division of Allergy-Immunology, Northwestern University, Chicago, Illinois, and 3Department of Otolaryngology–Head and Neck Surgery,
Northwestern University Feinberg School of Medicine, Chicago, Illinois
RA Settipane is on the Speakers Bureau and/or a consultant and/or a research grant
recipient for Bausch & Lomb, Meda, Sunovion, and Teva Respiratory. AT Peters is a
speaker for Baxter. R Chandra is a consultant/advisor for Intersect ENT, Gyrus/
Olympus, and Sunovion and received a research grant from Intersect ENT
Address correspondence and reprint requests to Russell A. Settipane, M.D., Allergy &
Asthma Center, 95 Pitman Street, Providence, RI 02906
E-mail address: [email protected]
Originally published in Am J Rhinol Allergy 27, S11–S15, 2013
Copyright © 2014, OceanSide Publications, Inc., U.S.A.
American Journal of Rhinology & Allergy
Copyright © Oceanside Publications, Inc. All rights reserved. DO NOT COPY
For permission to copy go to https://www.oceansidepubl.com/permission.htm
S11
blockage, postnasal drip, nasal discharge, pain or pressure, and hyposmia in
50%.2,30 Of clinical importance, many patients with facial pain are misdiagnosed as having sinusitis when migraine or facial headache is the more
common etiology.2 Anterior rhinoscopy, although of limited value, remains
the first step in physical examination. Nasal endoscopy provides improved
visualization and information helpful to confirm diagnosis such as the
observation of edema and mucosal obstruction of the middle meatus or
purulent discharge from the middle meatus.2
Table 1 Associated or predisposing conditions of CRS
Host factors
Ciliary impairment
Immunodeficiency
Aspirin-exacerbated respiratory disease
Granulomatous disorders (sarcoid)
Vasculitis (Churg-Straus syndrome, granulomatosis with
polyangiitis 关Wegener’s兴)
Cystic Fibrosis
Asthma
Allergic rhinitis
Environmental factors
Allergic (allergic rhinitis and allergic fungal sinusitis)
Irritants (tobacco smoke, primary and secondary)
Cocaine
LABORATORY TESTING IN CRS
Source: Adapted from Ref. 14.
CRS ⫽ chronic rhinosinusitis.
membrane-bound pattern recognition receptors, which include Toll-like
receptors.20 Other defects that may increase susceptibility to pathogens
include decreased levels of sinonasal epithelium–derived antimicrobial
proteins, such as the S100 protein family, S100A7 (psoriasin) and
S100A8/A9 (calprotectin).26 Signal transducer and activator of transcription 3 (STAT3), which is a transcriptional mediator for the IL-6 family
cytokines, plays a critical role in regulating host defense. Defects in this
pathway may contribute to excessive inflammatory response of CRS.27
Taken together, various defects in the immune barrier result in increased
microbial colonization,28,29 accentuated barrier damage, and a compensatory and damaging immune response.
Although there are no routine laboratory tests recommended for
CRS patients, testing may be helpful in specific settings. Cultures
taken from the middle meatus under endoscopy control have ⬎80%
accuracy and may be helpful in the treatment of acute bacterial
exacerbations of CRS as an alternative to empiric antibiotic selection.31
Nasal cytology is of limited value in the diagnosis of CRS but may
help to identify underlying eosinophilic inflammation. Nasal biopsy
is indicated for the evaluation of unidentified nasal masses or to
diagnose inflammatory diseases such as granulomatous disorders
and vasculitis. If cystic fibrosis is suspected, consider a sweat chloride
test. Ciliary dysfunction may be primary or acquired32; consider
ciliary beat frequency analysis (available at only a few centers)
and/or electron microscopy. Nasal fractional concentration of exhaled nitric oxide, a marker of sinus mucosal inflammation, may be
elevated in CRS with patent sinus ostia33 but is consistently low in the
setting if ciliary dyskinesia. Medically refractory CRS, particularly if
associated with recurrent lower airway disease, warrants evaluation
for common variable immunodeficiency or specific pneumococcal
antibody deficiency.34,35 Although the role of antibiotics in CRS is
uncertain, if the history suggests potential environmental allergy,
consider assessment for specific IgE hypersensitivity.36,37
SYMPTOMS AND PHYSICAL EXAMINATION
OF CRS
PARANASAL SINUS IMAGING IN CRS
The symptoms of CRS are typically of lesser intensity than with acute
rhinosinusitis. The most common presenting symptoms of CRS are nasal
CT scanning is the imaging modality of choice for the paranasal
sinuses; whereas sinus x rays have been deemed to be of limited
Table 2 Treatment evidence and recommendations for adults with CRSsNPs*#
Therapy
Level
Grade§
Relevance
Steroid—topical
Nasal saline irrigation
Bacterial lysates (OM-85 BV)
Oral antibiotic therapy short term, ⬍4 wk
Oral antibiotic therapy long term, ⱖ12 wk¶
Steroid—oral
Mucolytics
Proton pump inhibitors
Decongestant oral/topical
Allergen avoidance in allergic patients
Oral antihistamine added in allergic patients
Herbal and probiotics
Immunotherapy
Probiotics
Antimycotics—topical
Antimycotics—systemic
Antibiotics—topical
Ia
Ia
Ib
II
Ib
i.v.
III
III
No data
i.v.
No data
No data
No data
Ib(⫺)
Ib(⫺)
No data
Ib(⫺)
A
A
A
B
C
C
C
D
D
D
D
D
D
A(⫺)
A(⫺)
A(⫺)
A(⫺)
Yes
Yes
Unclear
During exacerbations
Yes, especially if IgE is not elevated
Unclear
No
No
No
Yes
No
No
No
No
No
No
No
Source: Adapted from Ref. 2.
*Some of these studies also included patients with CRS with nasal polyps.
#Acute exacerbations of CRS should be treated like acute rhinosinusitis.
§Grade of recommendation.
¶Level of evidence for macrolides in all patients with CRSsNP is Ib, and strength of recommendation C, because the two double-blind placebo-controlled studies are
contradictory; indication exists for better efficacy in CRSsNP patients with normal IgE the recommendation A. No randomized controlled trials exist for other antibiotics.
Ib(⫺) ⫽ Ib study with a negative outcome; A(⫺) ⫽ grade A recommendation not to use; CRS ⫽ chronic rhinosinusitis; CRSsNPs ⫽ chronic rhinosinusitis
without nasal polyps.
S12
May–June 2014, Vol. 28, No. 3
Copyright © Oceanside Publications, Inc. All rights reserved. DO NOT COPY
For permission to copy go to https://www.oceansidepubl.com/permission.htm
usefulness.2 Despite advances in cone beam CT technology, which has
allowed for reduction in radiation dosages,38 significant cost and
radiation exposure may still be incurred. Consider CT when unilateral signs and symptoms are present, when there are signs and
symptoms suggestive of extrasinus involvement, or when the patient
fails to respond to medical therapy.2 It is important to note that
abnormal CT imaging findings, such as air–fluid levels, mucosal
thickening, or opacification of the sinus cavities, are consistent with
but not specific for rhinosinusitis. In fact, incidental abnormalities can
be found in up to a fifth of the “normal” population and in the
absence of symptoms, are not diagnostic of CRS.39 Adding to the
diagnostic challenge, an 87% incidence of sinus disease has been
2 symptoms: one of which should be nasal obstrucon
or discolored discharge
+/- frontal pain, headache
+/- smell disturbance
Physical examinaon
CT scan and/or endoscopy
Consider evaluaon for allergy and potenal predisposing condions*
Consider other diagnosis
•
•
•
Orbital symptoms
•
•
•
•
Mild (based on severity
assessment**)
No serious mucosal disease at
endoscopy
Moderate/severe (based on
severity assessment**)
Mucosal disease at endoscopy
Unilateral symptoms
Bleeding crusng
Cacosmia
Peri-orbital edema/
erythema
Displaced globe
Double or reduced
vision
Ophthalmoplegia
Severe frontal headache
Frontal swelling
Signs of meningis
Neurological signs
Urgent invesgaon and
intervenon
Topical steroids
Nasal saline irrigaon
No
improvement
aer 3 months
Topical steroids
Nasal saline irrigaon
Culture
Consider short term
anbiocs for
exacerbaons
Consider long term
anbiocs (if IgE is Normal)
CT scan
Improvement
Consider surgery
CT scan
if not done before
Follow-up +
Nasal saline irrigaon
Topical steroids
Consider long term anbiocs
No improvement
Consider surgery
Follow-up +
Topical steroids
Nasal saline irrigaon
Culture
Consider long term
anbiocs
Figure 1. Chronic rhinosinusitis without nasal polyps (CRSsNPs) in adults: Management algorithm for rhinologists. *See Table 1 for predisposing conditions; **For
example: Visual analog score or SNOT-20. CT ⫽ computed tomography. (Source: Adapted from Ref. 2.)
American Journal of Rhinology & Allergy
Copyright © Oceanside Publications, Inc. All rights reserved. DO NOT COPY
For permission to copy go to https://www.oceansidepubl.com/permission.htm
S13
reported in the early stages of the common cold as documented by CT
scan.40 Still, CT scan is cost-effective in ruling out sinusitis when the
diagnosis is in doubt after physical (endoscopic) examination. A
negative CT scan and endoscopy in the setting of active symptoms
effectively rules out CRS. Validated tools, such as the Lund-Mackay
system, have become the standard to grade severity of sinus opacification and osteomeatal complex occlusion on CT.41
Magnetic resonance imaging is an alternative that offers improved
definition of soft tissue with an ability to differentiate between soft
tissue masses (including malignancy) and retained/obstructed secretions. However, in comparison with CT, it does not provide optimal
discrimination of air, bone, and soft tissue.
MEDICAL TREATMENT OF CRSsNPs
Medical treatment options for CRSsNPs have been studied; and evidencebased recommendations (Table 2) and treatment algorithms have been
established (Fig. 1).2 Treatment should be based on severity of symptomology using an assessment tool such as the 20-item Sino-Nasal Outcome Test
or using a 0- to 10-cm visual analog scale in the context of endoscopic
findings.2,42 The treatment of CRS is not curative. The underlying principle
of medical management in CRS is control of inflammation and reduction of
infectious exacerbations. The goal of treatment is to achieve and maintain
“clinical control,” defined as “a disease state in which the patient does not
suffer from bothersome symptoms, combined with a healthy or near
healthy mucosa and only the need for local medication.”2 “Difficult-to-treat
rhinosinusitis” is defined as “those patients who have persistent symptoms
of rhinosinusitis despite appropriate treatment” or, more specifically, “patients who do not reach an acceptable level of control despite adequate
surgery, intranasal corticosteroid treatment and up to two short courses of
antibiotics or systemic corticosteroids in the last year.”2
The first step in CRS management is to identify and address any
contributing factors (Table 1). Treatment modalities for which evidence-based “grade A” recommendations exist include intranasal
corticosteroids (INCSs) and saline irrigation. Corticosteroids (CS)
work by modulating the often eosinophilic inflammatory process
underlying CRS.43 Because of insufficient evidence, the use of oral CS
in CRSsNPs has earned only a “grade C” evidence-based recommendation and, consequently, oral CS use has been deemed optional.2,44
Evidence for the efficacy of antibiotic treatment of CRSsNPs is generally less robust than for INCS45 and, consequently, their role in the
treatment of CRS is limited and most often reserved for patients with
purulent sinus drainage. Oral antibacterial antibiotics and prolonged
macrolide antibiotics are considered therapeutic options.46 Although
there are no Food and Drug Administration–approved antibiotics for the
treatment of CRS, a “grade B” evidence-based recommendation exists
for antibiotic treatment (⬍4 weeks duration) of acute exacerbations of
CRS.2 Antibiotics commonly used are those that are currently Food and
Drug Administration approved for the treatment of acute sinusitis. The
use of long-term antibiotics (for ⱖ3 months) is less strongly recommended (grade C).2 The therapeutic benefit/risk ratio of antibiotic therapy should be carefully weighed given known associated adverse effects
including clostridium difficile colitis, quinolone-induced arrhythmias,47
and risk of tendonitis/tendon rupture,48 as well as macrolide-associated
cardiovascular death risk.49 Neither topical nor systemic antifungal treatment is advocated for the management of CRS.2,50 Herbal treatments
have been issued a grade D recommendation.2,51
Although nasal saline irrigation can provide symptomatic relief and
results in improved QOL, the addition of antibiotics to the irrigation
solution has failed to show benefit in double-blind, placebo-controlled
trials.2 Alternate irrigation solutions include dilute baby shampoo, which
has surfactant effects capable of attacking biofilms.52,53 To prevent potentially lethal amebic meningoencephalitis infection from tap water,
only boiled, distilled, or filtered water should be used.54 Additionally,
care should be taken to reduce irrigation bottle contamination.55–57
S14
SURGICAL TREATMENT OF CRS
Surgery is reserved for patients who fail to respond to medical
therapy; and only a small proportion of patients go on to need
surgery.58 The primary surgical treatment option for CRS is functional
endoscopic sinus surgery (FESS), the benefits of which include the
relief of ostial obstruction, removal of inflammatory mucous and
biofilm, and improved postoperative delivery of topical CSs to the
sinus cavities, which may assist in long-term anti-inflammatory control.59 Balloon sinuplasty has been studied in CRSsNPs and offers an
alternate surgical approach.60 Although trials providing high-level
evidence of the efficacy of FESS are missing, there is levels II–III
evidence that FESS is associated with improved symptoms (nasal
obstruction and discharge) and QOL.2,61 Perioperative complications
have been reported in 6%, most commonly intraoperative hemorrhage, less commonly cerebrospinal fluid leak.62
CRS SUMMARY
In summary, CRS is the second most common chronic medical condition in the United States. Although it is not likely to be cured by either
medical or surgical therapy, it can generally be controlled. The best
medical evidence supports maintenance therapy with INCSs and saline
irrigation. For exacerbations, short courses of antibiotics (up to 4-weeks)
with or without oral CSs are recommended. For patients with difficultto-treat CRS, FESS provides an adjunctive therapeutic option.
CLINICAL PEARLS
• Twelve-week duration of symptoms is required to meet the definition of CRS.
• Rhinosinusitis represents a multifactorial inflammatory disorder of
the sinonasal mucosa.
• Patients with “difficult-to-treat CRS” should be evaluated for immunodeficiency, particularly if there is a history of recurrent otitis
media and/or pneumonia.
• Based on the relative degree of tissue infiltration, CRSsNPs is more distinctly a neutrophilic process, whereas CRSwNPs is more eosinophilic.
• Conditions commonly associated with CRS include asthma and NPs.
• Consider CT when unilateral signs and symptoms are present,
when there are signs and symptoms suggestive of extrasinus involvement, or when the patient fails to respond to medical therapy.
ACKNOWLEDGEMENT
The authors wish to thank Davis Settipane for his technical assistance in the creation of Figure 1.
REFERENCES
1.
Benninger MS, Ferguson BJ, Hadley JA, et al. Adult chronic rhinosinusitis: Definitions, diagnosis, epidemiology, and pathophysiology.
Otolaryngol Head Neck Surg 129:S1–S32, 2003.
2. Fokkens WJ, Lund VJ, Mullol J, et al. European position paper on
rhinosinusitis and nasal polyps 2012. Rhinol Suppl 23:1–298, 2012.
3. Dalgorf DM, and Harvey RJ. Sinonasal anatomy and function. Am J
Rhinol Allergy 27:S3–S6, 2013.
4. Nakayama T, Asaka D, Yoshikawa M, et al. Identification of chronic
rhinosinusitis phenotypes using cluster analysis. Am J Rhinol Allergy
26:172–176, 2012.
5. Han JK. Subclassification of chronic rhinosinusitis. Laryngoscope.
123(suppl 2):S15–S27, 2013.
6. Settipane RA, Peters AT, and Chiu AG. Nasal polyps. Am J Rhinol Allergy
27:S20–S25, 2013.
7. Laury AM, and Wise SK. Allergic fungal rhinosinusitis. Am J Rhinol Allergy 27:S26–S27, 2013.
8. Dykewicz MS, and Hamilos DL. Rhinitis and sinusitis. J Allergy Clin
Immunol 125:S103–S115, 2010.
9. Collins JG. Prevalence of selected chronic conditions: United States,
1990–1992. Vital Health Stat 10:1–89, 1997.
10. Soler ZM, Mace JC, Litvack JR, and Smith TL. Chronic rhinosinusitis,
race, and ethnicity. Am J Rhinol Allergy 26:110–116, 2012.
May–June 2014, Vol. 28, No. 3
Copyright © Oceanside Publications, Inc. All rights reserved. DO NOT COPY
For permission to copy go to https://www.oceansidepubl.com/permission.htm
11.
12.
13.
14.
15.
16.
17.
18.
19.
20.
21.
22.
23.
24.
25.
26.
27.
28.
29.
30.
31.
32.
33.
34.
35.
36.
37.
38.
Bhattacharyya N. Functional limitations and workdays lost associated with chronic rhinosinusitis and allergic rhinitis. Am J Rhinol
Allergy 26:120–122, 2012.
Bhattacharyya N. Incremental health care utilization and expenditures for chronic rhinosinusitis in the United States. Ann Otol Rhinol
Laryngol 120:423–427, 2011.
Bhattacharyya N. Contemporary assessment of the disease burden of
sinusitis. The economic burden and symptom manifestations of chronic
rhinosinusitis. Am J Rhinol Allergy 23:392–395, 2009.
Georgy MS, and Peters AT. Rhinosinusitis. Allergy Asthma Proc
33:S24–S27, 2012.
Feng CH, Miller MD, and Simon RA. The united allergic airway:
Connections between allergic rhinitis, asthma, and chronic sinusitis.
Am J Rhinol Allergy 26:187–190, 2012.
Hellings PW, and Hens G. Rhinosinusitis and the lower airways.
Immunol Allergy Clin North Am 29:733–740, 2009.
Lin DC, Chandra RK, Tan BK, et al. Association between severity of asthma
and degree of chronic rhinosinusitis. Am J Rhinol Allergy 25:205–208, 2011.
Kohanski MA and Reh DD. Granulomatous diseases and chronic sinusitis.
Am J Rhinol Allergy 27:S39–S41, 2013.
Alqudah M, Graham SM, and Ballas ZK. High prevalence of humoral
immunodeficiency patients with refractory chronic rhinosinusitis.
Am J Rhinol Allergy 24:409–412, 2010.
Sun Y, Zhou B, Wang C, et al. Biofilm formation and Toll-like receptor 2,
Toll-like receptor 4, and NF-kappaB expression in sinus tissues of patients
with chronic rhinosinusitis. Am J Rhinol Allergy 26:104–109, 2012.
Madeo J, and Frieri M. Bacterial biofilms and chronic rhinosinusitis.
Allergy Asthma Proc DOI: 10.2500/aap.2013.34.3665 [Epub ahead of
print date March 7, 2013].
Van Crombruggen K, Zhang N, Gevaert P, et al. Pathogenesis of chronic
rhinosinusitis: Inflammation. J Allergy Clin Immunol 128:728–732, 2011.
Kim DW, Khalmuratova R, Hur DG, et al. Staphylococcus aureus enterotoxin
B contributes to induction of nasal polypoid lesions in an allergic rhinosinusitis murine model. Am J Rhinol Allergy 25:e255–e261, 2011.
Kim ST, Chung SW, Jung JH, et al. Association of T cells and eosinophils with Staphylococcus aureus exotoxin A and toxic shock syndrome toxin 1 in nasal polyps. Am J Rhinol Allergy 25:19–24, 2011.
Kern R, Conley D, Walsh W, et al., Perspectives on the etiology of
chronic rhinosinusitis: An immune barrier hypothesis. Am J Rhinol
22:549–559, 2008.
Tieu DD, Peters AT, Carter RT, et al. Evidence for diminished levels
of epithelial psoriasin and calprotectin in chronic rhinosinusitis. J
Allergy Clin Immunol 25:667–675, 2010.
Peters AT, Kato A, Zhang N, et al. Evidence for altered activity of the
IL-6 pathway in chronic rhinosinusitis with nasal polyps. J Allergy
Clin Immunol 125:397–403.e10, 2010.
Stressmann FA, Rogers GB, Chan SW, et al. Characterization of bacterial
community diversity in chronic rhinosinusitis infections using novel culture-independent techniques. Am J Rhinol Allergy 25:e133–e140, 2011.
Wood AJ, Fraser JD, Swift S, et al. Intramucosal bacterial microcolonies exist in chronic rhinosinusitis without inducing a local immune
response. Am J Rhinol Allergy 26:265–270, 2012.
Gaines A. Olfactory disorders . Am J Rhinol Allergy 27:S45–S47, 2013.
Benninger MS, Payne SC, Ferguson BJ, et al. Endoscopically directed middle meatal cultures versus maxillary sinus taps in acute bacterial maxillary
rhinosinusitis: A meta-analysis. Otolaryngol Head Neck Surg 134:3–9, 2006.
Gudis D, Zhao KQ, and Cohen NA. Acquired cilia dysfunction in
chronic rhinosinusitis. Am J Rhinol Allergy 26:1–6, 2012.
Noda N, Takeno S, Fukuiri T, and Hirakawa K. Monitoring of oral
and nasal exhaled nitric oxide in eosinophilic chronic rhinosinusitis:
A prospective study. Am J Rhinol Allergy 26:255–259, 2012.
Carr TF, Koterba AP, Chandra R, et al. Characterization of specific
antibody deficiency in adults with medically refractory chronic rhinosinusitis. Am J Rhinol Allergy 25:241–244, 2011.
Ocampo CJ, and Peters AT. Antibody deficiency in chronic rhinosinusitis:
Epidemiology and burden of illness. Am J Rhinol Allergy 27:34–38, 2013.
Kennedy JL, and Borish L. Chronic sinusitis pathophysiology: The role
of allergy. Am J Rhinol Allergy DOI: 10.2500/ajra.2013.27.3906 [Epub
ahead of print date April 18, 2013].
Settipane RA, Borish L, and Peters AT. Determining the role of
allergy in sinonasal disease. Am J Rhinol Allergy 27:S56–S58, 2013.
Leung R, Chaung K, Kelly JL, and Chandra RK. Advancements in
computed tomography management of chronic rhinosinusitis. Am J
Rhinol Allergy 25:299–302, 2011.
American Journal of Rhinology & Allergy
39.
40.
41.
42.
43.
44.
45.
46.
47.
48.
49.
50.
51.
52.
53.
54.
55.
56.
57.
58.
59.
60.
61.
62.
Fokkens W, Lund V, Mullol J, et al. European position paper on
rhinosinusitis and nasal polyps group. European position paper on
rhinosinusitis and nasal polyps 2007. Rhinol Suppl 20:1–136, 2007.
Gwaltney JM Jr, Phillips CD, Miller RD, and Riker DK. Computed tomographic study of the common cold. N Engl J Med 330:25–30, 1994.
Metson R, Gliklich RE, Stankiewicz JA, et al. Comparison of sinus
computed tomography staging systems. Otolaryngol Head Neck
Surg 117:372–379, 1997.
Piccirillo JF, Merritt MG Jr, and Richards ML. Psychometric and
clinimetric validity of the 20-Item Sino-Nasal Outcome Test (SNOT20). Otolaryngol Head Neck Surg 126:41–47, 2002.
Ocampo CJ, and Peters AT. Medical therapy as the primary modality
for the management of chronic rhinosinusitis. Allergy Asthma Proc
34:132–137, 2013.
Poetker DM, Jakubowski LA, Lal D, et al. Oral corticosteroids in the
management of adult chronic rhinosinusitis with and without nasal
polyps: An evidence-based review with recommendations. Int Forum
Allergy Rhinol 3:104–120, 2013.
Zeng M, Long XB, Cui YH, and Liu Z. Comparison of efficacy of
mometasone furoate versus clarithromycin in the treatment of
chronic rhinosinusitis without nasal polyps in Chinese adults. Am J
Rhinol Allergy 25:e203–e207, 2011.
Soler ZM, Oyer SL, Kern RC, et al. Antimicrobials and chronic rhinosinusitis
with or without polyposis in adults: an evidenced-based review with recommendations. Int Forum Allergy Rhinol 3:31–47, 2013.
Frothingham R. Rates of torsades de pointes associated with ciprofloxacin, ofloxacin, levofloxacin, gatifloxacin, and moxifloxacin. Pharmacotherapy 21:1468–1472, 2001.
van der Linden PD, Sturkenboom MC, Herings RM, et al. Increased
risk of achilles tendon rupture with quinolone antibacterial use,
especially in elderly patients taking oral corticosteroids. Arch Intern
Med 163:1801–1807, 2003.
Ray WA, Murray KT, Hall K, et al. Azithromycin and the risk of
cardiovascular death. N Engl J Med 366:1881–1890, 2012.
Sacks PL IV, Harvey RJ, Rimmer J, et al. Antifungal therapy in the
treatment of chronic rhinosinusitis: A meta-analysis. Am J Rhinol
Allergy 26:141–147, 2012.
Jiang RS, Wu SH, Tsai CC, et al. Efficacy of Chinese herbal medicine
compared with a macrolide in the treatment of chronic rhinosinusitis
without nasal polyps. Am J Rhinol Allergy 26:293–297, 2012.
Isaacs S, Fakhri S, Luong A, et al. The effect of dilute baby shampoo
on nasal mucociliary clearance in healthy subjects. Am J Rhinol
Allergy 25:e27–e29, 2011.
Rosen PL, Palmer JN, ÓMalley BW, and Cohen NA. Surfactants in the
management of rhinopathologies. Am J Rhinol Allergy 27:177–180, 2013.
Yoder JS, Straif-Bourgeois S, Roy SL, et al. Primary amebic meningoencephalitis deaths associated with sinus irrigation using contaminated tap water. Clin Infect Dis 55:e79–e85, 2012.
Morong S, and Lee JM. Microwave disinfection: Assessing the risks of
irrigation bottle and fluid contamination. Am J Rhinol Allergy 26:398–400,
2012.
Psaltis AJ, Foreman A, Wormald PJ, and Schlosser RJ. Contamination
of sinus irrigation devices: A review of the evidence and clinical
relevance. Am J Rhinol Allergy 26:201–203, 2012.
Woods CM, Hooper DN, Ooi EH, et al. Fungicidal activity of lysozyme is inhibited in vitro by commercial sinus irrigation solutions.
Am J Rhinol Allergy 26:298–301, 2012.
McNally PA, White MV, and Kaliner MA. Sinusitis in an allergist’s office:
Analysis of 200 consecutive cases. Allergy Asthma Proc 18:169–175, 1997.
Snidvongs K, Kalish L, Sacks R, et al. Sinus surgery and delivery
method influence the effectiveness of topical corticosteroid for
chronic rhinosinusitis: Systematic review and meta-analysis. Am J
Rhinol Allergy 27:221–233, 2013.
Koskinen A, Penttilä M, Myller J, et al. Endoscopic sinus surgery
might reduce exacerbations and symptoms more than balloon sinuplasty. Am J Rhinol Allergy 26:e150–e156, 2012.
Smith TL, Kern R, Palmer JN, et al. Medical therapy vs surgery for chronic
rhinosinusitis: A prospective, multi-institutional study with 1-year followup. Int Forum Allergy Rhinol 3:4–9, 2013.
Asaka D, Nakayama T, Hama T, et al. Risk factors for complications
of endoscopic sinus surgery for chronic rhinosinusitis. Am J Rhinol
Allergy 26:61–64, 2012.
e
Copyright © Oceanside Publications, Inc. All rights reserved. DO NOT COPY
For permission to copy go to https://www.oceansidepubl.com/permission.htm
S15
Nasal polyps
Russell A. Settipane, M.D.,1 Anju T. Peters, M.D.,2 and Alexander G. Chiu, M.D.3
ABSTRACT
Nasal polyps occur in 1– 4% of the population, usually occurring in the setting of an underlying local or systemic disease. The most common associated
condition is chronic rhinosinusitis (CRS). A high prevalence of nasal polyps is also seen in allergic fungal rhinosinusitis, aspirin-exacerbated respiratory
disease, Churg-Strauss syndrome, and cystic fibrosis. In the setting of CRS, nasal polyps are not likely to be cured by either medical or surgical therapy;
however, control is generally attainable. The best medical evidence supports the use of intranasal corticosteroids for maintenance therapy and short courses of
oral corticosteroids for exacerbations. The evidence for short- and long-term antibiotics is much less robust. For patients with symptomatic nasal polyposis
nonresponsive to medical therapies, functional endoscopic sinus surgery provides an adjunctive therapeutic option.
N
asal polyps, which were first described over 5000 years ago as
“grapes coming down the nose” by the ancient Greeks,1 are
inflammatory outgrowths of sinonasal epithelium. They manifest as
edematous semitransluscent lobular masses in the nasal and paranasal
cavities (typically bilateral), originating from the mucosal lining of the
sinuses, most commonly middle nasal meatus and ethmoid cells.2 The
severity of polyposis is often classified based on the degree of polyp extension beyond the middle meatus: confined within the middle meatus, extending beyond the middle meatus, or filling the entire nasal cavity.
scribed in patients with nasal polyps compared with the general
population, because higher rates of nasal polyps have not been consistently observed in allergic rhinitis, the role of allergy in CRSwNPs
remains unclear5,16–18 This is supported by the epidemiologic observation that the prevalence of nasal polyps in allergic rhinitis is comparable with that seen in the normal population.5 However, when
atopy is present in the setting of nasal polyps, it is associated with
lower quality of life scores and a higher incidence of asthma.18
HISTOPATHOLOGY OF NASAL POLYPS
EPIDEMIOLOGY OF NASAL POLYPS
Nasal polyps are common, affecting from 1 to 4% of the general
population.3 They occur in all races,4 have a male predominance, and
are more common after age 40 years.5 Childhood presentation (aged
⬍16–20 years) is rare and in the pediatric setting raises suspicion for
the diagnosis of cystic fibrosis.6
CONDITIONS ASSOCIATED WITH NASAL POLYPS
Nasal polyps usually occur in the setting of an underlying local or
systemic disease (Table 1),7 with the most commonly observed association being chronic rhinosinusitis (CRS).8 Although many different
CRS phenotypes exist,9–11 up to one-third of all cases are associated
with nasal polyps.12 Compared with CRS without nasal polyps (CRSsNPs), in CRS with nasal polyps (CRSwNPs), patients generally suffer
more severely, have a greater burden of symptoms, more prior surgery, higher CT scan scores, and greater use of medications.13,14 High
rates of nasal polyp prevalence are also seen in allergic fungal rhinosinusitis,15 aspirin-exacerbated respiratory disease (AERD), ChurgStrauss syndrome, cystic fibrosis, primary ciliary dyskinesia, asthma,
and Young’s syndrome (Table 1).7 Of note, allergy is omitted from the
aforementioned list. Although a higher rate of atopy has been deFrom the 1Department of Medicine, Warren Alpert Medical School of Brown University Providence, Rhode Island, 2Division of Allergy-Immunology, Northwestern University, Chicago, Illinois, Division of Otolaryngology, 3Department of Surgery, University of Arizona, Tucson, Arizona
RA Settipane is on the Speakers Bureau and/or a consultant and/or a research grant
recipient for Bausch & Lomb, Meda, Sunovion, and Teva Respiratory. AT Peters is a
speaker for Baxter. AG Chiu is a consultant for Olympus Gyrus
Address correspondence and reprint requests to Russell A. Settipane, M.D., Allergy &
Asthma Center, 95 Pitman Street, Providence, RI 02906
E-mail address: [email protected]
Originally published in Am J Rhinol Allergy 27, S20 –S25, 2013
Copyright © 2014, OceanSide Publications, Inc., U.S.A.
S16
Histologically, the mucosal surface of nasal polyps consists of respiratory epithelium and an edematous stroma infiltrated by inflammatory cells.20 Polyp mucosa exhibits a markedly greater inflammatory infiltrate and a lesser density of mucous glands than inferior
turbinate mucosa. In the United States and Europe, nasal polyps are
accompanied by an eosinophilic infiltrate in ⬃80% of cases; whereas
in Asia they are more likely to be associated with a noneosinophilic or
neutrophilic infiltrate and Th1/17 cytokine skewing.21 Correlating
with reduced eosinophilic inflammation, a very low rate of AERD has
been observed in Chinese patients with CRSwNPs.22 It is interesting
to note that over a 12-year period (from 1999 to 2011), a shift from
predominantly neutrophilic to eosinophilic inflammation was recently observed in Asian (Thai) patients with CRSwNPs in association
with an increase in intramucosal presence of Staphylococcus aureus.23
Neutrophilic inflammation is also a consistent finding in the nasal
polyps found in cystic fibrosis.24
PATHOGENESIS OF NASAL POLYPS
Because CRSwNPs is the most common condition in which polyps
are observed, the discussion of the etiology, pathogenesis, and treatment of nasal polyps will be limited to this association.
There is an emerging consensus that the persistent inflammation
that defines CRSwNP results from dysfunctional host–environment
interaction involving various exogenous agents and changes in the
sinonasal mucosa.5 Several pathophysiological hypotheses have been
put forth (Table 2)25; however, the initiating event, which triggers the
formation of nasal polyps, remains unknown.
Investigation of the potential contributory role of microorganisms
to CRSwNP pathogenesis has focused on several areas including the
role of sinus microbiology and the immune response to S. aureus
superantigens, biofilms, and/or fungi. Subject to debate is the clinical
significance of the microbiology of CRSwNPs, which is similar to
CRSsNPs. Although biofilms have been observed to be highly prevalent in association with high rates of S. aureus colonization and
May–June 2014, Vol. 28, No. 3
Copyright © Oceanside Publications, Inc. All rights reserved. DO NOT COPY
For permission to copy go to https://www.oceansidepubl.com/permission.htm
Table 1 Prevalence of nasal polyposis in association with other
conditions
Condition
Prevalence
AERD
Adult asthma
IgE mediated
Non-IgE mediated
CRS in adults
Rhinitis
Nonallergic rhinitis
Allergic rhinitis
Childhood asthma/sinusitis
Cystic fibrosis
Children
Adults
Churg-Strauss syndrome
Allergic fungal sinusitis
Primary ciliary dyskinesia (Kartagener’s)
Young’s syndrome (azoospermia)
15–23%
7%
5%
13%
33%
5%
1.5%
0.1%
10%
50%
50%
66–100%
40%
?
Source: Adapted from Ref. 7.
AERD ⫽ aspirin exacerbated respiratory disease; CRS ⫽ chronic rhinosinusitis.
Table 2 CRSwNPs: Pathophysiological hypotheses
Microorganisms
Staphylococcal superantigen hypothesis
Biofilms
Fungal hypothesis
Immune barrier hypothesis
Excessive Th2 response
Defects in eicosanoid pathway
Source: Adapted from Ref. 25.
CRSwNPs ⫽ chronic rhinosinusitis with nasal polyps.
Table 3 Benign lesions simulating nasal polyps
Anatomic
Concha bullosa
Tumors
Epithelial
Papilloma—inverted, everted, and cylindric
Minor salivary—pleomorphic adenoma
Mesenchymal
Neurogenic—meningioma, schwannoma, and neurofibroma
Vascular—hemangioma and angiofibroma
Fibro-osseous—ossifying fibroma muscular—leiomyoma and
angioleiomyoma
Granulomatous/inflammatory
Wegener’s granulomatosis
Sarcoidosis
Crohn’s disease
Source: Adapted from Refs. 41 & 70.
Table 4 Malignant lesions simulating polyps
Epithelial
Squamous cell carcinoma
Adenocarcinoma
Adenoid cystic carcinoma
Acinic cell carcinoma
Mucoepidermoid carcinoma
Olfactory neuroblastoma
Malignant melanoma
Metastatic tumors, e.g., kidney, breast, and pancreas
Undifferentiated carcinoma
Mesenchymal tumors
Lymphoreticular—lymphoma and plasmacytoma
Rhabdomyosarcoma
Chondrosarcoma
Ewing’s sarcoma
Source: Adapted from Refs. 41 & 71.
production of specific IgE to S. aureus,26 their role is not clear. The
“staphylococcal superantigen hypothesis” proposes that exposure to
S. aureus enterotoxins induces a Th2 lymphocytic response with inhibition of regulatory T cells, localized polyclonal IgE formation, and
amplification of eosinophilic mucosal inflammation.27–29 Except for
the CRS subtype known as allergic fungal rhinosinusitis,15 evidence
for fungi in the pathogenesis of CRSwNPs appears to be limited to
playing a role as a disease modifier.5
The “immune barrier hypothesis” suggests that a multitude of
potential defects in mechanical (epithelial) and immunologic (innate
and adaptive) barriers contribute to CRS.30 Among these defects is
dysfunction of membrane bound pattern recognition receptors, which
include Toll-like receptors.31 Other defects that may increase susceptibility to pathogens include decreased levels of sinonasal epithelium–
derived antimicrobial proteins, such as the S100 protein family,
S100A7 (psoriasin), and S100A8/A9 (calprotectin).32 Signal transducer and activator of transcription 3 (STAT3), which is a transcriptional mediator for the IL-6 family cytokines, plays a critical role in
regulating host defense. Defects in this pathway may contribute to the
excessive inflammatory response of CRS.33 Taken together, various
defects in the immune barrier result in increased microbial colonization,34,35 accentuated barrier damage, and a compensatory and damaging immune response.
B-cell activating factor of the TNF family levels are elevated in nasal
polyp tissue and are important in B-cell survival, proliferation, and
antibody production.36 Overproduction of B-cell activating factor of
the TNF family may lead to B-cell proliferation and contribute to the
excessive Th2 skewing as well as the generation of local antibodies
American Journal of Rhinology & Allergy
that may further accentuate tissue damage. Nasal polyps from patients with CRS possess increased levels of both B cells and plasma
cells, which actively produce local antibodies.37
Finally, aberrant arachidonic acid metabolism has been implicated
in the pathogenesis of CRSwNPs in the subset of patients with AERD,
which represents 15–23% of nasal polyp patients.38,39 Inflammation in
these patients is characterized by higher respiratory tract levels of
cyclooxygenase and 5-lipoxygenase products (leukotrienes) as well as
increased respiratory tract expression of the cysteinyl leukotriene 1
receptor and leukotriene C4 synthase).40
DIFFERENTIAL DIAGNOSIS OF NASAL POLYPS
Although nasal polyps have a characteristic appearance, other
nasal masses, benign or malignant, may be mistaken for polyps
(Tables 3 and 4).41 Although nasal polyps in CRS are almost always
bilateral, a unilateral polypoid nasal mass raises the potential of
malignancy.12 Common benign lesions mistaken for nasal polyps
in adults include concha bullosa (pneumatization of the middle
turbinate) and inverted papilloma,42 whereas malignant lesions
such as esthesioneuroblastomas and squamous cell carcinoma often present as a unilateral mass. In children, a juvenile angiofibroma may be mistaken for a nasal polyp. The differential diagnosis of nasal masses in childhood differs from adults (Table 5).41
The occurrence of true nasal polyps in children warrants an evaluation for cystic fibrosis.7
Copyright © Oceanside Publications, Inc. All rights reserved. DO NOT COPY
For permission to copy go to https://www.oceansidepubl.com/permission.htm
S17
Table 5 Differential diagnosis of a nasal mass in a child
Congenital
Encephalocele
Glioma
Dermoid cyst
Nasolacrimal duct cyst
Neoplasia
Benign—craniopharyngioma, hemangioma, and neurofibroma
Malignant—rhabdomyosarcoma
Table 6 Treatment evidence and recommendations for adults
with CRSwNPs*
Therapy
Level
Grade#
Topical steroids
Oral steroids
Oral antibiotics short
term, ⬍4 wk
Oral antibiotics long
term ⱖ12 wk
Ia
Ia
1b and 1b(⫺)
A
A
C§
Yes
Yes
Yes, small effect
III
C
Capsaicin
Proton pump inhibitors
Aspirin desensitization
Furosemide
Immunosuppressants
Nasal saline irrigation
II
II
II
III
i.v.
Ib, No data
in single
use
No data
No data
No data
No data in
single use
No data
No data
C
C
C
D
D
D
D
D
D
D
Yes, especially if
IgE is not
elevated,
small effect
No
No
Unclear
No
No
Yes, for
symptomatic
relief
No
Unclear
No
No
D
D
No
No
Ia (⫺)
Ib(⫺)
Ib(⫺)
Ib(⫺)
A(⫺)
A(⫺)
A(⫺)
A(⫺)
No
No
No
No
Source: Adapted from Ref. 41.
SIGNS AND SYMPTOMS OF NASAL POLYPS
Symptoms of nasal polyps range from partial to complete nasal
obstruction and from hyposmia to complete anosmia.43 Symptom
severity can be assessed on a 10-cm visual analog scale.5 On exam,
polyps appear as semitranslucent, pale gray or pink, lobular mucosal
tissue within the nasal cavity, and possess a smooth and glistening
surface when light is applied. On probing, polyp tissue is mobile and
easily depressed in comparison with the firm texture of the inferior
turbinate or other masses. Nasal endoscopy is recommended to fully
evaluate the size and location of the nasal polyps, as well as to gauge
their response to medication.
LABORATORY TESTING AND PARANASAL
SINUS IMAGING
Recommendations for laboratory testing and sinus imaging are similar
for both CRSwNPs and CRSsNPs.8
MEDICAL TREATMENT OF CRSwNP
Medical treatment options have been fairly well established for
corticosteroids but less well for antibiotics and other immunomodulatory agents.5,44 Evidence-based recommendations are described in
Table 6.5 Treatment should be based on severity of symptomology
(Fig. 1).5
“Grade A” evidence-based recommendations support the use of
both intranasal corticosteroids (INCSs) and oral corticosteroids, as
treatments for CRSwNPs.5 Corticosteroids have broad anti-inflammatory effects.45–48 INCSs are recommended as first-line therapy for
moderate to severe disease both before and after polypectomy.5 Except for aqueous mometasone,49 most topical INCS formulations are
used “off –Food and Drug Administration (FDA)–approved labeling”
for CRSwNPs, including both aerosol and aqueous corticosteroid
preparations, as well as drops and irrigations. In a recent Cochrane
review of 40 studies involving 3624 patients, topical corticosteroids
improved overall symptom scores, decreased polyp score, decreased
polyp size, and prevented polyp recurrence after surgery.50 In a recent
meta-analysis report of the effect of topical nasal steroid therapy on
symptoms of nasal polyposis, all three topical steroid preparations
that were studied (fluticasone, mometasone, and budesonide) resulted in significant improvement.51 Despite the efficacy of topical
steroids, when treating severely symptomatic patients, it is often
preferable to prescribe short courses (ⱕ2 weeks) of oral steroids to
acutely reduce polyp size.52–55 Maintenance therapy with INCSs is
necessary for long-term control of the inflammation that underlies
CRSwNPs.
The evidence for the use of oral antibiotics is less robust.56 “Grade
C” evidence-based recommendations support treatment with shortterm antibiotics (⬍4weeks) for CRS exacerbations when features suggestive of acute bacterial rhinosinusitis are present. Longer-term macrolide therapy may be an option, particularly in the setting of low
circulating IgE (“grade C” recommendation).57 Oral doxycycline has
been shown to have a modest effect in reducing polyp size and
decreasing inflammation but the effect is less than that of oral ste-
S18
Relevance
Topical antibiotics
Anti-IL-5
Phytotherapy
Decongestant
topical/oral
Mucolytics
Oral antihistamine in
allergic patients
Antimycotics—topical
Antimycotics—systemic
Anti leukotrienes
Anti-IgE
⬎Source: Adapted from Ref. 5.
*Some of these studies also included patients with CRS with nasal polyps.
#Grade of recommendation.
§Short-term antibiotics shows one positive and one negative study; therefore,
recommendation C.
¶Ia(⫺) ⫽ Ia level of evidence that treatment is not effective; Ib(⫺) ⫽ Ib study
with a negative outcome; A(⫺) ⫽ grade A recommendation not to use;
CRSwNPs ⫽ chronic rhinosinusitis with nasal polyps.
roids.55 The therapeutic benefit/risk ratio of antibiotic therapy should
be carefully weighed given known antibiotic-associated adverse effects including Clostridium difficile colitis, quinolone-induced arrhythmias,58 and risk of tendonitis/tendon rupture59 as well as macrolideassociated cardiovascular death risk.60 With regard to topical
antibiotics, only “grade D” evidence exists; and antifungal therapy,
whether intranasal or oral, is not advised.5
Adequate studies of allergy immunotherapy for CRSwNPs are
nonexistent.16 In AERD patients, aspirin desensitization has been
shown to modulate aberrant arachidonic acid metabolism by reducing leukotriene C4 levels in nasal secretions, lowering the expression
of cysteinyl leukotriene receptors and decreasing polyp recurrence
rates.61,62 Anti-IgE for CRSwNPs holds promise, but trials thus far
have resulted in inconsistent outcomes.63,64 Anti-IL5 warrants further
investigation because of its ability to reduce eosinophilic inflammation and polyp size.65
SURGICAL TREATMENT OF NASAL POLYPS
Surgery is reserved for cases when polyps are associated with
severe symptoms, recurrent sinusitis, and for patients refractory to
medical therapy.44 The primary surgical treatment option for CRS
is functional endoscopic sinus surgery (FESS), the benefits of
which include the removal of polyps, inflammatory mucin and
May–June 2014, Vol. 28, No. 3
Copyright © Oceanside Publications, Inc. All rights reserved. DO NOT COPY
For permission to copy go to https://www.oceansidepubl.com/permission.htm
2 symptoms: one of which should be nasal obstrucon
or discolored discharge
+/- frontal pain, headache
+/- smell disturbance
Physical examinaon
CT scan and/or endoscopy (size of polyps)
Consider evaluaon for allergy and associated condions*
Consider other diagnosis
•
•
•
Orbital symptoms
•
•
•
•
Mild (based on
severity assessment**)
No serious mucosal
disease at endoscopy
Moderate (based on
severity assessment**)
Mucosal disease at
endoscopy
Topical CS spray
Topical steroid spray
Consider increase dose
Consider CS drops
Consider oral CS
Severe (based on
severity assessment**)
Mucosal disease at
endoscopy
Topical CS
Oral CS
(short course)
Review aer 1-3
months
Improvement
Unilateral symptoms
Bleeding crusng
Cacosmia
Peri-orbital edema/
erythema
Displaced globe
Double or reduced
vision
Ophthalmoplegia
Severe frontal headache
Frontal swelling
Signs of meningis
Neurological signs
Urgent invesgaon and
intervenon
Review aer 1 month
No improvement
Consider aspirin
desensizaon for AERD
Consider AFRS
Improvement
Connue with
topical CS
No improvement
Consider aspirin
desensizaon for
AERD
Consider AFRS
CT scan
Review every 6
months
Follow up
+ Nasal irrigaon
+ Topical ± oral CS
± Long term anbiocs
Surgery
Figure 1. CRSwNPs in adults: Management algorithm for rhinologists. *See Table 1 for associated conditions; **For example: Visual analog score or SNOT-20. CRSwNPs ⫽
chronic rhinosinusitis with nasal polyps; SNOT-20 ⫽ 20-item Sino-Nasal Outcome test; VAS ⫽ visual analog score; CS ⫽ corticosteroid; CT ⫽ computed tomography;
AERD ⫽ aspirin-exacerbated respiratory disease; AFRS ⫽ allergic fungal rhinosinusitis. (Source: Adapted from Ref. 5.)
biofilms; the elimination of ostial obstruction66; and the improved
postoperative delivery of topical therapy into the sinuses.5 Although trials providing high-level evidence of the efficacy of FESS
are missing, there is levels II–III evidence that FESS is associated
American Journal of Rhinology & Allergy
with improved symptoms (nasal obstruction and discharge) and
quality of life.5,67,68 Perioperative complication has been reported
in 6%, most commonly intraoperative hemorrhage, less commonly
CSF leak.69 Despite optimal medical and surgical therapy for nasal
Copyright © Oceanside Publications, Inc. All rights reserved. DO NOT COPY
For permission to copy go to https://www.oceansidepubl.com/permission.htm
S19
polyps, the underlying inflammation often remains difficult to
control; consequently, there is a fairly high recurrence rate for
nasal polyps.
NASAL POLYP SUMMARY
In summary, nasal polyps occur in 1–4% of the population, usually
occurring in the setting of an underlying local or systemic disease.
The most common associated condition is CRS. High prevalence of
nasal polyps are also seen in allergic fungal sinusitis, AERD, ChurgStrauss syndrome, cystic fibrosis, CRS, primary ciliary dyskinesia,
asthma, and Young’s syndrome. In the setting of CRS, nasal polyps
are not likely to be cured by either medical or surgical therapy;
however, control is generally attainable. The best medical evidence
supports the use of INCSs for maintenance therapy and short courses
of oral corticosteroids (⬍4weeks) for exacerbations. The evidence for
short- and long-term antibiotics is much less robust. For patients with
difficult-to-treat CRSwNPs, FESS provides an adjunctive therapeutic
option.
11.
12.
13.
14.
15.
16.
17.
18.
19.
CLINICAL PEARLS
• Nasal polyps typically arise from the middle meatus and ethmoid
region.
• Nasal polyps are most commonly associated with CRS.
• Fifteen to 23% of nasal polyp patients have AERD.
• The presence of a unilateral polypoid mass suggests a higher risk
for malignancy.
• Prominent symptoms of nasal polyps include nasal obstruction and
hypo/anosmia.
• In the pediatric population, the occurrence of nasal polyps warrants
a sweat test evaluation for cystic fibrosis.
• Intranasal steroids and intermittent oral steroids are the mainstay of
medical therapy.
• FESS is reserved for patients whose condition proves difficult to
treat and who manifest persistent symptoms despite medical therapy.
20.
ACKNOWLEDGEMENT
26.
The authors wish to thank Davis Settipane for his technical assistance in the creation of Figure 1.
27.
21.
22.
23.
24.
25.
REFERENCES
1.
2.
3.
4.
5.
6.
7.
8.
9.
10.
S20
Brain DJ. Historical background. In Nasal Polyps: Epidemiology,
Pathogenesis and Treatment. Settipane GA, Lund VJ, Bernstein JM,
and Tos M (Eds). Providence, RI: OceanSide Pubications, 7–15, 1997.
Dalgorf DM, and Harvey RJ. Sinonasal anatomy and function. Am J
Rhinol Allergy 27:S3–S6, 2013.
Bateman ND, Fahy C, and Woolford TJ. Nasal Polyps: Still more
questions than answers. J Laryngol Otol 117:1–9, 2003.
Soler ZM, Mace JC, Litvack JR, and Smith TL. Chronic rhinosinusitis,
race, and ethnicity. Am J Rhinol Allergy. 26:110–116, 2012.
Fokkens WJ, Lund VJ, Mullol J, et al. European position paper on
rhinosinusitis and nasal polyps 2012. Rhinol Suppl 23:1–298, 2012.
Virgin FW, Rowe SM, Wade MB, et al. Extensive surgical and comprehensive postoperative medical management for cystic fibrosis
chronic rhinosinusitis. Am J Rhinol Allergy 26:70–75, 2012.
Settipane GA. Epidemiology of nasal polyps. In Nasal Polyps: Epidemiology, Pathogenesis and Treatment. Settipane GA, Lund VJ,
Bernstein JM, and Tos M (Eds). Providence, RI: OceanSide Pubications, 17–24, 1997.
Settipane RA, Peters AT, and Chandra R. Chronic rhinosinusitis.
Am J Rhinol Allergy. 27:S11–S15, 2013.
Nakayama T, Asaka D, Yoshikawa M, et al. Identification of chronic
rhinosinusitis phenotypes using cluster analysis. Am J Rhinol Allergy
26:172–176, 2012.
Onerci M, Elsurer C, Guzel EE, and Dagdeviren A. Distribution of
inflammatory cells, adhesion molecules, intermediate filaments, and
chemokine receptors in subgroups of nasal polyp patients. Am J
Rhinol Allergy 25:e176–e180, 2011.
28.
29.
30.
31.
32.
33.
34.
35.
Han JK. Subclassification of chronic rhinosinusitis. Laryngoscope.
123(suppl 2):S15–S27, 2013.
Dykewicz MS, and Hamilos DL. Rhinitis and sinusitis. J Allergy Clin
Immunol 125:S103–S115, 2010.
Banerji A, Piccirillo JF, Thawley SE, et al. Chronic rhinosinusitis
patients with polyps or polypoid mucosa have a greater burden of
illness. Am J Rhinol 21:19–26, 2007.
Pearlman AN, Chandra RK, Chang D, et al. Relationships between
severity of chronic rhinosinusitis and nasal polyposis, asthma, and
atopy. Am J Rhinol Allergy 23:145–148, 2009.
Laury AM and Wise SK. Allergic fungal rhinosinusitis. Am J Rhinol
Allergy 27:S26–S27, 2013.
Tan BK, Zirkle W, Chandra RK, et al. Atopic profile of patients failing
medical therapy for chronic rhinosinusitis. Int Forum Allergy Rhinol
1:88–94, 2011.
Settipane RA, Borish L, and Peters AT. Determining the role of
allergy in sinonasal disease. Am J Rhinol Allergy 27:S56–S58, 2013.
Kennedy JL, and Borish L. Chronic sinusitis pathophysiology: The
role of allergy. Am J Rhinol Allergy DOI: 10.2500/ajra.2013.27.3906
[Epub ahead of print date April 18, 2013].
Dávila I, Rondón C, Navarro A, et al. Aeroallergen sensitization
influences quality of life and comorbidities in patients with nasal
polyposis. Am J Rhinol Allergy 26:e126–e131, 2012.
Hellquist HB. In Nasal Polyps: Epidemiology, Pathogenesis and
Treatment. Settipane GA, Lund VJ, Bernstein JM, and Tos M (Eds).
Providence, RI: OceanSide Pubications, 17–24, 1997.
Cao PP, Li HB, Wang BF, et al. Distinct immunopathologic characteristics of various types of chronic rhinosinusitis in adult Chinese. J
Allergy Clin Immunol 124:478–484.e1–2, 2009.
Fan Y, Feng S, Xia W, et al. Aspirin-exacerbated respiratory disease in
China: A cohort investigation and literature review. Am J Rhinol
Allergy 26:e20–e22, 2012.
Katotomichelakis M, Tantilipikorn P, Holtappels G, et al. Inflammatory patterns in upper airway disease in the same geographical area
may change over time. Am J Rhinol Allergy 2013. DOI:10.2500/
ajra.2013.27.3922.
Sobol SE, Christodoulopoulos P, Manoukian JJ, et al. Cytokine profile
of chronic sinusitis in patients with cystic fibrosis. Arch Otolaryngol
Head Neck Surg 128:1295–1298, 2002.
Hsu J, and Peters AT. Pathophysiology of chronic rhinosinusitis with
nasal polyp. Am J Rhinol Allergy 25:285–290, 2011.
Madeo J, and Frieri M. Bacterial biofilms and chronic rhinosinusitis.
Allergy Asthma Proc DOI: 10.2500/aap.2013.34.3665 [Epub ahead of
print date March 7, 2013].
Van Crombruggen K, Zhang N, Gevaert P, et al. Pathogenesis of
chronic rhinosinusitis: Inflammation. J Allergy Clin Immunol 128:
728–732, 2011.
Kim DW, Khalmuratova R, Hur DG, et al. Staphylococcus aureus
enterotoxin B contributes to induction of nasal polypoid lesions in an
allergic rhinosinusitis murine model. Am J Rhinol Allergy 25:e255–
e261, 2011.
Kim ST, Chung SW, Jung JH, et al. Association of T cells and eosinophils with Staphylococcus aureus exotoxin A and toxic shock syndrome toxin 1 in nasal polyps. Am J Rhinol Allergy 25:19–24, 2011.
Kern R, Conley D, Walsh W, et al., Perspectives on the etiology of
chronic rhinosinusitis: An immune barrier hypothesis. Am J of Rhinol
22:549–559, 2008.
Sun Y, Zhou B, Wang C, et al. Biofilm formation and Toll-like
receptor 2, Toll-like receptor 4, and NF-kappaB expression in sinus
tissues of patients with chronic rhinosinusitis. Am J Rhinol Allergy
26:104–109, 2012.
Tieu DD, Peters AT, Carter RT, et al. Evidence for diminished levels
of epithelial psoriasin and calprotectin in chronic rhinosinusitis. J
Allergy Clin Immunol 25:667–675, 2010.
Peters AT, Kato A, Zhang N, et al. Evidence for altered activity of the
IL-6 pathway in chronic rhinosinusitis with nasal polyps. J Allergy
Clin Immunol 125:397–403.e10, 2010.
Stressmann FA, Rogers GB, Chan SW, et al. Characterization of
bacterial community diversity in chronic rhinosinusitis infections
using novel culture-independent techniques. Am J Rhinol Allergy
25:e133–e140, 2011.
Wood AJ, Fraser JD, Swift S, et al. Intramucosal bacterial microcolonies exist in chronic rhinosinusitis without inducing a local immune
response. Am J Rhinol Allergy 26:265–270, 2012.
May–June 2014, Vol. 28, No. 3
Copyright © Oceanside Publications, Inc. All rights reserved. DO NOT COPY
For permission to copy go to https://www.oceansidepubl.com/permission.htm
36.
37.
38.
39.
40.
41.
42.
43.
44.
45.
46.
47.
48.
49.
50.
51.
52.
53.
Kato A, Peters A, Suh L, et al. Evidence of a role for B cell-activating
factor of the TNF family in the pathogenesis of chronic rhinosinusitis
with nasal polyps. J Allergy Clin Immunol 121:1385–1392, 2008.
Hulse KE, Norton JE, Suh L, et al. Chronic rhinosinusitis with nasal
polyps is characterized by B-cell inflammation and EBV-induced
protein 2 expression. J Allergy Clin Immunol 131:1075–1083, 2013.
(DOI: pii: S0091-6749(13)00252-2. 10.1016/j.jaci. 2013.01.043.)
Chang JE, White A, Simon RA, and Stevenson DD. Aspirin-exacerbated respiratory disease: Burden of disease. Allergy Asthma Proc
33:117–121, 2012.
Bavbek S, Dursun B, Dursun E, et al. The prevalence of aspirin
hypersensitivity in patients with nasal polyposis and contributing
factors. Am J Rhinol Allergy 25:411–415, 2011.
Laidlaw TM, Kidder MS, Bhattacharyya N, et al. Cysteinyl leukotriene overproduction in aspirin-exacerbated respiratory disease is
driven by platelet-adherent leukocytes. Blood 119:3790–3798, 2012.
Bernstein JM. The immunohistopathology and pathophysiology of
nasal polyps. (The differential diagnosis of nasal polyps.) In Nasal
Polyps: Epidemiology, Pathogenesis and Treatment. Settipane GA,
Lund VJ, Bernstein JM, and Tos M (Eds). Providence, RI: OceanSide
Pubications, 85–95, 1997.
Wood JW, and Casiano RR. Inverted papillomas and benign nonneoplastic lesions of the nasal cavity. Am J Rhinol Allergy 26:157–163,
2012.
Gaines A. Olfactory disorders. Am J Rhinol Allergy 27:S45–S47, 2013.
Aouad RK, and Chiu AG. State of the art treatment of nasal polyposis. Am J Rhinol Allergy 25:291–298, 2011.
Ocampo CJ, and Peters AT. Medical therapy as the primary modality
for the management of chronic rhinosinusitis. Allergy Asthma Proc
34:132–137, 2013.
Salman S, Akpinar ME, Yigit O, and Gormus U. Surfactant protein A
and D in chronic rhinosinusitis with nasal polyposis and corticosteroid response. Am J Rhinol Allergy 26:e76–e80, 2012.
Park SK, Hur DY, and Urm SH. Effects of dexamethasone or methotrexate on chitinolytic activity in nasal polyps. Am J Rhinol Allergy
26:183–186, 2012.
Acıoğlu E, Yigit O, Alkan Z, et al. The effects of corticosteroid on
tissue lactoferrin in patients with nasal polyposis. Am J Rhinol Allergy 26:e28–e31, 2012.
Small CB, Hernandez J, Reyes A, et al. Efficacy and safety of mometasone furoate nasal spray in nasal polyposis. J Allergy Clin Immunol
116:1275–1281, 2005.
Kalish L, Snidvongs K, Sivasubramaniam R, et al. Topical steroids for
nasal polyps. Cochrane Database Syst Rev 12:CD006549, 2012.
Rudmik L, Schlosser RJ, Smith TL, and Soler ZM. Impact of topical
nasal steroid therapy on symptoms of nasal polyposis: A metaanalysis. Laryngoscope 122:1431–1437, 2012.
Vaidyanathan S, Barnes M, Williamson P, et al. Treatment of chronic
rhinosinusitis with nasal polyposis with oral steroids followed by
topical steroids: A randomized trial. Ann Intern Med 154:293–302,
2011.
Kirtsreesakul V, Wongsritrang K, and Ruttanaphol S. Does oral prednisolone increase the efficacy of subsequent nasal steroids in treating
nasal polyposis? Am J Rhinol Allergy 26:455–462, 2012.
American Journal of Rhinology & Allergy
54.
55.
56.
57.
58.
59.
60.
61.
62.
63.
64.
65.
66.
67.
68.
69.
70.
71.
Poetker DM, Jakubowski LA, Lal D, et al. Oral corticosteroids in the
management of adult chronic rhinosinusitis with and without nasal
polyps: An evidence-based review with recommendations. Int Forum
Allergy Rhinol 3:104–120, 2012.
Van Zele T, Gevaert P, Holtappels G, et al. Oral steroids and doxycycline: Two different approaches to treat nasal polyps. J Allergy Clin
Immunol 125:1069–1076.e4, 2010.
Soler ZM, Oyer SL, Kern RC, et al. Antimicrobials and chronic
rhinosinusitis with or without polyposis in adults: An evidencedbased review with recommendations. Int Forum Allergy Rhinol 3:31–
47, 2013.
Wallwork B, Coman W, Mackay-Sim A, et al. A double-blind, randomized, placebo-controlled trial of macrolide in the treatment of
chronic rhinosinusitis. Laryngoscope 116:189–193, 2006.
Frothingham R. Rates of torsades de pointes associated with ciprofloxacin, ofloxacin, levofloxacin, gatifloxacin, and moxifloxacin. Pharmacotherapy 21:1468–1472, 2001.
van der Linden PD, Sturkenboom MC, Herings RM, et al. Increased
risk of achilles tendon rupture with quinolone antibacterial use,
especially in elderly patients taking oral corticosteroids. Arch Intern
Med 163:1801–1807, 2003.
Ray WA, Murray KT, Hall K, et al. Azithromycin and the risk of
cardiovascular death. N Engl J Med 366:1881–1890, 2012.
White AA, and Stevenson DD. Aspirin-exacerbated respiratory disease: Update on pathogenesis and desensitization. Semin Respir Crit
Care Med 33:588–594, 2012.
Moebus RG, and Han JK. Immunomodulatory treatments for aspirin
exacerbated respiratory disease. Am J Rhinol Allergy 26:134–140,
2012.
Pinto JM, Mehta N, DiTineo M, et al. A randomized, double-blind,
placebo-controlled trial of anti-IgE for chronic rhinosinusitis. Rhinology 48:318–324, 2010.
Gevaert P, Calus L, Van Zele T, et al. Omalizumab is effective in
allergic and nonallergic patients with nasal polyps and asthma. J
Allergy Clin Immunol. 131:110–116.e1, 2013.
Gevaert P, Van Bruaene N, Cattaert T, et al. Mepolizumab, a humanized anti-IL-5 mAb, as a treatment option for severe nasal polyposis.
J Allergy Clin Immunol 128:989–995.e1–8, 2011.
Leung RM, Kern RC, Conley DB, et al. Osteomeatal complex obstruction is not associated with adjacent sinus disease in chronic rhinosinusitis with polyps. Am J Rhinol Allergy 25:401–403, 2011.
Smith TL, Kern R, Palmer JN, et al. Medical therapy vs surgery for
chronic rhinosinusitis: A prospective, multi-institutional study with
1-year follow-up. Int Forum Allergy Rhinol 3:4–9, 2013.
Gunhan K, Zeren F, Uz U, et al. Impact of nasal polyposis on erectile
dysfunction. Am J Rhinol Allergy 25:112–115, 2011.
Asaka D, Nakayama T, Hama T, et al. Risk factors for complications
of endoscopic sinus surgery for chronic rhinosinusitis. Am J Rhinol
Allergy 26:61–64, 2012.
Hennessey PT, and Reh DD. Benign sinonasal neoplasms. Am J
Rhinol Allergy 27:S31–S34, 2013.
Harvey RJ, and Dalgorf DM. Sinonasal malignancies. Am J Rhinol
Allergy 27:S35–S38, 2013.
e
Copyright © Oceanside Publications, Inc. All rights reserved. DO NOT COPY
For permission to copy go to https://www.oceansidepubl.com/permission.htm
S21
Allergic fungal rhinosinusitis
Adrienne M. Laury, M.D., and Sarah K. Wise, M.D., M.S.C.R.
ABSTRACT
Allergic fungal rhinosinusitis (AFRS) is a type of chronic rhinosinusitis in which patients classically exhibit nasal polyps, type I IgE-mediated
hypersensitivity, characteristic findings on computed tomography scans, eosinophilic mucin, and positive fungal stain. New research has sought to further
understand the pathophysiology of AFRS. However, this has also led to debate about the classification and predominance of this interesting disease process.
Historically, patients with AFRS are immunocompetent. The disease is most prevalent in the southeast and south central United States and typically presents
with sinus pressure, hyposmia, and congestion. Radiographically, cases of AFRS have a distinct appearance, often exhibiting unilateral heterogeneously dense
material, which may erode and expand the paranasal sinus bony walls. Treatment typically consists of surgery, sinonasal irrigations, and topical and systemic
steroids, all with the effort to decrease the fungal load and antigenic response. Immunotherapy is also often included in the treatment regimen for AFRS.
A
llergic fungal rhinosinusitis (AFRS) is a class of chronic rhinosinusitis (CRS) first described 30 years ago by Miller and colleagues.1 It was later characterized in 1994 by Bent and Kuhn based
on the presence of five criteria: (1) atopic history, (2) nasal polyposis,
(3) characteristic radiographic findings, (4) eosinophilic mucin without fungal invasion, and (5) positive fungal stain.2 Pathological findings in AFRS will often include a dense inflammatory response,
predominance of eosinophils, and Charcot-Leyden crystals (a byproduct of eosinophil degranulation), as well as hyphal elements on
fungal stains.3 Fungal species can vary, including Aspergillus, Alternaria, Bipolaris, and Curvularia, as well as others.
Over the past 20 years several studies have expanded on our
original understanding of the pathogenesis of AFRS. In patients with
AFRS, evidence of increased IgE production (consistent with a type I
hypersensitivity) has been established in both systemic serum levels
and local sinonasal tissue, although not always in both.4 Furthermore,
increased sinonasal tissue production of antigen-specific IgE includes
reactivity to both fungal and nonfungal antigens.5 Additionally, Carney et al. have identified multiple Th-2 cytokines including IL-4, IL-5,
IL-13, and major basic protein, as well as eosinophils and mast cells in
the inflammatory cascade of AFRS patients.6 Nonetheless, the classification of AFRS as a distinct clinical entity has been called into
question. Certain studies have debated the association of AFRS with
eosinophilic mucin CRS (EMCRS). Ferguson has claimed AFRS to be
distinct based on the diagnostic findings of unilaterality, fungal presence, and decreased incidence of asthma and aspirin sensitivity compared with EMCRS.7 In addition, Ferguson noted that patients with
EMCRS have a higher incidence of IgG1 deficiency compared with
AFRS patients.7 In contrast, Pant and colleagues have suggested
categorization of AFRS as a subclass of EMCRS based on the findings
that EM appears to be the predominant characteristic to predict the
pattern of disease presentation and severity.8
Regardless of nomenclature, AFRS patients are often identified by
a characteristic clinical history, presentation, and radiographic findings. Patients are typically immunocompetent, young adults who
From Emory University, Sinus, Nasal, and Allergy Center, Atlanta, Georgia
The authors have no conflicts of interest to declare pertaining to this article
Address correspondence and reprint requests to Sarah K. Wise, M.D., M.S.C.R., Emory
University, Sinus, Nasal and Allergy Center, 550 Peachtree Street, MOT 9th Floor,
Atlanta, GA 30308
E-mail address: [email protected]
Originally published in Am J Rhinol Allergy 27, S26 –S27, 2013
Copyright © 2014, OceanSide Publications, Inc., U.S.A.
S22
present with a prolonged report of nasal congestion, hyposmia or
anosmia, and facial pressure. In the southeastern United States, where
up to 32% of patients undergoing functional endoscopic sinus surgery
have been reported to have AFRS, African Americans and patients of
a lower socioeconomic status have been shown to have a greater
incidence and severity of AFRS.9 On physical exam, polyps are an
almost universal endoscopic finding, but, additionally, in more severe
cases, physical findings such as diplopia, proptosis, and telecanthus
can be identified. Radiographically, AFRS patients frequently present
with asymmetric, heterogeneously dense material filling and expanding one or more of their paranasal sinuses (Fig. 1). In soft tissue
windows, foci of near metallic density are seen, reflecting chelation of
metal salts by the fungal organisms. In more severe cases, this mucin
can actually erode through surrounding bone, encroaching into
spaces occupied by vital organs such as brain, orbit, and major
vessels. A radiological staging system, which specifically analyzes
this degree of bony remodeling, has also been developed specifically
for AFRS.10 As opposed to the Lund-Mackay radiological staging
system, which is based on sinus opacification, the AFRS radiological
staging system uses degree of bony erosion and expansion as the
defining criteria. In this system, points are assigned for each sinus
wall that has undergone bony remodeling up to a maximum of ⬃3
points per sinus.
Treatment of AFRS is based primarily on surgical debridement of
fungal mucin and polyps, as well as the use of saline irrigations and
systemic and topical steroids, all with an effort to reduce the antigenic
load and inflammatory response. Systemic steroids, especially after
surgical debridement, have been considered a mainstay of the treatment approach, but optimal dosing and treatment length remain
controversial. It is also notable that systemic antifungal therapy is
thought to have little role in the treatment of AFRS and is reserved for
selected refractory cases. Specific allergen immunotherapy has also
been used as a treatment in the long-term control of AFRS. The
rationale behind the use of immunotherapy in the treatment paradigm for AFRS is based on the premise that immunotherapy modifies
the basic allergic mechanism by inducing desensitization and an
anergy state.11 Although, prospective studies are limited, one retrospective study did show a drop in reoperation rates down to 11% in
AFRS patients who underwent immunotherapy versus 33% in those
treated with placebo.12
CLINICAL PEARLS
• AFRS has been characterized by Bent and Kuhn based on the
presence of five distinct criteria: (1) atopic history, (2) nasal polyp-
May–June 2014, Vol. 28, No. 3
Copyright © Oceanside Publications, Inc. All rights reserved. DO NOT COPY
For permission to copy go to https://www.oceansidepubl.com/permission.htm
Figure 1. Coronal computed tomography
(CT) scan in (A) soft tissue and (B) bone
window algorithms showing left maxillary
and ethmoid opacification. There are heterogeneous densities within the expanded
sinus cavities, as well as erosion of the left
medial maxillary wall, lamina papyracea,
and ethmoid skull base. This is a classic
appearance of allergic fungal rhinosinusitis (AFRS) on CT imaging.
osis, (3) characteristic radiographic findings, (4) EM without fungal
invasion, and (5) positive fungal stain.
• Relationship of AFRS to EMCRS is debated.
• Bony erosion and expansion of the sinuses is a relatively common
finding in AFRS and is the criteria on which the AFRS radiographic
staging system is based.
• Treatment of AFRS includes surgical debridement, nasal rinses, and
topical and systemic steroids. Immunotherapy has shown promise
in helping to decrease reoperation rates.
2.
3.
4.
6.
7.
8.
9.
REFERENCES
1.
5.
Miller JW, Johnston A, and Lamb D. Allergic aspergillosis of the
maxillary sinuses. Thorax 36:710, 1981.
Bent JP III, and Kuhn FA. Diagnosis of allergic fungal sinusitis.
Otolaryngol Head Neck Surg 111:580–588, 1994.
Corey JP, Delsupehe KG, and Ferguson GJ. Allergic fungal sinusitis:
Allergic, infectious, or both? Otolaryngol Head Neck Surg 113:110,
1995.
Collins M, Nair S, Smith W, et al. Role of local immunoglobulin E
production in the pathophysiology of noninvasive fungal sinusitis.
Laryngoscope 114:1242–1246, 2004.
American Journal of Rhinology & Allergy
10.
11.
12.
Wise S, Ahn C, Lathers D, et al. Antigen-specific IgE in sinus mucosa
of allergic fungal rhinosinusitis patients. Am J Rhinol 22:451–456,
2008.
Carney A, Tan L, Adams D, et al. Th2 immunological inflammation
in allergic fungal sinusitis, nonallergic eosinophilic fungal sinusitis,
and chronic rhinosinusitis. Am J Rhinol 20:145–149, 2006.
Ferguson BJ. Eosinophilic mucin rhinosinusitis: A distinct clinicopathological entity. Laryngoscope 110:799–813, 2000.
Pant H, Schembri MA, Wormald PJ, and Macardle PJ. IgE-mediated
fungal allergy in allergic fungal sinusitis. Laryngoscope 119:1046–
1052, 2009.
Wise SK, Ghegan MD, Gorham E, and Schlosser RJ. Socioeconomic
factors in the diagnosis of allergic fungal rhinosinusitis. Otolaryngol
Head Neck Surg 138:38–42, 2008.
Wise SK, Rogers GA, Ghegan MD, et al. Radiologic staging system
for allergic fungal rhinosinusitis (AFRS). Otolaryngol Head Neck
Surg 140:735–740, 2009.
Chang H, Han DH, Mo J, et al. Early compliance and efficacy of
sublingual immunotherapy in patients with allergic rhinitis for house
dust mites. Clin Exp Otorhinolaryngol 2:136–140, 2009.
Bassichis BA, Marple BF, Mabry RL, et al. Use of immunotherapy in
previously treated patients with allergic fungal sinusitis. Otolaryngol
Head Neck Surg 125:487–490, 2001.
e
Copyright © Oceanside Publications, Inc. All rights reserved. DO NOT COPY
For permission to copy go to https://www.oceansidepubl.com/permission.htm
S23
Invasive fungal rhinosinusitis
Praveen Duggal, M.D., and Sarah K. Wise, M.D., M.S.C.R.
ABSTRACT
Invasive fungal rhinosinusitis (IFRS) is a disease of the paranasal sinuses and nasal cavity that typically affects immunocompromised patients in the acute
fulminant form. Early symptoms can often mimic rhinosinusitis, while late symptoms can cause significant morbidity and mortality. Swelling and mucosal
thickening can quickly progress to pale or necrotic tissue in the nasal cavity and sinuses, and the disease can rapidly spread and invade the palate, orbit,
cavernous sinus, cranial nerves, skull base, carotid artery, and brain. IFRS can be life threatening if left undiagnosed or untreated. While the acute fulminant
form of IFRS is the most rapidly progressive and destructive, granulomatous and chronic forms also exist. Diagnosis of IFRS often mandates imaging studies
in conjunction with clinical, endoscopic, and histopathological examination. Treatment of IFRS consists of reversing the underlying immunosuppression,
antifungal therapy, and aggressive surgical debridement. With early diagnosis and treatment, IFRS can be treated and increase patient survival.
A
cute fulminant invasive fungal rhinosinusitis (IFRS) is a lifethreatening condition that requires immediate medical attention. Patients with this disease previously had abysmal survival rates
of 20–50%.1 With improvements in diagnosis and treatment, recent
studies reveal much improved survival, with mortality rates around
18%.2 Acute fulminant IFRS occurs almost exclusively in immunocompromised patients. These patients include those suffering from
conditions such as leukemia; acquired immunodeficiency syndrome;
poorly controlled diabetes mellitus or diabetic ketoacidosis; or those
undergoing bone marrow transplant, chemotherapy, or long-term
corticosteroid use. The most common predisposing factor to IFRS is
neutropenia, especially those with ⬍1000 neutrophils/␮L of blood,
which significantly reduces the inflammatory response and the
body’s ability to battle the infection.
This article will review the classification of IFRS and briefly discuss
the two chronic forms of IFRS, chronic invasive fungal sinusitis and
granulomatous fungal sinusitis. We will then discuss acute fulminant
IFRS in more detail, including pathophysiology, diagnosis, and management.
BACKGROUND
IFRS is typically classified into three categories: (1) chronic, (2)
granulomatous, and (3) acute fulminant.3 Chronic IFRS is a very rare
disease normally defined as an invasive paranasal sinus condition
occurring over a course of weeks to months. Patients may or may not
be immunocompetent and can present early with rhinosinusitis-like
symptoms that progress with fungal hyphae invading the sinus mucosa. Often, there are less inflammatory cells noted on biopsy compared with more acute infections. Symptoms can include periorbital
edema, proptosis, blindness, cranial nerve palsies, and soft tissue
involvement. Granulomatous IFRS infections have a gradual onset
and have been noted to occur mainly in the Sudan, India, and Pakistan.4 The condition often affects immunocompetent patients as well.
Symptoms may consist of proptosis or an enlarging mass within the
nose, orbit, or paranasal sinuses. Noncaseating granulomas with giant
cells and hyphae are noted. Aspergillus flavus is often the most isolated
species in granulomatous cases.
Acute fulminant IFRS is the most common form of invasive fungal
paranasal sinus infection.5 Infection is often attributed to invasion by
fungi that have previously colonized the sinuses or fungal spores that
have been inhaled. The disease occurs almost exclusively in immunosuppressed and quantitatively or functionally neutropenic patients. Functional neutropenia can be caused by poorly managed
diabetic mellitus and chronic steroid use. The lack of a functional and
normal immune system can lead the host response to be delayed or
nonexistent. Acute fulminant IFRS can progress very quickly in these
patients, often in a matter of hours to days. Early symptoms include
fever, nasal congestion, facial pain, epistaxis, and headache. Late
symptoms may include numbness, blindness, central neurological
symptoms, and even death. The disease can be rapidly fatal in 50–80%
of patients if left untreated.
PATHOPHYSIOLOGY
Spores from various fungi are found throughout the environment
and fungal exposure is common. Items such as decaying fruit, vegetables, plants, soil, old bread, and manure can all carry fungal spores.
However, infection is often avoided when patients are immunocompetent. Fungal infections that are aggressive and affect immunocompromised patients are species from the genus Aspergillus and the
Mucorales order, which includes Absidia, Mucor, Rhizomucor, and
Rhizopus. Mucor has been highly associated with infecting diabetic
ketoacidosis patients.6 Mucor and Aspergillus spread by invading arterial blood vessels. Tissue necrosis secondary to obstructed blood
flow leads to pale, gray, or black infarcted tissue. Infection can also
cause perineural invasion and spread across tissue planes.7 Without a
properly functioning immune system, the infection is left unimpeded
and can continue to progress to more vital structures.
DIAGNOSIS
From Emory University, Sinus, Nasal, and Allergy Center, Atlanta, Georgia
The authors have no conflicts of interest to declare pertaining to this article
Address correspondence and reprint requests to Sarah K. Wise, M.D., M.S.C.R., Emory
University, Sinus, Nasal and Allergy Center, 550 Peachtree Street, MOT 9th Floor,
Atlanta, GA 30308
E-mail address: [email protected]
Originally published in Am J Rhinol Allergy 27, S28 –S30, 2013
Copyright © 2014, OceanSide Publications, Inc., U.S.A.
S24
Proposed diagnostic criteria for IFRS include (1) mucosal thickening or air fluid levels consistent with sinusitis on imaging and (2)
histopathological evidence of fungal hyphae within sinus mucosa,
submucosa, blood vessels, or bone.3 Physical examination and nasal
endoscopy are essential to look for signs of significant edema, pallor,
ischemia, or necrosis of the nasal and paranasal sinus mucosa. Imaging, in conjunction with physical exam and endoscopy with biopsy, is
crucial in the workup up of IFRS.
May–June 2014, Vol. 28, No. 3
Copyright © Oceanside Publications, Inc. All rights reserved. DO NOT COPY
For permission to copy go to https://www.oceansidepubl.com/permission.htm
Figure 2. Gomori methenamine silver (GMS) fungal stain showing blackstained fungal hyphae invading sinonasal tissue. (Photograph courtesy of
Dr. Susan Muller.)
Figure 1. Invasive fungal rhinosinusitis (IFRS) involving the nasal cavity
and turbinate mucosa. Note the central necrotic IFRS area surrounded by
pale mucosa, in contrast to the healthy, pink areas of normal nasal mucosa.
(Photograph courtesy of Dr. John Delgaudio.)
Presentation
Early symptoms can mimic acute rhinosinusitis (nasal drainage,
congestion, headaches, and facial pain). Recognition of evolving IFRS
in this initial phase requires a high index of suspicion.8 After initially
presenting with these acute symptoms, patients may experience a
“latent” phase where pain transiently subsides, further confounding
the diagnosis if clinical suspicion is insufficient. This rapidly transitions to the onset of ominous signs and symptoms that include facial
or palate numbness, proptosis, visual changes, facial swelling, and
dental pain. IFRS is best visualized on nasal endoscopy and most
commonly affects the nasal septum, turbinates, and paranasal sinuses
(Fig. 1). Swelling and mucosal thickening can quickly progress to pale
or necrotic tissue in the nasal cavity and sinuses, and the disease can
rapidly spread and invade the palate, orbit, cavernous sinus, cranial
nerves, skull base, carotid artery, and brain.
Imaging
Computed tomographic (CT) imaging should quickly be used in
the workup for IFRS. Findings suggestive of IFRS include edema of
nasal cavity, sinus mucoperiosteal thickening, bone erosion, orbital
invasion, facial soft tissue swelling, and periantral or retroantral soft
tissue infiltration. None of these findings are pathognomonic for
IFRS, however. The most common finding on CT for patients with
IFRS is severe unilateral thickening of the nasal cavity mucosa and
soft tissues including the turbinates, septum, and nasal floor.9 Bone
destruction can be easily noted on CT scan and is highly suggestive of
IFRS, but it is often a late finding.
Magnetic resonance imaging (MRI) is another imaging modality
that is useful in detecting IFRS. MRI should be considered if the orbit
or intracranial involvement is suspected.10 MRI is more sensitive than
CT in the early screening and diagnosis of acute fulminant IFRS.11
Although having a higher cost, MRI offers zero radiation exposure
and provides slightly higher sensitivity to subtle infiltration of facial
fat planes anterior and posterior to the maxillary sinus. Despite the
imaging tool, suspicious clinical and radiographic findings should
prompt endoscopic evaluation and biopsy.
American Journal of Rhinology & Allergy
Pathology
The gold standard in diagnosis of IFRS is examination of the
pathology with permanent section. However, permanent section and
fungal stains can be time-consuming and delay diagnosis and treatment. Fungal cultures are an alternative means of diagnosis but have
a low sensitivity and can take days for growth and speciation to
occur. With the noted aggressiveness of IFRS, delay in immediate
treatment can lead to significant morbidity and possible mortality.
Frozen section pathology can provide an early diagnosis in the
clinical setting.12 Bedside and intraoperative frozen section has an
extremely high positive predictive value. With a positive finding of
fungal hyphae invading soft tissue on frozen section, the patient
should be taken to the operating room for surgical debridement.
Frozen section for IFRS diagnosis has a high sensitivity and negative
predictive value. Initial biopsy specimens should be taken at the
border between pale/necrotic mucosa and normal-appearing mucosa, areas questionable on CT imaging, and/or the middle turbinate.13 The frozen section is a tool that can guide thorough surgical
debridement and prevent further morbidity such as orbital exenteration or added invasive procedures by providing diagnosis of negative margins during surgery. Although not only helping with early
diagnosis and initial treatment, the frozen section can also guide
further postoperative and conservative therapy by focusing further
debridement to infected areas. Permanent pathological sections often
incorporate fungal stains that allow for excellent visualization of
fungal hyphae within the tissue sections (Fig. 2).
TREATMENT
Treatment of IFRS consists of (1) reversing the underlying immunosuppression, (2) antifungal therapy, and (3) aggressive surgical
debridement. Raising the absolute neutrophil count in those with
quantitative neutropenia is an important step in treatment of invasive
fungal disease. Improved diabetes management and control of fluctuating blood sugars is vital to helping those patients suffering from
IFRS secondary to their diabetic ketoacidosis. The prognosis is extremely poor if the host immune response does not improve.
Patients suffering from immunosuppressive treatment should
likely halt their therapy. An attempt to resolve neutropenia may be
made by using granulocyte colony stimulating factor.10 Isolation of
the fungal organism is also necessary to guide antifungal therapy.
Oral and i.v. antifungal agents can be implemented based on the
severity of the infection, the speciation of the offending agent, and the
Copyright © Oceanside Publications, Inc. All rights reserved. DO NOT COPY
For permission to copy go to https://www.oceansidepubl.com/permission.htm
S25
presence of ongoing immunosuppression or the underlying disorder.
Surgical debridement is the gold standard to remove invasive fungal
disease. Surgical treatment often can be approached via endoscopic
techniques with or without a combination of external approaches
depending on the disease extent. The most common finding at surgery is mucosal edema and/or pale mucosa.14 All pale or necrotic
tissue should be removed from the paranasal sinuses and nasal cavity. Resection should be performed until bleeding tissue is visualized.
Further frozen section biopsy specimens can be sent intraoperatively
to show negative margins. Fungal cultures should also be sent at the
time of surgery to help with identification and speciation of the
fungus. Orbital exenteration may be indicated if removal of the eye
may prevent spread intracranially. The underlying prognosis of IFRS
is dictated by the ability to reverse the quantitative or functional
neutropenic state. Surgery and antifungal therapy can help relieve the
fungal burden until the immune status recovers. Patients should be
monitored after their initial surgery with follow-up endoscopy, imaging, and serial debridements as needed. Long-term follow-up is
indicated until remucosalization of the sinuses, resolution of crusting,
and cessation of bony sequestration has occurred.15
CLINICAL PEARLS
• Acute fulminant IFRS is a serious medical condition that can be
life-threatening and requires urgent attention. A high index of
suspicion is required in susceptible patients (e.g., immunocompromised) who present with symptoms of rhinosinusitis.
• Quantitative or functional neutropenia is the most common predisposing risk factor for acute fulminant IFRS.
• The most common finding on CT is severe unilateral thickening of
the sinonasal mucosa and soft tissue.
• Sinonasal endoscopy with biopsy, in conjunction with CT or MRI, is
critical in diagnosing and evaluating the extent of IFRS and planning surgical intervention.
• Reversing the underlying disorder causing immunosuppression or
neutropenia, as well as emergent surgical debridement, is essential
to minimize morbidity and mortality from IFRS.
S26
REFERENCES
1.
Gillespie MB, O’Malley BW Jr, and Francis HW. An approach to
fulminant invasive fungal rhinosinusitis in the immunocompromised host. Arch Otolaryngol Head Neck Surg 124:520–526, 1998.
2. Parikh SL, Venkatraman G, and DelGaudio JM. Invasive fungal
sinusitis: A 15-year review from a single institution. Am J Rhinol
18:75–81, 2004.
3. deShazo RD, O’Brien M, Chapin K, et al. A new classification and
diagnostic criteria for invasive fungal sinusitis. Arch Otolaryngol
Head Neck Surg 123:1181–1188, 1997.
4. Thompson GR III, and Patterson TF. Fungal disease of the nose and
paranasal sinuses. J Allergy Clin Immunol 129:321–326, 2012.
5. Epstein VA, and Kern RC. Invasive fungal sinusitis and complications of
rhinosinusitis. Otolaryngol Clin North Am 41:497–524, viii, 2008.
6. Dhong HJ, Lee JC, Ryu JS, and Cho DY. Rhinosinusitis in transplant
patients. Clin Otolaryngol Allied Sci 26:329–333, 2001.
7. Frater JL, Hall GS, and Procop GW. Histologic features of zygomycosis: Emphasis on perineural invasion and fungal morphology. Arch
Pathol Lab Med 125:375–378, 2001.
8. DelGaudio JM, and Clemson LA. An early detection protocol for
invasive fungal sinusitis in neutropenic patients successfully reduces
extent of disease at presentation and long term morbidity. Laryngoscope 119:180–183, 2009.
9. DelGaudio JM, Swain RE Jr, Kingdom TT, et al. Computed tomographic findings in patients with invasive fungal sinusitis. Arch
Otolaryngol Head Neck Surg 129:236–240, 2003.
10. Gillespie MB, and O’Malley BW. An algorithmic approach to the
diagnosis and management of invasive fungal rhinosinusitis in the
immunocompromised patient. Otolaryngol Clin North Am 33:323–
334, 2000.
11. Groppo ER, El-Sayed IH, Aiken AH, and Glastonbury CM. Computed tomography and magnetic resonance imaging characteristics
of acute invasive fungal sinusitis. Arch Otolaryngol Head Neck Surg
137:1005–1010, 2011.
12. Ghadiali MT, Deckard NA, Farooq U, et al. Frozen-section biopsy
analysis for acute invasive fungal rhinosinusitis. Otolaryngol Head
Neck Surg 136:714–719, 2007.
13. Gillespie MB, Huchton DM, and O’Malley BW. Role of middle turbinate biopsy in the diagnosis of fulminant invasive fungal rhinosinusitis. Laryngoscope 110:1832–1836, 2000.
14. DelGaudio JM. Endoscopic transnasal approach to the pterygopalatine fossa. Arch Otolaryngol Head Neck Surg 129:441–446, 2003.
15. Otto KJ, and Delgaudio JM. Invasive fungal rhinosinusitis: What is
the appropriate follow-up? Am J Rhinol 20:582–585, 2006.
e
May–June 2014, Vol. 28, No. 3
Copyright © Oceanside Publications, Inc. All rights reserved. DO NOT COPY
For permission to copy go to https://www.oceansidepubl.com/permission.htm
Benign sinonasal neoplasms
Patrick T. Hennessey, M.D. and Douglas D. Reh, M.D.
ABSTRACT
Benign sinonasal neoplasms are a heterogeneous group of tumors that present with similar symptoms including nasal obstruction, anosmia, rhinorrhea, and
epistaxis. The proper workup and accurate diagnosis is essential for these tumors so that the appropriate treatment plan can be established. In this article of
benign sinonasal neoplasms, we discuss their typical clinical presentation, histological and radiographic findings, and treatment options.
I
nverted papillomas occur at a rate of 0.2–6 cases per 100,000 patients per year and represent 0.5–4.0% of sinonasal tumors. Inverted papillomas occur more commonly in male subjects with a 3:1
male/female ratio and occur most commonly in the fifth and sixth
decades of life. The majority of inverted papillomas (95.1%) are unilateral and most commonly originate from the lateral nasal wall
(89%), with specific sites of origin being the maxillary sinus (53.9%),
the ethmoid sinuses (31.6%), the septum (9.9%), the frontal sinus
(6.5%), and the sphenoid sinus (3.9%).1
Symptoms of sinonasal inverted papilloma are typically nondescript and include unilateral nasal obstruction (58%), epistaxis (17%),
nasal drainage (14%), and sinusitis (9%).1 Other symptoms may include headache, facial numbness, facial swelling, diplopia, or anosmia. Because of the nonspecific nature of these symptoms, many
patients with inverted papilloma are not diagnosed until they have
advanced disease.
On nasal endoscopy inverted papillomas appear as lobulated, mucosa-covered polypoid masses with a more vascular appearance than
nasal polyps, which typically are more translucent in appearance
(Fig. 1). Histologically inverted papillomas are characterized by invagination of the epithelium into the underlying stroma without
invasion into or through an intact basement membrane. Additionally,
inflammatory changes including periosteal thickening, osteoblastic
rimming, and the presence of immature bone, are often found in the
bone underlying inverted papillomas.2
Although the etiology of inverted papilloma is unknown, several
different mechanisms for its development have been proposed. One
theory suggests that inverted papilloma is a result of sinonasal inflammation and form in much the same way as nasal polyps.3,4
Human papilloma virus has also been implicated in the pathogenesis
of inverted papilloma. Human papilloma virus DNA has been identified in 32% of early stage inverted papillomas5 and up to 67% of
inverted papillomas associated with squamous cell carcinoma.6
Computed tomography (CT) is a critical component of the preoperative evaluation of patients with inverted papilloma. Inverted papillomas appear on CT as a soft tissue density that heterogeneously
enhances with contrast. Evidence of osteitis adjacent to inverted papilloma has been shown to be useful in identifying the site of attachment.7 However, there is no study that can definitively identify the
From the Department of Otolaryngology–Head and Neck Surgery, The Johns Hopkins
Sinus Center, Baltimore, Maryland
The authors have no conflicts of interest to declare pertaining to this article
Address correspondence and reprint to Douglas D. Reh, M.D., Department of Otolaryngology–Head and Neck Surgery, The Johns Hopkins Sinus Center, 601 North
Caroline Street, Baltimore, MD 21287
Originally published in Am J Rhinol Allergy 27, S31–S34, 2013
Copyright © 2014, OceanSide Publications, Inc., U.S.A.
American Journal of Rhinology & Allergy
site of attachment preoperatively, and others have debated the significance of this radiological finding. Magnetic resonance imaging can
also be useful in the preoperative assessment of inverted papilloma
because this modality has a superior ability to differentiate between
papilloma, adjacent inflammation, and inspissated secretions.8 Other
associated findings typically seen on CT include sclerosis, calcification, lobulation, and bony erosion. It has been reported that using
these features in conjunction with opacification increases the accuracy
of diagnosis of sinus involvement to 83–97%.9
Multiple staging systems have been proposed for inverted papilloma. Krouse proposed a commonly used staging system based
on radiographic characteristics (Table 1).10 This system is widely
used because it correlates well with inverted papilloma prognosis
with operative difficulty; however, it is limited by the fact that it
groups inverted papillomas with extrasinonasal extension with
those that have a focus of squamous cell carcinoma. These two
categories of inverted papillomas can differ substantially in terms
of their prognosis and surgical management. A newer staging
system proposed by Cannady et al. includes three groups of inverted papillomas based on recurrence rates (Table 2).11 Although
this system provides useful prognostic information, it shares some
of the same limitations as the Krouse system.10
The primary treatment modality for inverted papilloma is surgical
resection. Historically, inverted papillomas were treated via open approaches, including lateral rhinotomy, Caldwell-Luc, and midface degloving approaches often combined with partial or total maxillectomies.
With the advent of modern endoscopic techniques the majority of inverted papillomas are now treated endoscopically. The risk of recurrence
using open approaches has been reported to be 5–30%.1 In addition to
the risk of local recurrence, there is up to a 7.1% risk of malignancy
associated with inverted papilloma.12 Because of the high risks of recurrence and malignancy, complete resection of these tumors is essential. A
meta-analysis of 10 studies with a total of 1060 inverted papilloma
patients found a statistically lower rate of recurrence in patients treated
endoscopically (12%) versus patients treated by open approaches
(20%).13 This finding must be interpreted with caution, however, because
elements of selection or sampling bias may be possible. Additionally,
multiple studies advocate the use of combined open and endoscopic
approaches for inverted papillomas with sites of attachment that are not
easily accessible solely by endoscopic endonasal approaches.14–16
OSSEOUS AND FIBRO-OSSEOUS TUMORS
Osteoma
Osteomas, the most common benign tumors of the sinonasal tract,
are reportedly present on 1% of routine sinus radiographic studies
and are most commonly found in the frontal sinuses.1 Histologically,
Copyright © Oceanside Publications, Inc. All rights reserved. DO NOT COPY
For permission to copy go to https://www.oceansidepubl.com/permission.htm
S27
Figure 1. Characteristic endoscopic endonasal appearance of middle meatal (a) sinonasal polyps and (b) inverted papillomas.
Note the contrast between the translucent
appearance of the polyp with the lobulated
mucosa and prominent vasculature of the
inverted papilloma.
Table 1 Krause staging for inverted papilloma
T1: Limited to one area of the nasal cavity
T2: Involvement of the medial wall of the maxillary or ethmoid
sinuses and/or the osteomeatal unit
T3: Involvement of the superior, inferior, posterior, anterior or
lateral walls of the maxillary sinus
T4: Tumors with extrasinonasal spread or malignancy
Source: Ref. 10.
Table 2 Cannaday staging for inverted papilloma with published
recurrence rates
Group A: Confined to the nasal cavity, medial maxillary sinus, and
ethmoid sinus (recurrence rate ⫽ 3%)
Group B: Involving the lateral maxillary sinus, sphenoid sinus, or
frontal sinus (recurrence rate ⫽ 19.8%)
Group C: Extrasinus extension (recurrence rate ⫽ 35.3%)
Source: Ref. 11.
osteomas are characterized by proliferation of dense cancellous bone.
Osteomas usually involve the craniofacial skeleton with 21% found
within the paranasal sinuses.17 Typically, osteomas are very slow
growing; however, they can have periods of rapid growth after years
of inactivity. Clinically, osteomas of the sinonasal tract are seen on
endoscopy as smooth mucosa covered masses that are firm on palpation. No intervention is indicated unless the osteoma is obstructing
the nasal passages or sinus outflow or it is compressing critical
structures such as the orbit or skull base. In patients with multiple
osteomas, Gardner’s syndrome should be considered. This autosomal
dominant syndrome includes multiple osteomas (at any location in
the body), colon polyps, and soft tissue tumors. Gardner’s syndrome
is important to recognize and diagnosis necessitates a referral to a
gastroenterologist because of the high rate of malignant transformation of the colon polyps.
Fibrous Dysplasia and Ossifying Fibroma
Fibrous dysplasia and ossifying fibroma are grouped together because they share histological similarities that qualify them as fibroosseous lesions. Notable clinical differences are discussed here.
Fibrous dysplasia is a benign but potentially disfiguring condition that occurs in 1:4000 to 1:10,000 people.18 The disease can be
monostotic, polyostotic, or can present as a component of McCuneAlbright syndrome. Craniofacial fibrous dysplasia typically involves the maxilla but can also involve the zygoma, skull base, and
mandible.19 It typically presents in childhood and growth tends to
cease in early adulthood. The characteristic radiographic finding
with fibrous dysplasia is a ground-glass appearance of the lesions
S28
on CT imaging.19 Histologically, fibrous dysplasia is characterized
by woven bone with nonlamellar trabeculae in S and C shapes,
often described as Chinese script characters. These tumors are
rarely isolated in the sinuses and surgical planning must take into
account the extent of the disease. For isolated sinonasal fibrous
dysplasia, observation is the favored surgical management.20 Surgery is indicated only when critical structures such as the orbit or
cranial nerves are affected.
Ossifying fibromas are similar to fibrous dysplasia in that both
are true fibro-osseous lesions, but with a more rapid and aggressive pattern of growth. They occur in the second to fourth decade
of life and occur more often in female subjects than male subjects.21
In the craniofacial skeleton, 70% of ossifying fibromas occur in the
mandible and the remainder occur in the maxilla and paranasal
sinuses.21 Clinically, these tumors present as ovoid or round, expansile, painless masses that are similar in radiographic appearance to fibrous dysplasia but are locally destructive.22 Unlike fibrous dysplasia, ossifying fibromas continue to grow after
adolescence and can exhibit locally aggressive and destructive
behavior23; therefore, attempts should be made to resect these
lesions early. Despite the clinical differences between these tumors,
they can be difficult to distinguish radiographically and histologically and are believed to be the same disease process on different
ends of a morphological spectrum.24 Although endoscopic approaches to removal of these tumors has been described, open
approaches are sometimes required to ensure adequate visualization of the tumors for complete removal.25
VASCULAR NEOPLASMS
Juvenile Nasopharyngeal Angiofibroma
Juvenile nasopharyngeal angiofibromas (JNAs) are rare vascular
tumors of the sinonasal cavity with an incidence of 1:150,000,
which occur primarily in male subjects between the age 14 and 25
years.26,27 The term implies a nasopharyngeal origin, which is a
misnomer, because JNAs originate in the posterior–superior margin of the sphenopalatine foramen. Histologically, the lesion expands submucosally and is nonencapsulated, with local destruction of adjacent structures. On CT scan, anterior bowing of the
posterior maxillary wall (Holman-Miller sign) is consistent with a
JNA. Histologically, JNAs have a benign-appearing fibrous stroma
with a high density of blood vessels. Although JNAs exhibit benign
histology, they are locally aggressive and frequently extend into
the orbit, paranasal sinuses, and through the skull base. They are
characterized by slow growth and often are not detected until they
have caused symptoms caused by invasion into adjacent structures
or when they present with intractable epistaxis.26 Diagnosis is
often established by history, nasal endoscopy, and radiography.
Office biopsy incurs significant bleeding risk and is often deferred
if clinical/radiographic features are characteristic for JNA. These
May–June 2014, Vol. 28, No. 3
Copyright © Oceanside Publications, Inc. All rights reserved. DO NOT COPY
For permission to copy go to https://www.oceansidepubl.com/permission.htm
lesions are usually treated by preoperative embolization followed
by endoscopic resection; however, combined open and endoscopic
approaches are often needed for large tumors.28
Pyogenic Granuloma
The etiology of pyogenic granuloma, sometimes referred to as
lobular capillary hemangioma, is unknown; however, they typically occur after minor nasal mucosal trauma and are thought to be
the result of a local inflammatory response.29 In the sinonasal tract
pyogenic granulomas are characteristically found on the anterior
nasal septum in Kiesselbach’s plexus and have often been associated with nasogastric tube placement and nose picking. Histologically, these lesions have an ulcerated surface, an underlying inflammatory infiltrate, and a core of fibrous tissue perforated by
blood vessels. Pyogenic granulomas can also occur during pregnancy and it is thought that these lesions are hormonally stimulated. This theory of hormonal stimulation is supported by the
observation that these lesions spontaneously regress after delivery.29 Smaller lesions can be observed and often spontaneously
regress. Larger lesions can be treated with chemical or electrocautery or can be surgically excised.
Sinonasal Hemangiopericytoma
Sinonasal hemangiopericytomas are rare perivascular tumors,
constituting ⬍1% of vascular neoplasms with only 194 descriptions
in the literature.30 They primarily occur on the extremities, but can
also be found in the sinonasal cavity. Sinonasal hemangiopericytomas commonly involve the ethmoid sinuses but can be found at
any site in the nasal cavity. These tumors typically occur in the fifth
or sixth decade and, as with other sinonasal tumors, the most
common presenting symptoms are nasal obstruction and epistaxis.30 Sinonasal hemangiopericytomas typically have uniform,
polygonal to spindle-shaped cells with vesicular nuclei and a
characteristic staghorn appearance to blood vessels.31 Although
these tumors are often benign, they can be locally invasive and
rarely can undergo malignant transformation. The diagnosis is
sometimes established by biopsy performed in an outpatient setting during workup of a sinonasal mass, after imaging has ruled
out an intracranial connection. If epistaxis is a predominant presenting symptom or workup suggests marked vascularity, biopsy
should be performed in an operating room. Unless the lesion is
extensive or malignant with significant orbitocranial invasion, endoscopic resection is usually possible. Additionally, patients require close postoperative follow-up because 20% of patients have
recurrence, which can occur up to 17 years after surgery.30
CONCLUSION
Benign sinonasal tumors are a clinically and pathologically heterogeneous group of neoplasms. Because the majority of patients with
these tumors present with nonspecific symptoms, it is important for
the otolaryngologist to have knowledge of these lesions to establish
proper diagnosis and treatment. The advent of modern endoscopic
surgical techniques has evolved and potentially improved the treatment of these lesions. Open surgical approaches may be required
for certain sinonasal tumors that can not be visualized endoscopically, but they are associated with a significant degree of morbidity as well as cosmetic deformity. Endoscopic techniques and
approaches can eliminate the need for external incisions and allow
for equivalent, or for some lesions better, rates of cure than traditional open approaches.
CLINICAL PEARLS
• Identification and resection of the nasal wall attachment are critical
when removing an inverted papilloma to minimize the chance of
recurrence, regardless of operative technique. Evidence of osteitis
American Journal of Rhinology & Allergy
adjacent to inverted papilloma has been shown to be useful in
identifying the site of attachment.
• In patients presenting with multiple osteomas, Otolaryngologists
should have a high clinical suspicion for Gardner’s Syndrome.
• Fibrous dysplasia and ossifying fibroma are histologically similar
lesions with markedly different clinical behavior. It is important to
differentiate fibrous dysplasia from ossifying fibroma because the
latter is a more aggressive process that warrants earlier surgical
intervention.
• JNAs originate in the posterior–superior margin of the sphenopalatine foramen. The radiographic hallmark of JNA is anterior
bowing of the posterior maxillary wall on CT scan (HolmanMiller sign).
REFERENCES
1.
Melroy CT, and Senior BA, Benign sinonasal neoplasms: A focus on
inverting papilloma. Otolaryngol Clin North Am 39:601–617, 2006.
2. Chiu AG, Jackman AH, Antunes MB,et al. Radiographic and histologic analysis of the bone underlying inverted papillomas. Laryngoscope 116:1617–1620, 2006.
3. Orlandi RR, Rubin A, Terrell JE, et al. Sinus inflammation associated
with contralateral inverted papilloma. Am J Rhinol 16:91–95, 2002.
4. Roh HJ, Procop GW, Batra PS, et al. Inflammation and the pathogenesis of inverted papilloma. Am J Rhinol 18:65–74, 2004.
5. Syrjanen KJ. HPV infections in benign and malignant sinonasal lesions. J Clin Pathol 56:174–181, 2003.
6. Katori H, Nozawa A, and Tsukuda M. Markers of malignant transformation of sinonasal inverted papilloma. Eur J Surg Oncol 31:905–
911, 2005.
7. Yousuf K, and Wright ED. Site of attachment of inverted papilloma predicted by CT findings of osteitis. Am J Rhinol 21:32–36,
2007.
8. Yousem DM, Fellows DW, Kennedy DW, et al. Inverted papilloma:
Evaluation with MR imaging. Radiology 185:501–505, 1992.
9. Sham CL, King AD, van Hasselt A, and Tong MC. The roles and
limitations of computed tomography in the preoperative assessment
of sinonasal inverted papillomas. Am J Rhinol 22:144–150, 2008.
10. Krouse JH. Development of a staging system for inverted papilloma.
Laryngoscope 110:965–968, 2000.
11. Cannady SB, Batra PS, Sautter NB, et al. New staging system for
sinonasal inverted papilloma in the endoscopic era. Laryngoscope
117:1283–1287, 2007.
12. von Buchwald C, and Bradley PJ. Risks of malignancy in inverted
papilloma of the nose and paranasal sinuses. Curr Opin Otolaryngol
Head Neck Surg 15:95–98, 2007.
13. Busquets JM, and Hwang PH. Endoscopic resection of sinonasal
inverted papilloma: A meta-analysis. Otolaryngol Head Neck Surg
134:476–482, 2006.
14. Woodworth BA, Bhargave GA, Palmer JN,et al. Clinical outcomes of
endoscopic and endoscopic-assisted resection of inverted papillomas:
A 15-year experience. Am J Rhinol 21:591–600, 2007.
15. Sautter NB, Cannady SB, Citardi MJ, et al. Comparison of open
versus endoscopic resection of inverted papilloma. Am J Rhinol
21:320–323, 2007.
16. Sauter A, Matharu R, Hörmann K, and Naim R. Current advances in
the basic research and clinical management of sinonasal inverted
papilloma (review). Oncol Rep 17:495–504, 2007.
17. Herford AS, Stoffella E, and Tandon R. Osteomas involving the facial
skeleton: A report of 2 cases and review of the literature. Oral Surg
Oral Med Oral Pathol Oral Radiol 115:e1–e6, 2013. [Epub ahead of
print June 26, 2012].
18. Ricalde P, and Horswell BB. Craniofacial fibrous dysplasia of the
fronto-orbital region: A case series and literature review. J Oral
Maxillofac Surg 59:157–167, 2001.
19. Anari S, and Carrie S. Sinonasal inverted papilloma: Narrative review. J Laryngol Otol 124:705–715, 2010.
20. Ooi EH, Glicksman JT, Vescan AD, and Witterick IJ. An alternative
management approach to paranasal sinus fibro-osseous lesions. Int
Forum Allergy Rhinol 1:55–63, 2011.
21. Su L, Weathers DR, and Waldron CA. Distinguishing features of focal
cemento-osseous dysplasia and cemento-ossifying fibromas. II. A
clinical and radiologic spectrum of 316 cases. Oral Surg Oral Med
Oral Pathol Oral Radiol Endod 84:540–549, 1997.
Copyright © Oceanside Publications, Inc. All rights reserved. DO NOT COPY
For permission to copy go to https://www.oceansidepubl.com/permission.htm
S29
22.
23.
24.
25.
26.
S30
Mintz S, and Velez I. Central ossifying fibroma: An analysis of 20
cases and review of the literature. Quintessence Int 38:221–227,
2007.
Schreiber A, Villaret AB, Maroldi R, and Nicolai P. Fibrous dysplasia
of the sinonasal tract and adjacent skull base. Curr Opin Otolaryngol
Head Neck Surg 20:45–52, 2012.
Toyosawa S, Yuki M, Kishino M, et al. Ossifying fibroma vs fibrous
dysplasia of the jaw: Molecular and immunological characterization.
Mod Pathol 20:389–396, 2007.
Wenig BM, Vinh TN, Smirniotopoulos JG, et al. Aggressive psammomatoid ossifying fibromas of the sinonasal region: A clinicopathologic study of a distinct group of fibro-osseous lesions. Cancer
76:1155–1165, 1995.
Scholtz AW, Appenroth E, Kammen-Jolly K, et al. Juvenile nasopharyngeal angiofibroma: Management and therapy. Laryngoscope 111:
681–687, 2001.
27.
Bremer JW, Neel HB 3rd, DeSanto LW, and Jones GC. Angiofibroma:
Treatment trends in 150 patients during 40 years. Laryngoscope
96:1321–1329, 1986.
28. Pryor SG, Moore EJ, and Kasperbauer JL. Endoscopic versus traditional approaches for excision of juvenile nasopharyngeal angiofibroma. Laryngoscope 115:1201–1207, 2005.
29. Neves-Pinto RM, Carvalho A, Araujo E, et al. Nasal septum giant
pyogenic granuloma after a long lasting nasal intubation: Case report. Rhinology 43:66–69, 2005.
30. Duval M, Hwang E, and Kilty SJ. Systematic review of treatment
and prognosis of sinonasal hemangiopericytoma. Head Neck DOI:
10.1002/hed.23074. [Epub ahead of print June 25, 2012].
31. Middleton LP, Duray PH, and Merino MJ. The histological spectrum
of hemangiopericytoma: Application of immunohistochemical analysis including proliferative markers to facilitate diagnosis and predict
prognosis. Hum Pathol 29:636–640, 1998.
e
May–June 2014, Vol. 28, No. 3
Copyright © Oceanside Publications, Inc. All rights reserved. DO NOT COPY
For permission to copy go to https://www.oceansidepubl.com/permission.htm
Sinonasal malignancies
Richard J. Harvey, M.D., and Dustin M. Dalgorf, M.D.
ABSTRACT
Malignant tumors of the sinonasal tract are uncommon tumors of the head and neck. Patients often present in the later years of life with unilateral symptoms
and potential involvement of nearby structures such as the orbit, brain, or cranial nerves. Presenting symptoms are similar to patients suffering from
inflammatory sinonasal disease and thus early diagnosis relies heavily on a high clinical suspicion. There are established risk factors based on exposure to the
by-products of woodworking, metal, textile, and leather industries. Sinonasal malignancies are generally divided into those of epithelial origin (squamous cell
carcinoma, adenocarcinoma, and adenoid cystic carcinoma) and nonepithelial origin (olfactory neuroblastoma, chondrosarcoma, and mucosal melanoma).
Accurate histopathology confirmation and staging of the tumor is critical prior to making treatment decisions. Both computed tomography and magnetic
resonance imaging are required to accurately determine the extent of local disease. Treatment is based on multimodality therapy, primarily surgical excision,
and postoperative radiotherapy. This article reviews the classification of malignant tumors of the paranasal sinuses, their clinical presentation, relevant
diagnostic investigations, and the principals of therapy and management.
M
alignant neoplasms arise from a variety of tissues of origin within
the sinonasal tract but are, in general, defined as epithelial or
nonepithelial in origin. Malignancy of the sinonasal tract is uncommon
and accounts for only 1%1,2 of all malignancies and ⬃5% of head and
neck malignancy.3,4The incidence varies across geographical areas because of differences in genetic predisposition and exposure to carcinogenic factors. In Europeans, this rate is 1 per 100,000 and in Asia 3 per
100,000.5,6 It is a condition affecting older patients with 75% ⬎50 years of
age at diagnosis7 and predominately male gender.8,9
As with other malignancies, the presentation may be related to
local, regional, or distant disease. Defining the histology of the tumor
and its stage are the key goals of investigations. Finally, treatment is
generally multimodal with a combination of surgery and radiotherapy as the mainstay for most lesions. The proximity of critical structures, specifically the orbit, brain, and cranial nerves, dictate the
morbidity from curative interventions.
PATHOPHYSIOLOGY
The World Health Organization classification10 is listed in Table 1
with the most common subtypes included. The epithelial versus
nonepithelial distinction is easy and reflects the frequency of tumors.
Epithelial tumors are the most common with squamous cell carcinoma (SCC), adenocarcinoma, and adenoid cystic carcinoma most
commonly reported.10 The nonepithelial tumors are lymphoma (hematologic), olfactory neuroblastoma (neuroectodermal), chondrosarcoma (bone/cartilage), and mucosal melanoma (neuroectodermal).
Undifferentiated nasopharyngeal carcinoma is often included in
discussions on upper airway malignancy. However, it is not considered a sinonasal tumor and its origins are from epithelial and b-cell
interactions of the nasopharynx. It is a common malignancy of young
Southeast Asian men compared with the uncommon nature of most
From the Applied Medical Research Center, St. Vincent’s Hospital and University of
New South Wales, and Macquarie University, Darlinghurst, Sydney, New South
Wales, Australia
The authors have no conflicts of interest to declare pertaining to this article
Address correspondence and reprint requests to Richard J. Harvey, M.D., Applied
Medical Research Center, St. Vincent’s Hospital and University of New South Wales,
354 Victoria Street, Darlinghurst, Sydney, New South Wales, Australia, 2010
E-mail address: [email protected]; alternative: [email protected]
Originally published in Am J Rhinol Allergy 27, S35–S38, 2013
Copyright © 2014, OceanSide Publications, Inc., U.S.A.
American Journal of Rhinology & Allergy
tumors discussed in this article. Its etiology is unique in that it is
thought to arise from early Epstein-Barr virus infection in a genetically susceptible host.
There are several associated risk factors for the development of
sinonasal malignancy. The most notable is wood dust exposure and
adenocarcinoma. The large-particle dust from certain hardwoods (ebony, oak, and beech) are thought to provide a 900-fold risk of developing adenocarcinoma.9,11. Less than 5 years exposure is still considered critical and the latency to tumor development is delayed (⬃40
years).9,12 Smoking has been linked to SCC.7 Metal industry products
(chromate and nickel), leather and boot products and the textile
industry (chrome pigments) and thorium dioxide, and imaging
agents, are all risk factors for SCC.13 Evidence for the role of human
papilloma virus as a primary carcinogen in the sinonasal tract is
strong but inconclusive and additional studies are required.13
DIAGNOSIS
The most important element in accurate diagnosis of sinonasal
malignancy is clinical suspicion. The insidious onset of unilateral
symptoms, the lack of previous inflammatory sinus disease or rhinitis, and the relative age of the patient (⬎50 years old for tumors
compared with ⬍50 years old for inflammatory disease) should be
key features that prompt exclusion of neoplasia as a cause for a
patient’s symptoms.
Presentation
Although the presentation can be with regional symptoms (neck
lumps, orbital changes, diplopia, epiphora, or cranial nerve dysfunction) and/or distant metastasis, this is relatively uncommon for most
tumors and local (nasal obstruction, bleeding, discharge, and hyposmia) are the more common presenting symptoms. These symptoms
share common presenting complaints of patients with inflammatory
sinonasal disease, which again highlights the importance of initial
clinical suspicion. Unilateral eustachian tube dysfunction can also
occur. Gross macroscopic changes to hard palate mucosa or the skin
are uncommon in developed countries. Endoscopic examination reveals a mass within the nasal cavity (Figs. 1 and 2).
Imaging
Imaging should always focus on what is trying to be achieved, viz.,
tumor staging. Accurate information on local tissue involvement is
Copyright © Oceanside Publications, Inc. All rights reserved. DO NOT COPY
For permission to copy go to https://www.oceansidepubl.com/permission.htm
S31
Table 1 The World Health Organization classification system for
sinonasal malignancies
1. Malignant epithelial
a. SCC
b. Adenoid cystic carcinoma
c. Adenocarcinoma
2. Neuroendocrine
a. Carcinoid
3. Malignant soft tissue
a. Sarcomas
4. Borderline and low malignant potential tumors of soft tissue
a. Hemangiopericytoma
5. Malignant tumors of bone and cartilage
a. Chondrosarcoma
6. Hematolymphoid
a. Lymphoma
7. Neuroectodermal
a. Olfactory neuroblastoma
b. Mucosal melanoma
8. Germ cell
9. Secondary
a. Renal
b. Lung
c. Breast
The main subtypes are described here with common pathologies.
SCC ⫽ squamous cell carcinoma.
critical for “T” staging. Most patients will undergo both computed
tomography (CT) and magnetic resonance imaging (MRI; Figs. 1 and
2). There are several key principals in rationale for both modalities.
First, distinguish tumor from retained mucus. The T2 MRI will highlight edematous mucus and retained secretions compared with the
tumor, seen on T1 postcontrast imaging (Fig. 2). Second, the involvement of the periobita is determined by bone loss on CT and fat
enhancement on MRI. Third, dural (and brain parenchyma) involvement is defined by bone loss on CT and dural enhancement on MRI
(Figs. 1, 2). Perineural involvement of cranial nerves is usually defined using fine slice fat-saturated T1 postcontrast MRI. Finally, the
relationship of the tumor to the intracranial course of the internal
carotid artery and its branches are defined by either CT angiography
or MR angiography.
Regional and distant disease is commonly defined with a positron
emission tomography/CT assessment. This technique is a combination of full-body CT and assessment of focal radioactive glucose
uptake (18FDG) by cells. This is the most efficient form of staging and
can provide standard uptake value information for subsequent follow-up.14 Imaging of the neck, chest, and abdomen as well as simple
blood tests for calcium and alkaline phosphatase would also suffice.
Specific investigations for bone, brain, or other metastasis is usually
driven by clinical suspicion rather than routine.
Pathology
Obtaining histopathological confirmation of malignancy and its
subtype is critical before therapy. This is usually done before treatment decision making and the use of “frozen” or intraoperative
specimens is ill-advised and not recommended. This is important
because some tumors such as lymphomas are unique in that they are
radiosensitive and do not require surgery whereas a diagnosis of
melanoma would prompt an aggressive search for metastasis before
local treatment.
Traditional teaching that “all” tumors should be biopsied in the
operative setting is neither necessary nor practical. Such statements
are designed to avoid disastrous consequences of biopsying an illdefined nasal mass. As a general rule, nasal masses that present
S32
without any imaging and when the diagnosis is uncertain should not
be biopsied in the office (avoiding biopsy of encephalocele or aneurysm). Likewise, masses deep within the nose (beyond the middle
turbinate) should not be biopsied unless there is a special setup for
controlling posterior nasal bleeding. There should also be appropriate
sampling for fresh tissue (for flow cytometry), when suspicion of
lymphoma exists, and needs to be available as well as formalin-fixed
tissue.
TREATMENT
Most patients with disseminated disease rarely undergo surgical
therapy. The focus of treatment is symptom control (palliation) and
short courses of radiotherapy are often given. With the exception of
lymphomas, chemotherapy, radiotherapy, or combinations are not
curative therapies but are performed as adjuncts to surgical resection
with curative intent. Radiotherapy is used to control local and regional disease. There are complex lymphatic channels in the paranasal sinuses and the skull base that prevent full excision of the
lymphatic compartments during surgery. Additionally, close margins occur next to orbits, carotids, and cranial nerves that benefit
from additional local therapy. Radiation therapy can be performed
before surgery, termed “neoadjuvant,” or after surgery, termed
“adjuvant.” When chemotherapy is added, the goal is to enhance
radiotherapy, reduce tumor growth, and manage potential micrometastasis.
Some centers use chemoradiotherapy, for sensitive tumors such
as olfactory neuroblastoma, before surgery because this is generally quick to initiate and can reduce tumor size making surgery
less technically demanding (less bulk and bleeding). With this
approach, it does not mean that a lesser region of the skull base can
be removed and the resection must still follow the originally
involved anatomic sites. The down side is that the surgeon is
working with postirradiated tissues with impaired healing. When
performed after surgery, adjuvant therapy starts 4–6 weeks postsurgery, when there is a balance between early residual tumor cell
killing and the skull base that has healed and the chance of wound
breakdown is minimal.
The orbit is a critical factor is decision making and morbidity.
The decision to remove the orbit is typically considered an “allor-none” approach. It is not practical to remove portions of the
orbital contents (periorbita, fat, and/or muscle) and subsequently
irradiate the remaining orbital contents. This results in a nonfunctional eye with restricted ocular movements, diplopia, visual loss,
poor cosmesis, and, potentially, pain. Thus, if the periorbita is not
breeched on MRI or intraoperatively, the eye is spared and the
close margin is acknowledged. Close observation is required and
early reoperation and orbital exoneration is required if recurrence
occurs.
Dura, frontal lobe, and cranial nerves are routinely removed if
they are involved by tumor. These structures often bring about the
morbidity imparted on the patient. However, if the eye is removed,
the ipsilateral cranial nerves 1–6 can be removed with minimal
impact on function. Mastication remains functional with an ipsilateral loss of the trigeminal nerve. This differs greatly to the
dysfunction caused by lower cranial nerve loss (7–12). The neck
generally is not empirically treated with either surgery or radiotherapy in the N0 (no clinically detectable disease in the cervical
lymph nodes) presentation because the risk of subclinical disease is
⬍20%. However, a case could be made for N0 therapy in olfactory
neuroblastoma.15 When nodal disease occurs at presentation, the
neck is typically managed with a local excision with modified
radical neck dissection, although there is little evidence to suggest
an advantage of selective neck or radical neck dissection as an
alternative.
The main complications of treatment include the early risks of
cerebrospinal fluid leak, epistaxis, pneumocephalus, and meningitis.
May–June 2014, Vol. 28, No. 3
Copyright © Oceanside Publications, Inc. All rights reserved. DO NOT COPY
For permission to copy go to https://www.oceansidepubl.com/permission.htm
Figure 1. Olfactory neuroblastoma is a neuroectodermal tumor. It is radiosensitive and
has a better outcome than epithelial tumors.
(A) The characteristic strawberry red appearance of a mass arising from medial to the
middle turbinate. (B) The postoperative magnetic resonance imaging (MRI) view. (C)
The typical bone loss of the skull base must
always be further assessed with an MRI (D)
because it is a marker of potential intracranial involvement.
Figure 2. Squamous cell carcinoma (SCC)
arising in a 62-year-old smoker. (A) The
endoscopic view of a friable hemorrhagic
mass filling the right nasal cavity. (B)
Computed tomography (CT) showing loss
of skull base bone and diffuse opacification
of the surrounding sinuses. The T1 postgadolinium magnetic resonance imaging
(MRI) (C) showing the sphenoid centered
mass and T2 series (D) showing that much
of the surround sinus changes is obstructive disease secondary to the tumor.
Delayed complications included sinonasal dysfunction, hypopituitarism (post-radiation therapy), post-radiation therapy cranial neuropathy, and osteoradionecrosis of the skull base.
PROGNOSIS
The prognosis for sinonasal malignancy is generally poor. Locoregional recurrence is a critical factor because many tumors
present late and wide en bloc removal is difficult. For some tumors
American Journal of Rhinology & Allergy
such as olfactory neuroblastoma, adenoid cystic carcinoma, and
mucosal melanoma, nodal and distant disease can occur late. Up to
15% of olfactory neuroblastomas will recur with neck nodes.15 The
overall survival for SCC and adenocarcinoma is 60% at 5 years.13
Some radiosensitive tumors, such as olfactory neuroblastoma, have
survival rates of ⬎80% for early stage tumors.16 Mucosal melanoma survival is worse than that for cutaneous origins with 20%
5-year rates.17
Copyright © Oceanside Publications, Inc. All rights reserved. DO NOT COPY
For permission to copy go to https://www.oceansidepubl.com/permission.htm
S33
CLINICAL PEARLS
6.
• Sinonasal malignancies are uncommon malignancies and nasopharyngeal carcinoma is not considered part of this group.
• Unilateral nasal symptoms in an older patient without a history of
inflammatory airway conditions should prompt clinical suspicion.
• Both CT and MRI are required to accurately stage local tissue
involvement.
• The involvement of the orbit, dura, brain, palate, and cranial nerves
will determine the degree of surgical resection.
• As with all tumors, accurate tissue diagnosis and local, regional,
and distant staging must occur before treatment decisions.
• Surgery and adjuvant radiotherapy are the mainstay of therapy.
Close surveillance every 3 months for 2 years and then every 6
months until 5 years is standard care.
7.
2.
3.
4.
5.
S34
9.
10.
11.
12.
13.
REFERENCES
1.
8.
Tufano RP, Mokadam NA, Montone KT, et al. Malignant tumors of
the nose and paranasal sinuses: Hospital of the University of Pennsylvania experience 1990–1997. Am J Rhinol 13:117–123, 1999.
Rinaldo A, Ferlito A, Shaha AR, and Wei WI. Is elective neck treatment indicated in patients with squamous cell carcinoma of the
maxillary sinus? Acta Otolaryngol 122:443–447, 2002.
Le QT, Fu KK, Kaplan M, et al. Treatment of maxillary sinus carcinoma: A comparison of the 1997 and 1977 American Joint Committee
on cancer staging systems. Cancer 86:1700–1711, 1999.
Tiwari R, Hardillo JA, Mehta D, et al. Squamous cell carcinoma of
maxillary sinus. Head Neck 22:164–169, 2000.
Magnani C, Ciambellotti E, Salvi U, et al. The incidence of tumors of
the nasal cavity and the paranasal sinuses in the district of Biella,
1970–1986. Acta Otorhinolaryngol Ital 9:511–519, 1989.
14.
15.
16.
17.
Muir CS, and Nectoux J. Descriptive epidemiology of malignant
neoplasms of nose, nasal cavities, middle ear and accessory sinuses.
Clin Otolaryngol Allied Sci 5:195–211, 1980.
Olsen KD. Nose and sinus tumors. In Rhinologic Diagnosis and
Treatment. McCaffrey T (Ed). New York. NY: Thieme, 334–359, 1997.
Wolf J, Schmezer P, Fengel D, et al. The role of combination effects on
the etiology of malignant nasal tumours in the wood-working industry. Acta Otolaryngol Suppl 535:1–16, 1998.
Nylander LA, and Dement JM. Carcinogenic effects of wood dust:
Review and discussion. Am J Ind Med 24:619–647, 1993.
Barnes L, Eveson JW, Reichart P, et al. Tumors of the nasal cavity and
paranasal sinuses. In Pathology and Genetics of Head and Neck
Tumors. Barnes L, Tse LLY, Hunt JL, Brandwein-Gensler M, Curtin
HD, Boffetta P (Eds). Lyon, France: IARC Press, 9–80, 2005.
Acheson ED, Cowdell RH, Hadfield E, and Macbeth RG. Nasal
cancer in woodworkers in the furniture industry. Br Med J 2:587–596,
1968.
Macbeth R. Malignant disease of the paranasal sinuses. J Laryngol
Otol 79:592–612, 1965.
Lund VJ, Stammberger H, Nicolai P, et al. European position paper
on endoscopic management of tumours of the nose, paranasal sinuses and skull base. Rhinol Suppl 22:1–143, 2010.
Zanation AM, Sutton DK, Couch ME, et al. Use, accuracy, and
implications for patient management of [18F]-2-fluorodeoxyglucosepositron emission/computerized tomography for head and neck tumors. Laryngoscope 115:1186–1190, 2005.
Zanation AM, Ferlito A, Rinaldo A, et al. When, how and why to treat
the neck in patients with esthesioneuroblastoma: A review. Eur Arch
Otorhinolaryngol 267:1667–1671, 2010.
Devaiah AK, and Andreoli MT. Treatment of esthesioneuroblastoma:
A 16-year meta-analysis of 361 patients. Laryngoscope 119:1412–
1416, 2009.
Dauer EH, Lewis JE, Rohlinger AL, et al. Sinonasal melanoma: A
clinicopathologic review of 61 cases. Otolaryngol Head Neck Surg
138:347–352.
e
May–June 2014, Vol. 28, No. 3
Copyright © Oceanside Publications, Inc. All rights reserved. DO NOT COPY
For permission to copy go to https://www.oceansidepubl.com/permission.htm
Granulomatous diseases and chronic sinusitis
Michael A. Kohanski, M.D., Ph.D., and Douglas D. Reh, M.D.
ABSTRACT
Nasal crusting, rhinitis, and sinusitis are presentations of common conditions; however, these can also be the presenting symptoms of an underlying systemic
disorder such as an infection, malignancy, or granulomatous disease. Granulomatous diseases with head and neck manifestations include Wegener’s
granulomatosis, Churg-Strauss syndrome, and sarcoidosis. These diseases are managed through a multidisciplinary approach that often includes otolaryngologists. This article presents a brief review of granulomatous diseases and their rhinologic manifestations and includes relevant diagnostic tests and systemic
and local treatment options.
C
hronic nasal obstruction, rhinorrhea, and nasal crusting are common symptoms seen by otolaryngologists in patients with
chronic rhinosinusitis (CRS). Although the management of such patients often begins with evaluation for and treatment of common
infectious or allergic etiologies, these symptoms can also be a manifestation of systemic granulomatous diseases such as Wegener’s granulomatosis (WG), Churg-Strauss syndrome (CSS), or sarcoidosis. The
rhinologic manifestations of these systemic diseases can often be the
presenting symptoms and these patients may also show concurrent
systemic symptoms relating to pulmonary, renal, or neurological
involvement. Granulomatous disorders should be considered in individuals with severe CRS that is minimally responsive to surgery
and/or medical therapy and these patients may require a more extensive workup.
WEGENER’S GRANULOMATOSIS
WG is a systemic small-to-medium vessel vasculitis characterized by
necrotizing granulomas. The head and neck, lungs, and kidneys are
the most common regions of the body affected by WG.1,2 Head and
neck signs and symptoms of WG are seen in early manifestations of
the disease. Later stages of WG are associated with progression of the
vasculitis and involvement of additional organ systems.
Pulmonary involvement can be in the form of asymptomatic pulmonary nodules or pulmonary infiltrates seen on chest x ray. Segmental necrotizing glomerulonephritis is often seen in the kidneys of
patients with WG.3 Head and neck manifestations of WG are seen in
⬃70–95% of patients.2,3 Otologic involvement as a result of WG is
relatively rare. Subglottic stenosis is also seen in people with WG and
can result in life-threatening dyspnea and stridor requiring surgical
intervention.
The granulomatous changes associated with WG can lead to a
broad range of rhinologic signs and symptoms. Mucosal inflammation (Fig. 1 a) can lead to nasal obstruction, granulation tissue, crusting, hyposmia, anosmia, rhinitis, and/or sinusitis. Chronic Staphylococcus aureus infections are often encountered in patients with WG as
a result of the mucus stasis that can occur from these inflammatory
From the Department of Otolaryngology–Head and Neck Surgery, The Johns Hopkins
Sinus Center, Baltimore, Maryland
The authors have no conflicts of interest to declare pertaining to this article
Address correspondence and reprint requests to Douglas D. Reh, M.D., Department of
Otolaryngology–Head and Neck Surgery, The Johns Hopkins Sinus Center, 601 North
Caroline Street, Baltimore, MD 21287
Originally published in Am J Rhinol Allergy 27, S39 –S41, 2013
Copyright © 2014, OceanSide Publications, Inc., U.S.A.
American Journal of Rhinology & Allergy
changes. Major nasal obstruction, epistaxis, and structural changes to
the nose can occur as a result of the vasculitic processes associated
with WG. A septal perforation or a saddle nose deformity in the
absence of a history of trauma or infection is suggestive of WG.
Granuloma formation in the nose can also cause outflow obstruction
of the nasolacrimal duct resulting in epiphora.
The diagnosis of WG is clinical and is based on findings suggestive
of a vasculitic/granulomatous disease state (Table 1). In a person with
a vasculitic disease, the American College of Rheumatology defines a
person as having WG when they meet two of the following four
criteria: (1) oral ulcers or nasal discharge, (2) abnormal chest x ray or
CT, (3) abnormal urine sediment (red cell casts, ⬎5 RBCs), and (4)
granulomatous inflammation on biopsy.4 A chest x ray may reveal
pulmonary nodules or infiltrates; urinalysis can be used to detect
glomerulonephritis. WG is almost always associated with a positive
c-ANCA state. In an acute flare-up, erythrocyte sedimentation rate
and C-reactive protein may be elevated and blood work may reveal
leukocytosis and thrombocytosis.
Histologically confirmed evidence of WG is one of the four American College of Rheumatology criteria for diagnosis of WG. A biopsy
specimen must show evidence of necrosis, granulomatous inflammation, and vasculitis to be diagnostic for WG. Specimens should be
obtained from suspicious lesions or sites of involvement, usually from
areas with sinonasal granulation tissue. Because the nose and paranasal sinuses are frequently involved in WG, the otolaryngologist
may be called on to obtain samples from intranasal or sinus contents.
It is important to keep in mind that ⬎50% of nasal biopsies are
nondiagnostic and the majority of specimens only show acute or
chronic inflammation, findings that are nonspecific for WG.2,3
Treatment of WG is geared toward suppression of the severe inflammatory cycle with a regimen of corticosteroids (e.g., prednisone)
and immunosuppressive agents (e.g., cyclophosphamide and methotrexate) to induce disease remission (Fig. 1 b). TNF-␣ inhibitors, which
have shown success in the treatment of rheumatoid arthritis, remain
controversial in the treatment of WG and do not enhance traditional
therapy with prednisone and cyclophosphamide.5
Nasal symptoms are managed conservatively with nasal irrigation,
topical steroids, and topical antibiotics when concurrent infection is
present. There is a limited role for surgery in the management of WG
and it is best performed in patients with quiescent disease. Surgery
can be helpful in relieving nasal obstruction by removing scar tissue;
however, patients continue to require systemic and/or local therapy
to maintain disease remission. Septal perforations may be addressed
through surgical repair or with the use of silastic septal buttons to
occlude the septal opening, and saddle nose deformities can be repaired with septorhinoplasty. Surgery to repair perforations and/or
Copyright © Oceanside Publications, Inc. All rights reserved. DO NOT COPY
For permission to copy go to https://www.oceansidepubl.com/permission.htm
S35
Figure 1. Nasal manifestations of granulomatous diseases. (a) Inflammatory
changes in nasal mucosa of a patient
with Wegener’s granulomatosis (WG).
(b) Induction of quiescent disease after
intranasal steroid treatment of patient
seen in panel a. (c) Cobblestoning in a
patient with sarcoidosis-associated rhinitis. (d) Pearly submucosal nodule seen in
the nasal passage of a patient with
sarcoidosis.
Table 1 Tests and findings associated with granulomatous disorders
p-ANCA
c-ANCA
CBC with differential
Urinalysis
Chest x ray
ACE level
Serum calcium
WG
CSS
Sarcoidosis
⫺
⫹
Thrombocytosis and leukocytosis
Red casts, RBCs, and proteinuria
Pulmonary nodules and pulmonary infiltrates
Normal
Normal
⫾
⫺
⬎10% Eosinophilia
Abnormal w/renal involvement
Pulmonary infiltrates
Normal
Normal
⫺
⫺
Normal
Elevated urine calcium
Hilar lymphadenopathy
Elevated (⬃50%)
Normal to elevated
ACE ⫽ angiotensin-converting enzyme; CSS ⫽ Churg-Strauss syndrome; CBC ⫽ complete blood cell count; RBC ⫽ red blood cell; WG ⫽ Wegener’s
granulomatosis.
saddle nose should be deferred until the disease process has stabilized for 6 months to 1 year.
CHURG-STRAUSS SYNDROME
CSS is a small-to-medium–sized vasculitis characterized by
marked eosinophilia that is associated with necrotizing granulomas with an eosinophilic core. The prodromal stage of CSS presents as asthma with nasal symptoms, including allergic rhinitis,
CRS, nasal crusting, and nasal polyps.1,6 As CSS progresses to the
second stage, hypereosinophilia and eosinophilic tissue infiltrates
develop. This is followed by the development of a systemic vasculitis and disseminated multiorgan disease. Those with prodromal
stage CSS may present to an otolaryngologist early on in the
disease course because these individuals are likely to have nasal
symptoms. Unfortunately, asthma and the associated nasal symptoms
of CSS are both commonplace and by themselves do not necessarily
indicate that a patient has CSS. It is notable that myocarditis often
develops in the second and disseminated stages and that heart failure is
a frequent cause of death in these patients.
S36
The American College of Rheumatology devised a set of criteria for
the diagnosis of CSS. Four of the following six criteria must be present
to confirm a diagnosis of CSS: (1) asthma, (2) eosinophilia of ⬎10%,
(3) paranasal sinus abnormality, (4) pulmonary infiltrates, (5) mononeuritis or polyneuropathy, and (6) tissue biopsy showing vasculitis
with extravascular eosinophils.7 When CSS is suspected, a complete
blood count with differential should be obtained to look for hypereosinophilia and a chest x ray or CT should be ordered to look for
pulmonary infiltrates (Table 1). Erythrocyte sedimentation rate
and C-reactive protein may be elevated in the acute phase of the
vasculitis, and a p-ANCA test may also be positive. If there is a
clinically suspicious lesion, a biopsy specimen can be obtained to
look for signs of vasculitis and extravascular eosinophils. Biopsies
are typically performed from skin or lung lesions and nasal polyp
tissue may also be helpful in establishing the diagnosis.
As with WG, systemic treatment of CSS involves use of corticosteroids and immunosuppressive drugs. The sinonasal manifestations of
CSS can be managed symptomatically with sinus irrigations and
topical steroids. In those with nasal polyps as a result of CSS, medical
May–June 2014, Vol. 28, No. 3
Copyright © Oceanside Publications, Inc. All rights reserved. DO NOT COPY
For permission to copy go to https://www.oceansidepubl.com/permission.htm
management and endoscopic sinus surgery can provide symptomatic
relief but is not curative.6
SARCOIDOSIS
Sarcoidosis is a noncaseating granulomatous disease that primarily
affects the lungs and lymphatic system. This disease is thought to be
caused by an inflammatory process involving T-helper cells, inflammatory cytokines, and TNF-␣.8 Sarcoidosis often presents with a
persistent dry cough. Other signs and symptoms include peripheral
lymphadenopathy, fatigue, weight loss, night sweats, eye involvement, and skin involvement.
Head and neck manifestations occur in 10–15% of patients.9 Patients can present with salivary gland swelling or xerostomia. Patients
with enlarged parotid glands, uveitis, facial nerve palsy, and fever
have Heerfordt’s syndrome, a rare manifestation of sarcoidosis. Sinonasal involvement is rare, occurring in ⬃1–4% of patients with sarcoidosis although there are reports that sinonasal involvement is
more common.10 Sarcoidosis can present with chronic nasal crusting,
rhinitis, nasal obstruction, anosmia, and epistaxis (Fig. 1 c). There can
be hypertrophy of the nasal mucosa, nasal polyps, and submucosal
granulation tissue that appears as a pearly nodule (Fig. 1 d).1,9,10
When evaluating a patient for sarcoidosis, it is important to rule out
tuberculosis and other infectious causes. Tissue biopsy specimens of
sarcoid lesions will show noncaseating granulomas and tissue cultures should also be obtained to rule out infectious etiologies. Angiotensin-converting enzyme levels can be elevated, serum calcium
and urine calcium levels may also be mildly elevated, and hilar
lymphadenopathy is present on chest radiography (Table 1).
Nasal sarcoidosis is typically treated with topical steroids such as
nasal steroid sprays, steroid irrigations, or, occasionally, intralesional
injections. Systemic steroids, immunosuppressive agents, and TNF-␣
inhibitors have a role in treating recalcitrant and severe sarcoidosis,
although sarcoidosis can resolve spontaneously without treatment.9
Endoscopic sinus surgery can be helpful in the management of
chronic sinusitis in patients with sarcoidosis, although topical steroids
are still necessary after surgical treatment to treat the associated
chronic inflammation.11
DISCUSSION
When considering granulomatous diseases in patients with CRS, it is
important to rule out infection and neoplasms.1 Fungal and bacterial
cultures as well as acid fast tissue staining should be obtained to rule
out infectious causes such as actinomyces, tuberculosis, fungal infections, or rhinoscleroma (caused by Klebsiella rhinoscleromatis), which
are treated with the appropriate antimicrobials or antifungals along
with judicious surgical debridement when appropriate. Histopathological analysis of biopsy specimens can help identify or rule out
neoplastic disease that may present as CRS. The treatment of sinus
manifestations of granulomatous diseases is usually conservative
with topical and systemic anti-inflammatory agents such as steroids
being the mainstay therapies. Surgery is reserved for specific considerations such as saddle nose deformity with WG or recalcitrant CRS
in patients with quiescent disease. The otolaryngologist should keep
American Journal of Rhinology & Allergy
in mind when evaluating patients with granulomatous diseases that
one half of biopsies will show nonspecific inflammatory changes and
there should be a strong clinical suspicion of a granulomatous disease
before obtaining a biopsy specimen.12 WG, CSS, and sarcoidosis are
systemic granulomatous diseases with nonspecific but often severe
sinonasal manifestations. By paying careful attention to clinical signs
and symptoms and providing thorough and accurate evaluations,
otolaryngologists can play an important role in the multidisciplinary
management and treatment of these diseases.
CLINICAL PEARLS
• CRS can be an early presentation for granulomatous disorders, and
these diseases should be considered in patients with severe CRS.
• WG is associated with severe sinonasal inflammation, granulation
tissue, and a positive c-ANCA. CSS is associated with nasal polyps,
asthma, and eosinophilia of ⬎10%. Sarcoidosis is often associated
with an elevated angiotensin-converting enzyme level.
• Nasal symptoms of WG, CSS, and sarcoidosis are usually managed
conservatively with nasal irrigation, topical steroids, and topical
antibiotics.
• It is important to rule out infection and neoplasms when considering granulomatous diseases in patients with CRS.
REFERENCES
1.
2.
3.
4.
5.
6.
7.
8.
9.
10.
11.
12.
Tami TA. Granulomatous diseases and chronic rhinosinusitis. Otolaryngol Clin North Am 38:1267–1278, 2005.
Gottschlich S, Ambrosch P, Kramkowski D, et al. Head and neck
manifestations of Wegener’s granulomatosis. Rhinology 44:227–233,
2006.
Erickson VR, and Hwang PH. Wegener’s granulomatosis: Current
trends in diagnosis and management. Curr Opin Otolaryngol Head
Neck Surg 15:170–176, 2007.
Leavitt RY, Fauci AS, Bloch DA, et al. The American College of
Rheumatology 1990 criteria for the classification of Wegener’s granulomatosis. Arthritis Rheum 33:1101–1107, 1990.
Wegener’s Granulomatosis Etanercept Trial (WGET) Research
Group. Etanercept plus standard therapy for Wegener’s granulomatosis. N Engl J Med 352:351–361, 2005.
Bacciu A, Bacciu S, Mercante G, et al. Ear, nose and throat manifestations of Churg-Strauss syndrome. Acta Otolaryngol 126:503–509,
2006.
Masi AT, Hunder GG, Lie JT, et al. The American College of Rheumatology 1990 criteria for the classification of Churg-Strauss syndrome (allergic granulomatosis and angiitis). Arthritis Rheum 33:
1094–1100, 1990.
Baughman RP, Lower EE, and du Bois RM. Sarcoidosis. Lancet
361:1111–1118, 2003.
Mrowka-Kata K, Kata D, Lange D, et al. Sarcoidosis and its otolaryngological implications. Eur Arch Otorhinolaryngol 267:1507–1514.
Zeitlin JF, Tami TA, Baughman R, and Winget D. Nasal and sinus
manifestations of sarcoidosis. Am J Rhinol 14:157–161, 2000.
Gulati S, Krossnes B, Olofsson J, and Danielsen A. Sinonasal involvement in sarcoidosis: A report of seven cases and review of literature.
Eur Arch Otorhinolaryngol 269:891–896.
van den Boer C, Brutel G, and de Vries N. Is routine histopathological
examination of FESS material useful? Eur Arch Otorhinolaryngol
267:381–384.
e
Copyright © Oceanside Publications, Inc. All rights reserved. DO NOT COPY
For permission to copy go to https://www.oceansidepubl.com/permission.htm
S37
Cystic fibrosis chronic rhinosinusitis:
A comprehensive review
Mohamad R. Chaaban, M.D.,1 Alexandra Kejner, M.D.,1 Steven M. Rowe, M.D.,2,3 and
Bradford A. Woodworth, M.D.1,3
ABSTRACT
Background: Advances in the care of patients with cystic fibrosis (CF) have improved pulmonary outcomes and survival. In addition, rapid developments
regarding the underlying genetic and molecular basis of the disease have led to numerous novel targets for treatment. However, clinical and basic scientific
research focusing on therapeutic strategies for CF-associated chronic rhinosinusitis (CRS) lags behind the evidence-based approaches currently used for
pulmonary disease.
Methods: This review evaluates the available literature and provides an update concerning the pathophysiology, current treatment approaches, and future
pharmaceutical tactics in the management of CRS in patients with CF.
Results: Optimal medical and surgical strategies for CF CRS are lacking because of a dearth of well-performed clinical trials. Medical and surgical
interventions are supported primarily by level 2 or 3 evidence and are aimed at improving clearance of mucus, infection, and inflammation. A number of novel
therapeutics that target the basic defect in the cystic fibrosis transmembrane conductance regulator channel are currently under investigation. Ivacaftor, a
corrector of the G551D mutation, was recently approved by the Food and Drug Administration. However, sinonasal outcomes using this and other novel drugs
are pending.
Conclusion: CRS is a lifelong disease in CF patients that can lead to substantial morbidity and decreased quality of life. A multidisciplinary approach will
be necessary to develop consistent and evidence-based treatment paradigms.
C
ystic fibrosis (CF) is an autosomal recessive disorder that affects
the upper and lower airways as well as the digestive system. It
is considered the most lethal autosomal recessive disorder among
Caucasians and is estimated to affect 1 in 2000 to 1 in 6000 births.1
Thirty years ago this disease regularly led to death in the first decade
of life, usually secondary to pulmonary deterioration from opportunistic bacteria. Advancements in therapy have led to substantial
improvements in survival with a current median life expectancy of
36.8 years.2 The underlying genetic basis of the disease is related to
dysfunction or deficiency of the CF transmembrane conductance
regulator (CFTR), an apical membrane anion (e.g., chloride and bicarbonate) channel present in respiratory and exocrine glandular epithelium.3,4 Early diagnosis of CF is crucial to allow for intervention
before lung disease ensues.5
Although chronic rhinosinusitis (CRS) is a serious cause of morbidity
and may drive pulmonary disease in patients with CF, it is rarely the
cause of mortality related to the disease. Sinonasal symptoms can be
severe and refractory despite a myriad of medical and surgical interventions leading to frustration for the patient. Quality of life indicators have
From the Departments of 1Surgery/Division of Otolaryngology and 2Medicine, and the
3
Gregory Fleming James Cystic Fibrosis Research Center, University of Alabama at
Birmingham, Birmingham, Alabama
Presented at the North American Allergy and Rhinology Conference, Puerto Rico,
February 5, 2012
BA Woodworth received funding from the Flight Attendant’s Medical Research Institute Young Clinical Scientist Award (072218) and NIH/NHLBI (1K08HL107142-01),
is a consultant for ArthroCare ENT and Gyrus ENT, and he is an inventor on a patent
submitted regarding the use of chloride secretagogues for therapy of sinus disease (35
U.S.C. n111(b) and 37 C.F.R n.53 (c)) in the United States Patent and Trademark
Office. SM Rowe received funding from R01HL105487-01 and 1R03DK084110-01 and
from Vertex Pharmaceuticals to conduct clinical trials in cystic fibrosis and he is an
inventor on a patent submitted regarding the use of chloride secretagogues for therapy
of sinus disease (35 U.S.C. n111(b) and 37 C.F.R n.53 (c)) in the United States Patent
and Trademark Office. The remaining authors have no conflicts of interest to declare
pertaining to this article
Address correspondence and reprint requests to Bradford A. Woodworth, M.D., BDB
563, 1530 3rd Avenue S, Birmingham, AL 35294-0012
E-mail address: [email protected]
Originally published in Am J Rhinol Allergy 27, 387–395, 2013
Copyright © 2014, OceanSide Publications, Inc., U.S.A.
S38
shown that sinus disease often mirrors pulmonary function and can be
predictive of pulmonary disease, particularly in the pediatric population.6 Additionally, few randomized controlled trials are currently available with regard to efficacy of therapies for CF CRS and many studies are
limited by the lack of long-term follow-up.7–9 Evidence-based guidelines
are deficient and management paradigms concerning disease interventions have not been standardized for all patients.
The purpose of the current review is to provide an update regarding management of this unique CRS population, present a summary
of the best available evidence for therapeutic interventions, and discuss exciting new strategies in drug development.
GENETICS
Over 1600 mutations have been described in the coding sequence of
the CFTR gene, messenger RNA splice signals, and other regions.10
Mutations are generally classified into six categories according to the
mechanistic basis by which they are believed to cause disease
(Table 1). The first three classes (I–III) are generally associated with
increased phenotypic severity. Class I mutations result in an absence of
CFTR gene synthesis and develop secondary to premature termination codons, nonsense mutations (e.g., G542X mutation, the “X” referring to existence of a premature stop codon), or other out of frame
mutations (insertions or deletions).11 CFTR is normally transcribed
and translated in class II mutations, but the protein folds incorrectly
and is recognized as defective in the endoplasmic reticulum during
intracellular trafficking. The protein is degraded before it reaches the
site of action at the cell surface. The class II F508del mutation (deletion
of a phenylalanine residue at the 508 position) is the most common
genetic mutation and accounts for ⬃70% of defective alleles.12 Class
III mutations consist of full-length CFTR protein present in normal
quantities at the cell surface, but disrupted regulation or gating of the
chloride transporter leads to a lack of ion channel activity (e.g., G551D
mutation).13 Although class III mutations possess minimal to no
CFTR-dependent transport, class IV defects represent abnormalities
of chloride conductance and may have partial activity in vivo. Such
mutations may lead to a CF pulmonary phenotype that is less severe
than other forms of the disease (e.g., R117H mutation).14 Class V
mutations produce decreased quantities of CFTR transcripts and,
thus, fewer functional CFTR channels at the cell surface.11,12 Finally,
May–June 2014, Vol. 28, No. 3
Copyright © Oceanside Publications, Inc. All rights reserved. DO NOT COPY
For permission to copy go to https://www.oceansidepubl.com/permission.htm
Table 1 CF genetic mutations
Mutation
Class
Class I
Class II
Class III
Class IV
Class V
Class VI
Mechanism
Premature truncation
or nonsense
mutation
Loss of
phenylalanine at
508 position
(del508)
Disrupted regulation
or gating of
channel
Abnormality of
chloride
conductance
Decreased quantities
of mature CFTR
transcripts
Defect in stability of
protein and high
turnover
Gene Result
Absence of CFTR gene
Protein Result
Chloride Channel
Phenotype
Example
Gene intact and with
normal transcription
Degraded before
reaching cell
membrane
Abnormal folding and
degradation in ER
Absent
Very severe
G542X
Absent (deficient in
mild forms)
Usually very
severe
F508del (most
common
mutation)
Gene intact and with
normal transcription
Present in normal
quantity
Minimal function
Severe
G551D
Gene intact and with
normal transcription
Present
Partial function
Less severe
R117H
Gene intact but aberrant
splicing at variable
frequency
Gene intact with normal
transcription
Decreased number of
full-length CFTRs
Full function but in
low quantity
Variable
2789 ⫹ 5G 3 A
Present but with
decreased stability
Functional but high
turnover with
overall decreased
number
Less severe
4236delTC
CF ⫽ cystic fibrosis; CFTR ⫽ cystic fibrosis transmembrane conductance regulator; ER ⫽ emergency room.
Table 2 Summary of therapies
Author’s Principals of Evaluation/Management/Therapy
Topical nasal saline irrigations improve QOL
Topical steroids decrease CF NPs
Topical antibiotics, postsurgical and culture directed
Macrolide therapy to decrease NPs
Postoperative topical dornase alfa
High-dose ibuprofen for CRS with NPs
Ivacaftor for G511D mutation
FESS (maxillary antrostomy and anterior ethmoidectomy)
improves QOL indicators
FESS improves pulmonary outcomes
Modified medial maxillectomy improves QOL indicators
Evidence Grade of Recommendation
B (extrapolated from level 1 evidence in CRS)
B (one level 1 study with high risk of bias ⫹ extrapolation from eosinophilic
CRS/NP level 1)
B
B
A (two level 1 studies showing benefit)
C (one retrospective study)
N/A
B
B
B
Grades of recommendation: A, consistent level 1 studies; B, consistent level 2 or 3 studies or extrapolations from level 1 studies; C, level 4 studies or
extrapolations from level 2 or 3 studies; D, level 5 evidence or troublingly inconsistent or inconclusive studies of any level.
QOL ⫽ quality of life; CF ⫽ cystic fibrosis; CRS ⫽ chronic rhinosinusitis; FESS ⫽ functional endoscopic sinus surgery; NPs ⫽ nasal polyps.
class VI mutations create defects in the stability of the protein leading
to accelerated turnover at the cell surface and insufficient quantities of
CFTR under steady-state conditions.15
SPECTRUM OF DISEASE: VARIANCE IN
PHENOTYPIC EXPRESSION
exhibit at least one characteristic phenotypic feature, have a family
history of CF, or have a positive neonatal screening test. In addition
to these requisites, the patients must also have positive testing with
either an increased sweat chloride concentration (⬎60 mmol/L), genetic tests showing two CF causing mutations, or demonstration of an
abnormal ancillary test indicative of CFTR abnormality (e.g., nasal
epithelial ion transport or intestinal current measurement).18
Classic CF
Patients with classic CF tend to have class I–III mutations and
develop upper and lower airway disease, exocrine pancreatic insufficiency, absence of the vas deferens, and highly elevated sweat
chloride concentration, although specific genetic mutations may be
associated with increased phenotypic severity (Table 2). For example,
the most common mutation F508del homozygosity does not predict
disease severity on its own, but this genotype is shown to be an
independent risk factor in other manifestations of CF, including reduced mineral bone density and earlier colonization with Pseudomonas aeruginosa.16,17 The diagnosis of classic CF is made when patients
American Journal of Rhinology & Allergy
Atypical CF
Individuals with atypical CF provide a diagnostic challenge as
standard diagnostics, including that sweat chloride testing can be
normal.19 This group is currently described in the Cystic Fibrosis
Foundation consensus document as individuals who show a CF phenotype in at least one organ system and have normal (⬍40 mmol/L)
or borderline (40–60 mmol/L) sweat chloride values.18 These patients
tend to have pancreatic exocrine sufficiency in the setting of milder
lung or sinonasal disease. They may present with a classic CF phenotype in only a single organ system. This underscores the impor-
Copyright © Oceanside Publications, Inc. All rights reserved. DO NOT COPY
For permission to copy go to https://www.oceansidepubl.com/permission.htm
S39
tance of CF testing in patients with persistent, refractory CRS who do
not exhibit classic pulmonary or gastrointestinal CF manifestations.
Individuals with nonclassic CF carry two CFTR mutations, at least
one of which is usually a “mild/variable” mutation.20
CFTR-Related Diseases
CFTR-related diseases encompass well-known pathological disorders that appear to be influenced by CFTR genotype, including allergic bronchopulmonary aspergillosis, idiopathic bronchiectasis, and
CRS. Wang et al.21 performed genetic testing on 147 non-CF patients
with severe CRS and discovered a 7% incidence of CFTR mutations
compared with the presence of 2% in 123 healthy controls (p ⫽ 0.04).
Nasal potential difference measurements also documented a slight
reduction in CFTR-mediated anion transport in carriers, but not to the
levels seen with CF. In another study of 58 children with CRS, seven
(12%) carried a single CF mutation, which was much higher than the
expected frequency of 4%.22 Although these illnesses appear to be
influenced by CFTR dysfunction, they are also influenced by nonCFTR genes and environmental exposures. However, the influence of
CFTR indicates that many treatment regimens may be directly applicable to the management of non-CF CRS—specifically, the importance of improving mucus clearance in susceptible individuals.
CYSTIC FIBROSIS CHRONIC RHINOSINUSITIS
Pathophysiology of CRS
The lining of the sinonasal epithelium is comprised of airway
surface liquid containing a low viscosity periciliary fluid layer (sol)
around the respiratory cilia and a superficial mucus (gel) layer, which
function to trap and sweep inhaled particles into the digestive tract
through coordinated mucociliary clearance (MCC). Intact MCC is
considered the airway’s innate defense against disease.23 MCC is
grossly impaired in CF because of alterations in the transepithelial
passage of anions (chloride and bicarbonate) caused by genetic mutations in the CFTR.24 Disturbances in anion transport result in increased viscosity of mucins that is 30–60 times higher than patients
without CF. Tenacious secretions obstruct sinus ostia and create
hypoxic conditions with increased edema, secondary ciliary dyskinesia, and subsequent bacterial overgrowth.25,26 Hypoxia has also been
shown to affect CFTR transcription and function in epithelium in
patients with normal CFTR.27 Patients with classic CF have a high
incidence of CRS approaching 100%.25
CF patients also have a high incidence of nasal polyposis associated
with CRS (7–48%).28 Nasal polyps (NPs) associated with CF are
typically mediated by neutrophilic Th1-mediated inflammation
rather than eosinophilic Th2-mediated inflammation seen in atopic
and aspirin-sensitive CRS with NPs.29 Claeys et al.30 showed that the
Th1 inflammatory mediators IL-8 and myeloperoxidase actually dominate in CF NP compared with eosinophilic cationic protein, eotaxin,
and IgE in non–CF NPs. In addition, the antimicrobial peptide (human defensin ␤2) and pattern recognition receptor Toll-like receptor
2 were significantly increased in CF NPs as well. Other innate defense
proteins such as surfactant protein (SP) A, SP-B, and SP-D are also
upregulated in CF.31–34 Other molecular differences between CF and
non CF polyps include a significantly higher level of lipoxin A4 and
slightly elevated cyclooxygenase 2 in CF polyps.35 Onset of bacterial
infection and colonization in CF CRS may instigate inflammatory
pathways with Toll-like receptors recognizing pathogen-associated
molecular patterns with production of these antimicrobial peptides.36–38 The chronic inflammation seen in the sinuses after bacterial
contamination also results in goblet cell hyperplasia, squamous metaplasia, and the loss of ciliated cells.26
Mucocele formation is common in CF patients39,40 and its presence
in children should be diagnostic of CF unless proven otherwise.41
Decreased paranasal sinus development (hypoplasia) is also a distinguishing characteristic noted in CF patients.42–45 The reasons for this
S40
lack of development and the effects of specific CF genotypes on
phenotypic expression of sinus development are unclear. One prevailing theory considers that ongoing inflammatory mucosal disease
leads to decreased pneumatization similar to poor temporal bone
pneumatization in chronic otitis media.46 The general lack of chronic
ear disease in CF patients and normal temporal bone development
argues against this hypothesis.47 However, genotype does appear to
influence paranasal sinus development as individuals homozygous
for the F508del mutation have shown a significantly increased frequency of underdeveloped frontal (98%), maxillary (70%), and sphenoid (100%) sinuses when compared with other genetic mutations (69,
8, and 50%, respectively), suggesting CFTR may be a primary contributor to sinus development.45 Studies from the recently developed
CF pig model support the premise that CFTR dysfunction as opposed
to chronic infection is responsible for decreased sinus pneumatization
because pigs lacking intact CFTR have sinus underdevelopment before the development of infection.48
Microbiology
Numerous bacteria are frequently isolated from sinus cultures of
CF patients including P. aeruginosa, Staphylococcus aureus, Escherichia
coli, Burkholderia cepacia, Acinetobacter species, Stenotrophomonas
maltophilia, Haemophilus influenza, Streptococci, and anaerobes.49–52
Muhlebach et al.53 studied lower airway and throat cultures and
reported that P. aeruginosa as well S. aureus are the most common
bacterial species found in CF patients. There is a higher frequency of
Pseudomonas colonization in the lower airways in patients who have
CRS with NPs and is more likely to start in the sinuses at a younger
age and progress to involve both the upper and the lower airways.54,55
This was confirmed by Godoy et al.56 who showed a significant
association between sinus cultures and lower airway cultures from
bronchoalveolar lavage. Genotypes of sinus bacteria were also shown
to be concordant with the lower airway, indicating the sinuses also
may serve as a reservoir for recurrent lung infection,57 increasing the
importance of maximizing sinus health. There are increased patterns
of antimicrobial resistance secondary to multiple antibiotic exposures
and increased prevalence of resistant bacteria within the community,
particularly S. aureus. This is problematic because methicillin-resistant
S. aureus colonization and infection in the respiratory tract of CF
patients is associated with significantly worse overall survival.58
In addition to bacteria, fungi are commonly isolated from CF patients with Candida species being the most prevalent. It is considered
a colonizer in pulmonary cultures.59 The use of inhaled steroids in CF
patients along with improved culture techniques are likely contributors to increased fungal recovery. Fungus was also retrieved in 33% of
patients in a study by Wise et al.,60 with two patients fulfilling the
criteria for allergic fungal rhinosinusitis. The implications of these
findings are unclear and further studies are required to examine the
potential pathogenic role of these fungi.
Clinical Manifestations
Symptoms of CRS, when present, frequently include rhinorrhea,
nasal obstruction, mouth breathing, headache, anosmia, and restless
sleep.61,62 Other symptoms include facial pain, activity intolerance,
halitosis, and voice changes.63–65 When associated with nasal CRS
with NPs, the most common complaint is nasal obstruction whereas
in patients without NPs, the most common complaint is headache or
facial pain.25
Mouth breathing coupled with thick anterior and posterior nasal
discharge may be the result of sinonasal polyposis. CF patients may
also have facial deformation such as broadening of the nasal bridge,
hypertelorism, and proptosis from chronic polyp expansion.26 NPs
are frequently seen on rhinoscopy, which are usually multiple and
bilateral.66 Rhinoscopy or nasal endoscopy may also show medial
bulging of the lateral nasal wall.67 Unfortunately, physical examina-
May–June 2014, Vol. 28, No. 3
Copyright © Oceanside Publications, Inc. All rights reserved. DO NOT COPY
For permission to copy go to https://www.oceansidepubl.com/permission.htm
tion findings do not correlate or fluctuate with ongoing changes in
clinical status.9
Imaging
Radiographic abnormalities are frequent and often multiple in CF
patients. The prevalence and detection of these abnormalities has
improved dramatically with the increased use and improvement in
CT scans. Findings on CT scan can aid in the diagnosis of CF,
particularly among children, including demineralization and medial
displacement of the uncinate process with inspissated secretions in
the maxillary sinuses. Imaging is often crucial in this patient population because of the loss of important anatomic landmarks in particularly severe cases. Imaging may also be of use during surgical
planning but also intraoperatively by using stereotactic imaging in
conjunction with an endoscopic view.
Management
Management of CF sinusitis can be a daunting task, but most
treatment recommendations dictate conservative management with
medical therapy in lieu of primary surgical intervention.68 Few prospective studies exist to address medical management strategy of
patients with CF sinusitis. Conservative therapy is favored with the
use of sinus irrigations, mucolytics, oral steroids, and oral or i.v.
antibiotics, as well as topical antibiotic and steroid therapeutic delivery. Patients who fail medical management or who present with
complications such as bone erosion are good candidates for sinus
surgery. The need for surgical management of the sinuses in CF
patients continues to be controversial. Several studies showing severe
CF sinusitis as a risk factor for forced expiratory volume in 1 second
(FEV1) decline and thus worsened prognosis in children suggest
aggressive treatment may have a role in select patients.69–71
Medical Management
Medical management consists of nasal saline irrigations as well as
medications including antibiotics, decongestants, antihistamines, topical and systemic steroids, dornase alfa, and N-acetyl cysteine as well
as surfactant lavage.72,73 Evidence regarding several interventions is
discussed later.
Nasal Saline Irrigations. Irrigations with isotonic and hypotonic saline solutions serve to mechanically debride crusting along the sinonasal mucosa while hypertonic saline has the added theoretical benefit of decongestion by osmosis.74 A Cochrane meta-analysis
concluded that quality of life in non-CF patients is improved with
saline irrigation when compared with nontreatment.72 Unfortunately,
no studies are available regarding the use of saline nasal irrigations in
the CF CRS population, and thus recommendations for its use are
extrapolated from studies in non-CF patients and the benefit is indicated in the pulmonary airways with nebulized hypertonic saline.75,76
The patients usually are instructed to use nasal irrigations using
larger-volume, low-pressure irrigation bottles for comfort. A cadaver
study showed the effects of irrigation are greatly enhanced after
endoscopic sinus surgery (ESS). The squeeze bottle/neti pot devices
provided the best saline irrigation delivery to the paranasal sinuses
and are probably the best devices to administer topical delivery of
antibiotics and steroids as well.77
Topical Steroids. Topical steroids are particularly effective for patients with allergic rhinitis as well as eosinophilic NPs in adults and
children.78 CF NPs are neutrophilic and generally less responsive to
steroids, but polyp size reduction and improvement in symptoms has
been noted, particularly when using the Mygind’s position or upside
down positioning.78–80 Hadfield et al.79 performed a randomized controlled trial (46 participants) comparing topical steroid (betamethasone) nasal drops with placebo and reported significant reduction in
polyp size. It was noted in a later Cochrane review that the risk of bias
was high in this study because ⬎50% of people enrolled did not
American Journal of Rhinology & Allergy
complete follow-up.81 However, high-dose topical steroid rinses with
budesonide have shown no alteration of the hypothalamic–pituitary
axis in several studies.82,83 Low-absorption topical steroid irrigations
appear to be a reasonable strategy in CF CRS, although further
randomized controlled trials are required.
Topical Antibiotics. In a systematic review on the use of topical
antimicrobials delivered in sinonasal irrigation by Lim et al.,84 the
authors noted that there was no sufficient evidence to justify their use
in CRS patients in general, but a high level of evidence was reported
regarding use in the CF CRS population (IIb).85 Topical antibiotics
have fewer adverse effects than oral antibiotics and may achieve a
higher drug concentration at the target site.26 Topical tobramycin has
been shown to be effective in reducing symptoms and reveals improvements in endoscopic scores in sinusitis.84 Because of the increased risk of recurrent CRS exacerbations after surgery, aggressive
topical management is generally recommended. The use of topical
antibiotics postoperatively has also been associated with reduced
recurrence of CF sinus exacerbations86 with another study showing
improved control of sinus disease for at least 2 years after surgery.52
Macrolide Antibiotics. Macrolides with 14 and 15 membered rings
(e.g., azithromycin and clarithromycin) down-regulate inflammatory
responses and are effective for the treatment of chronic airway inflammatory diseases including diffuse panbronchiolitis, CRS, and CF.
Besides clinical improvement in nasal obstruction and nasal secretions,87 macrolides have been shown to decrease production of IL-8 by
nasal epithelial cells88 and they correlate with improvement in eosinophilic CRS with NPs.89 Although there are no discrete studies establishing clinical benefit for CRS in the CF population, similar CFrelated pathophysiology between the sinonasal and pulmonary
airways indicates this medication class is likely a valuable therapeutic
addition for CF-associated sinus inflammation.90 Proper dosage and
scheduling is controversial, but use for CF-associated pulmonary
disease is 500 mg, 3 times/wk and is supported by randomized,
controlled trials.91
Oral Antibiotics. In addition to macrolides, ciprofloxacin has been
used prophylactically in patients with CF. Although studies for CFassociated sinus disease are lacking, an analysis of individuals progressing to pulmonary exacerbation who were provided oral antibiotics for an average of 13.4 days were found to have circumvented the
need for i.v. antibiotics in up to 80% of the time92
Ibuprofen. There have been positive results with the use of highdose ibuprofen on the progression of pulmonary disease in children
with CF.93 In a small series of 12 CF patients with NPs treated with
high-dose ibuprofen therapy for pulmonary disease, the absence of
polyps was noted at some point during treatment. In addition, clinical
regression of NPs was noted in five patients during ibuprofen therapy.94 Confirmatory studies are required to evaluate the effectiveness
of this drug in CF CRS.
Dornase Alfa. Dornase alfa is a recombinant human deoxyribonuclease that hydrolyzes DNA polymers, reduces DNA fragment
length, and reduces the viscosity of CF purulent secretions.95 Clinically, it has been shown to reduce the risks of pulmonary exacerbations, improve FEV1, and slow the continued rate of decline of lung
function in CF patients ⬎5 years old.96–98 Nasal nebulized dornase
alfa has also shown clinical efficacy in CF CRS. Cimmino et al.,99 in a
double-blind placebo-controlled trial, reported on the use of nebulized dornase alfa in early postoperative CF ESS. There was a significant improvement in the nasal symptoms and endoscopic findings,
as well as FEV1. Mainz et al.100 also reported significant improvement
in quality of life (as measured by the 20-item Sino-Nasal Outcome
Test) in a double-blind placebo-controlled crossover trial with nebulized dornase alfa compared to normal saline.
Novel Therapeutics: Targeting the Basic Defect
New therapeutic strategies for CF that target rescue of CFTR activity have recently been approved for select CF patients, and are in
development in other groups of patients. Based on a dramatic ad-
Copyright © Oceanside Publications, Inc. All rights reserved. DO NOT COPY
For permission to copy go to https://www.oceansidepubl.com/permission.htm
S41
vancement in our understanding of the production, processing, and
function of the CFTR channel, small molecules identified by high
throughput drug screening that restore activity to the mutant CFTR
protein have been discovered and developed for clinical use.101–104
The three drugs that have entered clinical testing in CF include
ivacaftor (Kalydeco, formerly VX-770; Vertex Pharmaceuticals, Cambridge, MA), lumacaftor (formerly VX-809), and ataluren (formerly
PTC124). Ivacaftor, which potentiates mutant CFTR already present
in the cell surface, caused significant improvement in lung function
and other outcomes in clinical trials for patients with the G551D CFTR
mutation and was recently approved by the Food and Drug Administration for use in individuals aged ⱖ6 years with at least one copy
of this mutation.105 In this form of CF, mutant protein is present in
normal quantities on the cell surface, but the channel exhibits severely
defective gating. Although the effect on the CF sinuses still has not
been studied, it is presumed that augmenting CFTR would generate
pronounced improvement in the MCC of the sinus cavities, resulting
in improved sinus disease outcomes. This will be evaluated in part
during a postapproval study in G551D patients currently in progress.
The F508del CFTR mutation is a more challenging target, because
channels are “misfolded” and degraded in the endoplasmic reticulum
before reaching the cell surface. Lumacaftor, another molecule discovered by high throughput screening, “corrects” the processing of
the protein to improve delivery of CFTR to the plasma membrane.
Clinical trials are currently using VX-770 and VX-809 together in an
attempt to improve therapeutic effects and will be advanced to phase
3 testing.106,107 Ataluren is another small molecule currently under
investigation in clinical trials.108 This drug induces translational readthrough of nonsense mutations in CFTR in vitro and in vivo and has
shown activity in some but not all proof of concept CF trials. Ivacaftor
and other CFTR modulators109are being advanced in CF patients with
additional CFTR mutations; if successful, these new treatments could
also provide relief of CFTR-mediated mucosal abnormalities that
drive CF CRS pathogenesis.
Other approaches designed to improve mucociliary transport in CF
include targeting other apical ion channels to improve airway surface
liquid hydration. Drugs that either inhibit epithelial sodium channels
or stimulate alternative chloride pathways such as calcium-activated
chloride channels are also under active investigation.110,111
Surgical Management
Surgical Indications. The low incidence of self-reported symptoms
(20%) despite the presence of radiographic and endoscopic sinus
disease in the vast majority of CF patients reflects the difficulties in
assessing the indications for surgical management. In general, CF
patients with persistent symptoms who have failed medical management are often considered appropriate candidates for functional ESS
(FESS). Surgical management of the sinuses in CF patients is also
thought to improve pulmonary outcomes and is used to justify intervention in asymptomatic individuals. This has been shown in other
diseases such as asthma, where Stammberger112 reported a 70% improvement in asthma symptoms and Lund113 reported pulmonary
improvement in two-thirds of the postsurgical patients after ESS.
However, pulmonary outcomes after surgical intervention for CF
sinusitis are mixed. Several studies reported no change in objective
outcomes for children and adults such as hospital admissions and
pulmonary status, but showed improvements in clinical symptoms
and quality of life.64,114 The best data to support surgical intervention
in asymptomatic individuals derive from the identification of identical P. aeruginosa clones isolated in the sputum and the bronchoalveolar lavage of CF lung transplant patients before and after their transplant.115 Several studies have reported on the effect of sinus surgery
as well as postoperative nasal care as a better means for controlling
the sinuses as a reservoir for pulmonary infection.86,116 A retrospective
chart review of patients who received lung transplantation and had
subsequent FESS found a significant decrease in rehospitalization
rates.117 Surgery coupled with daily nasal irrigations led to a signifi-
S42
cant reduction in the incidence of tracheobronchitis as well as pneumonia and bronchiolitis obliterans syndrome in this population.117
With conflicting evidence regarding surgical indications, it is understandable that the percentage of CF patients requiring surgical
management of their disease varies considerably. Virgin et al.118 used
the pediatric health information services database to investigate the
number patients with CF who had sinus surgery during a 3-year
period at the 43 largest pediatric hospitals in the United States. The
frequency of FESS in CF patients varied from 3 to 47% among centers
with a positive correlation between hospital size, number of CF
patients, and percentage of patients that adhered to the Cystic Fibrosis Foundation guidelines.
FESS Outcomes. Standard FESS includes maxillary antrostomy, anterior and posterior ethmoidectomy, sphenoidotomy, and, depending
on the age of the patient and presence of frontal sinuses, a frontal
sinusotomy. Multiple studies have been conducted to report on the
safety and effectiveness of FESS in CF patients.9,119–123 Complication
rate after FESS in CF patients (11.5%) was found to be similar to the
rate of non-CF FESS complications (0–17%).121 Khalid et al. reported
on the outcomes of sinus surgery in adult patients both with and
without CF.114 Although baseline CT and endoscopy scores were
significantly worse in CF patients, the overall quality of life improvements as well as the degree of endoscopic improvement was similar
between the two groups. The quality of life scales used in this study
were the Rhinosinusitis Disability Index and the Chronic Sinusitis
Survey. However, overall failure rates requiring revision surgery
range from 13 to 89% in the literature.52,61,122,124,125 As a result, many
CF patients have had multiple surgeries by the time they reach
adulthood.
Often used as a quality indicator, the need for revision surgery can
be difficult to assess. In one study, CT findings were a significant
predictor for revision sinus surgery.126 Patients with higher LundMckay scores were found to require revision surgery. However,
almost all patients with CF have abnormal findings on CT scan,127
including asymptomatic non-CF patients (18–72%),128,129 but not all
require surgery. In CRS patients, CT does not correlate well with
symptom scores.130,131 McMurphy et al.132 found no significant difference between the preoperative and postoperative Lund-MacKay
scores of pediatric CF patients after initial surgery or in subsequent
scans despite medical or surgical interventions. Persistence or worsening of radiographic abnormalities after FESS has been shown in
several other studies.120,133 Thus, CT imaging changes alone is probably not an appropriate indicator for recurrence/failure or predicting
patient perception of disease except in the development of a mucocele
or orbital complication.125,134–136
The presence of NP was found to assess the future likelihood of
requiring revision FESS in several studies. In a study by Rowe-Jonce
and Mackay122 regarding FESS for CF sinusitis with NPs, the reported
rate of revision or return to preoperative symptom severity was 50%
with 18–24 months of clinical follow-up. Rickert et al.137 found that
preoperative grading of NPs in CF patients was predictive of the need
for future surgical revisions. In their study with a longer follow-up of
7.3 years, the reported revision rate for patients with severe polyps
was significantly higher (58%) compared with patients with no polyps (28%). The time for surgery was also significantly different between the groups tested with patients who had severe polyps requiring surgery in a shorter period of time.
Pediatric FESS Outcomes. The pediatric patient population is unique
because of the anatomic development peculiar to each age group.
Treatment algorithms focus on maximizing medical therapy although
surgical intervention has shown to improve patients whose disease
course is recalcitrant to therapy. Most CF patients who undergo FESS
exhibit improved symptom profile; however, radiographic and endoscopic scores are rarely significantly changed postoperatively.138 Similarly, data on pulmonary outcomes is mixed. Although short-term
improvement in lung function has been observed in children, longterm effects have not been found to be significant.139 Other retrospec-
May–June 2014, Vol. 28, No. 3
Copyright © Oceanside Publications, Inc. All rights reserved. DO NOT COPY
For permission to copy go to https://www.oceansidepubl.com/permission.htm
Figure 1. Coronal CT scans showing the
preoperative appearance of a patient with
cystic fibrosis (CF) after traditional maxillary antrostomies with completely opacified maxillary sinuses (left, white arrow)
and postoperative appearance after bilateral modified endoscopic medial maxillectomies and revision sinus surgery (right).
The coronal CT image is posterior to the
anterior one-third of the inferior turbinate.
(Adapted with permission from Ref. 145.)
tive studies on CF patients fail to show improved pulmonary function
tests after sinus surgery.139,140 One pediatric study showed significant
improvement in pulmonary function tests after FESS up to the first 2
years postoperatively.141 This was not shown in patients with low
socioeconomic status. So far, there are no reports of adverse effects of
FESS on facial growth in pediatric patients.123
Extensive Surgical Intervention Outcomes. The maxillary sinus is a
chronic refractory problem area in CF patients because the normal
MCC pathway is through the natural ostium and against gravity.
Because MCC is impaired, accumulation of mucopurulence in the
largest of the sinus cavities is commonplace on CT imaging and
endoscopic findings despite previous “adequate” maxillary antrostomies. This is one reason CT findings are unchanged after FESS in CF
patients because the maxillary sinus reaccumulates mucopurulence
quickly despite the use of nasal irrigations. In essence, the sinuses in
CF patients are similar to large abscesses that may be drained
temporarily but ultimately lack real change within the cavities
because the mucosa will never have normal function because of
their underlying genetic defect. Thus, the ultimate treatment goals
of aggressive surgical intervention are to establish permanent “access” rather than “ventilation” to the sinus cavities, permitting
additional medicinal, mechanical, and physical means for the removal of desiccated mucus.
The modified endoscopic medial maxillectomy (MEMM) involves
removal of the medial maxillary wall marsupializing the maxillary
sinus into the nasal cavity, but without sacrificing the head of the
inferior turbinate or lacrimal system (Figs. 1 and 2). Accumulation of
secretions becomes less frequent due to the open cavity and elimination of the physiological requirement of drainage through narrow
anatomic ostia. In addition, the procedure allows physical debridement of mucus and polypoid edema in the clinic, improved clearance
of mucus with nasal saline irrigations, and increased access for topical
delivery of therapeutics.
Multiple studies have been conducted regarding the role of this
more aggressive surgery in the management of CF sinusitis. A retrospective study in 2006 was the first investigation regarding the use of
MEMM in patients with CF CRS and it showed a low complication
rate.142 Shatz143 also reported on a very aggressive surgical approach
to the maxillary sinuses in CF children with prior history of FESS. In
this study, there was a significant reduction in symptoms and duration of hospitalization as well as FEV1 after bilateral Caldwell-Luc
and endoscopic medial maxillectomies in a cohort of 15 pediatric CF
patients.143 Cho et al.144 reported results of this technique (referred to
as a maxillary mega-antrostomy) and also found it to be safe and
effective. In a recent prospective study, FESS and MEMM combined
with a comprehensive postoperative medical management regimen
(culture-directed antibiotics, oral steroid taper, and topical steroid/
American Journal of Rhinology & Allergy
Figure 2. Transnasal endoscopic view of a left maxillary sinus after modified
endoscopic medial maxillectomy. A 30° endoscope is inserted past the anterior one-third of the inferior turbinate revealing a well-healed maxillary
cavity with no secretions retained in the floor of the sinus (arrow). (Adapted
with permission from Ref. 145.)
antibiotic irrigations) was associated with marked improvement in
sinus disease outcomes including a decrease in symptoms (22-item
Sino-Nasal Outcome Test) and objective findings (Lund Kennedy scores)
at 1 year of clinical follow-up.145 In this study, FEV1 was not significantly
changed, but there was significant reduction in the hospital admissions
for pulmonary exacerbations in the year postsurgery compared with the
year before. Results from these investigations lends support to a more
extensive surgical approach in CF sinusitis, but further studies are warranted to determine whether this treatment paradigm will provide longterm symptom improvement and confer advantages in CF pulmonary
outcomes.
CONCLUSION
CRS continues to be an important issue in the management of
patients with CF, particularly given the improved survival rates
associated with this disease. CF is a lifelong disease that requires
long-term surveillance, vigilance, and compliance. Quality of life and
decreased hospitalization will potentially become quality indicators
Copyright © Oceanside Publications, Inc. All rights reserved. DO NOT COPY
For permission to copy go to https://www.oceansidepubl.com/permission.htm
S43
in the management of this patient group, and a multidisciplinary
approach is necessary to develop consistent treatment paradigms for
this difficult entity. With improved treatments, MCC may be able to
be augmented in the future, reducing symptoms and their contribution to pulmonary progression.
26.
27.
28.
REFERENCES
1.
Rowe SM, Miller S, and Sorscher EJ. Cystic fibrosis. N Engl J Med
352:1992–2001, 2005.
2. Cystic Fibrosis Foundation. Patient Registry 2011 Annual Report.
Bethesda, MD: Cystic Fibrosis Foundation, pg 6, 2012.
3. Riordan JR, Rommens JM, Kerem B, et al. Identification of the cystic
fibrosis gene: Cloning and characterization of complementary DNA.
Science 245:1066–1073, 1989.
4. Collins FS. Cystic fibrosis: Molecular biology and therapeutic implications. Science 256:774–779, 1992.
5. Rosen MJ. Chronic cough due to bronchiectasis: ACCP evidencebased clinical practice guidelines. Chest 129(suppl):122S–131S, 2006.
6. Friedman EM, and Stewart M. An assessment of sinus quality of life
and pulmonary function in children with cystic fibrosis. Am J Rhinol 20:568–572, 2006.
7. Rosbe KW, Jones DT, Rahbar R, et al. Endoscopic sinus surgery in
cystic fibrosis: Do patients benefit from surgery? Int J Pediatr Otorhinolaryngol 61:113–119, 2001.
8. Rickert S, Banuchi VE, Germana JD, et al. Cystic fibrosis and endoscopic sinus surgery: Relationship between nasal polyposis and
likelihood of revision endoscopic sinus surgery in patients with
cystic fibrosis. Arch Otolaryngol Head Neck Surg 136:988–992, 2010.
9. Keck T, and Rozsasi A. Medium-term symptom outcomes after
paranasal sinus surgery in children and young adults with cystic
fibrosis. Laryngoscope 117:475–479, 2007.
10. Cystic Fibrosis Mutation Database. Available online at www.genet.sickkids.on.ca/cftr/Home.html; accessed August 19, 2012.
11. Zielenski J, and Tsui LC. Cystic fibrosis: Genotypic and phenotypic
variations. Annu Rev Genet 29:777–807, 1995.
12. Welsh MJ, and Smith AE. Molecular mechanisms of CFTR chloride
channel dysfunction in cystic fibrosis. Cell 73:1251–1254, 1993.
13. Accurso FJ, Rowe SM, Clancy JP, et al. Effect of VX-770 in persons
with cystic fibrosis and the G551D-CFTR mutation. N Engl J Med
363:1991–2003, 2010.
14. Gan KH, Veeze HJ, van den Ouweland AM, et al. A cystic fibrosis
mutation associated with mild lung disease. N Engl J Med 333:95–99, 1995.
15. Haardt M, Benharouga M, Lechardeur D, et al. C-terminal truncations destabilize the cystic fibrosis transmembrane conductance regulator without impairing its biogenesis. A novel class of mutation.
J Biol Chem 274:21873–21877, 1999.
16. King SJ, Topliss DJ, Kotsimbos T, et al. Reduced bone density in
cystic fibrosis: DeltaF508 mutation is an independent risk factor. Eur
Respir J 25:54–61, 2005.
17. Maselli JH, Sontag MK, Norris JM, et al. Risk factors for initial acquisition
of Pseudomonas aeruginosa in children with cystic fibrosis identified by
newborn screening. Pediatr Pulmonol 35:257–262, 2003.
18. Rosenstein BJ, and Cutting GR. The diagnosis of cystic fibrosis: A
consensus statement. Cystic Fibrosis Foundation Consensus Panel.
J Pediatr 132:589–595, 1998.
19. Boyle MP. Nonclassic cystic fibrosis and CFTR-related diseases.
Curr Opin Pulm Med 9:498–503, 2003.
20. Rowntree RK, and Harris A. The phenotypic consequences of CFTR
mutations. Ann Hum Genet 67:471–485, 2003.
21. Wang X, Moylan B, Leopold DA, et al. Mutation in the gene responsible for cystic fibrosis and predisposition to chronic rhinosinusitis
in the general population. JAMA 284:1814–1819, 2000.
22. Raman V, Clary R, Siegrist KL, et al. Increased prevalence of mutations in the cystic fibrosis transmembrane conductance regulator in
children with chronic rhinosinusitis. Pediatrics 109:E13, 2002.
23. Knowles MR, and Boucher RC. Mucus clearance as a primary innate
defense mechanism for mammalian airways. J Clin Invest 109:571–577,
2002.
24. Regnis JA, Robinson M, Bailey DL, et al. Mucociliary clearance in
patients with cystic fibrosis and in normal subjects. Am J Respir Crit
Care Med 150:66–71, 1994.
25. Gentile VG, and Isaacson G. Patterns of sinusitis in cystic fibrosis.
Laryngoscope 106:1005–1009, 1996.
S44
29.
30.
31.
32.
33.
34.
35.
36.
37.
38.
39.
40.
41.
42.
43.
44.
45.
46.
47.
48.
49.
50.
51.
52.
Gysin C, Alothman GA, and Papsin BC. Sinonasal disease in cystic
fibrosis: Clinical characteristics, diagnosis, and management. Pediatr Pulmonol 30:481–489, 2000.
Blount A, Zhang S, Chestnut M, et al. Transepithelial ion transport
is suppressed in hypoxic sinonasal epithelium. Laryngoscope 121:
1929–1934, 2011.
Robertson JM, Friedman EM, and Rubin BK. Nasal and sinus disease in cystic fibrosis. Paediatr Respir Rev 9:213–219, 2008.
Van Zele T, Claeys S, Gevaert P, et al. Differentiation of chronic
sinus diseases by measurement of inflammatory mediators. Allergy
61:1280–1289, 2006.
Claeys S, Van Hoecke H, Holtappels G, et al. Nasal polyps in patients with
and without cystic fibrosis: A differentiation by innate markers and inflammatory mediators. Clin Exp Allergy 35:467–472, 2005.
Woodworth BA, Lathers D, Neal JG, et al. Immunolocalization of
surfactant protein A and D in sinonasal mucosa. Am J Rhinol
20:461–465, 2006.
Woodworth BA, Neal JG, Newton D, et al. Surfactant protein A and
D in human sinus mucosa: A preliminary report. ORL J Otorhinolaryngol Relat Spec 69:57–60, 2007.
Woodworth BA, Wood R, Baatz JE, and Schlosser RJ. Sinonasal
surfactant protein A1, A2, and D gene expression in cystic fibrosis:
A preliminary report. Otolaryngol Head Neck Surg 137:34–38, 2007.
Woodworth BA, Wood R, Bhargave G, et al. Surfactant protein B
detection and gene expression in chronic rhinosinusitis. Laryngoscope 117:1296–1301, 2007.
Rozsasi A, Heinemann A, and Keck T. Cyclooxygenase 2 and lipoxin A(4) in nasal polyps in cystic fibrosis. Am J Rhinol Allergy
25:e251–e254.
Rampey AM, Lathers DM, Woodworth BA, and Schlosser RJ. Immunolocalization of dendritic cells and pattern recognition receptors in chronic rhinosinusitis. Am J Rhinol 21:117–121, 2007.
Raoust E, Balloy V, Garcia-Verdugo I, et al. Pseudomonas aeruginosa LPS or
flagellin are sufficient to activate TLR-dependent signaling in murine
alveolar macrophages and airway epithelial cells. PLoS One 4:e7259, 2009.
Laube DM, Yim S, Ryan LK, et al. Antimicrobial peptides in the
airway. Curr Top Microbiol Immunol 306:153–182, 2006.
Nicollas R, Facon F, Sudre-Levillain I, et al. Pediatric paranasal sinus
mucoceles: Etiologic factors, management and outcome. Int J Pediatr
Otorhinolaryngol 70:905–908, 2006.
Alvarez RJ, Liu NJ, and Isaacson G. Pediatric ethmoid mucoceles in
cystic fibrosis: Long-term follow-up of reported cases. Ear Nose
Throat J 76:538–539, 1997.
Olze H, Matthias C, and Degenhardt P. Paediatric paranasal sinus
mucoceles. Eur J Pediatr Surg 16:192–196, 2006.
Eggesbo HB, Sovik S, Dolvik S, et al. CT characterization of developmental variations of the paranasal sinuses in cystic fibrosis. Acta
Radiol 42:482–493, 2001.
Eggesbo HB, Sovik S, Dolvik S, et al. Proposal of a CT scoring
system of the paranasal sinuses in diagnosing cystic fibrosis. Eur
Radiol 13:1451–1460, 2003.
Shah RK, Dhingra JK, Carter BL, and Rebeiz EE. Paranasal sinus development: A radiographic study. Laryngoscope 113:205–209, 2003.
Woodworth BA, Ahn C, Flume PA, and Schlosser RJ. The delta F508
mutation in cystic fibrosis and impact on sinus development. Am J
Rhinol 21:122–127, 2007.
Milczuk HA, Dalley RW, Wessbacher FW, and Richardson MA.
Nasal and paranasal sinus anomalies in children with chronic sinusitis. Laryngoscope 103:247–252, 1993.
Seifert CM, Harvey RJ, Mathews JW, et al. Temporal bone pneumatization and its relationship to paranasal sinus development in
cystic fibrosis. Rhinology 48:233–238, 2010.
Chang EH, Pezzulo AA, Meyerholz DK, et al. Sinus hypoplasia
precedes sinus infection in a porcine model of cystic fibrosis. Laryngoscope 18:1898–1905, 2012.
Shapiro ED, Milmoe GJ, Wald ER, et al. Bacteriology of the maxillary
sinuses in patients with cystic fibrosis. J Infect Dis 146:589–593, 1982.
Halvorson DJ, Dupree JR, and Porubsky ES. Management of chronic
sinusitis in the adult cystic fibrosis patient. Ann Otol Rhinol Laryngol 107:946–952, 1998.
Mak GK, and Henig NR. Sinus disease in cystic fibrosis. Clin Rev
Allergy Immunol 21:51–63, 2001.
Moss RB, and King VV. Management of sinusitis in cystic fibrosis by
endoscopic surgery and serial antimicrobial lavage. Reduction in
May–June 2014, Vol. 28, No. 3
Copyright © Oceanside Publications, Inc. All rights reserved. DO NOT COPY
For permission to copy go to https://www.oceansidepubl.com/permission.htm
53.
54.
55.
56.
57.
58.
59.
60.
61.
62.
63.
64.
65.
66.
67.
68.
69.
70.
71.
72.
73.
74.
75.
76.
77.
78.
79.
recurrence requiring surgery. Arch Otolaryngol Head Neck Surg
121:566–572, 1995.
Muhlebach MS, Miller MB, Moore C, et al. Are lower airway or
throat cultures predictive of sinus bacteriology in cystic fibrosis?
Pediatr Pulmonol 41:445–451, 2006.
Henriksson G, Westrin KM, Karpati F, et al. Nasal polyps in cystic
fibrosis: Clinical endoscopic study with nasal lavage fluid analysis.
Chest 121:40–47, 2002.
Roby BB, McNamara J, Finkelstein M, and Sidman J. Sinus surgery
in cystic fibrosis patients: Comparison of sinus and lower airway
cultures. Int J Pediatr Otorhinolaryngol 72:1365–1369, 2008.
Godoy JM, Godoy AN, Ribalta G, and Largo I. Bacterial pattern in
chronic sinusitis and cystic fibrosis. Otolaryngol Head Neck Surg
145:673–676, 2011.
Mainz JG, and Koitschev A. Management of chronic rhinosinusitis
in CF. J Cyst Fibros 8(suppl 1):S10–S14, 2009.
Dasenbrook EC, Checkley W, Merlo CA, et al. Association between
respiratory tract methicillin-resistant Staphylococcus aureus and survival in cystic fibrosis. JAMA 303:2386–2392, 2010.
Haase G, Skopnik H, Groten T, et al. Long-term fungal cultures from
sputum of patients with cystic fibrosis. Mycoses 34:373–376, 1991.
Wise SK, Kingdom TT, McKean L, et al. Presence of fungus in sinus
cultures of cystic fibrosis patients. Am J Rhinol 19:47–51, 2005.
Cepero R, Smith RJ, Catlin FI, et al. Cystic fibrosis–An otolaryngologic perspective. Otolaryngol Head Neck Surg 97:356–360, 1987.
Naqvi SK, Sotelo C, Murry L, and Simakajornboon N. Sleep architecture in
children and adolescents with cystic fibrosis and the association with
severity of lung disease. Sleep Breath 12:77–83, 2008.
Nishioka GJ, Barbero GJ, Konig P, et al. Symptom outcome after functional
endoscopic sinus surgery in patients with cystic fibrosis: A prospective
study. Otolaryngol Head Neck Surg 113:440–445, 1995.
Jones JW, Parsons DS, and Cuyler JP. The results of functional
endoscopic sinus (FES) surgery on the symptoms of patients with
cystic fibrosis. Int J Pediatr Otorhinolaryngol 28:25–32, 1993.
Hulka GF. Head and neck manifestations of cystic fibrosis and ciliary
dyskinesia. Otolaryngol Clin North Am 33:1333–1341, vii–viii, 2000.
Neely JG, Harrison GM, Jerger JF, et al. The otolaryngologic aspects of
cystic fibrosis. Trans Am Acad Ophthalmol Otolaryngol 76:313–324, 1972.
Brihaye P, Clement PA, Dab I, and Desprechin B. Pathological
changes of the lateral nasal wall in patients with cystic fibrosis
(mucoviscidosis). Int J Pediatr Otorhinolaryngol 28:141–147, 1994.
Loebinger MR, Bilton D, and Wilson R. Upper airway 2: Bronchiectasis, cystic fibrosis and sinusitis. Thorax 64:1096–1101, 2009.
Amadori A, Antonelli A, Balteri I, et al. Recurrent exacerbations
affect FEV(1) decline in adult patients with cystic fibrosis. Respir
Med 103:407–413, 2009.
Sanders DB, Bittner RC, Rosenfeld M, et al. Pulmonary exacerbations are
associated with subsequent FEV1 decline in both adults and children with
cystic fibrosis. Pediatr Pulmonolr 46:393–400, 2010.
Konstan MW, Morgan WJ, Butler SM, et al. Risk factors for rate of decline
in forced expiratory volume in one second in children and adolescents
with cystic fibrosis. J Pediatr 151:134–139, 2007.
Harvey R, Hannan SA, Badia L, and Scadding G. Nasal saline
irrigations for the symptoms of chronic rhinosinusitis. Cochrane
Database Syst Rev 3:CD006394, 2007.
Fokkens WJ, Lund VJ, Mullol J, et al. EPOS 2012: European position
paper on rhinosinusitis and nasal polyps 2012. A summary for
otorhinolaryngologists. Rhinology 50:1–12, 2012.
Talbot AR, Herr TM, and Parsons DS. Mucociliary clearance and buffered
hypertonic saline solution. Laryngoscope 107:500–503, 1997.
Elkins MR, and Bye PT. Inhaled hypertonic saline as a therapy for
cystic fibrosis. Curr Opin Pulm Med 12:445–452, 2006.
Elkins MR, Robinson M, Rose BR, et al. A controlled trial of longterm inhaled hypertonic saline in patients with cystic fibrosis.
N Engl J Med 354:229–240, 2006.
Harvey RJ, Goddard JC, Wise SK, and Schlosser RJ. Effects of
endoscopic sinus surgery and delivery device on cadaver sinus
irrigation. Otolaryngol Head Neck Surg 139:137–142, 2008.
Costantini D, Di Cicco M, Giunta A, and Amabile G. Nasal polyposis in cystic fibrosis treated by beclomethasone dipropionate. Acta
Univ Carol Med (Praha) 36:220–221, 1990.
Hadfield PJ, Rowe-Jones JM, and Mackay IS. A prospective treatment trial of nasal polyps in adults with cystic fibrosis. Rhinology
38:63–65, 2000.
American Journal of Rhinology & Allergy
80.
Mainz JG KA. Management of chronic rhinosinusitis in CF. J Cyst
Fibros 8(suppl 1):S10–S14, 2009.
81. Beer H, Southern KW, and Swift AC. Topical nasal steroids for
treating nasal polyposis in people with cystic fibrosis. Cochrane
Database Syst Rev 5:CD008253, 2011.
82. Bhalla RK, Payton K, and Wright ED. Safety of budesonide in saline
sinonasal irrigations in the management of chronic rhinosinusitis
with polyposis: Lack of significant adrenal suppression. J Otolaryngol Head Neck Surg 37:821–825, 2008.
83. Welch KC, Thaler ER, Doghramji LL, et al. The effects of serum and
urinary cortisol levels of topical intranasal irrigations with budesonide added to saline in patients with recurrent polyposis after
endoscopic sinus surgery. Am J Rhinol Allerg 24:26–28, 2010.
84. Lim M, Citardi MJ, and Leong JL. Topical antimicrobials in the
management of chronic rhinosinusitis: A systematic review. Am J
Rhinol 22:381–389, 2008.
85. Vaughan WC, and Carvalho G. Use of nebulized antibiotics for
acute infections in chronic sinusitis. Otolaryngol Head Neck Surg
127:558–568, 2002.
86. Davidson TM, Murphy C, Mitchell M, et al. Management of chronic
sinusitis in cystic fibrosis. Laryngoscope 105:354–358, 1995.
87. Majima Y. Clinical implications of the immunomodulatory effects of
macrolides on sinusitis. Am J Med 117(suppl 9A):20S–25S, 2004.
88. Suzuki H, Shimomura A, Ikeda K, et al. Inhibitory effect of macrolides on interleukin-8 secretion from cultured human nasal epithelial cells. Laryngoscope 107:1661–1666, 1997.
89. Yamada T, Fujieda S, Mori S, et al. Macrolide treatment decreased
the size of nasal polyps and IL-8 levels in nasal lavage. Am J Rhinol
14:143–148, 2000.
90. Jaffe A, Francis J, Rosenthal M, and Bush A. Long-term azithromycin may improve lung function in children with cystic fibrosis.
Lancet 7 351:420, 1998.
91. Saiman L, Marshall BC, Mayer-Hamblett N, et al. Azithromycin in patients
with cystic fibrosis chronically infected with Pseudomonas aeruginosa: A
randomized controlled trial. JAMA 290:1749–1756, 2003.
92. Briggs EC, Nguyen T, Wall MA, and MacDonald KD. Oral antimicrobial use in outpatient cystic fibrosis pulmonary exacerbation
management: a single-center experience. Clin Respir J 6:56–64, 2012.
93. Konstan MW, Schluchter MD, Xue W, and Davis PB. Clinical use of
Ibuprofen is associated with slower FEV1 decline in children with
cystic fibrosis. Am J Respir Crit Care Med 1 176:1084–1089, 2007.
94. Lindstrom DR, Conley SF, Splaingard ML, and Gershan WM. Ibuprofen therapy and nasal polyposis in cystic fibrosis patients. J
Otolaryngol 36:309–314, 2007.
95. Shak S, Capon DJ, Hellmiss R, et al. Recombinant human DNase I
reduces the viscosity of cystic fibrosis sputum. Proc Natl Acad Sci U
S A 87:9188–9192, 1990.
96. Fuchs HJ, Borowitz DS, Christiansen DH, et al. Effect of aerosolized
recombinant human DNase on exacerbations of respiratory symptoms and on pulmonary function in patients with cystic fibrosis. The
Pulmozyme Study Group. N Engl J Med 331:637–642, 1994.
97. Quan JM, Tiddens HA, Sy JP, et al. A two-year randomized, placebo-controlled trial of dornase alfa in young patients with cystic
fibrosis with mild lung function abnormalities. J Pediatr 139:813–
820, 2001.
98. Harms HK, Matouk E, Tournier G, et al. Multicenter, open-label
study of recombinant human DNase in cystic fibrosis patients with
moderate lung disease. DNase International Study Group. Pediatr
Pulmonol 26:155–161, 1998.
99. Cimmino M, Nardone M, Cavaliere M, et al. Dornase alfa as postoperative therapy in cystic fibrosis sinonasal disease. Arch Otolaryngol Head Neck Surg 131:1097–1101, 2005.
100. Mainz JG, Schiller I, Ritschel C, et al. Sinonasal inhalation of dornase
alfa in CF: A double-blind placebo-controlled cross-over pilot trial.
Auris Nasus Larynx 38:220–227, 2011.
101. Rowe SM, Accurso F, and Clancy JP. Detection of cystic fibrosis
transmembrane conductance regulator activity in early-phase clinical trials. Proc Am Thorac Soc 4:387–398, 2007.
102. Rowe SM, Clancy JP, and Sorscher EJ. A breath of fresh air. Sci Am
305:68–73, 2011.
103. Rowe SM, Pyle LC, Jurkevante A, et al. DeltaF508 CFTR processing
correction and activity in polarized airway and non-airway cell
monolayers. Pulm Pharmacol Ther 23:268–278, 2010.
Copyright © Oceanside Publications, Inc. All rights reserved. DO NOT COPY
For permission to copy go to https://www.oceansidepubl.com/permission.htm
S45
104.
105.
106.
107.
108.
109.
110.
111.
112.
113.
114.
115.
116.
117.
118.
119.
120.
121.
122.
123.
124.
S46
Rowe SM, Varga K, Rab A, et al. Restoration of W1282X CFTR
activity by enhanced expression. Am J Respir Cell Mol Biol 37:347–
356, 2007.
Accurso FJ, Rowe SM, Clancy JP, et al. Effect of VX-770 in persons
with cystic fibrosis and the G551D-CFTR mutation. N Engl J Med
363:1991–2003, 2010.
Kim Chiaw P, Eckford PD, and Bear CE. Insights into the mechanisms underlying CFTR channel activity, the molecular basis for
cystic fibrosis and strategies for therapy. Essays Biochem 50:233–
248, 2011.
Pettit RS. Cystic fibrosis transmembrane conductance regulatormodifying medications: The future of cystic fibrosis treatment. Ann
Pharmacother 46:1065–1075, 2012.
Peltz SW, Welch EM, Jacobson A, et al. Nonsense suppression
activity of PTC124 (ataluren). Proc Natl Acad Sci U S A 106:E64,
2009.
Zhang S, Smith N, Schuster D, et al. Quercetin increases cystic
fibrosis transmembrane conductance regulator-mediated chloride
transport and ciliary beat frequency: Therapeutic implications for
chronic rhinosinusitis. Am J Rhinol Allergy 25:307–312, 2011.
Jones AM, and Helm JM. Emerging treatments in cystic fibrosis.
Drugs 69:1903–1910, 2009.
Kellerman D, Rossi Mospan A, Engels J, et al. Denufosol: A review
of studies with inhaled P2Y(2) agonists that led to Phase 3. Pulm
Pharmacol Ther 21:600–607, 2008.
Stammberger H. Endoscopic endonasal surgery–Concepts in treatment of recurring rhinosinusitis. Part II. Surgical technique. Otolaryngol Head Neck Surg 94:147–156, 1986.
Lund VJ, Holmstrom M, and Scadding GK. Functional endoscopic
sinus surgery in the management of chronic rhinosinusitis. An
objective assessment. J Laryngol Otol 105:832–835, 1991.
Khalid AN, Mace J, and Smith TL. Outcomes of sinus surgery in
adults with cystic fibrosis. Otolaryngol Head Neck Surg 141:358–
363, 2009.
Mainz JG, Hentschel J, Schien C, et al. Sinonasal persistence of
Pseudomonas aeruginosa after lung transplantation. J Cyst Fibrosr
11:158–161, 2011.
Lewiston N, King V, Umetsu D, et al. Cystic fibrosis patients who
have undergone heart-lung transplantation benefit from maxillary
sinus antrostomy and repeated sinus lavage. Transplant Proc 23:
1207–1208, 1991.
Holzmann D, Speich R, Kaufmann T, et al. Effects of sinus surgery
in patients with cystic fibrosis after lung transplantation: A 10-year
experience. Transplantation 77:134–136, 2004.
Virgin FW, Huang L, Roberson D, and Sawicki G. Inter-hospital
variation in the frequency of sinus surgery in pediatric patients with
cystic fibrosis. Pediatr Pulmonol Suppl 47:358, 2012.
Schulte DL, and Kasperbauer JL. Safety of paranasal sinus surgery
in patients with cystic fibrosis. Laryngoscope 108:1813–1815, 1998.
Cuyler JP. Follow-up of endoscopic sinus surgery on children with
cystic fibrosis. Arch Otolaryngol Head Neck Surg 118:505–506, 1992.
Albritton FD, and Kingdom TT. Endoscopic sinus surgery in patients with cystic fibrosis: An analysis of complications. Am J Rhinol
14:379–385, 2000.
Rowe-Jones JM, and Mackay IS. Endoscopic sinus surgery in the
treatment of cystic fibrosis with nasal polyposis. Laryngoscope 106:
1540–1544, 1996.
Van Peteghem A, and Clement PA. Influence of extensive functional
endoscopic sinus surgery (FESS) on facial growth in children with
cystic fibrosis. Comparison of 10 cephalometric parameters of the
midface for three study groups. Int J Pediatr Otorhinolaryngol 70:
1407–1413, 2006.
Jaffe BF, Strome M, Khaw KT, and Shwachman H. Nasal polypectomy and sinus surgery for cystic fibrosis–A 10 year review. Otolaryngol Clin North Am 10:81–90, 1977.
125.
126.
127.
128.
129.
130.
131.
132.
133.
134.
135.
136.
137.
138.
139.
140.
141.
142.
143.
144.
145.
Yung MW, Gould J, and Upton GJ. Nasal polyposis in children with
cystic fibrosis: A long-term follow-up study. Ann Otol Rhinol
Laryngol 111:1081–1086, 2002.
Becker SS, de Alarcon A, Bomeli SR, et al. Risk factors for recurrent
sinus surgery in cystic fibrosis: Review of a decade of experience.
Am J Rhinol 21:478–482, 2007.
April MM, Zinreich SJ, Baroody FM, and Naclerio RM. Coronal CT
scan abnormalities in children with chronic sinusitis. Laryngoscope
103:985–990, 1993.
Lesserson JA, Kieserman SP, and Finn DG. The radiographic incidence of chronic sinus disease in the pediatric population. Laryngoscope 104:159–166, 1994.
Diament MJ, Senac MO Jr, Gilsanz V, et al. Prevalence of incidental
paranasal sinuses opacification in pediatric patients: A CT study.
J Comput Assist Tomogr 11:426–431, 1987.
Smith TL, Mendolia-Loffredo S, Loehrl TA, et al. Predictive factors
and outcomes in endoscopic sinus surgery for chronic rhinosinusitis. Laryngoscope 115:2199–2205, 2005.
Stewart MG, Donovan DT, Parke RB Jr, and Bautista MH. Does the
severity of sinus computed tomography findings predict outcome in
chronic sinusitis? Otolaryngol Head Neck Surg 123:81–84, 2000.
McMurphy AB, Morriss C, Roberts DB, and Friedman EM. The
usefulness of computed tomography scans in cystic fibrosis patients
with chronic sinusitis. Am J Rhinol 21:706–710, 2007.
Eggesbo HB, Sovik S, Dolvik S, and Kolmannskog F. CT characterization of inflammatory paranasal sinus disease in cystic fibrosis.
Acta Radiol 43:21–28, 2002.
Tandon R, and Derkay C. Contemporary management of rhinosinusitis and cystic fibrosis. Curr Opin Otolaryngol Head Neck Surg
11:41–44, 2003.
Nishioka GJ, and Cook PR. Paranasal sinus disease in patients with
cystic fibrosis. Otolaryngol Clin North Am 29:193–205, 1996.
Krzeski A, Kapiszewska-Dzedzej D, Jakubczyk I, et al. Extent of
pathological changes in the paranasal sinuses of patients with cystic
fibrosis: CT analysis. Am J Rhinol 15:207–210, 2001.
Rickert S, Banuchi VE, Germana JD, et al. Cystic fibrosis and endoscopic sinus surgery: Relationship between nasal polyposis and
likelihood of revision endoscopic sinus surgery in patients with
cystic fibrosis. Arch Otolaryngol Head Neck Surg 136:988–992, 2010.
Rosbe KW, Jones DT, Rahbar R, et al. Endoscopic sinus surgery in
cystic fibrosis: Do patients benefit from surgery? Int J Pediatr Otorhinolaryngol 1 61:113–119, 2001.
Jarrett WA, Militsakh O, Anstad M, and Manaligod J. Endoscopic
sinus surgery in cystic fibrosis: effects on pulmonary function and
ideal body weight. Ear Nose Throat J 83:118–121, 2004.
Madonna D, Isaacson G, Rosenfeld RM, and Panitch H. Effect of
sinus surgery on pulmonary function in patients with cystic fibrosis.
Laryngoscope 107:328–331, 1997.
Kovell LC, Wang J, Ishman SL, et al. Cystic fibrosis and sinusitis in
children: Outcomes and socioeconomic status. Otolaryngol Head
Neck Surg 145:146–153, 2011.
Woodworth BA, Parker RO, and Schlosser RJ. Modified endoscopic
medial maxillectomy for chronic maxillary sinusitis. Am J Rhinol
20:317–319, 2006.
Shatz A. Management of recurrent sinus disease in children with
cystic fibrosis: A combined approach. Otolaryngol Head Neck Surg
135:248–252, 2006.
Cho DY, and Hwang PH. Results of endoscopic maxillary megaantrostomy in recalcitrant maxillary sinusitis. Am J Rhinol 22:658–
662, 2008.
Virgin F, Rowe SM, Wade MB, et al. Extensive surgical and comprehensive postoperative medical management for severe, recalcitrant cystic fibrosis chronic rhinosinusitis. Am J Rhinol Allergy
26:70–75, 2012.
e
May–June 2014, Vol. 28, No. 3
Copyright © Oceanside Publications, Inc. All rights reserved. DO NOT COPY
For permission to copy go to https://www.oceansidepubl.com/permission.htm
Pediatric rhinosinusitis: Definitions, diagnosis
and management—An overview
Swapna K. Chandran, M.D.,1 and Thomas S. Higgins, M.D., M.S.P.H.2
ABSTRACT
Rhinosinusitis is common in the pediatric population; however, diagnostic and management techniques often differ when compared with adult rhinosinusitis.
Multidisciplinary guidelines have outlined the diagnostic criteria for pediatric rhinosinusitis. Although acute rhinosinusitis is a more infectious phenomenon,
chronic sinusitis involves a more multifactorial etiology. This article outlines some of the definitions of rhinosinusitis, diagnosis and management of pediatric
sinusitis, and the complications of rhinosinusitis seen in the pediatric population.
R
hinosinusitis is a common clinical problem with significant morbidity, resulting in 31 million1 office and emergency room visits
per year in the United States. Children have between six and eight
viral upper respiratory infections per year and up to 13% can be
complicated by bacterial rhinosinusitis.2 Defining the varied manifestations of rhinosinusitis has proven to be difficult, secondary to the
numerous causes of the condition: viral, bacterial, allergic, nonallergic, fungal, and even idiopathic. Commonly divided into acute and
chronic, other terms have also been used, including subacute and
recurrent acute rhinosinusitis. Acute sinusitis is often thought to be
infectious in nature, whereas chronic rhinosinusitis is often from
other inflammatory processes that are not necessarily infectious in
nature, such as gastroesophageal reflux disease and cystic fibrosis.3
The paranasal sinuses are common sites of infection for children
and adolescents. Viral upper respiratory infections (rhinitis) often
predisposes to infectious rhinosinusitis.2 These infections are typically
not considered life-threatening; however, serious sequelae can occur
rarely.
This article will give an overview of the current definitions of
rhinosinusitis according to recently published guidelines and also
outline some of the rare but life-threatening complications that may
occur in pediatric rhinosinusitis. Current guidelines and controversies in diagnosis and management will also be discussed.
DEFINITIONS (AMERICAN ACADEMY OF
PEDIATRICS GUIDELINE WITH AMERICAN
ACADEMY OF OTOLARYNGOLOGY GUIDELINE)
Definitions of the infections involving the upper respiratory tract
and paranasal sinuses have been described in published guidelines by
both the American Academy of Pediatrics (AAP) as well as the
American Academy of Otolaryngology.2,4 Table 1 outlines the types of
infections that are mainly classified by duration and severity of symptoms. Associated symptoms of cough, fatigue, hyposmia, anosmia,
1
From the Department of Otolaryngology Head and Neck Surgery, University of
Louisville School of Medicine, Louisville, Kentucky, and 2Kentuckiana Ear, Nose &
Throat, P.S.C., Louisville, Kentucky
The authors have no conflicts of interest to declare pertaining to this article
Address correspondence and reprint requests to Thomas S. Higgins, M.D., M.S.P.H.,
Kentuckiana Ear, Nose & Throat, P.S.C., 6420 Dutchmans Parkway, Number 380,
Louisville, KY 40205
E-mail address: [email protected]
Originally published in Am J Rhinol Allergy 27, S16 –S19, 2013
Copyright © 2014, OceanSide Publications, Inc., U.S.A.
American Journal of Rhinology & Allergy
maxillary dental pain, and ear fullness and pressure also are present
with acute rhinosinusitis.
A sinus infection can be caused by one or more bacteria isolated in
a high-density field.2–4 Streptococcus pneumoniae, Haemophilus influenzae, and Moraxella catarrhalis are the most commonly isolated pathogens. Pseudomonas aeruginosa and Staphylococcus aureus are also commonly isolated organisms,5 although more often isolated in chronic
rhinosinusitis. The role of bacterial infections in chronic rhinosinusitis
is controversial, although bacterial superantigen5biofilms and osteitis
of the sinuses may play a role.3
METHODS OF DIAGNOSIS
The multidisciplinary task forces strongly recommend the use of
clinical diagnostic criteria as outlined previously in making a diagnosis of rhinosinusitis.
Imaging studies are often controversial in young patients, secondary to radiation exposure and often there is a need for sedation in
obtaining a suitable radiographic image. In general, for both adults
and children, imaging studies are not recommended in acute sinusitis
unless a complication or alternative diagnosis is suspected. In children, imaging studies are not necessary to confirm a diagnosis of
clinical sinusitis in children ⬍6 years of age (recommendation 2a of
the AAP guidelines2). Computed tomography of the paranasal sinuses should be reserved for patients in whom surgery is being
considered (recommendation 2b of the AAP guidelines2). It should be
noted, however, that abnormal findings on a radiograph do not make
a diagnosis of rhinosinusitis, but merely serves to confirm a diagnosis.6
MANAGEMENT
In general, it is well accepted that viral upper respiratory infections
can be treated symptomatically with analgesic, antipyretic, and decongestant medications (either topical or systemic).2 Decongestants in
children should be used cautiously because there are few studies on
the efficacy and side effect profile of decongestants in pediatric patients. Concomitant decongestant and antihistamine use remains controversial in the treatment of infection and inflammation of the pediatric upper respiratory system.
Nasal saline irrigation in the pediatric population has both been
shown to be effective and well tolerated in children with rhinosinusitis.7,8
Copyright © Oceanside Publications, Inc. All rights reserved. DO NOT COPY
For permission to copy go to https://www.oceansidepubl.com/permission.htm
S47
Figure 1. Chandler’s classification of orbital complications of acute sinusitis. (A) Periorbital inflammatory edema caused by venous congestion.
(B) Orbital cellulitis. (C) Subperiosteal abscess
(purulence between the ethmoid bone and orbital
periosteum. (D) Orbital abscess (purulence within
the orbit). (E) Cavernous sinus thrombosis.
(Adapted from Ref. 14.)
S48
May–June 2014, Vol. 28, No. 3
Copyright © Oceanside Publications, Inc. All rights reserved. DO NOT COPY
For permission to copy go to https://www.oceansidepubl.com/permission.htm
Table 1. Types of infections classified by duration and severity of symptoms
Type
Duration
Acute viral
⬍10 days
Acute bacterial
⬎10 days
⬍30 days
Subacute bacterial
Recurrent acute bacterial
30–90 days (1–3 mo)
Episodic ⬍30 days separated by at
least 10 days without symptoms
⬎90 days (3 mo)
Chronic
Symptoms
Nasal drainage
Symptoms do not worsen
Purulent nasal drainage
Nasal obstruction
Facial pain/pressure/fullness
Worsening symptoms after 10
days
Mucopurulent drainage
New respiratory symptoms on residual
respiratory
Acute Rhinosinusitis
Antibiotics are recommended for the management of acute bacterial sinusitis to achieve a more rapid clinical cure (recommendation 3
of the AAP guidelines2). In most cases, amoxicillin should be used as
first-line therapy2,4 unless there are factors indicating presence of
bacterial species that are likely to be resistant to penicillins. These
factors include (1) attendance in day care, (2) recent antimicrobial
treatment, and (3) age ⬍2 years.2
Chronic Rhinosinusitis
Long-term, broad-spectrum antibiotics have been the cornerstone
of management of pediatric chronic rhinosinusitis, in contrast to
adults. The use of long-term broad-spectrum antibiotics is variable
and many patients are refractory perhaps secondary to biofilm
growth or other inflammatory causes of disease.5,9 Both oral and i.v.
antibiotics have been studied and have been shown to have successful
results. The most commonly used antibiotic is amoxicillin/clavulanate or ampicillin sodium with sulbactam sodium when i.v. studies
were performed.
The role of surgery in the pediatric population is controversial.
Although functional endoscopic sinus surgery is shown to be effective
in up to 60–80%9,10 of pediatric patients who underwent surgery,
some authors have expressed concerns of interference with facial
growth and sinus development. Adenoidectomy as first-line surgical
treatment with or without maxillary sinus irrigation and/or antrostomy has also evolved in the management of pediatric chronic rhinosinusitis. This offers a less invasive treatment at reduction of bacterial
load and aeration of the paranasal sinus. Results have shown 60–85%
efficacy at symptom improvement.10,11
Complications of Pediatric Rhinosinusitis
The most common complication of acute rhinosinusitis is orbital
extension of infection.12 The progression of orbital complications is
illustrated in Fig. 1.13 These complications most often occur in older
American Journal of Rhinology & Allergy
Complete
Complete
Episodic
Nasal obstruction
Facial pain/pressure/fullness
Decreased sense of smell
AND
Purulent mucus or edema in the
middle meatus or ethmoid
region
Polyps
Radiographic imaging showing
inflammation of the paranasal
sinuses
Acute bacterial sinusitis
superimposed on
chronic sinusitis
Symptom Resolution
Complete
Persistent respiratory symptoms such
as cough and nasal obstruction
Rhinorrhea
New symptoms resolve with
treatment and old remain
children. Diagnosis includes computed tomography to evaluate the
extent of disease. Management should include ophthalmology consultation and broad-spectrum antibiotics. Surgical management is
indicated in certain cases. Studies have recently shown that a trial of
antibiotics can be used, however, in small subperiosteal abscesses.
Intracranial complications also arise, although are much rarer than
orbital complications. The spectrum progresses from meningitis, epidural, and subdural empyemas to frontal lobe abscess. Treatment
again requires broad-spectrum antibiotics and a combination of sinus
and neurosurgical procedures to eradicate infection.14
CONCLUSION
Pediatric rhinosinusitis is a disease distinct from adult rhinosinusitis. Rhinosinusitis is difficult to define secondary to the varied pathophysiological mechanisms involved, especially in chronic disease.
Management is also difficult because of controversies surrounding
pediatric patients. Guidelines have helped to make diagnosis and
management easier and it has become clear that multidisciplinary
approaches to management are necessary.
CLINICAL PEARLS
• Children experience an average of 6 – 8 viral upper respiratory
infections per year, of which up to 13% can be complicated by
bacterial rhinosinusitis
• Streptococcus pneumoniae, Haemophilus influenzae, and Moraxella catarrhalis are the most commonly isolated pathogens in acute bacterial
rhinosinusitis of children
• Factors associated with penicillin resistant bacteria in acute rhinosinusitis in children include (1) attendance in day care, (2) recent
antimicrobial treatment, and (3) age ⬍ 2 years
• The most common complication of acute rhinosinusitis in children
is orbital extension of infection
• Orbital complications of acute rhinosinusitis in children include
Copyright © Oceanside Publications, Inc. All rights reserved. DO NOT COPY
For permission to copy go to https://www.oceansidepubl.com/permission.htm
S49
eyelid edema, subperiosteal abscess, orbital cellulitis, orbital abscess
and cavernous sinus thrombosis.
• Intracranial complication of acute rhinosinusitis in children are rare
and include meningitis, epidural and subdural empyemas and frontal lobe abscess.
REFERENCES
1.
2.
3.
4.
5.
6.
S50
International Rhinosinusitis Advisory Board. Infectious rhinosinusitis in adults: Classification, etiology, and management. Ear Nose
Throat J 76:1–22, 1997.
American Academy of Pediatrics, Subcommittee on Management of
Sinusitis and Committee on Quality Improvement. Clinical management guideline: Management of sinusitis. Pediatrics 108:798–808,
2001.
Meltzer EO, Hamilos DL, Hadley JA, et al. Rhinosinusitis: Establishing definitions for clinical research and patient care. Otolaryngol
Head Neck Surg 131:S1–S62, 2004.
Rosenfeld RM, Andes D, Bhattacharyya N, et al. Clinical practice
guideline: Adult sinusitis. Otolaryngol Head Neck Surg 137:S1–S31,
2007.
Criddle MW, Stinson A, Savliwala M, and Coticchia J. Pediatric
chronic rhinosinusitis: A retrospective review. Am J Otolaryngol
29:372–378, 2008.
Kronemer KA, and McAlister WH. Sinusitis and its imaging in the
pediatric population. Pedatr Radiol 27:837–846, 1997.
7.
Wang YH, Yang CP, Ku MS, et al. Efficacy of nasal irrigation in the
treatment of acute sinusitis in children. Int J Pediatr Otorhinolaryngol
73:1696–1701, 2009.
8. Jeffe JS, Bhushan B, and Schroeder JW Jr. Nasal saline irrigation in
children: A study of compliance and tolerance. Int J Pediatr Otorhinolaryngol 76:409–413, 2012.
9. Adappa ND, and Coticchia JM. Management of refractory chronic
rhinosinusitis in children. Am J Otolaryngol 27:384–389, 2006.
10. Thottam PJ, Haupert M, Saraiya S, et al. Functional endoscopy sinus
surgery (FESS) alone vs. balloon catheter sinuplasty (BCS) and ethmoidectomy: A comparative outcome analysis in pediatric chronic
rhinosinusitis. Int J Pediatr Otorhinolaryngol 76:1355–1360, 2012.
11. Ramadan HH, Bueller H, Hester ST, and Terrell AM. Sinus balloon
catheter dilation after adenoidectomy failure for children with
chronic rhinosinusitis. Arch Otolaryngol Head Neck Surg 138:635–
637, 2012.
12. Brook I. Microbiology and antimicrobial treatment of orbital and
intracranial complications of sinusitis in children and their management. Int J Pediatr Otorhinolaryngol 73:1183–1186, 2009.
13. Ward RF, and April MM. Complications of sinusitis in the pediatric population. Op Tech Otolaryngol Head Neck Surg 7:305–309,
1996.
14. Hicks CW, Weber JG, Reid JR, et al. Identifying and managing
intracranial complications of sinusitis in children. Pedatr Infect Dis J
30:222–226, 2011.
e
May–June 2014, Vol. 28, No. 3
Copyright © Oceanside Publications, Inc. All rights reserved. DO NOT COPY
For permission to copy go to https://www.oceansidepubl.com/permission.htm
Surgery for sinonasal disease
Thomas S. Higgins, M.D., M.S.P.H.,1 and Andrew P. Lane, M.D.2
ABSTRACT
Surgery for chronic rhinosinusitis is an effective complement to a well-designed medical regimen. Functional endoscopic sinus surgery is among the most
common surgeries performed for sinonasal disease refractory to maximal medical therapy. Nasal surgery techniques, such as septoplasty and inferior turbinate
surgery, may assist in both relieving the symptom of nasal obstruction and providing access for sinus surgery. Although rare, open sinus techniques are
occasionally required.
S
urgery for sinonasal disease is effective for recalcitrant rhinosinusitis after failed medical therapy. Sinus surgery has evolved
over the past few decades to a safer and more effective treatment
modality. Before the 1980s, sinus surgery was mainly performed with
open approaches using skin incisions for access. Functional endoscopic sinus surgery (FESS) is now the standard of care for most cases
of surgical sinus disease; open approaches are rarely required.
PREOPERATIVE ASSESSMENT
As with any surgery, a discussion with the patient of the risks,
benefits, and alternatives of sinus surgery is important. The risks
include bleeding, infection, anesthetic risks, scarring, recurrence of
disease, cerebrospinal fluid leak, nasolacrimal duct leak, vision/eye
injury, and, if a septoplasty is planned, nasoseptal perforation. A
systems-based medical evaluation, including but not limited to careful assessment of cardiac, pulmonary, and bleeding risk factors,
should be performed before undergoing surgery. Patients are candidates for sinus surgery if they meet the criteria for chronic rhinosinusitis (CRS) and are resistant to maximal medical therapy. It is
important to note, however, that surgery should be considered an
adjunct therapeutic modality performed along with ongoing medical
therapy, not a replacement to medical therapy. A computed tomography of the sinuses should be obtained to define the extent of disease
and for surgical planning. Stereotactic navigation, which requires
fine-cut images, should be considered for complex cases, especially
for revision sinus surgery, extensive polyposis, and difficult anatomy.
SINUS SURGERY TECHNIQUES
Sinus surgery can be divided by approach and location: (1) approach
(endoscopic versus open) and (2) location (maxillary sinus, ethmoid
sinus, sphenoid sinus, frontal sinus, etc.). Table 1 summarizes the
classification of sinus surgery.
ENDOSCOPIC SINUS SURGERY
The goal of ESS is to enlarge the natural drainage pathways of the
sinuses to improve mucociliary clearance and to permit better peneFrom 1Kentuckiana Ear, Nose & Throat, P.S.C., Louisville, Kentucky, and 2Division of
Rhinology, Department of Otolaryngology–Head and Neck Surgery, Johns Hopkins
School of Medicine, Baltimore, Maryland
The authors have no conflicts of interest to declare pertaining to this article
Address correspondence and reprint requests to Andrew P. Lane, M.D., Department of
Otolaryngology–Head and Neck Surgery, Johns Hopkins Outpatient Center, 6th Floor,
601 North Caroline Street, Baltimore, MD 21287
E-mail address: [email protected]
Originally published in Am J Rhinol Allergy 27, S42–S44, 2013
Copyright © 2014, OceanSide Publications, Inc., U.S.A.
American Journal of Rhinology & Allergy
tration of medical therapy. Several outcome studies have shown
improvement in sinonasal symptoms, signs, and quality of life after
FESS. A systematic review of outcome studies after FESS, including 11
prospective studies and 5 studies using validated quality-of-life measures, found that all 45 studies showed improvement in CRS-related
symptoms and quality of life.1
Techniques important to any ESS operation include mucosal preservation and control of intraoperative bleeding. Mucosal stripping
disrupts the small submucosal vascular system, leading to oozing into
the surgical field. Topical vasoconstrictors may be used to improve
visualization of the surgical field. A systematic review of topical
vasoconstrictors showed that most topical vasoconstrictors have very
few side effects and reduce bleeding. Some commonly used vasoconstrictor agents include oxymetazoline/xylometazoline, cocaine, and
epinephrine. It is important to note that these agents should be used
judiciously in patients with cardiovascular risk factors and the pediatric population.2
Endoscopic maxillary antrostomy is the most common endoscopic
sinus procedure performed. The important aspects of this surgery
include an adequate uncinectomy and identification of the natural
ostium with incorporation into the surgical antrostomy. Maxillary
antrostomy with tissue removal is documented when extensive debris
or a mass is removed along with the antrostomy. Another technique,
termed endoscopic maxillary mega-antrostomy or endoscopic medial
maxillectomy, in which the posterior aspect of inferior turbinate and
a wide antrostomy are performed, may be beneficial in cases of
refractory disease and tumor excisions.3
Endoscopic ethmoidectomy may be performed as a partial (or
anterior) or total ethmoidectomy, depending on the extent of disease.
The ethmoid bulla is removed to reveal the basal lamina, the dividing
point of the anterior and posterior ethmoid sinuses. The lamina
papyracea and basal lamella should be identified, and care should be
taken to not obstruct the frontal outflow tract. If a concha bullosa (an
aerated middle turbinate) is encountered, resection of its lateral wall
may be performed to open the sinus and widen the middle meatus.
The basal lamella of the middle turbinate is entered to reach the
posterior ethmoid cells, with care taken to avoid destabilization of the
horizontal attachment during inferior dissection. The surgeon should
also recognize that the skull base typically slants inferiorly as the
ethmoid dissection continues posteriorly; therefore, the trajectory of
dissection should be more horizontal to avoid causing an iatrogenic
cerebrospinal fluid leak. In addition, overzealous superomedial dissection can risk injury to the thin bone of the lateral lamella of
cribriform plate, which is the most common site of iatrogenic cerebrospinal fluid leak. Complete dissection of the ethmoid sinus includes removing all partitions with visual identification of lamina
Copyright © Oceanside Publications, Inc. All rights reserved. DO NOT COPY
For permission to copy go to https://www.oceansidepubl.com/permission.htm
S51
Table 1 Summary of common sinus surgery techniques
Location
Maxillary sinus
Ethmoid sinus
Sphenoid sinus
Frontal sinus
Septum
Inferior turbinates
Endoscopic
papyracea, superior turbinate, anterior face of the sphenoid sinus,
and the skull base.
Endoscopic sphenoidotomy may be performed via transnasal or
transethmoidal approaches. The transnasal approach is typically used
for isolated sphenoid sinus disease with favorable anatomy. The
endoscope is guided to the sphenoethmoid recess until the natural os
of the sphenoid is identified and enlarged transnasally. This approach
has the advantage of not requiring an ethmoidectomy; however, the
nasal anatomy is not always favorable for this approach and access
for postoperative debridements can be difficult. The transethmoidal
approach is performed more commonly. After a total ethmoidectomy,
the sphenoid face and superior turbinate are identified. At this point,
the sphenoid sinus may be opened either by removing the lower
portion of the superior turbinate with identification of the sphenoid
os or through the anterior face of the sphenoid sinus. Definitive
identification of the skull base, medial orbital wall, and superior
turbinate is essential to performing a safe sphenoidotomy.
Endoscopic frontal sinusotomy is the most complex procedure in
ESS. The anatomy can be highly variable and imprecise dissection can
lead to scarring and recalcitrant frontal sinus disease. Because the
frontal outflow tract drains to the middle meatus, an anterior ethmoidectomy is the first-line treatment for frontal sinus disease; however, certain patients, such as those with aspirin triad disease or a
frontal intersinus septal cell, are more likely to fail with this procedure alone. A frontal sinusotomy requires careful analysis of the
radiographic anatomy to obtain a three-dimensional understanding
of frontal recess cells before surgery. Intraoperatively, the relationship
of the frontal sinus outflow tract to the orbit, skull base, and anterior
ethmoid artery must be delineated. The agger nasi cell, if present, is
removed. Depending on the specific anatomic features, other frontal
cells may need to be opened. Endoscopic frontal sinus procedures
have been divided into Draf types, which were described by Dr.
Wolfgang Draf. Draf type 1 involves removal of anterior ethmoid cells
and uncinate process to open the frontal outflow tract and removal of
any frontal cells. Draf type 2 (standard endoscopic frontal sinusotomy) involves resection of the frontal sinus floor from the nasal
septum to the lamina papyracea to enlarge each frontal outflow tract
maximally. Draf type 3, or modified Lothrop procedure, involves all
components of a Draf type 2 procedure with additional resection of
anterior–superior nasal septum and connection of both frontal sinuses
into a single cavity.
SINUS DILATION PROCEDURES
Sinus dilation procedures using balloon technology can be used in
select patients for the maxillary, sphenoid, and frontal sinuses. This
S52
Open
Maxillary antrostomy
Maxillary antrostomy with tissue removal
Maxillary dilation procedure
Endoscopic medial maxillectomy
Ethmoidectomy
Concha bullosa resection
Sphenoidotomy
Sphenoidotomy with tissue removal
Draf 1 frontal sinusotomy
Draf 2 frontal sinusotomy
Draf 3 frontal sinusotomy (modified Lothrop procedure)
Septoplasty
Inferior turbinate outfracture
Inferior turbinate cauterization
Inferior turbinate submucosal resection
Inferior turbinate resection
Caldwell-Luc procedure
Maxillectomy
External ethmoidectomy (Lynch approach)
Frontal sinus trephination
Osteoplastic flap with obliteration
Osteoplastic flap without obliteration
procedure is not approved for dilation of the ethmoid sinuses. Studies
are currently underway to evaluate the benefits, risks, and indications
of this technique as an alternative or adjunct to ESS. A recent Cochrane review to evaluate studies comparing balloon sinuplasty and
conventional ESS found only one study that had not yet been published in a peer-reviewed journal that met the reviews inclusion
criteria. The authors concluded that there was an urgent need for
randomized controlled trials to better evaluate these modalities.4
Although efficacy studies are ongoing, the procedure so far seems to
have a good safety profile in general,5 although surgical complications have been reported.6
OPEN SINUS PROCEDURES
Open approaches to the sinuses are occasionally used as an adjunct
during complicated cases. They also may be used in acute sinusitis
with impending complications, to rapidly drain an infected sinus or
mucocele under pressure, where an endoscopic approach would be
difficult because of severe inflammation. A Caldwell-Luc approach
may be used to access to the anterior and inferior maxillary sinus. A
frontal sinus trephination may be used to access difficult-to-reach
areas of the frontal sinus. Refractory frontal sinus disease may also be
managed with osteoplastic flap with or without obliteration. Other
external techniques are rarely required unless endoscopic sinus
equipment is not available.
SEPTOPLASTY AND INFERIOR TURBINATE
REDUCTION
Concomitant nasal obstruction surgery may be indicated in patient
chronic rhinitis or deviated septum. Septoplasty is a common procedure performed in which mucoperichondrial and mucoperiosteal
flaps are raised and the portions of deviated septum are removed.
This procedure is typically performed for nasal airway obstruction
but may also be performed in conjunction with sinus surgery for
improved access. Inferior turbinate surgery is a commonly performed
procedure for nasal airway obstruction. Inferior turbinate outfracture
is a mucosa-preserving technique performed to lateralize the bony
portion of the inferior turbinate. The inferior turbinates may also be
reduced in size using techniques that may remove bone or erectile
submucosal tissue. If possible, preserving the inferior turbinate mucosa is preferred to limit the risk of empty nose syndrome in which
the lack of inferior turbinate causes a paradoxical sensation of nasal
congestion even though the appearance of the airway is extremely
wide.
May–June 2014, Vol. 28, No. 3
Copyright © Oceanside Publications, Inc. All rights reserved. DO NOT COPY
For permission to copy go to https://www.oceansidepubl.com/permission.htm
CONCLUSION
CRS is a common inflammatory disease with a ubiquitous etiologic
profile. Surgical intervention is commonly required for recalcitrant
disease, not as a cure, but as a means of permitting better penetration
of medical therapy. This article provides an overview of the techniques of sinus surgery.
• Stereotactic navigation may be beneficial when dissection is performed along the skull base and orbit, especially in cases of revision
surgery.
REFERENCES
1.
CLINICAL PEARLS
• Surgery is effective for rhinosinusitis refractory to medical therapy.
Surgery should be considered an adjunct modality used along with
medical therapy, not in place of medical therapy.
• Preoperative workup for ESS in recalcitrant rhinosinusitis includes
a thorough medical evaluation; CT scan of the sinuses; and a
discussion of the risks, benefits, alternatives, and complications of
the procedures.
• Sinonasal surgical techniques are categorized by the approach and
sinus location.
• ESS is safe and effective for the management of most cases of
rhinosinusitis. Sinus dilation techniques allow in-office dilation of
the sinuses and as an adjunct to ESS in the operating room. Open
approaches are still occasionally required.
American Journal of Rhinology & Allergy
2.
3.
4.
5.
6.
Smith TL, Batra PS, Seiden AM, and Hannley M. Evidence supporting endoscopic sinus surgery in the management of adult chronic
rhinosinusitis: A systematic review. Am J Rhinol 19:537–543, 2005.
Higgins TS, Hwang PH, Kingdom TT, et al. Systematic review of
topical vasoconstrictors in endoscopic sinus surgery. Laryngoscope
121:422–432, 2011.
Cho DY, and Hwang PH. Results of endoscopic maxillary megaantrostomy in recalcitrant maxillary sinusitis. Am J Rhinol 22:658–
662, 2008.
Ahmed J, Pal S, Hopkins C, and Jayaraj S. Functional endoscopic
balloon dilation of sinus ostia for chronic rhinosinusitis. Cochrane
Database Syst Rev 6:CD008515, 2011.
Taghi AS, Khalil SS, Mace AD, and Saleh HA. Balloon sinuplasty:
Balloon-catheter dilation of paranasal sinus ostia for chronic rhinosinusitis. Expert Rev Med Devices 6:377–382, 2009.
Tomazic PV, Stammberger H, Koele W, and Gerstenberger C. Ethmoid roof CSF-leak following frontal sinus balloon sinuplasty. Rhinology 48:247, 2010.
e
Copyright © Oceanside Publications, Inc. All rights reserved. DO NOT COPY
For permission to copy go to https://www.oceansidepubl.com/permission.htm
S53
Augmenting the nasal airway: Beyond septoplasty
Patrick Simon, M.D., and Douglas Sidle, M.D., F.A.C.S.
ABSTRACT
Background: Nasal airway obstruction is a common complaint of patients presenting to otolaryngology clinics and can be caused by a variety of anatomic
factors. A number of advances in the surgical management of nasal airway obstruction have been made over the last century. The objective of this article is
to provide descriptions of the surgical procedures used to augment specific anatomic deviations that lead to obstruction of the nasal airway.
Methods: The descriptions of surgical procedures were derived from a literature review as well as the empiric knowledge of the senior author. Preoperative
considerations of nasal anatomy, the nasal airway, and the L-strut are detailed.
Results: Functional rhinoplasty techniques are reviewed including septoplasty, extracorporeal septoplasty, spreader grafts, batten grafts, alar rim grafts,
and correction of caudal septal deviation.
Conclusion: The symptom, nasal obstruction, may arise from a number of different anatomic and physiological elements. The rhinoplasty surgeon must
consider these contributing elements and manage accordingly, to achieve optimal results.
N
asal obstruction is one of the most common complaints of patients presenting to otolaryngology clinics. A number of different anatomic factors may contribute to the subjective sensation of
decreased nasal airflow. Septoplasty is frequently performed on patients with anatomic changes of the nasal septum that impinge on the
nose’s function as an airway. Although this procedure will produce
improvement in nasal function in a majority of patients with simple
deviation, augmentation of the nasal valves must also be considered.
The salient nasal anatomy of the nasal septum, internal nasal valve
(INV), and external nasal valve (ENV) is reviewed in regard to their
contributions to the nasal airway. The common surgical approaches
to address the problems of nasal valve stenosis, nasal valve collapse,
and other anatomic distortions of the nasal airway are described.
PREOPERATIVE NASAL EVALUATION
The nasal airway serves as the primary conduit for inspired air to
reach the lower respiratory tract. The effects of ecogeographical evolution has produced significant individual variation in the size and shape of
the human nose.1 Thus, the anatomic contribution to the complaint of nasal
obstruction must be elucidated before embarking on surgical repair.
Historical information that must be ascertained includes prior surgeries,
trauma, allergic, and sinus symptoms. Furthermore, subjective questionnaires such as the Nasal Obstruction Symptom Evaluation2 should be used
to characterize the patient’s complaint. Preoperative assessment should include anterior rhinoscopy to find septal deviation, nasal endoscopy to rule
out polyposis, Cottle maneuvers and external dilators to define dynamical
valve collapse, and trials of decongestants to determine the contribution of
mucosal congestion. Care must be taken to rule out previously undiagnosed
congenital abnormalities such as choanal atresia/stenosis or pyriform aperture stenosis. Objective testing such as rhinomanometry and acoustic rhinometry may be performed to provide a numerical stratification of the
patient’s complaints and have been shown to predict postoperative patient
satisfaction.3 Based on these assessments the clinician may develop a surgical plan individualized to the patient’s anatomic disposition.
From the Northwestern Memorial Hospital, Department of Otolaryngology–Head and
Neck Surgery, Chicago, Illinois
Presented at the Northwestern University Feinberg School of Medicine, Summer Sinus
Course, July 22–23, 2011, Chicago, Illinois
The authors have no conflicts of interest to declare pertaining to this article
Address correspondence and reprint requests to Douglas Sidle, M.D., F.A.C.S., Northwestern Memorial Hospital, Department of Otolaryngology–Head and Neck Surgery,
Northwestern University Feinberg School of Medicine, 676 North St. Clair, Suite 1325,
Chicago, IL 60611
E-mail address: [email protected]
(Am J Rhinol Allergy 26, S326 –S331, 2012; doi: 10.2500/ajra.2012.26.3786)
Copyright © 2014, OceanSide Publications, Inc., U.S.A.
S54
ANATOMY OF THE NASAL AIRWAY
External Nasal Valve
The axially positioned nostrils guard the entrance to the bilateral
nasal cavities. The nostrils are bordered laterally by the ala and
medially by the columella. The nasal vestibule defines the area within
the external nasal aperture. The soft tissue envelope and the medial
footplates of the lower lateral cartilages (LLCs) support the most
caudal portion of the columella.
The alar subunit is composed of the soft tissue envelope and the
lateral crura of the LLCs. The skin of the ala and lower third of the
nose is thick, relative to the upper nose, contains an abundance of
sebaceous glands, and is intimately associated with the attached
musculature.
The bony correlate for the posterior termination of the ENV is the
pyriform aperture. The cartilaginous extent of the ENV ends at the scroll
region joining the LLC and the upper lateral cartilage (ULC; Fig. 1).
Internal Nasal Valve
The INV is the point of greatest resistance in the nasal airway. The
caudal inferior turbinate forms the inferiolateral border of the INV.
The ULC provides the lateral border of the INV as it continues
superiorly toward the septum. At its junction with the nasal septum
an angle of 10–15° is created. Medially, the INV is defined by the nasal
septum and the valve is completed at the maxillary crest and floor of
the nose.
Septum
The nasal septum is a midline structure that divides the nasal
airway into two nasal cavities. It is firmly invested by mucoperichondrium, anteriorly, and mucoperiosteum, posterior and inferiorly. Its
cartilaginous component, the quadrangular cartilage, forms its caudal-most extent, contributing to both the ENV and the INV. Additionally, the quadrangular cartilage defines the anterior nasal dorsum. Along the dorsum the ULCs are supported medially and are
separated from the septum by fibrous attachments and its mucosal
investment. At its most cephalic position the cartilage meets the
paired nasal bones. At this junction, the “keystone” region of the nose,
an area of stability essential to the support and structure of the nose,
is located.
Inferiorly, the cartilaginous septum firmly rests on the maxillary
crest and is bound by the decussating fibrous attachments at the
junction of the perichondrium with the periosteum. The inferior
septum is continued posteriorly by the vomer. Superior to the vomer,
the septum approaches the nasal bones, the floor of the frontal sinus,
May–June 2014, Vol. 28, No. 3
Copyright © Oceanside Publications, Inc. All rights reserved. DO NOT COPY
For permission to copy go to https://www.oceansidepubl.com/permission.htm
Figure 1. This series of pictures represents the nasal keystone area (solid black circle), internal nasal valve (dashed black oval), and external nasal valve (dashed
white oval).
and the anterior skull base through the perpendicular plate of the
ethmoid.
The dynamic contribution of the septum to nasal airflow is the septal
body. This poorly understood vasoerectile structure has been localized to
the region anterior to the middle turbinate, above the nasal floor, and
caudally approaching the nasal valve region.4 The septal body invests an
observed thickening at the junction of the cartilaginous and bony septum. Furthermore, this region has a rich venous sinusoidal composition, thus, indicating a compliment to the nasal turbinates in regulating airflow.5
Neurovascular Anatomy
The vascular contributions that are significant to rhinoplasty primarily arise from the facial, sphenopalatine, and ophthalmic arteries.
The facial arteries provide blood supply to the caudal nasal septum
and nasal sidewall through the superior labial and the angular arteries, respectively. The anterior ethmoid branch of the ophthalmic
artery contributes to the dorsal septum and the dorsal nasal tip
through dorsal nasal artery. The sphenopalatine artery contributes to
the septal blood supply through its posterior septal branches.
The first and second branches of the trigeminal nerve provide
sensation of temperature, pain, and changes in pressure (i.e., airflow).
The sensation of airflow is most profound at the skin-lined vestibule
where end-sensory mechanoreceptors serve to refine the tactile perception.6 Alternatively, beyond the vestibule, the nasal mucosa has a
more primitive end-sensory arrangement, where no specialization of
the nerve endings or overlying epithelium exists, and arborization of
the nerve occurs terminally.7 Previous studies have shown that anesthetizing the anterior nose results in a significant subjective sensation
of nasal obstruction when compared with anesthesia of the nasal
mucosa.8
SURGICAL MANAGEMENT
Deviated Nasal Septum and Septoplasty
Deviation of the nasal septum is a common finding in patients
seeking attention for nasal obstruction and is commonly secondary to
three etiologies: congenital, traumatic, or iatrogenic. Previous epidemiological studies have revealed that the finding of a straight septum
is present in only 42% of newborns and in adults, only 21%.9 Trauma
American Journal of Rhinology & Allergy
Figure 2. The “L-strut” is depicted here in a sagittal orientation. The
importance of preserving of at least 1cm of cartilage both caudally and
dorsally is demonstrated here. The strut maintains the articulation cephalically at the keystone, and caudally at the anterior nasal spine, while providing support along the dorsum, and at the nasal tip.
to the nose may result in a variety of bony and cartilaginous fractures
as well as dislocation of the cartilage off the maxillary crest.10 The
fractured cartilage may heal in a variety of orientations but often
results in anatomic deviation. Previous nasal surgeries may predis-
Copyright © Oceanside Publications, Inc. All rights reserved. DO NOT COPY
For permission to copy go to https://www.oceansidepubl.com/permission.htm
S55
Figure 3. (Upper left) During extracorporeal septoplasty the quandrangular cartilage is resected en bloc. (Upper right) The PDS™ Flexible Plate (Ethicon
Inc., Somerville, NJ) is fashioned into the desired reconstructed septum. (Lower left) Septal remnants are positioned onto the plate to achieve a straightened
profile along its caudal and dorsal sides. (Lower right) The reconstructed septum is secured to the maxillary crest and nasal dorsum.
Figure 4. Spreader grafts are depicted here in lateral, frontal and in cross-sectional views. The functional contribution of these grafts to the INV is best
demonstrated on the cross-sectional view.
pose the septum to deviation through asymmetrical external forces
and scar formation during healing.11
The standard approach to correction of cartilaginous septal deviation, first popularized by Killian12 and Freer,13 involves a submucous
dissection of the quadrangular cartilage and removal of the deviation
with preservation of mucoperichondrial flaps. Once the deviated
segment of the septum has been exposed bilaterally it may be
straightened through a variety of techniques. Conservatively, the
deviated cartilage may be weakened on its concave side by crosshatching with partial thickness incisions to relieve intracartilaginous
tension. Alternatively, the deviation may be submucosally resected
leaving a caudal-dorsal “L-strut” for support (Fig. 2).
Caudal Septal Deviation
The deviated caudal septum requires attention beyond the traditional septoplasty approach. These deviations are important on both
S56
the esthetic and the functional levels. The caudal septum, if significantly deviated, may be noticeable on both frontal and lateral views
of the face given its relationship to the lobule and columella. Furthermore, the septum contributes to both the ENV and the INV, and the
caudal septum provides that contribution. Finally, the caudal septum
provides essential structure to the nose and without an appropriate
⬃1–2 cm of caudal strut significant deformities such as saddle nose
and tip ptosis may develop.
Correction of caudal septal deviation has been approached in a
number of different ways, depending on the nature of the deformity.
In the situation where the caudal septum has excessive vertical length
and is positioned lateral to the anterior nasal spine, the swinging door
technique, a technique first popularized by Metzenbaum,14 can be
used. A complete transfixion incision may be necessary to raise bilateral mucoperichondrial flaps and expose the caudal septum from the
anterior septal angle to the anterior nasal spine. Sharp incision may be
May–June 2014, Vol. 28, No. 3
Copyright © Oceanside Publications, Inc. All rights reserved. DO NOT COPY
For permission to copy go to https://www.oceansidepubl.com/permission.htm
necessary to maintain a continuous flap through the dense decussating fibers. The redundant cartilage is resected, leaving the caudal
septum only attached superiorly. The now freed inferior portion of
the caudal septum is anchored to the anterior nasal spine with sutures.
Pastorek15 proposed a modification of the swinging door technique.
The deviated caudal septum may be transposed over the anterior
nasal spine to the nasal cavity opposite the deviation without further
resection of cartilage; this appropriately named “doorstop” technique, prevents the cartilage from returning to its original position.
Sedwick et al. found resolution of subjective nasal obstruction in
51/62 of his patients with caudal deviations treated with the aforementioned techniques.16
Mild to moderate deviations may be dealt with in a manner similar
to that previously described. Likewise, the deviated portion of the
septum may be scored or morselized on the concave side to weaken
the cartilage. The limitation of this technique is the propensity of the
deviations to recur over time, this may be improved with the placement of Mustarde-type sutures through the deviation.17 Alternatively,
batten grafting may be applied to the weakened caudal septum. These
grafts are typically harvested from the posterior quadrangular cartilage or the perpendicular plate of the ethmoid. The batten grafts are
then secured along the weakened cartilage to support and stabilize its
corrected position. Extension of longer spreader grafts from the ULC
onto the caudal septum may also be used to stabilize the cartilage. The
main point of criticism of the use of the grafting techniques is the
tendency of the overlapping grafts to widen the caudal septum and
subsequently narrow the INV and ENV; thus, these grafts must be
adequately thinned before securing to the septum.18
Kridel19 popularized the tongue-in-groove technique for the management of caudal septal deviation. This technique requires the
cephaloposterior advancement of the medial crura of the LLCs onto
the caudal septum. The medial crura are then secured to the caudal
septum providing enhanced stability and correction of the deviation.
In Kridel et al.’s series of 108 patients with caudal septal deviation
good functional outcomes were noted.19 The main criticism of this
technique, again, is widening of the columella.
Extracorporeal Septoplasty
More severe deviations or loss of significant portions of the septum
necessitate reconstruction through extracorporeal septoplasty. The
execution of this procedure requires en bloc removal of the residual
cartilaginous and bony septum for extracorporeal reshaping, followed by reinsertion in a straightened dorsal and caudal septal configuration. Gubish,20 who performed the procedure ⬎2000 times,
found the open approach superior to the endonasal approach for the
improved visualization it provided for dissection and replantation.
Subperichondrial dissection was performed to expose the cartilaginous and bony septum. The ULCs are sharply incised extramucosally
from their junction with the dorsal septum and dissected laterally.
The dorsal septum is then freed from the aforementioned “keystone”
area where there is essential attachment of the dorsal septal cartilage
to the nasal bones and perpendicular plate of the ethmoid. An inferiorly based osteotomy may be necessary to separate the caudal
septum from the anterior nasal spine and maxillary crest.
Once the septum has been freed from its bony attachments, it is
removed and its structure is examined. The reconstructed septum must
contain straight sections caudally and through the dorsum to recreate the
L-strut (Fig. 2). This may be achieved by rotation of the septal specimen
to reorient a straight posterior septum or by weakening deviated cartilage through scoring techniques previously described.
Most21 has described a modification of this technique that preserves
the dorsal septum at the keystone area, which minimizes the destabilization of the nose and irregularities of the dorsum. The current
authors prefer to maintain a 1.5- to 2-cm keel of intact dorsal septal
cartilages attached to the nasal bones and ethmoid plate to aid in
repositioning of the reconstructed cartilaginous septum.
American Journal of Rhinology & Allergy
Figure 5. Alar batten grafts are secured in an underlay fashion. The grafts
extend toward the pyriform aperture, beyond the lateral crus, for enhanced
stability.
In circumstances where only weakened or crooked cartilage segments remain the septal plate may be supported with grafts from the
bony septum, auricular or costal cartilage grafts. Alternatively, a
polydioxanone, PDS Flexible Plate (Ethicon, Inc., Somerville, NJ) may
be used to augment reconstructed septum18,20(Fig. 3). Boenisch,22,23 in
a series of 369 extracorporeal septoplasty patients, in which the PDS
foil was used, reported no short- or long-term complications such as
rejection, infection, or necrosis.
Moreover, residual cartilage fragments are often sutured along the
reconstructed dorsal septum, as spreader grafts, to widen the INV at
the ULC. The reconstructed septal plate is then secured in place to the
columella/medial crura, ULC, keystone area, and maxillary crest
with nonresorbable sutures.18,20,24,25 In 404 patients undergoing extracorporeal septoplasty, Gubish found that 96% of his patients reported
improvement in their nasal breathing. The most common postoperative complaint was irregularity of the dorsum, seen in 8% of patient.20
Surgical Management of the INV
Obstruction of nasal airflow through the INV is usually a result of
changes in its major components: the dorsal septum, the ULC, or the
caudal inferior turbinate. There are a variety of procedures available
to manage obstruction caused by inferior turbinate hypertrophy and
they are beyond the scope of this article. Obstruction at the level of the
INV caused by dorsal septal deviation is typically resolved through
the procedures previously described such as submucosal resection, as
well as extracorporeal septoplasty. Once these two key areas are
properly addressed, the contribution of the ULC may become the
focus of the surgeon.
Weakness or absence of the ULC typically arises in the patient
who has previously undergone septorhinoplasty. An obvious sign
of insufficiency of the ULC is the inverted-V deformity where the
caudal end of the nasal bones is visible and creates a discontinuity
with the middle nasal vault. This defect is created when resection
of the broad dorsal septum allows the ULC to collapse medially,
thus narrowing the middle nasal vault and INV to less than its
normal 10–15°.
The common surgical management of this situation is accomplished through the use of spreader grafts, first described by Sheen.26
The spreader grafts are matchstick-shaped pieces of autologous cartilage typically harvested from septal or costal cartilage. They will sit
in an extramucosal pocket between the caudal septum and ULC and
are secured in place with nonresorbable sutures (Fig. 4). These grafts
serve to broaden the dorsal septum and widen the angle between the
septum and ULC, thus enlarging the INV.
Copyright © Oceanside Publications, Inc. All rights reserved. DO NOT COPY
For permission to copy go to https://www.oceansidepubl.com/permission.htm
S57
Figure 6. Alar rim grafts are depicted here in lateral, frontal, and basal views. They are inserted through the marginal incisions and improve the strength and
stability of the ala.
A second procedure for dealing with weakened or deficient ULC,
which are causing dynamic inspiratory collapse, is the splay or butterfly graft. In this procedure, described by Clark and Cook,27 cartilage grafting material is typically harvested from conchal cartilage.
The graft is placed over the septal dorsum with its V-orientation
facing caudally. The graft is secured to the lateral wings of the ULC.
The resulting splay created by the inherent strength of the cartilage
serves to stabilize the ULC from collapsing during inspiration. To
prevent pollybeak deformity during this procedure, the graft must be
adequately thinned or the dorsal septum reduced to maintain a
smooth dorsal line on profile. Similarly, flaring sutures use the same
principle as butterfly graft. Sutures are secured to the lateral/caudal
ULC, pass over the dorsal septum, and are secured to the contralateral
ULC. With tightening along the midline a flaring effect is created.28
Care must be taken to avoid the risk of a “cheese-wire” effect and
loosening.
In the situation where reduction of the bony and cartilaginous
dorsum leaves an open middle vault, Gassner29 describes a dorsal
onlay graft that simultaneously reconstructs the INV. In this procedure a posterior septal graft is harvested between ⬃8 and 9 mm wide,
with length variable depending on the amount of dorsum to be
reconstructed. The dorsal septum must be reduced along its length to
receive the graft and maintain a continuous dorsal profile. The graft
fits horizontally within the middle vault and is secured to the septum
and ULC with sutures. The ULCs are sutured to the undersurface of
the onlay graft, thus laterally rotating the graft and maintaining a
wider, more natural, septal-ULC angle. Similarly, the modified Skoog
dorsal reduction proposed by Hall et al.30 resects an en bloc dorsal graft
that is reshaped extracorporeally, replanted, and secured to the ULC
and septum. These two procedures provide functional reconstruction
of the middle vault and INV while concurrently addressing irregularities of the dorsal profile.
Surgical Management of the ENV
Unlike obstruction at the level of the INV, obstruction of the ENV
and intervalve area tend not to be related to previous nasal surgery.
Weakness and instability of the ENV is often a byproduct of normal
S58
anatomic development and aging. These patients will present with
nasal morphology such as narrow nostrils, recurvature of the lateral
crus of the LLC, overprojected tip, and weak sidewalls that predispose the patient to obstruction.31 Toriumi noted that cephalic orientation of the lateral crus will decrease the support of the ala during
inspiration and allow for collapse.32 Based on these preoperative
findings, augmentation of the support of the lateral crura and ala are
the primary interventions to correct ENV dysfunction.
Lateral crural strut grafts are cartilaginous grafts that are sutured
along the lateral crura of the LLC to improve the strength of the
cartilage. The indications for their use are weak or deformed lateral
crura. They typically will extend laterally out over the pyriform
aperture for enhanced support. They are secured either over (subcutaneous) or under (submucosal) the length of the lateral crus by
nonresorbable suture. In some cases, they may be used to completely
replace a deficient or missing lateral crus of the LLC.
Alar batten grafts have been the staple of augmentation of the ENV.
They are autologous cartilage grafts typically taken from septal or
conchal cartilage. Depending on the point of maximal collapse on
preoperative dynamic testing, the batten grafts may be placed at
different points along the nasal sidewall. Weakness may be seen at the
level of the lateral crus, the intervalve area, or at the caudal end of the
ULC. The batten grafts are placed in subcutaneous or submucosal
pockets that are tailored to the exact dimensions of the graft, and the
dissection is carried laterally toward the pyriform aperture. If precise
pockets are created, the grafts may be simply placed and do not
require suturing. In his review, Toriumi32 noted that subcutaneous
grafts may add fullness to the supraalar region but was acceptable by
his patients. Alternatively, Kenyon advocated a submucosal positioning of the graft for cosmesis as well as the empiric mechanical advantage of an underlay graft in improving the support and stability of
the ala (Fig. 5).31 Using these techniques in 80 patients, Cervelli et al.
described a 90% functional postoperative score of excellent.21,33
Finally, alar rim grafts are an additional measure are used during
rhinoplasty to augment the ENV. They are often used as adjuncts to
the previously mentioned surgical techniques.34 Common indications
include alar retraction, dynamic collapse of the ala during inspiration,
May–June 2014, Vol. 28, No. 3
Copyright © Oceanside Publications, Inc. All rights reserved. DO NOT COPY
For permission to copy go to https://www.oceansidepubl.com/permission.htm
or alar contour deformity. The grafts are fashioned from quadrangular or costal cartilage, and suggested dimensions are a 2-to 3-mm
width and 15- to 25-mm length.35 They are placed within a pocket
created along the alar margin. If the pocket is formed to the correct
dimension, they may be simply placed and the incision closed (Fig. 6).
The tip of the rim graft is thinned to prevent contour deformity in the
area of the soft tissue triangle. Alternatively, a resorbable stitch may
be placed around the graft to prevent migration.
CONCLUSION
Nasal obstruction is a common complaint and will present in a
number of anatomic variations. Performing submucous resection septoplasty in patients with concomitant nasal valve obstruction will
often result in dissatisfied patients who are now without the convenient abundance of autologous quadrangular cartilage. The use and
indications for open versus endonasal approaches must be thoroughly reviewed preoperatively so that the operation undertaken is
appropriate for the intended outcomes.36 It is imperative that the
rhinological surgeon considers the multiple contributing elements
that lead to anatomic obstruction and is knowledgeable of the complete armamentarium of techniques used to functionally improve the
nasal airway.
ACKNOWLEDGMENTS
The authors thank Shelia Macomber for her work on the illustrations used in this article.
REFERENCES
1.
2.
3.
4.
5.
6.
7.
8.
9.
10.
11.
Churchill SE, Shackelford LL, Georgi JN, and Black MT. Morphological variation and airflow dynamics in the human nose. Am J Hum
Biol 16:625–638, 2004.
Stewart MG, Witsell DL, Smith TL, et al. Development and validation
of the Nasal Obstruction Symptom Evaluation (NOSE) scale. Otolaryngol Head Neck Surg 130:157–163, 2004.
Pirila T, and Tikanto J. Acoustic rhinometry and rhinomanometry in
the preoperative screening of septal surgery patients. Am J Rhinol
Allergy 23:605–609, 2009.
Cole P. The four components of the nasal valve. Am J Rhinol 17:107–
110, 2003.
Costa DJ, Sanford T, Janney C, et al. Radiographic and anatomic
characterization of the nasal septal swell body. Arch Otolaryngol
Head Neck Surg 136:1107–1110, 2010.
Wrobel BB, and Leopold DA. Olfactory and sensory attributes of the
nose. Otolaryngol Clin North Am 38:1163–1170, 2005.
Cauna N, Hinderer KH, and Wentges RT. Sensory receptor organs of
the human nasal respiratory mucosa. Am J Anat 124:187–209, 1969.
Jones AS, Wight RG, Crosher R, and Durham LH. Nasal sensation of
airflow following blockade of the nasal trigeminal afferents. Clin
Otolaryngol Allied Sci 14:285–289, 1989.
Gray LP. Deviated nasal septum. Incidence and etiology. Ann Otol
Rhinol Laryngol Suppl 87:3–20, 1978.
Lee M, Inman J, Callahan S, and Ducic Y. Fracture patterns of the
nasal septum. Otolaryngol Head Neck Surg 143:784–788, 2010.
Yeo NK, and Jang YJ. Rhinoplasty to correct nasal deformities in
postseptoplasty patients. Am J Rhinol Allergy 23:540–545, 2009.
American Journal of Rhinology & Allergy
12.
Killian G. Die submukose Fensterresektion der Nasenscheiderwand.
Arch Laryngol Rhinol (Berl) 16:326, 1904.
13. Freer O. The correction of deflections of the nasal septum with
minimum of traumatism. JAMA 38:636–692, 1902.
14. Metzenbaum M. Replacement of the lower end of the dislocated
septal cartilage vs. submucous resection of the dislocated end of the
septal cartilage. Arch Otolaryngol 1929:282–292.
15. Pastorek NJ, and Becker DG. Treating the caudal septal deflection.
Arch Facial Plast Surg 2:217–220, 2000.
16. Sedwick JD, Lopez AB, Gajewski BJ, and Simons RL. Caudal septoplasty for treatment of septal deviation: Aesthetic and functional
correction of the nasal base. Arch Facial Plast Surg 7:158–162, 2005.
17. Ellis MS. Suture technique for caudal septal deviations. Laryngoscope 90:1510–1512, 1980.
18. Haack J, and Papel ID. Caudal septal deviation. Otolaryngol Clin
North Am 42:427–436, 2009.
19. Kridel RW, Scott BA, and Foda HM. The tongue-in-groove technique
in septorhinoplasty. A 10-year experience. Arch Facial Plast Surg
1:246–256, 1999.
20. Gubisch W. Twenty-five years experience with extracorporeal septoplasty. Facial Plast Surg 22:230–239, 2006.
21. Most SP. Anterior septal reconstruction: Outcomes after a modified extracorporeal septoplasty technique. Arch Facial Plast Surg 8:202–207, 2006.
22. Boenisch M, and Nolst Trenite GJ. Reconstruction of the nasal septum
using polydioxanone plate. Arch Facial Plast Surg 12:4–10, 2010.
23. Boenisch M, and Nolst Trenite GJ. Reconstructive septal surgery.
Facial Plast Surg 22:249–254, 2006.
24. Heppt W, and Gubisch W. Septal surgery in rhinoplasty. Facial Plast
Surg 27:167–178, 2011.
25. Gubisch W. Extracorporeal septoplasty for the markedly deviated
septum. Arch Facial Plast Surg 7:218–226, 2005.
26. Sheen JH. Spreader graft: A method of reconstructing the roof of the
middle nasal vault following rhinoplasty. Plast Reconstr Surg 73:230–
239, 1984.
27. Clark JM, and Cook TA. The “butterfly” graft in functional secondary
rhinoplasty. Laryngoscope 112:1917–1925, 2002.
28. Park SS. The flaring suture to augment the repair of the dysfunctional
nasal valve. Plast Reconstr Surg 101:1120–1122, 1998.
29. Gassner HG, Friedman O, Sherris DA, and Kern EB. An alternative
method of middle vault reconstruction. Arch Facial Plast Surg 8:432–
435, 2006.
30. Hall JA, Peters MD, and Hilger PA. Modification of the Skoog dorsal
reduction for preservation of the middle nasal vault. Arch Facial Plast
Surg 6:105–110, 2004.
31. Ballert JA, and Park SS. Functional considerations in revision rhinoplasty. Facial Plast Surg 24:348–357, 2008.
32. Toriumi DM, Josen J, Weinberger M, and Tardy ME Jr. Use of alar
batten grafts for correction of nasal valve collapse. Arch Otolaryngol
Head Neck Surg 123:802–808, 1997.
33. Cervelli V, Spallone D, Bottini JD, et al. Alar batten cartilage graft:
Treatment of internal and external nasal valve collapse. Aesthetic
Plast Surg 33:625–634, 2009.
34. Apaydin F. Nasal valve surgery. Facial Plast Surg 27:179–191, 2011.
35. Boahene KD, and Hilger PA. Alar rim grafting in rhinoplasty: Indications, technique, and outcomes. Arch Facial Plast Surg 11:285–289,
2009.
36. Phillips PS, Stow N, Timperley DG, et al. Functional and cosmetic
outcomes of external approach septoplasty. Am J Rhinol Allergy
25:351–357, 2011.
e
Copyright © Oceanside Publications, Inc. All rights reserved. DO NOT COPY
For permission to copy go to https://www.oceansidepubl.com/permission.htm
S59
The role of the nose in sleep-disordered breathing
Eric K. Meen, M.D., and Rakesh K. Chandra, M.D.
ABSTRACT
Background: Sleep-disordered breathing (SDB) is a spectrum of airway collapse, ranging from primary snoring to profound obstructive sleep apnea (OSA).
Studies have shown an association between impaired nasal breathing and SDB; consequently, treatments of nasal obstruction are often used in an attempt to
improve disease severity. The authors performed a review of the literature to determine the impact of nasal obstruction and the effectiveness of nonsurgical and
surgical interventions on SDB.
Methods: Relevant literature up to 2012 on the association between nasal obstruction and SDB and effectiveness of nonsurgical and surgical treatment
of the nose in SDB were reviewed.
Results: The literature is mostly limited to uncontrolled case series in which patient groups, interventions, disease definitions, and outcome measures are
not standardized. Nasal medications, including intranasal steroids and nasal decongestants, have not been shown to improve either snoring or OSA. Nasal
dilators have no impact on OSA but may improve snoring. Surgery for nasal obstruction does not improve objective indicators of SDB but can improve
subjective elements of disease, such as snoring, sleepiness, and quality of life. Nasal surgery can facilitate continuous positive airway pressure use in cases where
nasal obstruction is the factor limiting compliance.
Conclusion: Nasal obstruction plays a modulating, but not causative, role in SDB. Nasal interventions may improve subjective aspects of snoring and OSA
but do not improve objective indicators of disease. Standardization of methods and higher evidence level studies will further clarify the benefit of nasal
interventions in the treatment of SDB.
S
leep-disordered breathing (SDB) represents a group of disorders
frequently encountered by otolaryngologists. It is characterized
as a spectrum of airway collapse, ranging from primary uncomplicated snoring at one extreme, to profound obstructive sleep apnea
(OSA) on the other. Both are common disorders whereby OSA affects
2–4% of men and 1–2% of women in middle age,1 and snoring can
affect up to 50% of middle-aged men and postmenopausal women.2
Both primary snoring and OSA are characterized by multisite obstruction, and the role of pharyngeal collapse is well established as
key to the underlying pathophysiology of the disease.3 In contrast, the
contribution of nasal obstruction to SDB and the effect of treating
nasal obstruction on SDB are less clear. Furthermore, nasal patency is
affected by a constellation of fixed and dynamic variables. Examples
of the former include the size of the piriform aperture and deviation
of the nasal septum, and the latter includes collapsibility of the upper
lateral and alar nasal cartilages (nasal valves) and inflammatory factors such as allergic rhinitis. The exact contribution of each of these
elements is subject to variation, such that the exact treatment of nasal
obstruction in the setting of SDB must be individualized. Some patients benefit from directed nasal surgery (e.g., septoplasty, inferior
turbinate reduction, and nasal valve augmentation), whereas others
require medical therapy (e.g., intranasal corticosteroids), and in practice, these treatment modalities are often used in combination. This
review will seek to define the role of the nose in the pathophysiology
of SDB and will examine current data regarding the efficacy of various medical and surgical treatments for nasal obstruction in the
overall management of SDB. Studies were acquired via a search of the
PubMed database for terms relating to nasal obstruction or treatment
thereof (nasal obstruction, rhinitis, intranasal steroids (INSs), decon-
From the Department of Otolaryngology–Head and Neck Surgery, Northwestern
University Feinberg School of Medicine, Chicago, Illinois
Presented at the North American Rhinology & Allergy Conference, February 4, 2012,
Puerto Rico
R Chandra is a consultant for Gyrus ACMI
The remaining author has no conflicts of interest to declare pertaining to this article
Address correspondence and reprint requests to Eric K. Meen, M.D., Department of
Otolaryngology, University of Manitoba, 601– 400 Tache Avenue, Winnipeg, Manitoba, Canada R2H3C3
E-mail address: [email protected]
Originally published in Am J Rhinol Allergy 27, 213–220, 2013
Copyright © 2014, OceanSide Publications, Inc., U.S.A.
S60
gestants, nasal dilators, nasal surgery, septoplasty, and turbinate
reduction) and any of the following: SDB, snoring, OSA, or continuous positive airway pressure (CPAP).
ROLE OF THE NOSE IN THE
PATHOPHYSIOLOGY OF SDB
The nose is the initial point of entry of air under normal conditions,
under which it contributes one-half to two-thirds of total airway
resistance.4 This has led to the hypothesis that nasal obstruction can
contribute to SDB. Some have likened the upper airway collapse in
OSA to a Starling resistor model, wherein the upper airway is characterized as a hollow tube, with the nose representing partial obstruction at the inlet and the pharynx representing a collapsible downstream segment. Based on this model, increased obstruction in the
nose will result in increased negative pressure in the pharynx, resulting in additional pharyngeal collapse.5 If nasal obstruction is sufficiently severe, a transition to oral breathing may occur. However,
studies have shown this transition to be physiologically disadvantageous. Under normal conditions, nasal breathing is the primary route
of airflow, responsible for ⬃92 and 96% of inhaled ventilation during
periods of wakefulness and sleep, respectively.6 These authors also
indicated that transitioning to a primarily oral airflow results in an
increase in upper airway resistance by 2.5 times.7 In these studies,
nasal airflow was measured from the fraction of inspired ventilation
that passed through the nasal portion of an oronasal mask, and upper
airway resistance was calculated from the differential pressure between the mask and a catheter tip placed in the supraglottic pharynx.
Anatomic factors that appear to be associated with increased resistance during oral breathing include increased collapsibility of the
pharyngeal lumen and posterior retraction of the tongue.8
Numerous studies have documented an association between nasal
obstruction and SDB. The Wisconsin Sleep Cohort Study, comprised
of 911 patients, showed that patients with nasal obstruction were
more likely to exhibit primary snoring and report increased daytime
sleepiness. The study failed, however, to show a statistically significant increase in the apnea hypopnea index (AHI) when symptoms of
congestion were present.9 A similar follow-up study of 1032 subjects
indicated that patients with nasal congestion every night were considerably more likely to exhibit simple and habitual snoring (odds
ratios, 3 and 3.33, respectively), and the authors concluded that nasal
obstruction was a strong independent risk factor for these conditions.
May–June 2014, Vol. 28, No. 3
Copyright © Oceanside Publications, Inc. All rights reserved. DO NOT COPY
For permission to copy go to https://www.oceansidepubl.com/permission.htm
Table 1 RCT of pharmacologic therapy for SDB
Reference
Craig et al.20
Patient Characteristics
Study Design and
Intervention
Subjective Outcomes
Objective Outcomes
Sixty-nine patients with Pooled results from three Decreased congestion, Measured only in
allergic rhinitis (OSA
double-blind, placebosleep problems, and
fluticasone study,
patients excluded)
controlled crossover
daytime
in which there was
studies; specific steroid
somnolence
no change in AHI
varied between studies
Meltzer et al.53 Thirty patients with
allergic rhinitis
Double-blind parallel
study; mometasone vs
placebo (saline)
Kiely et al.24
Ten simple snorers
(AHI, ⬍10; median,
3.0), 13 OSA patients
(AHI, ⬎ 10; median,
26.5)
Double-blind crossover
study; fluticasone vs
placebo (saline)
Kerr et al.25
Ten OSA patients; six
had significant nasal
obstruction
McLean
et al.26
Ten OSA patients, all
with significant nasal
obstruction
Clarenbach
et al.27
Twelve OSA patients,
all with nasal
obstruction
Single-blind crossover
study; topical
oxymetazoline and
internal nasal dilator
for one night vs
placebo
Single-blind crossover
design; oxymetazoline
and external nasal
dilator vs placebo
Double-blind crossover
study; xylometazoline
vs placebo
Comments
Significant negative
correlation between
reduction in
congestion and
improvement in
sleep
AHI of patients in both
groups ranged from
0 to 19.5
Decreased nasal
No change in AHI or
congestion, daytime
snoring
sleepiness, and
improved QOL
measures
Improved daytime
Decrease in AHI from Rhinitis present in all
alertness in non30.3 to 23.3 in OSA
patients (type not
OSA patients; no
group on
specified); no OSA
change in OSA
fluticasone (p ⬍
patient cured
patients; no change
0.05); no change in
in snoring
non-OSA group
Improved nasal
Small decrease in
No data on snoring
respiration and
arousals/hr; no
sleep quality
change in AHI
No change in
sleepiness
No change in sleep
quality or
sleepiness
Improvement in nasal Only one patient had
resistance, AHI,
AHI decreased to
and sleep
⬍15; no data on
architecture
snoring
Improved nasal
No data on snoring
resistance; no
improvement in
AHI
AR ⫽ allergic rhinitis; OSA ⫽ obstructive sleep apnea; AHI ⫽ apnea hypopnea index; QOL ⫽ quality of life; SDB ⫽ sleep-disordered breathing; RCT ⫽
tandomized controlled trial.
Other cross-sectional studies have also found nasal resistance to be a
strong determinant of snoring.10,11
Several series have evaluated the connection between nasal obstruction and AHI. In a prospective study of 528 subjects with snoring, Lofaso et al. showed that patients with OSA (defined in the study
by an AHI of ⬎15) had greater nasal resistance when compared with
those with an AHI of ⬍15, although the nasal obstruction was only
responsible for 2.3% of the total AHI variance.12 Virkulla et al. evaluated cephalometric factors, AHI, and nasal resistance, and showed
that increased nasal resistance was an independent predictor of increased AHI in nonobese patients.13 In a study of patients with nasal
obstruction secondary to allergic rhinitis, McNicholas et al. found a
direct association between nasal resistance and the frequency and
duration of obstructive apneas during sleep.14 Taken together, this
series of investigations supports the hypothesis that nasal obstruction
may contribute to the overall AHI.
Studies assessing the effect of induced nasal obstruction have further supported the correlation between impaired nasal breathing and
SDB. In a study on eight healthy subjects, Suratt et al. showed that
nasal packing with petroleum gauze caused a marked increase in
obstructive apneas and hypopneas during sleep.15 In another study in
healthy volunteers, Lavie et al. showed an increase in number of
apneas, nonapneic-related microarousals, and wake time within sleep
after artificially occluding the nasal passages.16 In a similar study of
healthy subjects, Zwillich et al. found that nasal occlusion caused
increased apneas, sleep arousals and awakenings, loss of deep-stage
sleep, and subjectively poorer sleep quality.17 In a more recent study
American Journal of Rhinology & Allergy
evaluating the impact of postoperative nasal packing on sleep parameters, Regli et al. noted significant worsening in the AHI in patients
with OSA and worsening in both the AHI and in the oxygen desaturation index in patients without OSA.18 Friedman et al.19 also indicated that postoperative nasal packing worsened the respiratory disturbance index, duration of snoring, and oxygen desaturation index,
but only in patients with mild OSA, and not in those with moderate/
severe OSA. The authors concluded that nasal obstruction therefore
had a greater effect on milder disease, but was not a factor as the
severity of OSA increased.
In summary, physiological and epidemiological studies support
that nasal obstruction contributes to snoring and is likely to impact
AHI in OSA, although to what extent remains unclear. Regarding the
overall effect on the pathophysiology of SDB, it is likely that nasal
obstruction functions as a disease modifier, rather than a primary
causative factor.
EFFECTIVENESS OF NONSURGICAL
TREATMENTS FOR NASAL OBSTRUCTION
ON SDB
Nasal Steroids
Several well-conducted randomized controlled trials (RCTs) have
evaluated the effect of INSs on sleep quality in allergic rhinitis, and
these are summarized in Table 1. In 2005, Craig et al. released a pooled
study of 69 patients with allergic rhinitis,20 combining the results of
Copyright © Oceanside Publications, Inc. All rights reserved. DO NOT COPY
For permission to copy go to https://www.oceansidepubl.com/permission.htm
S61
Table 2 RCTs of nasal dilators for SDB
Reference
Patient Characteristics
Study Design and
Intervention
Subjective Outcomes
Objective Outcomes
Comments
Höijer et al.28
Ten patients with snoring
or observed apneas;
none had nasal
obstruction; mean
apnea index of 18
Eighteen snorers with
UARS (defined as AHI
of ⬍15) and excessive
daytime sleepiness; no
data on nasal
complaints
Twelve snorers with
nasal obstruction and
chronic rhinitis; mean
AHI of 6
Nonblinded crossover;
Nozovent vs no
intervention
No change in daytime
somnolence
Decreased snoring events,
apnea index, and
minimum oxygen
saturation
Decreased nasal
resistance with
dilator
Double-blind crossover
study; Breathe Right
vs placebo strips
No data on change in
sleepiness
Decreased desaturation
time, improved sleep
architecture; no change
in AHI or arousal index
Double-blind crossover
study; Breathe Right
vs placebo strips
Not assessed
Eighteen snorers with
nocturnal nasal
obstruction; range of
AHI (0–26.5)
Thirty-eight OSA patients
(RDI ⬎ 20) undergoing
CPAP titration; no data
on nasal complaints
Single-blind crossover
study; Breathe Right
vs placebo strips
Not assessed
Nonblinded crossover
study; Nozovent vs
no intervention
Not assessed
Decrease in snoring
frequency not but
loudness; no change in
AHI, arousal index, or
sleep architecture
No improvement in ODI,
snoring, or sleep
architecture; AHI
increased with dilator
No change in RDI with
dilator; small decrease
in CPAP pressure with
dilator (⬍1 cm H2O)
Increased nasal
cross-sectional
area with
dilator; no
data on change
in snoring
Decrease in nasal
resistance with
dilator nearly
significant
with dilator
Increased nasal
cross-sectional
area with
dilator
Bahammam
et al.29
Pevernagie
et al.30
Djupesland
et al.32
Schönhofer
et al.31
UARS ⫽ upper airway resistance syndrome; AHI ⫽ apnea hypopnea index; OSA ⫽ obstructive sleep apnea; RDI ⫽ respiratory disturbance index; CPAP ⫽
continuous positive airway pressure; ODI ⫽ oxygen desaturation index; RCT ⫽ randomized controlled trial; SDB ⫽ sleep-disordered breathing.
three prior studies done by their group,21–23 which all used the same
methodology, differing only in which nasal steroid was used. In all of
their studies, OSA was an exclusion criterion, and the presence of
snoring was not assessed. The data, however, were illustrative of a
significant decrease in not only nasal congestion, but also in daytime
somnolence. The authors also observed improved sleep quality and
noted a significant negative correlation between the decrease in nasal
congestion and improved sleep quality. In the sole study that evaluated AHI,21 no difference was observed between steroid and placebo
arms. More recently, in a study assessing the effect of intranasal
mometasone on rhinitis-disturbed sleep, Meltzer et al. also showed an
improvement in daily peak nasal inspiratory flow, sleep quality, and
quality of life (QOL) measures. All patients in their study were
assessed with polysomnography (PSG) and had a variable AHI ranging from 0 to 19.5; no change in AHI or snoring was noted between
treatment and placebo arms. Taken together, the aforementioned
studies support the ability of INSs to improve nasal congestion and
sleep quality in patients with allergic rhinitis (so-called rhinitis-disturbed sleep) but showed no improvement in objective sleep parameters (e.g., AHI), when that was included as an outcome measure.
Although the impairment in sleep quality associated with nasal congestion due to allergic rhinitis is not technically considered SDB, these
results do highlight the effect of impaired nasal breathing on overall
sleep quality.
In the sole study evaluating the effect of INS therapy on objective
sleep measures of SDB patients, Kiely et al.24 assessed the effect of INS
on patients with either primary snoring or OSA, all of whom had
concurrent rhinitis. In patients with OSA, they found a significant
decrease in AHI (from 30.3 to 23.3; p ⬍ 0.05) and in nasal resistance;
changes in the primary snoring group were not significant, and the
degree of snoring was not significant in either group. Minor subjective improvements were seen in the snoring group, but not the OSA
group. The authors concluded that intranasal fluticasone was sometimes effective in reducing AHI in some OSA patients, and this was
S62
hypothesized as being attributable to the impact of the topical steroid
on nasal airflow. Notably, however, in none of the OSA patients did
the AHI decrease to a normal level.
Decongestants
Three RCTs have evaluated the effect of topical decongestants in
SDB—one combined with an internal nasal dilator,25 one combined
with an external nasal dilator,26 and another with decongestant
alone.27 An improvement in AHI26 was noted in only one of those
studies, and only one patient showed a decrease to ⬍15. Small improvements in sleep architecture were found in two of the studies.25,26
Improved sleep quality was only noted in the series by Kerr et al.,25
but improvements in sleepiness were not seen in any of the three
studies, and none addressed the symptoms of snoring. Although
Clarenbach et al.27 found an improvement in nasal resistance, the lack
of meaningful improvement on other subjective or objective parameters suggests that nasal decongestants in the treatment of SDB at
night are of limited usefulness.
Nasal Dilators
If the nose plays a major role in the pathophysiology of SDB, it
stands to reason that dilating its most narrow part, the internal nasal
valve, could improve snoring or OSA. Both external (Breathe Right,
GlaxoSmithKline, Philadelphia, PA) and internal (Nozovent, Scandinavian Formulas, Inc., Sellersville, PA) nasal dilators have been developed to accomplish this, and five RCTs have evaluated their effects
in the setting of SDB (Table 2). Results between the studies are not
consistent. Höijer et al.28 showed a decrease in the apnea index from
18 to 6.4 in a group of mainly mild OSA patients; no subjective
improvement in sleepiness was noted, however. In the only other
study evaluating subjective sleep quality, Bahammam et al.29 also
reported no improvement in sleepiness. Although three of the studies
indicated no change in AHI with use of a nasal dilator,29–31 Djupe-
May–June 2014, Vol. 28, No. 3
Copyright © Oceanside Publications, Inc. All rights reserved. DO NOT COPY
For permission to copy go to https://www.oceansidepubl.com/permission.htm
sland et al.32 actually found an increased AHI with the dilator when
compared with the placebo strip. In a study of OSA patients undergoing CPAP titration, Schönhofer et al.31 found a small decrease in
CPAP pressure that was not clinically significant (⬍1 cm H2O). That
study was the only one of the five to not assess nasal patency at all,
whereas the other four all showed an increase in nasal patency. Two
of the studies indicated a decrease in snoring events.28,30 In summary,
nasal dilators have been shown to improve nasal patency in patients
with SDB, and may reduce the severity of snoring, but the current
evidence does not support their use in the treatment of OSA, including as an adjunct to facilitating CPAP tolerance.
EFFECTIVENESS OF SURGICAL TREATMENT OF
NASAL OBSTRUCTION FOR SDB
The intent of nasal surgery for SDB includes improvement of nasal
obstruction, snoring, and OSA. Additionally, the primary goal may be
to improve tolerance of CPAP in patients with nasal obstruction.
Literature addressing the success of nasal surgery for treatment of
SDB is mostly limited to uncontrolled case series. Patients groups
within these studies are heterogeneous, with respect to background
details, as well as type or degree of SDB. Definitions for SDB are also
not standardized, and so inclusion criteria of “OSA” may capture
relatively different patient populations between studies. Surgical
treatments are not standardized between studies or within individual
studies, in which patients may have undergone several different
operations, or combinations thereof. Reported outcome measures are
highly variable, with differential emphasis on both subjective and
objective measures, and specific data collected within each category is
often different (e.g., nonstandardized reporting of PSG results). Definitions of surgical success also vary widely and are not evidence
based.33 Despite the inherent limitations, however, a review of the
available literature still permits some conclusions to be drawn.
To evaluate the effect of nasal surgery on SDB, 15 studies, conducted between 2002 and 2012, were assessed. Thirteen of these were
assessed in a meta-analysis conducted by Li et al. in 2011, and two
newer suitable studies were evaluated as well. Table 3 contains a
summary of these reports. Together, despite ubiquitous success in
improving nasal resistance, studies have established a consistent failure of nasal surgery to improve AHI and highly variable success in
treating most other objective sleep indices. In their meta-analysis, Li et
al. indicated that nasal surgery led to no change in the weighted AHI
(35.3–33.5; p ⫽ 0.69),34 and when considered individually, with one
exception,35 none of the studies showed a decrease in AHI. When
considering all patients who met the criteria for success in individual
studies, the pooled success rate was 16.7%. As alluded to previously,
definitions of success varied widely and included reduction of AHI
by 50% and to ⬍20,36 reduction of AHI by 50% and to ⬍15,37 respiratory disturbance index of ⬍15,35 AHI of ⬍5,38 and having at least a
50% “apnea improvement ratio.”39 Because of the variable definition
and low rate of success, predictors of surgical success still have not
been convincingly elucidated. Studies have also inconsistently shown
improvement in other objective indicators of OSA, including lowest
oxygen saturation,40,41 improved arousal index,42 improved sleep architecture and efficiency,38,41,43,44 and shortened apnea and hypopnea
duration.38,41 Despite reaching statistical significance in some of these
parameters, the changes were often small and may not have reached
the level that is clinically important. This limitation, in addition to the
inconsistency of the results, renders the overall value of nasal surgery
in addressing these objective sleep parameters unclear. In summary,
although individual patients may be surgically “cured,” the weight of
the evidence suggests that nasal surgery is ineffective in reliably
improving AHI and other objective sleep parameters of OSA.
When snoring is assessed objectively, the effect of nasal surgery is
unclear. Several studies have shown failure of nasal surgery to improve duration35,43,45 and intensity45 of snoring, and Choi et al.
showed a modest decrease in snoring duration.44 In contrast, when
American Journal of Rhinology & Allergy
snoring is assessed subjectively, patients consistently report improvement after nasal surgery. Friedman et al.46 reported an improvement
in snoring in 28% of patients and complete cessation in 6%, and
Virkkula et al. showed an improvement in subjective snoring frequency, irritation, and quality in 26, 50, and 38% of patients, respectively.43 In another study, comprised of 27 patients with snoring and
nasal obstruction due to nasal polyps, most of whom did not have
OSA, all patients reported improvement of snoring after endoscopic
sinus surgery, with improvement in 67%, and complete resolution in
the other 33%.47 Using a similar visual analog scale, Sufioğlu et al.
showed a statistically significant drop in snoring severity after nasal
surgery.38 In three studies of patients with OSA and snoring, Li et al.
showed a consistent improvement in subjective snoring as reported
by patients and their bed partners.36,48,49 In one of these investigations,
they also reported improvement in 86% of patients, and complete
snoring cessation in another 12%.49 In summary, snoring has been
shown to consistently improve subjectively but not objectively after
nasal surgery. The degree of subjective improvement is widely variable, and studies still have not revealed subgroups in which snoring
improvement is more likely.
In contrast to the consistent failure of nasal surgery to improve
objective OSA parameters, more focus has been placed recently on the
impact of nasal surgery on subjective outcomes of OSA. Of the
reviewed studies, 11 addressed the degree of sleepiness. Only two
failed to show an improvement,43,45 and a significant improvement
was found in nine.36–38,40–42,46–48 In the majority of these, sleepiness
was assessed with the Epworth Sleepiness Scale, and in most cases,
scores fell to within the normal range, indicating that the decreases
were also clinically significant. Li et al. also assessed the impact of
nasal surgery on overall QOL in OSA patients and found that nasal
surgery resulted in significant improvements in six of eight domains
of the 36-item short form health survey. Despite the failure of nasal
surgery to reliably lower the AHI and other objective indices of OSA,
subjective improvements should not be discounted. Sleepiness is
often the most troublesome symptom in OSA patients, and any reduction in it likely represents an important clinical difference to the
individual patient, who often presents seeking subjective improvement of symptoms rather than a numerical improvement in their
AHI. An additional limitation of focusing exclusively on PSG parameters is that they have shown little correlation with daytime somnolence and QOL.50
Nasal surgery appears to play a clear role in improving CPAP
tolerance in patients with concurrent nasal obstruction. CPAP tolerability remains a significant obstacle in the management of OSA.
Compliance rates of ⬍70% have been reported, and nasal obstruction
or discomfort is frequently cited as a factor in CPAP intolerance.51
Several studies have shown a positive effect of nasal surgery on CPAP
pressures and compliance. In a study of 40 OSA patients with nasal
obstruction, Friedman et al. found a significant reduction in required
CPAP pressure levels after nasal surgery.46 In a similar study group of
patients who were refractory to CPAP, Nakata et al. showed a significant decrease in CPAP pressure after nasal surgery and ultimate
device compliance in all 12 patients.40 In a randomized, double-blind,
placebo-controlled trial assessing the effect of radiofrequency treatment of inferior turbinate hypertrophy, Powell et al. observed that
patients in the treatment arm reported significant improvement in
both nasal patency and CPAP tolerance, changes that were not present in the control group.52 Taken together, the available literature
reveals that although nasal surgery does not improve SDB objectively,
it can reliably improve objective CPAP titration levels, often leading
to improved device compliance.
CONCLUSION
Physiological and epidemiological studies support that the nose
plays a modulating role in the pathophysiology of SDB but does not
act as a primary causative factor. Further investigation will likely
Copyright © Oceanside Publications, Inc. All rights reserved. DO NOT COPY
For permission to copy go to https://www.oceansidepubl.com/permission.htm
S63
S64
Copyright © Oceanside Publications, Inc. All rights reserved. DO NOT COPY
For permission to copy go to https://www.oceansidepubl.com/permission.htm
May–June 2014, Vol. 28, No. 3
Forty patients with snoring
and nasal obstruction; some
with OSA (mean AHI, 13.6)
Forty patients with snoring
and nasal obstruction; some
with OSA (AHI, 14)
Forty-nine patients with OSA
and nasal obstruction from
septal deviation (mean
AHI, 31.5 vs 30.6)
Virkkula et al.45
Virkkula et al.43
Case series; patients underwent
septo ⫾ ITR, or ESS
Forty-nine patients with OSA
and nasal obstruction
(mean AHI, 44.6)
Fifty-two patients with OSA,
snoring, and nasal
obstruction
Nakata et al.41
Li et al.49
Case series; patients underwent
septo and ITR
Case series; patients underwent
septo ⫾ ITR
Fifty-one patients with OSA
and nasal obstruction
(mean AHI, 37.4)
Case series; patients underwent
septo and/or ITR, or
septorhinoplasty
Randomized, single-blinded
control trial; septo ⫾ ITR, vs
sham surgery
Case series; patients underwent
septo and/r ITR
Case series; patients underwent
septo and/or ITR
Li et al.48
Koutsourelakis et al.37
Nakata et al.40
Kim et al.35
Case series; patients underwent
various combinations of
septo, ITR, septorhino, nasal
tip surgery, and ESS
Case series; patients underwent
septo ⫾ ITR
Case series; patients underwent
septoplasty (septo) ⫾ ITR
Fifty patients with OSA, nasal
obstruction, and snoring
(mean RDI, 31.6)
Twenty-six patients with
snoring and nasal
obstruction; some with
OSA (mean AHI, 29.2)
Twenty-one patients with
OSA, nasal obstruction, and
snoring (mean RDI, 39)
Twelve OSA patients with
nasal obstruction and
refractory to CPAP
Friedman et al.46
Verse et al.42
Study Design and
Intervention
Patient Characteristics
Reference
Table 3 Selected trials assessing effect of surgery on SDB
Improvement in snoring on
patient and bed partners
scales
Improvement in snoring on
patient and bed partners
scales; improved sleepiness;
improved QOL
Decreased sleepiness
Decrease in sleepiness with
surgery
Improved snoring intensity; no
change in sleepiness
No difference in sleepiness
Substantial improvement in
daytime sleepiness
Not addressed
Significant decrease in daytime
sleepiness
Decreased snoring; increased
daytime energy
Subjective Outcomes
Decreased nasal resistance;
improved sleep
architecture and efficiency;
small increase in lowest O2
saturation; no change in
AHI, but shortened apnea
time
Decreased nasal resistance;
no change in AHI
No change in AHI or lowest
O2 saturation
Decrease in RDI (39–29.1)
and ODI; duration of
snoring unchanged
No difference in AHI;
modest improvement in
lowest O2 saturation;
decrease in nasal
resistance
Decrease nasal resistance; no
change in snoring time or
intensity, or in AHI, ODI,
or arousals; improved
architecture
No change in snoring time;
no change in AHI, ODI, or
arousals
No change in AHI or oxygen
saturation
RDI increased (31.6–39.5); no
change in oxygen
saturation
Decrease in nasal resistance;
mean AHI unchanged;
decrease in arousal index
Objective Outcomes
Snoring improved in 86%
of patients and
eliminated in 12%;
small tonsil size
predicted greater
snoring improvement
Snoring time was the only
objective outcome of
snoring evaluated
Decrease in nasal
resistance in surgery
group; snoring not
assessed; success in four
patients (15%)
QOL improvement in six
of eight SF-36 subscales
Snoring not evaluated
subjectively
CPAP titration levels
decreased after surgery
Snoring not assessed
CPAP titration levels
decreased after surgery
Comments
American Journal of Rhinology & Allergy
Copyright © Oceanside Publications, Inc. All rights reserved. DO NOT COPY
For permission to copy go to https://www.oceansidepubl.com/permission.htm
S65
Improved sleep efficiency,
architecture, and snoring
time; no change in AHI or
arousals or minimum O2
levels
Improved apnea–hypopnea
duration and sleep
architecture; no change in
AHI, lowest O2 levels, or
arousals
Twenty-eight patients with
OSA and nasal obstruction
(mean AHI, 32.5)
Case series; patients underwent
various combinations of
septo, ITR, septorhino, and
ESS
Improved sleepiness and
snoring
Improved snoring and
sleepiness in surgical group
compared with preoperative
scores and to medical group
Not assessed
Nonrandomized, parallel study;
surgical (septoplasty and ITR)
vs medical (INS, decongestant,
saline lavage, or oral
antihistamines)
Case series; patients underwent
combination of
septo ⫾ ITR ⫾ ESS
Not assessed
Case series; patients underwent
septo ⫾ ITR or ESS
Sixty-six patients OSA,
snoring, and nasal
obstruction (mean AHI, 38
in surgical group and 25.9
in medical group)
Twenty-two patients with
OSA and nasal obstruction
(mean AHI, 40.4)
Improved nasal resistance;
no change in AHI, lowest
O2 saturation, arousals, or
sleep architecture
Preoperative and
postoperative AHI and
nasal resistance not
compared; decrease in
AHI of responders vs
nonresponders; no other
data reported
No change in AHI,
minimum O2 saturation, or
sleep architecture
Improvement in sleepiness and
snoring
Case series; patients underwent
ESS
Objective Outcomes
Twenty-seven patients with
nasal obstruction due to
nasal polyps and snoring
(mean AHI, 6.7)
Thirty-five patients with OSA
and nasal obstruction; AHI
reported for responders
(43.7) and nonresponders
(43.4)
Subjective Outcomes
Study Design and
Intervention
Patient Characteristics
Decrease in CPAP titration
levels, but not significant
(p ⫽ 0.062)
Subjective snoring not
assessed
Eight (23%) responders
(mean AHI decreased to
18.5); 27 nonresponders
(mean AHI unchanged);
high soft palate and
wide retroglossal space
predicted success
Seven (16%) patients
classified as meeting
“success”
All patients reported
improved snoring
postoperatively
Comments
UARS ⫽ upper airway resistance syndrome; AHI ⫽ apnea hypopnea index; OSA ⫽ obstructive sleep apnea; RDI ⫽ respiratory disturbance index; CPAP ⫽ continuous positive airway pressure; ITR ⫽
inferior turbinate reduction; ESS ⫽ endoscopic sinus surgery; INS ⫽ intranasal steroids; ODI ⫽ oxygen desaturation index; QOL ⫽ quality of life; SDB ⫽ sleep-disordered breathing.
Sufioğlu et al.38
Choi et al.44
Li et al.36
Morinaga et al.39
Tosun et al.47
Reference
Table 3 Continued
continue to show the variable, idiosyncratic relationship between
nasal airflow and the spectrum of SDB, although it may clarify
subgroups of SDB patients in whom nasal obstruction plays a more
prominent role. Treatment of nasal obstruction can improve elements
of the disease, in addition to improving baseline nasal obstruction.
Specifically:
• Nasal steroids can improve subjective sleep quality and sleepiness
in rhinitis patients, but their ability to successfully treat snoring and
OSA has not been established.
• Nasal decongestants do not effectively treat OSA. Their efficacy in
the management of snoring has not been assessed.
• Nasal dilators may provide small improvements in snoring but are
not effective in the treatment of OSA.
• Nasal surgery has not been convincingly shown to improve snoring
objectively, but the majority of studies show a subjective improvement. The odds of snoring improvement and complete cessation
vary widely, and the literature currently does not allow accurate
prediction of which patients will respond.
• Nasal surgery is very unlikely to improve objective parameters of
OSA, particularly AHI. Although individual patients may achieve
successful results depending on the outcome measures considered,
cure of OSA is the exception, not the rule. Conversely, ample
evidence supports subjective improvement of OSA patients. This
contradiction suggests that although nasal surgery can be offered to
improve symptoms of nasal obstruction, sleepiness, and QOL, nasal
surgery is not effective as a primary treatment for OSA. Nasal
surgery nonetheless appears to reliably augment CPAP compliance
when nasal patency is the limiting issue.
Efforts to conduct higher evidence level studies and to standardize
disease definitions, patient groups, surgical interventions, and outcome measures will likely further clarify the role of nasal interventions in the overall treatment of SDB in the future.
REFERENCES
1.
2.
3.
4.
5.
6.
7.
8.
9.
10.
11.
12.
13.
S66
Stradling J. Obstructive sleep apnoea: Definitions, epidemiology, and
natural history. Thorax 50:683–689, 1995.
Young T, Paltq M, Dempsey J, et al. The occurrence of sleep-disordered breathing among middle-aged adults. . N Engl J Med 328:1230–
1235, 1993.
Quinn S, Huang L, Ellis P, et al. The differentiation of snoring
mechanisms using sound analysis. Clin Otolaryngol 21:119–123,
1996.
Ferris B, Mead J, and Opie L. Partitioning of respiratory flow resistance in man. J Appl Physiol 19:653–658, 1964.
Park S. Flow-regulatory function of upper airway in health and
disease: A unified pathogenetic view of sleep-disordered breathing.
Lung 64:311–333, 1993.
Fitzpatrick M, Driver H, Chatha N, et al. Partitioning of inhaled
ventilation between the nasal and oral routes during sleep in normal
subjects. J Appl Physiol 94:883–890, 2003.
Fitzpatrick M, McLean H, Urton A, et al. Effect of nasal or oral
breathing route on upper airway resistance during sleep. Eur Respir
J 22:827–832, 2003.
Meurice J, Marc I, Carrier G, et al. Effects of mouth opening on upper
airway collapsibility in normal sleeping subjects. Am J Respir Crit
Care 153:255–259, 1996.
Young T, Finn L, and Kim H. Nasal obstruction as a risk factor for
sleep-disordered breathing. J Allergy Clin Immunol 99:S757–S762,
1997.
Metes A, Ohki M, Cole P, et al. Snoring, apnea and nasal resistance in
men and women. J Otolaryngol 20:57–61, 1991.
Virkkula P, Bachour A, Hytönen M, et al. Patient- and bed partnerreported symptoms, smoking, and nasal resistance in sleep-disordered breathing. Chest 128:2176–2182, 2005.
Lofaso F, Coste A, d’Ortho M, et al. Nasal obstruction as a risk factor
for sleep apnoea syndrome. Eur Respir J 16:639–643, 2000.
Virkkula P, Hurmerinta K, Loytonen M, et al. Postural cephalometric
analysis and nasal resistance in sleep-disordered breathing. Laryngoscope 113:1166–1174, 2003.
14.
15.
16.
17.
18.
19.
20.
21.
22.
23.
24.
25.
26.
27.
28.
29.
30.
31.
32.
33.
34.
35.
36.
37.
McNicholas W, Tarlo S, Cole P, et al. Obstructive apneas during sleep
in patients with seasonal allergic rhinitis. Am Rev Respir Dis 126:
625–628, 1982.
Suratt P, Turner B, and Wilhoit S. Effect of intranasal obstruction on
breathing during sleep. Chest 90:324–329, 1986.
Lavie P, Fischel N, Zomer J, and Eliaschar I. The effects of partial and
complete mechanical occlusion of the nasal passages on sleep structure and breathing in sleep. Acta Otolaryngol 95:161–166, 1983.
Zwillich C, Pickett C, Hanson F, and Weil J. Disturbed sleep and
prolonged apnea during nasal obstruction in normal men. Am Rev
Respir Dis 124:158–160, 1981.
Regli A, von Ungern-Sternberg B, Strobel W, et al. The impact of
postoperative nasal packing on sleep-disordered breathing and nocturnal oxygen saturation in patients with obstructive sleep apnea
syndrome. Anaesth Analg 102:615–620, 2006.
Friedman M, Maley A, Kelley K, et al. Impact of nasal obstruction on
obstructive sleep apnea. Otolaryngol Head Neck Surg 144:1000–1004,
2011.
Craig T, Hanks C, and Fisher L. How do topical nasal corticosteroids
improve sleep and daytime somnolence in allergic rhinitis? J Allergy
Clin Immunol 116:1264–1266, 2005.
Craig T, Mende C, Hughes K, et al. The effect of topical nasal
fluticasone on objective sleep testing and the symptoms of rhinitis,
sleep, and daytime somnolence in perennial allergic rhinitis. Allergy
Asthma Proc 24:53–58, 2003.
Craig T, Teets S, Lehman E, et al. Nasal congestion secondary to
allergic rhinitis as a cause of sleep disturbance and daytime fatigue
and the response to topical nasal corticosteroids. J Allergy Clin
Immunol 101:633–637, 1998.
Hughes K, Glass C, Ripchinski M, et al. Efficacy of the topical nasal
steroid budesonide on improving sleep and daytime somnolence in
patients with perennial allergic rhinitis. Allergy 58:380–385, 2003.
Kiely J, Nolan P, and McNicholas W. Intranasal corticosteroid therapy for obstructive sleep apnoea in patients with co-existing rhinitis.
Thorax 59:50–55, 2004.
Kerr P, Millar T, Buckle P, and Kryger M. The importance of nasal
resistance in obstructive sleep apnea syndrome. J Otolaryngol 21:189–
195, 1992.
McLean H, Urton A, Driver H, et al. Effect of treating severe nasal
obstruction on the severity of obstructive sleep apnoea. Eur Respir J
25:521–527, 2005.
Clarenbach C, Kohler M, Senn O, et al. Does nasal decongestion
improve obstructive sleep apnea? J Sleep Res 17:444–449, 2008.
Höijer U, Ejnell H, Hedner J, et al. The effects of nasal dilation on
snoring and obstructive sleep apnea. Arch Otolaryngol Head Neck
Surg 1992:281–284, 1992.
Bahammam A, Tate R, Manfreda J, and Kryger M. Upper airway
resistance syndrome: Effect of nasal dilation, sleep stage, and sleep
position. Sleep 22:592–598, 1999.
Pevernagie D, Hamans E, Van Cauwenberge P, and Pauwels R.
External nasal dilation reduces snoring in chronic rhinitis patients: A
randomized controlled trial. Eur Respir J 15:996–1000, 2000.
Schönhofer B, Kerl J, Suchi S, et al. Effect of nasal valve dilation on
effective CPAP level in obstructive sleep apnea. Respir Med 97:1001–
1005, 2003.
Djupesland P, Skatvedt O, and Borgersen A. Dichotomous physiological effects of nocturnal external nasal dilation in heavy snorers:
the answer to a rhinologic controversy? Am J Rhinol 15:95–103, 2001.
Kezirian E, Weaver E, Criswell M, et al. Reporting results of obstructive sleep apnea syndrome surgery trials. Otolaryngol Head Neck
Surg 144:496–499, 2011.
Li H, Wang P, Chen Y, et al. Critical appraisal and meta-analysis of
nasal surgery for obstructive sleep apnea. Am J Rhinol Allergy 25:
45–49, 2011.
Kim S, Choi J, Jeon H, et al. Polysomnographic effects of nasal
surgery for snoring and obstructive sleep apnea. Acta Otolaryngol
124:297–300, 2004.
Li H, Lee L, Wang P, et al. Can nasal surgery improve obstructive
sleep apnea: Subjective or objective? Am J Rhinol Allergy 23:e51–e55,
2009.
Koutsourelakis I, Georgoulopoulos G, Perraki E, et al. Randomized
trial of nasal surgery for fixed nasal obstruction in obstructive sleep
apnoea. Eur Respir J 31:110–117, 2008.
May–June 2014, Vol. 28, No. 3
Copyright © Oceanside Publications, Inc. All rights reserved. DO NOT COPY
For permission to copy go to https://www.oceansidepubl.com/permission.htm
38.
Sufioğlu M, Ozmen O, Kasapoglu F, et al. The efficacy of nasal
surgery in obstructive sleep apnea syndrome: A prospective clinical
study. Eur Arch Otorhinolaryngol 269:487–494, 2012.
39. Morinaga M, Nakata S, Yasuma F, et al. Pharyngeal morphology: A
determinant of successful nasal surgery for sleep apnea. Laryngoscope 119:1011–1016, 2009.
40. Nakata S, Noda A, Yagi H, et al. Nasal resistance for determinant
factor of nasal surgery in CPAP failure patients with obstructive
sleep apnea syndrome. Rhinology 44:296–299, 2005.
41. Nakata S, Noda A, Yasuma F, et al. Effects of nasal surgery on sleep
quality in obstructive sleep apnea syndrome with nasal obstruction.
Am J Rhinol 22:59–63, 2008.
42. Verse T, Maurer J, and Pirsig W. Effect of nasal surgery on sleeprelated breathing disorders. Laryngoscope 112:64–68, 2002.
43. Virkkula P, Hytönen M, Bachour A, et al. Smoking and improvement
after nasal surgery in snoring men. Am J Rhinol 21:169–173, 2007.
44. Choi J, Kim E, Kim Y, et al. Effectiveness of nasal surgery alone on
sleep quality, architecture, position, and sleep-disordered breathing
in obstructive sleep apnea syndrome with nasal obstruction. Am J
Rhinol Allergy 25:338–341, 2011.
45. Virkkula P, Bachour A, Hytönen M, et al. Snoring is not relieved by
nasal surgery despite improvement in nasal resistance. Chest 129:81–
87, 2006.
46. Friedman M, Tanyeri H, Lim J, et al. Effect of improved nasal breathing on
obstructive sleep apnea. Otolaryngol Head Neck Surg 122:71–74, 2000.
American Journal of Rhinology & Allergy
47.
Tosun F, Kemikli K, Yetkin S, et al. Impact of endoscopic sinus
surgery on sleep quality in patients with chronic nasal obstruction
due to nasal polyposis. J Craniofac Surg 20:446–449, 2009.
48. Li H, Lin Y, Chen N, et al. Improvement in quality of life after
nasal surgery alone for patients with obstructive sleep apnea and
nasal obstruction. Arch Otolaryngol Head Neck Surg 134:429–433,
2008.
49. Li H, Lee L, Wang P, et al. Nasal surgery for snoring in patients with
obstructive sleep apnea. Laryngoscope 118:354–359, 2008.
50. Weaver E, Tucker Woodson B, and Steward D. Polysomnography
indexes are discordant with quality of life, symptoms, and reaction
times in sleep apnea patients. Otolaryngol Head Neck Surg 132:255–
262, 2005.
51. Hoffstein V, Viner S, Mateika S, and Conway J. Treatment of obstructive sleep apnea with nasal continuous positive airway pressure.
Patient compliance, perception of benefits, and side effects. Am Rev
Respir Dis 145:841–845, 1992.
52. Powell N, Zonato A, Weaver E, et al. Radiofrequency treatment of
turbinate hypertrophy in subjects using continuous positive airway
pressure: A randomized, double-blind, placebo-controlled clinical
pilot trial. Laryngoscope 111:1783–1790, 2001.
53. Meltzer E, Munafo D, Chung W, et al. Intranasal mometasone furoate
therapy for allergic rhinitis symptoms and rhinitis-disturbed sleep.
Ann Allergy Asthma Immunol 105:65–74, 2010.
e
Copyright © Oceanside Publications, Inc. All rights reserved. DO NOT COPY
For permission to copy go to https://www.oceansidepubl.com/permission.htm
S67
Olfactory disorders
Alan Gaines, M.D.
ABSTRACT
Decreased sense of smell can lead to significant impairment of quality of life, including taste disturbance and loss of pleasure from eating with resulting
changes in weight, and difficulty in avoiding health risks such as spoiled food or leaking natural gas. Recent epidemiological reports have shown that despite
fairly low self-reported prevalence of these disorders in large population studies, when validated smell identification or threshold tests are used they reveal quite
a high prevalence of hyposmia and anosmia in certain groups, especially the elderly. Several different pathophysiological processes, such as head trauma, aging,
autoimmunity, and toxic exposures, can contribute to smell impairment, with distinct implications concerning prognosis and possible treatment. Otolaryngologists are most likely to see this symptom in patients with chronic rhinosinusitis, and this now appears to be caused more by the mucosal inflammation
than by physical airway obstruction.
A
lthough standardized, validated tests of vision and hearing are
long-accepted tools, the use of validated tests for patients with
possible smell disorders has lagged behind. In part, this could be
because of the lack of a “quick fix” to ameliorate a deficit if one is
found (such as eyeglasses or hearing aids). However, there are several
tests available to diagnose and quantify a patient’s sense of smell as
discussed in detail in a recent article.1 The University of Pennsylvania
Smell Identification Test, a 40-item “scratch and sniff” test with the
patient forced to identify each odor as one of four available responses
for each item, is common in U.S. studies. Another commonly used
approach is to determine the detection threshold of a specific odor,
given increasingly diluted vials to sniff according to specific protocols
to see the lowest concentration that the patient can reliably detect.
However, this process requires some technician training and takes
somewhat longer. Subjective questions or visual analog scales are less
satisfactory.
EPIDEMIOLOGY
Several studies have shown that there are gender-related differences in smell identification, with most studies showing women as
having superior ability to detect, identify, and discriminate smells.1,2
These gender differences seem to be especially prominent in the
young and the elderly subjects.2
Although it is not always a presenting complaint, many studies
show the incidence of decreased olfactory function increasing very
significantly with age, to 50% or more of those ⬎65 years of age.1,2
Interestingly, a significant disconnect between self-reported problems
and measured dysfunction has been reported,3 which highlights the
need for awareness of smell impairment in the elderly even if not
reported by the patient. At least a portion of this age-related loss of
smell function can be attributed to the association of anosmia and
hyposmia with several neurodegenerative diseases, including Parkinson’s disease and multiple sclerosis.1,3,4
From the Department of Medicine, Warren Alpert Medical School of Brown University,
Providence, Rhode Island
The author has no conflicts of interest to declare pertaining to this article
Address correspondence and reprint requests to Alan Gaines, M.D., Allergy and
Asthma Center, 95 Pitman Street, Providence, RI 02906
E-mail address: [email protected]
Originally published in Am J Rhinol Allergy 27, S45–S47, 2013
Copyright © 2014, OceanSide Publications, Inc., U.S.A.
S68
COMMON ETIOLOGIES/ASSOCIATIONS
Anosmia and hyposmia can be related to several different presumed etiologies.5 These include fairly rare congenital problems,
associations with neurodegenerative and autoimmune diseases,
symptoms that develop after head trauma or after exposure to a toxin,
either local or systemic, postviral disease, and, finally, symptoms
associated with sinonasal inflammation, which will be addressed in
its own section.
Congenital anosmia in its more extreme form can be found in
association with certain genetic syndromes such as Kallman syndrome,6 but there are also twin studies7 and other research indicating
more subtle genetic influences on smell discrimination, and there will
likely be more work forthcoming in this area in the near future
because large-scale population studies screening hundreds of different gene polymorphisms are becoming more common.
An association has long been recognized between smell dysfunction and neurodegenerative and psychiatric diseases, and there are
indications that these may be connected in part through autoimmune
mechanisms.1,4,8 Odor information gathered from the olfactory bulb is
normally transmitted both to the cortex9 and to the limbic system,
where it appears pleasant or unpleasant odors trigger neurochemical
changes in different areas of the amygdala. Olfactory disorders are
among the earliest signs of Parkinson’s disease, as well as Alzheimer’s
disease, multiple sclerosis, and even schizophrenia and depression,
and cortex and amygdala are frequently involved in these neuropsychiatric diseases.
Smell dysfunction related to head trauma can be present in as
many as 15–30% of cases1,10 and has long been thought to be
associated with stretching or shearing of the olfactory nerves in the
course of a sudden head contusion.11 According to this theory, the
observed findings of decreased volume of the olfactory bulb in
these patients may be caused by decreased sensory input.12 However, a recent report of delayed anosmia several weeks after
trauma and magnetic resonance imaging (MRI) showing scarring
in the region of the olfactory bulb indicate that direct brain trauma
could also be a relevant factor in some cases.11 A role for direct
brain injury is also supported by many studies showing abnormalities on MRI in frontal lobes as well as olfactory bulbs, and a recent
study with single-photon emission computed tomography (CT)
even implicating hypoperfusion of parietal and temporal lobes in
patients with posttraumatic anosmia compared with patients with
similar trauma but normal sense of smell.10 Sports concussions can
also have an apparent gradual effect on olfaction as well.13 MRI of
May–June 2014, Vol. 28, No. 3
Copyright © Oceanside Publications, Inc. All rights reserved. DO NOT COPY
For permission to copy go to https://www.oceansidepubl.com/permission.htm
the brain, to rule out the aforementioned entities, should be considered in all cases where history is suggestive or where the
clinical picture is not suggestive of a specific cause (e.g., chronic
rhinosinusitis and nasal polyposis). This will also reveal intracranial lesions such as an olfactory meningioma.
Local exposures to toxins such as ammonia, gasoline, hairdressing
chemicals, and others can cause permanent smell dysfunction.14 Adverse reactions to pharmaceuticals can also cause toxic damage to the
olfactory function, both systemically15 (especially with chemotherapy) and locally, such as the cases of anosmia found to be caused by
the topical application of intranasal preparations containing zinc,
which were marketed as a treatment for upper respiratory infections.14
It is also not uncommon to have onset of anosmia/hyposmia
during a particularly severe upper respiratory infection, but with
the olfactory dysfunction persisting long after the other symptoms
have resolved.5 This postviral anosmia is also poorly responsive to
treatment, and it may be another of the factors behind the cumulative increase in smell disorders with increasing age. There is
some evidence this postviral anosmia may be centrally mediated.16
RHINOSINUSITIS AND ANOSMIA ASSOCIATED
WITH INFLAMMATION
The subgroup of patients with olfactory disorders likely to be most
relevant to the practicing allergist and rhinologist is the patient with
chronic rhinosinusitis, with or without nasal polyps, who complains
of decreased or even total loss of their sense of smell. Although it may
be tempting to assume that this may be related to airflow with the
odor-related molecules not having access to the olfactory mucosa, this
is rarely the case because there is little correlation between airway
patency and olfactory function except in marked obstruction.1 There
is a wider correlation, although, between disease severity as measured by overall symptoms, endoscopy, CT scans, etc., and olfactory
function.1,17,18 Unfortunately, however, functional endoscopic sinus
surgery even with polypectomy results in limited improvement in
olfactory symptoms as measured by simple report or visual analog
scales1,18–20 and significantly less if any improvement in more detailed
assessments of olfaction.1,19,20
One explanation for this is that rather than the impairment in smell
being just from blocked access to the olfactory mucosa, the mucosal
inflammation itself contributes to the decreased function.1,21 In nasal
biopsy specimens obtained from patients with chronic rhinosinusitis
and control patients, histopathological examination of the olfactory
mucosa revealed erosion of the olfactory epithelium as well as squamous metaplasia and intermixing of goblet cells.21 The percentage of
sensory neurons in the olfactory epithelium was also decreased in
patients with chronic rhinosinusitis. Even compared with the other
patients with chronic rhinosinusitis, those with anosmia had the most
epithelial erosion and the highest density of infiltrating eosinophils.21,22
detailed measures of olfactory function.1,19,20 A recent study
showed that in nasal polyposis the status of the olfactory cleft,
especially the anterior portion, on CT could help predict olfactory
response to endoscopic surgery.24 However, ethmoid histology did
not help with such a prediction.22
Given the major role of inflammation in this type of olfactory
dysfunction, it is not surprising that there is significant improvement in symptoms often seen with oral corticosteroids, and much
of this effect is retained with continued use of topical steroids.25,26
Topical steroids have also been shown to help with the hyposmia
of allergic rhinitis.27 As in other inflammatory diseases, such as
asthma, the effects of even a strong anti-inflammatory medications
such as corticosteroids may be limited by remodeling and longerterm changes that have taken place as well. Although this treatment may result in less than complete resolution, it does currently
appear to be the most effective medical treatment for olfactory
disorders associated with chronic rhinosinusitis.
CLINICAL PEARLS
• Olfactory disorders are common, especially in elderly people, and
can have a significant effect on quality of life.
• Although there are many possible contributors to the loss of the
sense of smell, the specialist frequently sees this in patients with
chronic rhinosinusitis, with and without nasal polyps. MRI
should be strongly considered in patients without this clinical
picture.
• Although some patients do benefit as far as restored olfaction from
sinus surgery when there may have been a component of obstruction impeding airflow to the olfactory epithelium, anti-inflammatory treatment with topical or occasionally systemic steroids appears to be the most consistently beneficial treatment.
REFERENCES
1.
2.
3.
4.
5.
6.
7.
8.
TREATMENT
Unfortunately, treatment of most forms of anosmia and hyposmia
is very limited. Very little has been shown effective to treat congenital
or age-related olfactory dysfunction or even that associated with toxic
exposures, although these may occasionally improve on their own.
Prognosis for recovery of normal function in head trauma–related
anosmia is also poor, with, generally, there being little if any response
even to systemic steroids.11,12,23 Early research currently underway
into the possibility of surgical approaches to restore olfactory function
such as transplantation of the olfactory epithelium does offer some
hope for future developments.23
As mentioned previously, there is some potential for improvement of anosmia associated with chronic rhinosinusitis after endoscopic sinus surgery and polyp removal in certain patients, especially if one looks at symptom scores, with less effect on more
American Journal of Rhinology & Allergy
9.
10.
11.
12.
13.
Doty RL. Office procedures for quantitative assessment of olfactory
function. Am J Rhinol 21:460–473, 2007.
Doty RL, Shaman P, Applebaum SL, et al. Smell identification ability:
Changes with age. Science 226:1441–1443, 1984.
Smith W, and Murphy C. Epidemiological studies of smell: Discussion and perspectives. Ann N Y Acad Sci 1170:569–573, 2009.
Fleiner F, Dahlslett SB, Schmidt F, et al. Olfactory and gustatory
function in patients with multiple sclerosis. Am J Rhinol Allergy
24:e93–e97, 2010.
Harris R, Davidson TM, Murphy C, et al. Clinical evaluation and
symptoms of chemosensory impairment: One thousand consecutive
cases from the Nasal Dysfunction Clinic in San Diego. Am J Rhinol
20:101–108, 2006.
Hasan KS, Reddy SS, and Barsony N. Taste perception in Kallmann
syndrome, a model of congenital anosmia. Endocr Pract 13:716–720,
2007.
Segal NL, Topolski TD, Wilson SM, et al. Twin analysis of odor
identification and perception. Physiol Behav 57:605–609, 1995.
Moscavitch SD, Szyper-Kravitz M, and Shoenfeld Y. Autoimmune
pathology accounts for common manifestations in a wide range of
neuro-psychiatric disorders: The olfactory and immune system interrelationship. Clin Immunol 130:235–243, 2009.
Kokan N, Sakai N, Doi K, et al. Near-infrared spectroscopy of orbitofrontal cortex during odorant stimulation. Am J Rhinol Allergy 25:
163–165, 2011.
Atighechi S, Salari H, Baradarantar MH, et al. A comparative study of
brain perfusion single-photon emission computed tomography and
magnetic resonance imaging in patients with post-traumatic anosmia. Am J Rhinol Allergy 23:409–412, 2009.
Wu AP, and Davidson T. Posttraumatic anosmia secondary to central
nervous system injury. Am J Rhinol 22:606–607, 2008.
Jiang RS, Chai JW, Chen WH, et al. Olfactory bulb volume in Taiwanese patients with posttraumatic anosmia. Am J Rhinol Allergy
23:582–584, 2009.
Charland-Verville V, Lassonde M, and Frasnelli J. Olfaction in athletes with concussion. Am J Rhinol Allergy 26:222–226, 2012.
Copyright © Oceanside Publications, Inc. All rights reserved. DO NOT COPY
For permission to copy go to https://www.oceansidepubl.com/permission.htm
S69
14.
Smith WM, Davidson TM, and Murphy C. Toxin-induced chemosensory dysfunction: A case series and review. Am J Rhinol Allergy
23:578–581, 2009.
15. Doty RL, and Bromley SM. Effects of drugs on olfaction and taste.
Otolaryngol Clin North Am 37:1229–1254, 2004.
16. Kim YK, Hong SL, Yoon EJ, et al. Central presentation of postviral
olfactory loss evaluated by positron emission tomography scan: A
pilot study. Am J Rhinol Allergy 26:204–208, 2012.
17. Litvack JR, Mace JC, and Smith TL. Olfactory function and disease severity
in chronic rhinosinusitis. Am J Rhinol Allergy 23:139–144, 2009.
18. Jiang RS, Lu FJ, Liang KL, et al. Olfactory function in patients with
chronic rhinosinusitis before and after functional endoscopic sinus
surgery. Am J Rhinol 22:445–448, 2008.
19. Soler ZM, Mace J, and Smith TL. Symptom-based presentation of
chronic rhinosinusitis and symptom-specific outcomes after endoscopic sinus surgery. Am J Rhinol 22:297–301, 2008.
20. Jiang RS, Su MC, Liang KL, et al. Preoperative prognostic factors for
olfactory change after functional endoscopic sinus surgery. Am J
Rhinol Allergy 23:64–70, 2009.
S70
21.
Yee KK, Pribitkin EA, Cowart BJ, et al. Neuropathology of the olfactory
mucosa in chronic rhinosinusitis. Am J Rhinol Allergy 24:110–120, 2010.
22. Soler ZM, Sauer DA, Mace JC, and Smith TL. Ethmoid histopathology
does not predict olfactory outcomes after endoscopic sinus surgery.
Am J Rhinol Allergy 24:281–285, 2010.
23. Yagi S, and Costanzo RM. Grafting the olfactory epithelium
to the olfactory bulb. Am J Rhinol Allergy 23:239–243,
2009.
24. Kim DW, Kim JY, and Jeon SY. The status of the olfactory cleft may
predict postoperative olfactory function in chronic rhinosinusitis
with nasal polyposis. Am J Rhinol Allergy 25:e90–e94,
2011.
25. Hellings PW, and Rombaux P. Medical therapy and smell dysfunction. B-ENT. 5(suppl 13):71–75, 2009.
26. Mullol J, Obando A, Pujols L, and Alobid I. Corticosteroid treatment
in chronic rhinosinusitis: The possibilities and the limits. Immunol
Allergy Clin North Am 29:657–668, 2009.
27. Sivam A, Jeswani S, Reder L, et al. Olfactory cleft inflammation is
present in seasonal allergic rhinitis and is reduced with intranasal
steroids. Am J Rhinol Allergy 24:286–290, 2010.
e
May–June 2014, Vol. 28, No. 3
Copyright © Oceanside Publications, Inc. All rights reserved. DO NOT COPY
For permission to copy go to https://www.oceansidepubl.com/permission.htm
Nonallergic rhinitis
Russell A. Settipane, M.D.,1 and Michael A. Kaliner, M.D.2
ABSTRACT
Rhinitis is characterized by one or more of the following nasal symptoms: congestion, rhinorrhea (anterior and posterior), sneezing, and itching. It is
classified as allergic or nonallergic, the latter being a diverse syndrome that is characterized by symptoms of rhinitis that are not the result of IgE-mediated
events. Excluding infectious rhinitis and underlying systemic diseases, clinical entities that can be classified among the disorders that make up the nonallergic
rhinitis syndromes include gustatory rhinitis, nonallergic rhinitis with eosinophilia syndrome (NARES), atrophic, drug-induced (rhinitis medicamentosa),
hormone induced, senile rhinitis (of the elderly), rhinitis associated with chronic rhinosinusitis with or without nasal polyps, and the idiopathic variant formerly
known as vasomotor rhinitis but more accurately denoted as nonallergic rhinopathy (NAR). The prevalence of nonallergic rhinitis has been observed to be
one-third that of allergic rhinitis, affecting ⬃7% of the U.S. population or ⬃22 million people. NAR is the most common of the nonallergic rhinitis subtypes,
comprising at least two-thirds of all nonallergic rhinitis sufferers. Although certain precipitants such as perfume, strong odors, changes in temperature or
humidity, and exposure to tobacco smoke are frequently identified as symptom triggers, NAR may occur in the absence of defined triggers. The diagnosis of
nonallergic rhinitis is purely clinical and relies on a detailed history and physical exam. Skin testing or in vitro testing to seasonal and perennial aeroallergens
is required to make the diagnosis of nonallergic rhinitis. Because of the heterogeneous nature of this group of disorders, treatment should be individualized to
the patient’s underlying pathophysiology and/or symptoms and is often empiric.
T
he Joint Task Force Rhinitis Practice Parameter defines rhinitis as
characterized by one or more of the following nasal symptoms:
congestion, rhinorrhea (anterior and posterior), sneezing, and itching.1 Rhinitis can be classified as allergic2 or nonallergic; the latter
being a diverse syndrome that is characterized by periodic or perennial symptoms of rhinitis that are not the result of IgE-dependent
events.1 Excluding infectious rhinitis and underlying systemic diseases that may be associated with chronic rhinitis symptoms (Table
1),3 there are a number of separate clinical entities that can be classified among the disorders that make up the nonallergic rhinitis syndromes (Table 2).3 As will be discussed in greater detail, these entities
include gustatory rhinitis, nonallergic rhinitis with eosinophilia syndrome (NARES), atrophic, drug-induced (rhinitis medicamentosa),
hormone induced, senile rhinitis (of the elderly), rhinitis associated
with chronic rhinosinusitis with or without nasal polyps,4,5 and the
idiopathic variant known as vasomotor rhinitis, which is more accurately defined as nonallergic rhinopathy (NAR).6
EPIDEMIOLOGY
The prevalence of nonallergic rhinitis has been observed to be
one-third that of allergic rhinitis, affecting ⬃7% of the U.S. population
or ⬃22 million people.7 Vasomotor rhinitis is the most common of the
nonallergic rhinitis subtypes, comprising at least two-thirds of all
nonallergic rhinitis sufferers.8 Many patients with rhinitis actually
suffer from a combination of both nonallergic and allergic rhinitis.
So-called “mixed rhinitis” occurs in ⬃44–87% of patients with allergic
From 1Department of Medicine, Warren Alpert Medical School of Brown University,
Providence, Rhode Island, and 2Department of Medicine, George Washington School Of
Medicine, Washington, D.C.
RA Settipane is on the Speakers Bureau and/or a consultant and/or a research grant
recipient for Baush & Lomb, Meda, Sunovion, and Teva Respiratory. MA Kaliner is a
consultant/advisor for ISTA, Sunovion, and Meda; speaker for Alcon, ISTA, Sunovion,
Meda, and Genentech; and honorarium for Meda, Genentech, and Sunovion
Address correspondence and reprint requests to Russell A. Settipane, M.D., Allergy &
Asthma Center, 95 Pitman Street, Providence, RI 02906
E-mail address: [email protected]
Originally published in Am J Rhinol Allergy 27, S48 –S51, 2013
Copyright © 2014, OceanSide Publications, Inc., U.S.A.
American Journal of Rhinology & Allergy
rhinitis; and this form of rhinitis is more common than either pure
allergic rhinitis or nonallergic rhinitis.9 Risk factors for nonallergic
rhinitis include female sex and age of ⬎40 years.10
VASOMOTOR RHINITIS (NAR)
Vasomotor rhinitis is alternatively referred to as idiopathic, nonallergic, noninfectious rhinitis, and, most recently, as NAR.6 The Joint
Task Force Rhinitis Practice Parameter defines vasomotor rhinitis as a
“heterogeneous group of patients with chronic nasal symptoms that
are not immunologic or infectious in origin and are usually not
associated with nasal eosinophilia.”1 The underlying pathophysiology is unknown and may involve incipient, local atopy (entopy),11
dysfunction of nociceptive nerve sensor and ion channel proteins, and
autonomic dysfunction as found in chronic fatigue syndrome and
other functional disorders.12 Essential to the diagnosis of vasomotor
rhinitis is the absence of conditions and other causes listed in Tables
1 and 2.
Primary symptoms are nasal congestion and/or rhinorrhea. In
contrast to allergic rhinitis, nasal pruritus, sneezing, and conjunctival
symptoms are rare. Patterns of symptom occurrence may be perennial, persistent, intermittent, or seasonal and may occur in response to
climatic changes in temperature, humidity, and barometric pressure.13,14 It is important to emphasize that although certain precipitants such as perfume or strong odors are frequently identified as
symptom triggers (Table 3), NAR (vasomotor rhinitis) may occur in
the absence of defined triggers.6
The first step in treatment is avoidance of factors that may be
contributing such as cigarette smoke and other environmental
triggers.15 Compared with allergic rhinitis, less rigorous safety and
efficacy pharmaceutical treatment data exist for nonallergic rhinitis. However, there is a growing body of evidence for the efficacy
of topical therapies, including topical antihistamine nasal sprays,16–18 intranasal steroids,19 and intranasal ipratropium bromide20 (for rhinorrhea). Recently, a combination product containing both a topical
antihistamine (azelastine) and a topical nasal steroid (fluticasone
propionate), was conducted in a mixed population of patients with
either perennial allergic rhinitis or vasomotor rhinitis, and was shown
to be efficacious.21
Copyright © Oceanside Publications, Inc. All rights reserved. DO NOT COPY
For permission to copy go to https://www.oceansidepubl.com/permission.htm
S71
Table 1 Medical conditions associated with NAR symptoms
Metabolic
Acromegaly
Pregnancy
Hypothyroidism
Autoimmune
Sjögren’s syndrome
SLE
Relapsing polychondritis
Churg-Straus
Granulomatous diseases (sarcoidosis and Wegener’s granulomatosis)
Other
Cystic fibrosis
Cilia dyskinesia syndromes
Immunodeficiency
Amyloidosis
Chronic fatigue syndrome
GERD/LPR
Source: Adapted from Ref. 3.
NAR ⫽ nonallergic rhinopathy; SLE ⫽ sytemic lupus erythematous;
GERD ⫽ gastroesophageal reflux disease; LPR ⫽ laryngeal pharyngeal
reflux.
Table 2 Nonallergic, noninfectious chronic rhinitis not caused by
anatomical/mechanical causes
Nonallergic rhinopathy (vasomotor rhinitis)
Gustatory rhinitis
NARES
Atrophic rhinitis
Drug-induced (including rhinitis medicamentosa)
Hormone induced (including pregnancy rhinitis)
Rhinitis of elderly subjects
Chronic rhinitis associated with chronic rhinosinusitis with or
without nasal polyps
Source: Adapted from Ref. 3.
NARES ⫽ nonallergic rhinitis with eosinophilia syndrome.
Table 3 Precipitants of nonallergic rhinopathy (vasomotor rhinitis)
Changes in climate (such as temperature, humidity, and barometric
pressure)
Strong smells (such as perfume, cooking smells, flowers, and
chemical odors)
Environmental tobacco smoke
Pollutants and chemicals (e.g., volatile organics)
Exercise
Alcohol ingestion
GUSTATORY RHINTIS
Beer and wine may produce nasal congestion by direct nasal vasodilation and may exacerbate most forms of rhinitis. The syndrome of
watery rhinorrhea occurring immediately after ingestion of foods,
particularly hot and spicy foods, is known as gustatory rhinitis and is
vagally mediated.22 Preprandial treatment with topically sprayed
ipratropium bromide is often efficacious.23,24
NONALLERGIC RHINITIS WITH EOSINPOHILIA
SYNDROME
NARES is a perennial disorder that usually manifests in middleaged adults and may be responsible for up to one-third of all cases of
nonallergic rhinitis.25 Common symptoms include sneezing paroxysms, profuse, clear rhinorrhea, nasal pruritus, and reduced sense of
S72
smell.1 NARES is marked by eosinophilia (5–20%) on nasal cytology
in the setting of negative assessment for aeroallergen-specific IgE.1
The etiology for this condition is unknown; however, in some settings, it may be a precursor to nasal polyps, asthma, and aspirinexacerbated respiratory disease.26 NARES patients characteristically
respond well to topical nasal corticosteroids.
ATROPHIC RHINITIS
The hallmark features of atrophic rhinitis include progressive atrophy of the nasal mucosa, the development of hardened nasal crusts,
anosmia, and the presence of a foul odor or fetor emanating from the
patient’s nose.27 Primary atrophic rhinitis is most prevalent in developing countries in subtropical and temperate climate zones. Etiology
is unknown but bacterial infection is thought to be primarily or
secondarily involved, including Klebsiella ozaenae, Staphylococcus aureus, Proteus mirabilis, and Escherichia coli.1
Secondary atrophic rhinitis, which is the more prevalent form in the
United States and developed countries, is less severe and less progressive. It may be iatrogenically induced (“empty nose syndrome”
due to aggressive sinonasal surgery, particularly inferior turbinectomy) or may result from underlying granulomatous disease28 (tuberculosis, leprosy, and syphilis, and autoimmune processes such as
sarcoidosis and Wegener’s granulomatosis.) Management of atrophic
rhinitis includes nasal saline irrigation, antibiotics, and surgical approaches to reduce the nasal cavity size by providing tissue augmentation as a means to help restore nasal anatomy toward the premorbid
state.27,29
RHINITIS MEDICAMENTOSA
Rhinitis medicamentosa is a term most commonly used to describe
the rebound nasal congestion that occurs with the repetitive and
prolonged use of a topical, ␣-adrenergic, decongestant/vasoconstrictor agent such as oxymetazoline and phenylephrine. Although these
medications are generally safe to use for up to 3 consecutive days,
continued use for ⬎5–7 days may result in tachyphylaxis and rebound congestion. Turbinate hypertrophy and a “beefy red” appearance of the nasal mucosa are classic findings. A similar rebound nasal
congestion may result from the use of cocaine. Rhinitis medicamentosa, manifesting as nasal congestion, may also occur with certain oral
medications, including ␣-receptor antagonists used in the treatment
of benign prostatic hypertrophy and phophodiesterase-5-selective inhibitors used to treat erectile dysfunction. Treatment consists of stopping the offending agent. For topical decongestants, symptom control
during the withdrawal process often requires a short course of systemic corticosteroids (oral preferred).
HORMONE-INDUCED RHINITIS
Pregnancy, menstrual cycle–associated hormonal changes, and,
more controversially, oral contraceptives may be associated with
nasal congestion and rhinorrhea.30 Preexisting rhinitis is often exacerbated by the physiological changes of pregnancy (expanded blood
volume, vascular pooling, plasma leakage, and smooth muscle relaxation). Because rhinitis occurs in approximately one-fifth of pregnancies, it is often referred to as “pregnancy rhinitis.” Most commonly,
this condition presents as nasal congestion peaking in the last 6 weeks
of pregnancy and resolving within 2 weeks of delivery䡠1 Impact on
quality of life can be significant.31 Although a complete discussion of
the treatment of rhinitis during pregnancy is beyond the scope of this
article and is reviewed elsewhere,1 it is sufficient to note that the main
treatment concern during pregnancy is safety and that nasal saline
alone can be efficacious.32
RHINITIS OF ELDERLY PATIENTS
Rhinitis in elderly patients may be caused by the same types of
rhinitis that are common in other age populations; however, the
May–June 2014, Vol. 28, No. 3
Copyright © Oceanside Publications, Inc. All rights reserved. DO NOT COPY
For permission to copy go to https://www.oceansidepubl.com/permission.htm
prevalence rates of the various types may differ.30 For example,
because elderly subjects are more likely to be receiving prescription
medications, rhinitis medicamentosa may be a more prevalent cause
or contributor to chronic rhinitis in this age group.1 Additionally, both
NAR and rhinitis sicca (dry nasal tissues) are more commonly seen in
elderly subjects. The latter may occur secondary to Sjögren’s syndrome, atrophic rhinitis, or as a normal part of aging. However, a
more common complaint of elderly subjects is chronic clear rhinorrhea, which may result from NAR and/or cholinergic hyperactivity. Treatment is specific to the underlying condition. Although for rhinitis sicca, treatment with the liberal use of nasal
saline sprays is appropriate, the watery rhinorrhea resulting from
NAR has been shown to respond to intranasal ipratropium bromide in subjects ⬎60 years of age.23 Ipratropium bromide should
be used with caution in patients with preexisting glaucoma or
prostatic hypertrophy.24
DIFFERENTIAL DIAGNOSIS
The differential diagnosis of rhinitis includes other conditions that
mimic rhinitis symptoms. These include systemic diseases with nasal
manifestations (Table 1) and anatomic abnormalities, such as a deviated septum, turbinate hypertrophy, nasal valve collapse, nasal tumors, and foreign bodies.33 Refractory unilateral clear rhinorrhea may
be the result of a cerebral spinal fluid (CSF) leak, resulting from head
trauma, basal skull fracture, a postoperative complication of sinus
surgery, or spontaneous CSF leak. Diagnostically, ␤2-transferrin is a
more sensitive and specific indicator of CSF fluid than is the presence
of glucose.34
The diagnosis of nonallergic rhinitis is purely clinical and relies on
a detailed history and physical exam. Skin testing35 or in vitro testing36
to seasonal and perennial aeroallergens with negative results is required to make the diagnosis. Recent research suggests the possibility
that in a small subset of patients, the synthesis of specific IgE may be
localized and limited to occurring only in the nasal mucosa.37 This
condition, referred to as “entopy,” can be diagnosed by a positive
nasal allergen provocation test and/or detection of nasal specific IgE
(research tools) and in the setting of a negative skin test or negative
assessment for serum specific IgE.38,39
3.
4.
5.
6.
7.
8.
9.
10.
11.
12.
13.
14.
15.
16.
17.
18.
19.
SUMMARY
20.
Nonallergic rhinitis is a common condition that results in a substantial burden of disease. Because of the heterogeneous nature of this
group of disorders, treatment should be individualized to the patient’s underlying pathophysiology and/or symptoms and is often
empiric.
21.
22.
23.
CLINICAL PEARLS
• Nonallergic rhinitis affects 20 million people in the United States, is
more prevalent in women.
• NAR (vasomotor rhinitis) is the most common type of nonallergic
rhinitis, representing more than two-thirds of sufferers.
• Classically, symptoms of NAR are nonspecific but may be triggered
by irritant odors, perfumes, alcohol, and weather changes.
• With regard to pharmacotherapy of vasomotor rhinitis, preferred
treatments include intranasal steroids and intranasal antihistamines. When symptom relief with either agent is not effective,
combination of the two may produce greater benefit.
2.
25.
26.
27.
28.
REFERENCES
1.
24.
Wallace DV, Dykewicz MS, Bernstein DI, et al. The diagnosis and
management of rhinitis: An updated practice parameter. J Allergy
Clin Immunol 122:S1–S84, 2008.
Settipane RA, and Schwindt C. Allergic rhinitis. Am J Rhinol Allergy
27:S52–S55, 2013.
American Journal of Rhinology & Allergy
29.
30.
Kaliner M. Classification of nonallergic rhinitis syndromes with a
focus on vasomotor rhinitis, proposed to be known henceforth as
nonallergic rhinopathy. WAO J 2:98–101, 2009.
Settipane RA, Peters AT, and Chandra R. Chronic rhinosinusitis.
Am J Rhinol Allergy 27:S11–S15, 2013.
Settipane RA, Peters AT, and Chiu AG. Nasal polyps. Am J Rhinol
Allergy 27:S20–S25, 2013.
Kaliner M, Baraniuk JN, Benninger M, et al. Consensus definition of
nonallergic rhinopathy, previously referred to as vasomotor rhinitis,
nonallergic rhinitis, and/or idiopathic rhinitis. WAO J 2:119–120,
2009.
Settipane RA, and Charnock DR. Epidemiology of rhinitis: Allergic
and nonallergic. Clin Allergy Immunol 19:23–34, 2007.
Settipane RA. Epidemiology of vasomotor rhinitis. WAO J 2:115–118,
2009.
Bernstein JA. Allergic and mixed rhinitis: Epidemiology and natural
history. Allergy Asthma Proc 31:365–369, 2010.
Settipane RA. Rhinitis: A dose of epidemiological reality. Allergy
Asthma Proc 24:147–154, 2003.
Çomoğlu Ş, Keles N, and Değer K. Inflammatory cell patterns in the
nasal mucosa of patients with idiopathic rhinitis. Am J Rhinol Allergy
26:e55–e62, 2012.
Baraniuk JN. Pathogenic mechanisms of idiopathic nonallergic rhinitis. WAO J 2:106–114, 2009.
Bernstein JA, Salapatek AM, Lee JS, et al. Provocation of nonallergic
rhinitis subjects in response to simulated weather conditions using an
environmental exposure chamber model. Allergy Asthma Proc 33:
333–340, 2012.
Kim YH, Oh YS, Kim KJ, and Jang TY. Use of cold dry air provocation
with acoustic rhinometry in detecting nonspecific nasal hyperreactivity. Am J Rhinol Allergy 24:260–262, 2010.
Håkansson K, von Buchwald C, Thomsen SF, et al. Nonallergic
rhinitis and its association with smoking and lower airway disease: A
general population study. Am J Rhinol Allergy 25:25–29, 2011.
Lieberman P. The role of antihistamines in the treatment of vasomotor rhinitis. WAO J 2:156–161, 2009.
Smith PK, and Collins J. Olopatadine 0.6% nasal spray protects from
vasomotor challenge in patients with severe vasomotor rhinitis. Am J
Rhinol Allergy 25:e149–e152, 2011.
Lieberman P, Meltzer EO, LaForce CF, et al. Two-week comparison
study of olopatadine hydrochloride nasal spray 0.6% versus azelastine hydrochloride nasal spray 0.1% in patients with vasomotor
rhinitis. Allergy Asthma Proc 32:151–158, 2011.
Meltzer EO. The treatment of vasomotor rhinitis with intranasal
corticosteroids. WAO J 2:166–176, 2009.
Naclerio R. Anticholinergic drugs in nonallergic rhinitis. WAO J
2:162–165, 2009.
Meda Pharmaceuticals, Inc. Dymista package insert. Somerset, NJ;
2013. Last revision April 2012.
Raphael G, Raphael MH, and Kaliner M. Gustatory rhinitis: A syndrome of food-induced rhinorrhea. J Allergy Clin Immunol 83:110–
115, 1989.
Malmberg H, Grahne B, Holopainen E, and Binder E. Ipratropium
(Atrovent) in the treatment of vasomotor rhinitis of elderly patients.
Clin Otolaryngol Allied Sci 8:273–276, 1983.
Bronsky EA, Druce H, Findlay SR, and Hampel FC. A clinical trial of
ipratropium bromide nasal spray in patients with perennial nonallergic rhinitis. J Allergy Clin Immunol 95:1117–1122, 1995.
Settipane GA, and Klein DE. Non allergic rhinitis: Demography of
eosinophils in nasal smear, blood total eosinophil counts and IgE
levels. N Engl Reg Allergy Proc 6:363–366, 1985.
Moneret-Vautrin DA, Hsieh V, Wayoff M, et al. Nonallergic rhinitis
with eosinophilia syndrome a precursor of the triad: Nasal polyposis,
intrinsic asthma, and intolerance to aspirin. Ann Allergy 64:513–518,
1990.
Banks TA, and Gada SM. Clinical pearls and pitfalls: atrophic rhinitis.
Allergy Asthma Proc 34:185–187, 2013.
Kohanski MA, and Reh DD. Granulomatous diseases and chronic
sinusitis. Am J Rhinol Allergy 27:S39–S41, 2013.
Modrzyński M. Hyaluronic acid gel in the treatment of empty nose
syndrome. Am J Rhinol Allergy 25:103–106, 2011.
Settipane RA. Other causes of rhinitis: mixed rhinitis, rhinitis medicamentosa, hormonal rhinitis, rhinitis of the elderly, and gustatory
rhinitis. Immunol Allergy Clin North Am 31:457–467, 2011.
Copyright © Oceanside Publications, Inc. All rights reserved. DO NOT COPY
For permission to copy go to https://www.oceansidepubl.com/permission.htm
S73
31.
32.
33.
34.
S74
Gilbey P, McGruthers L, Morency AM, and Shrim A. Rhinosinusitis-related
quality of life during pregnancy. Am J Rhinol Allergy 26:283–286, 2012.
Hermelingmeier KE, Weber RK, Hellmich M, et al. Nasal irrigation as
an adjunctive treatment in allergic rhinitis: A systematic review and
meta-analysis. Am J Rhinol Allergy 26:e119–e125, 2012.
Shah R, and McGrath KG. Nonallergic rhinitis. Allergy Asthma Proc
33(suppl 1):S19–S21, 2012.
Sampaio MH, de Barros-Mazon S, Sakano E, and Chone CT. Predictability of quantification of beta-trace protein for diagnosis of cerebrospinal fluid leak: Cutoff determination in nasal fluids with two
control groups. Am J Rhinol Allergy 23:585–590, 2009.
35.
36.
37.
38.
39.
Carr TF, and Saltoun C. Skin testing in allergy. Allergy Asthma Proc
33:S6–S8, 2012.
Settipane RA, Borish L, and Peters AT. Determining the role of
allergy in sinonasal disease. Am J Rhinol Allergy 27:S56–S58, 2013.
Rondón C, Campo P, Galindo L, et al. Prevalence and clinical relevance of local allergic rhinitis. Allergy 67:1282–1288, 2012.
Broide DH. Allergic rhinitis: Pathophysiology. Allergy Asthma Proc
31:370–374, 2010.
Rondón C, Campo P, Togias A, et al. Local allergic rhinitis: Concept,
pathophysiology, and management. J Allergy Clin Immunol. 129:
1460–1467, 2012.
e
May–June 2014, Vol. 28, No. 3
Copyright © Oceanside Publications, Inc. All rights reserved. DO NOT COPY
For permission to copy go to https://www.oceansidepubl.com/permission.htm
Allergic rhinitis
Russell A. Settipane, M.D.,1 and Christina Schwindt, M.D.2
ABSTRACT
Allergic rhinitis affects 60 million of the U.S. population, 1.4 billion of the global population, and its prevalence appears to be increasing. The duration and
severity of allergic rhinitis symptoms place a substantial burden on patient’s quality of life, sleep, work productivity, and activity. The health impact of allergic
rhinitis is compounded by associated complications and comorbidities including asthma, otitis media, sinusitis, and nasal polyps. Allergic rhinitis symptoms
result from a complex, allergen-driven mucosal inflammatory process, modulated by immunoglobulin E (IgE), and caused by interplay between resident and
infiltrating inflammatory cells and a number of vasoactive and proinflammatory mediators, including cytokines. This allergic response may be characterized
as three phases: IgE sensitization, allergen challenge, and elicitation of symptoms. A thorough allergic history is the best tool for the diagnosis of allergic
rhinitis, the establishment of which is achieved by correlating the patient’s history and physical exam with an assessment for the presence of specific IgE
antibodies to relevant aeroallergens determined by skin testing or by in vitro assay. Management of allergic rhinitis includes modifying environmental
exposures, implementing pharmacotherapy, and, in select cases, administering allergen-specific immunotherapy. Intranasal therapeutic options include
antihistamines, anticholinergic agents, corticosteroids (aqueous or aerosol), mast cell stabilizers, saline, and brief courses of decongestants. Selection of
pharmacotherapy is based on the severity and chronicity of symptoms with the most effective medications being intranasal corticosteroids and intranasal
antihistamines, which can be used in combination (separately or in fixed dose) for more difficult to control allergic rhinitis.
T
he Joint Task Force Rhinitis Practice Parameter defines rhinitis as
characterized by one or more of the following nasal symptoms:
congestion, rhinorrhea (anterior and posterior), sneezing, and itching.1 Rhinitis is classified as allergic or nonallergic, the latter being a
diverse syndrome.2 Allergic rhinitis is classified as seasonal (commonly known as hay fever), resulting from sensitivity to pollen
allergens (tree, grass, or weed); perennial, resulting from indoor
allergens (such as animal dander and/or dust mites); intermittent; or
occupational.1 An international rhinitis guideline, Allergic Rhinitis
and Its Impact on Asthma, alternatively categorizes allergic rhinitis as
intermittent or persistent and in terms of severity as mild or moderate–severe, which is useful to help guide initial therapy.3
EPIDEMIOLOGY AND BURDEN OF ILLNESS
Allergic rhinitis affects ⬃10–30% of the U.S. population (totaling 60
million), 1.4 billion of the global population, and its prevalence appears to be increasing.4,5 Mixed rhinitis (combined allergic and nonallergic rhinitis) occurs in ⬃44–87% of patients with allergic rhinitis
and is more common than either pure allergic rhinitis or nonallergic
rhinitis.6 The duration and severity of allergic rhinitis symptoms have
been shown to place a substantial burden on patient’s quality of life
(QoL), sleep, work productivity, and activity.7,8 Risk factors for allergic rhinitis include1 family history of atopy,2 serum IgE of ⬎100
IU/mL before age 6 years,3 higher socioeconomic class, and4 presence
of a positive allergy skin-prick test.1
The health impact of allergic rhinitis is further compounded by associated complications and comorbidities including asthma, otitis media,
From 1Department of Medicine, Warren Alpert Medical School of Brown University,
Providence, Rhode Island, and 2Allergy and Asthma Associates, Southern California
Research, Mission Viejo, California
RA Settipane is on the Speakers Bureau and/or a consultant and/or a research grant
recipient for Baush & Lomb, Meda, Sunovion, and Teva Respiratory. C Schwindt has
no conflicts of interest to declare pertaining to this article
Address correspondence and reprint requests to Russell A. Settipane, M.D., Allergy &
Asthma Center, 95 Pitman Street, Providence, RI 02906
E-mail address: [email protected]
Originally published in Am J Rhinol Allergy 27, S52–S55, 2013
Copyright © 2014, OceanSide Publications, Inc., U.S.A.
American Journal of Rhinology & Allergy
sinusitis, and nasal polyps.9 Allergic rhinitis and associated comorbid
illnesses result in substantial costs, both direct (medical treatment expenditures) and indirect (because of reduced work productivity and lost days
from work.)10,11 When these costs are added, allergic rhinitis is estimated to
be the fifth most costly chronic disease in the United States.3,12
Pathophysiology
Allergic rhinitis symptoms result from a complex, allergen-driven
mucosal inflammatory process, modulated by immunoglobulin E
(IgE), and caused by interplay between resident and infiltrating inflammatory cells and a number of vasoactive and proinflammatory
mediators, including cytokines.1,13 The immunologic reaction that
underlies the acute symptoms of allergic rhinitis is defined as a type
I reaction according to the 1963 Gell and Coombs classification of
hypersensitivity reactions.14,15 This type of reaction takes place when
aeroallergens (pollens, molds, animal danders, dust mite fecal particles, cockroach residues, etc.) enter the nose and precipitate the release
of inflammatory mediators from tissue mast cells residing in the nasal
mucosa.16 This allergic response may be characterized as three distinct phases: sensitization, challenge, and elicitation.17
Sensitization refers to the production of the specific IgE antibody
that occurs when a genetically predisposed individual inhales aeroallergen, which is subsequently presented by antigen-presenting cells
to CD4⫹ T cells in local lymph nodes. In the setting of a Th2 cytokinerich milieu, allergen-stimulated Th2 cells proliferate and release cytokines (IL-4, IL-5, IL-9, and IL-13), which lead to the plasma cell
production of allergen-specific IgE antibodies.17
The challenge phase occurs on reexposure, when the same aeroallergen is recognized by specific IgE antibodies that have become
bound to mast cells in the nasal mucosa. Cross-linking of two adjacent
Fc␧RI on mediator cell membranes initiates a signal, leading to the
degranulation and release of mediators that elicit symptoms. The
third phase of the allergic response is referred to as the elicitation
phase, which, in the laboratory, has been characterized as comprised
of an early phase and late-phase response.17 Clinically, nasal symptoms of the early phase response occur within 5–30 minutes and
correlate with the mast cell mediator response (histamine, tryptase,
chymase, prostaglandin D2, and cysteinyl leukotrienes). This is followed in 60–70% of subjects by increased nasal symptoms 4–8 hours
Copyright © Oceanside Publications, Inc. All rights reserved. DO NOT COPY
For permission to copy go to https://www.oceansidepubl.com/permission.htm
S75
later because of a late-phase response characterized by influx into the
nasal mucosa of inflammatory cells (eosinophils, basophils, and T
lymphocytes), which become activated and further produce mediators. These mediators contribute to vasodilation, plasma leakage,
increased mucous secretion, and stimulation of afferent nerves. As a
consequence, both the early phase and late-phase responses are characterized by pruritus, sneezing, congestion, and rhinorrhea; however,
in the late phase, congestion is predominantly observed.1 Another
important clinical observation, which is a manifestation of the latephase reaction, is a phenomenon referred to as priming, where repeated exposure to an allergen results in the nasal mucosa becoming
increasingly allergen sensitive such that exposure to lesser amounts of
allergen is capable of eliciting symptoms.18 Priming also contributes
to a general increase in nasal hyperreactivity to nonallergic triggers
(irritants) as well.
Recent research suggests that in a small subset of patients, the
synthesis of specific IgE may be localized and limited to occurring
only in the nasal mucosa.19 This condition, referred to as “entopy,”
can be diagnosed in the setting of a positive nasal allergen provocation test and/or detection of nasal-specific IgE (research tool) in the
absence of systemic atopy (negative skin test and in vitro assessment
for serum specific IgE).13
Evaluation and Diagnostic Studies
More so than the physical exam, a thorough allergic history is the
best tool for the diagnosis of allergic rhinitis. It should include assessment of the following: a determination of the pattern, chronicity,
seasonality and triggers of nasal and related symptoms, family history, current medications, response to previous treatments, presence
of coexisting conditions, occupational exposure, and a detailed environmental history.1 The history should also consider the impact of
rhinitis symptoms on the patient’s QoL, including symptoms of fatigue, sleep disturbances, learning and attention problems, absenteeism, and presenteeism (impaired functionality) at work and/or
school.1 Provocation of symptoms by allergen exposure and occurrence of associated ocular symptoms (itching, redness, and tearing)
are key historical features that strongly suggest allergic causation.
It is important to note that findings on nasal examination are not
specific for the diagnosis of allergic rhinitis.1 Although the nasal
mucosa of the symptomatic allergic rhinitis patient commonly appears pale and boggy, with turbinate swelling, and clear nasal secretions, these findings may also be seen in nonallergic rhinitis.
Although an otoscope is commonly used for examination of the
nose, better assessment is achieved by the use of nasal speculum and
light source. When necessary, a more thorough exam may be achieved
using special techniques such as rigid or flexible nasal endoscopy. The
nasal exam should include assessment for the following abnormalities: reduced patency of nasal valve, alar collapse, transverse external
crease, external deformity such as saddle nose, septal abnormalities
(deviation, spurs, erosions, ulcers, perforation, prominent vessels, or
excoriation), nasal turbinate abnormalities (hypertrophy, edema, pallor, erythema, and crusting), discharge (amount, color, and consistency), nasal polyps, nasal masses, and foreign bodies.1 To assess
septal integrity, translumination of the septum from the contralateral
nares should result a pink glow; visualization of a white light indicates septal perforation (“white light test”). It is important to document the integrity of the nasal septum before instituting intranasal
therapy.
The diagnosis of allergic rhinitis is established by correlating the
patient’s history and physical exam with an assessment for the presence of specific IgE antibodies to relevant aeroallergens as preferably
determined by skin testing20 or by in vitro assay.21 Nasal cytology is
not routinely used in the assessment of allergic rhinitis, but may
confirm the presence of eosinophilic inflammation, which has predictive value for bronchial hyperresponsiveness.22
S76
Table 1 Oral and intranasal medications used for the treatment
of allergic rhinitis
Oral Therapies
Sedating antihistamines
Cetirizine/levocetirizine
Diphenhydramine
Carbinoxamine maleate
Clemastine
Chlorpheniramine
Brompheniramine
Triprolidine
Acrivastine
Nonsedating antihistamines
Loratadine/desloratadine
Fexofenadine
Decongestants
Pseudoephedrine
Phenylephrine
Leukotriene receptor antagonists
Montelukast35,36
Zafirlukast
Fixed combination products
Antihistamine/decongestants
Systemic corticosteroids
Intranasal Therapies
Intranasal corticosteroids24
Beclomethasone dipropionate
HFA25,26
Ciclesonide HFA27–29
Ciclesonide aqueous
Flunisolide
Fluticasone furoate30,31
Fluticasone propionate
Mometasone furoate32
Triamcinolone acetonide
Intranasal antihistamines
Azelastine33
Olopatadine34
Decongestants
Oxymetazoline
Phenylephrine
Ipratropium bromide 0.03%
Mast cell stabilizers
Cromolyn sodium
Fixed combination products
FP/azelastine44,45
Saline42
FP ⫽ fluticasone propionate; HFA ⫽ hydrofluoroalkane.
Treatment
Management of allergic rhinitis includes modifying environmental
exposures, implementing pharmacotherapy, and, in select cases, administering allergen-specific immunotherapy.
Environmental management is specific to the allergens identified
by history and corroborated by diagnostic testing for allergic sensitivities. For dust-mite allergen, the key area to address is the bedroom, where exposure is proportional to the quantity and age of the
fabric as well as the indoor humidity. Mattresses, pillows, box
springs, and comforters should be encased in material impermeable
to dust-mite particles. Bed linens should be washed in hot water
(120°F), carpets removed, and indoor humidity kept below 50%. For
animal dander allergen, there is no substitute for complete removal of
the pet from the home. For mold spore allergen, interventions include
moisture control, improved ventilation, leak repair, and, in severe
cases, extensive remediation. Unfortunately, in exquisitely sensitive
patients, despite best efforts to implement environmental control
measures, symptoms may persist because of small amounts of retained allergen. Therefore, treatment of allergic rhinitis frequently
necessitates pharmacotherapy. Physicians must take into account
safety; efficacy; cost-effectiveness; severity of symptoms; and patient
preference, satisfaction, and adherence when recommending medications.23 Two routes of administration exist for the administration of
medications used for the treatment of allergic rhinitis: oral and intranasal (see Table 1).24–36 By the time the patient sees a rhinologist, they
have frequently exhausted oral options (antihistamines and decongestants) because many of these agents are available without a prescription. Other oral options include leukotriene receptor antagonists
for mild disease and short courses of systemic corticosteroids for the
most severe cases. Intranasal therapeutic options include H1-receptor
antagonists, anticholinergic agents, corticosteroids (aqueous or aerosol), mast cell stabilizers, saline, and brief courses of decongestants.
Selection of pharmacotherapy is usually based on the severity and
chronicity of symptoms with the most effective single agent medications being intranasal corticosteroids and intranasal antihistamines.37,38 If rhinorrhea is the primary symptom, an anticholinergic
May–June 2014, Vol. 28, No. 3
Copyright © Oceanside Publications, Inc. All rights reserved. DO NOT COPY
For permission to copy go to https://www.oceansidepubl.com/permission.htm
agent may be added. For ocular symptoms, although some relief
frequently results from an intranasal antihistamine or an intranasal
corticosteroid, an ocular H1-receptor antagonist/mast cell stabilizer is
recommended for optimal benefit.39,40
In pregnancy, impact of rhinitis on QoL can be significant.41 Although a complete discussion of the treatment of allergic rhinitis
during pregnancy is beyond the scope of this article and is reviewed
elsewhere,1 it is sufficient to note that the main treatment concern
during pregnancy is safety. In this regard, nasal saline irrigation has
been shown to be efficacious in allergic rhinitis.42
After initial therapeutic intervention, ongoing monitoring of rhinitis control should guide therapeutic changes. A recently validated
tool, the Rhinitis Control Assessment Test, can help identify patients
who are inadequately controlled.43,44
When control is not adequately achieved, the environment should
be reassessed and medical therapy maximized. Additionally, stepping up to the combination of an intranasal antihistamine and an
intranasal corticosteroid (either separately or fixed-dose combination)
has been shown to provide greater symptomatic relief than monotherapy with either of the individual agents.45,46 Another therapeutic
alternative, allergen-specific immunotherapy, should be considered
when symptoms can not be adequately controlled by environmental
avoidance measures combined with maximal pharmacotherapy.47,48
The decision to begin immunotherapy may also depend on the patient’s preference/acceptability, adherence, and the adverse effects of
medications.49
6.
7.
8.
9.
10.
11.
12.
13.
14.
15.
CLINICAL PEARLS
16.
• The diagnosis of allergic rhinitis is established by correlating the
patient’s history and physical exam with an assessment for the
presence of specific IgE antibodies to relevant aeroallergens as
preferably determined by skin testing.
• To assess nasal septal integrity, translumination of the septum from
the contralateral nares should result in a pink glow; visualization of
a white light indicates septal perforation.
• The phenomenon referred to as “priming” is a manifestation of the
late-phase reaction, where repeated exposure to an allergen results
in the nasal mucosa becoming increasingly allergen sensitive such
that exposure to lesser amounts of allergen are capable of eliciting
symptoms
• A recently validated tool, the Rhinitis Control Assessment Test, can
help identify patients who are inadequately controlled.
• Stepping up to the combination of an intranasal antihistamine and
an intranasal corticosteroid (either separately or fixed dose combination) has been shown to provide greater symptomatic relief than
monotherapy with the individual agents.
• Allergen-specific immunotherapy should be considered when symptoms can not be adequately controlled by environmental avoidance
measures combined with maximal pharmacotherapy and may also
depend on the patient’s preference/acceptability, adherence, and the
adverse effects of medications.
17.
18.
19.
20.
21.
22.
23.
24.
25.
26.
REFERENCES
1.
2.
3.
4.
5.
Wallace DV, Dykewicz MS, Bernstein DI, et al. The diagnosis and
management of rhinitis: An updated practice parameter. J Allergy
Clin Immunol 122:S1–S84, 2008.
Settipane RA, and Kaliner MA. Nonallergic rhinitis. Am J Rhinol
Allergy 27:S48–S51, 2013.
Bousquet J, Khaltaev N, Cruz AA, et al. Allergic Rhinitis and its
Impact on Asthma (ARIA) 2008 update (in collaboration with the
World Health Organization, GA(2)LEN and AllerGen). Allergy 63
(suppl):8–160, 2008.
Settipane RA, Charnock DR. Epidemiology of rhinitis: allergic and
nonallergic. Clin Allergy Immunol 19:23–34, 2007.
Katelaris CH, Lai CK, Rhee CS, et al. Nasal allergies in the AsianPacific population: results from the Allergies in Asia-Pacific Survey.
Am J Rhinol Allergy 25 Suppl 1:S3–15, 2011.
American Journal of Rhinology & Allergy
27.
28.
29.
30.
Bernstein JA. Allergic and mixed rhinitis: Epidemiology and natural
history. Allergy Asthma Proc 31:365–369, 2010.
Meltzer EO, Blaiss MS, Naclerio RM, et al. Burden of allergic rhinitis:
Allergies in America, Latin America, and Asia-Pacific adult surveys.
Allergy Asthma Proc 33(suppl 1):S113–S141, 2012.
Abdulrahman H, Hadi U, Tarraf H, et al. Nasal allergies in the
Middle Eastern population: Results from the “Allergies in Middle
East Survey.” Am J Rhinol Allergy 26(suppl 1):S3–S23, 2012.
Settipane RA. Complications of allergic rhinitis. Allergy Asthma Proc
20:209–213, 1999.
Bhattacharyya N. Functional limitations and workdays lost associated with chronic rhinosinusitis and allergic rhinitis. Am J Rhinol
Allergy 26:120–122, 2012.
de la Hoz Caballer B, Rodríguez M, Fraj J, et al. Allergic rhinitis and
its impact on work productivity in primary care practice and a
comparison with other common diseases: The Cross-sectional study
to evAluate work Productivity in allergic Rhinitis compared with
other common dIseases (CAPRI) study. Am J Rhinol Allergy 26:390–
394, 2012.
Blaiss MS. Allergic rhinitis: Direct and indirect costs. Allergy Asthma
Proc 31:375–380, 2010.
Broide DH. Allergic rhinitis: Pathophysiology. Allergy Asthma Proc
31:370–374, 2010.
Coombs RRA, and Gell PGH. Classification of allergic reactions
responsible for clinical hypersensitivity and disease. In Clinical Aspect of Immunology, 3rd ed. Gell PGH, Coombs RRA, and Lachman
PJ (Eds). Oxford, U.K.: Blackwell Scientific Publications, 575–596,
1975.
Uzzaman A, and Cho SH. Classification of hypersensitivity reactions.
Allergy Asthma Proc 33(suppl 1):S96–S99, 2012.
Shah R, and Grammer LC. An overview of allergens. Allergy Asthma
Proc 33(suppl 1):S2–S5, 2012.
Luccioli S, Escobar-Gutierrez A, Bellanti JA. Allergic diseases and
asthma. In Immunology IV: Clinical applications in heath and disease. Bethesda: I Care, 685–765, 2012.
Connell JT. Quantitative intranasal pollen challenges. 3. The priming
effect in allergic rhinitis. J Allergy 43:33–44, 1969.
Rondón C, Campo P, Galindo L, et al. Prevalence and clinical relevance of local allergic rhinitis. Allergy 67:1282–1288, 2012.
Carr TF, and Saltoun CA. Skin testing in allergy. Allergy Asthma
Proc 33(suppl 1):S6–S8, 2012.
Settipane RA, Borish L, and Peters AT. Determining the role of
allergy in sinonasal disease. Am J Rhinol Allergy 27:S56–S58, 2013.
Canbaz P, Uskudar-Teke H, Aksu K, et al. Nasal eosinophilia can
predict bronchial hyperresponsiveness in persistent rhinitis: Evidence for united airways disease concept. Am J Rhinol Allergy 25:
120–124, 2011.
Bukstein D, Luskin AT, and Farrar JR. The reality of adherence to
rhinitis treatment: Identifying and overcoming the barriers. Allergy
Asthma Proc 32:265–271, 2011.
Blaiss MS. Safety update regarding intranasal corticosteroids for the
treatment of allergic rhinitis. Allergy Asthma Proc 32:413–418, 2011.
van Bavel JH, Ratner PH, Amar NJ, et al. Efficacy and safety of
once-daily treatment with beclomethasone dipropionate nasal aerosol in subjects with seasonal allergic rhinitis. Allergy Asthma Proc
33:386–396, 2012.
Meltzer EO, Jacobs RL, LaForce CF, et al. Safety and efficacy of
once-daily treatment with beclomethasone dipropionate nasal aerosol in subjects with perennial allergic rhinitis. Allergy Asthma Proc
33:249–257, 2012.
Berger WE, Mohar DE, LaForce C, et al. A 26-week tolerability study
of ciclesonide nasal aerosol in patients with perennial allergic rhinitis.
Am J Rhinol Allergy 26:302–307, 2012.
Ratner PH, Andrews C, Martin B, et al. A study of the efficacy and
safety of ciclesonide hydrofluoroalkane nasal aerosol in patients with
seasonal allergic rhinitis from mountain cedar pollen. Allergy
Asthma Proc 33:27–35, 2012.
Mohar D, Berger WE, Laforce C, et al. Efficacy and tolerability study
of ciclesonide nasal aerosol in patients with perennial allergic rhinitis.
Allergy Asthma Proc 33:19–26, 2012.
Han D, Liu S, Zhang Y, et al. Efficacy and safety of fluticasone furoate
nasal spray in Chinese adult and adolescent subjects with intermittent or persistent allergic rhinitis. Allergy Asthma Proc 32:472–481,
2011.
Copyright © Oceanside Publications, Inc. All rights reserved. DO NOT COPY
For permission to copy go to https://www.oceansidepubl.com/permission.htm
S77
31.
32.
33.
34.
35.
36.
37.
38.
39.
S78
Fokkens WJ, Rinia B, van Drunen CM, et al. No mucosal atrophy and
reduced inflammatory cells: Active-controlled trial with yearlong
fluticasone furoate nasal spray. Am J Rhinol Allergy 26:36–44, 2012.
Yamada T, Yamamoto H, Kubo S, et al. Efficacy of mometasone
furoate nasal spray for nasal symptoms, quality of life, rhinitisdisturbed sleep, and nasal nitric oxide in patients with perennial
allergic rhinitis. Allergy Asthma Proc 33:e9–e16, 2012.
Lieberman P, Meltzer EO, LaForce CF, et al. Two-week comparison
study of olopatadine hydrochloride nasal spray 0.6% versus azelastine hydrochloride nasal spray 0.1% in patients with vasomotor
rhinitis. Allergy Asthma Proc 32:151–158, 2011.
Meltzer EO, Blaiss M, and Fairchild CJ. Comprehensive report of
olopatadine 0.6% nasal spray as treatment for children with seasonal
allergic rhinitis. Allergy Asthma Proc 32:213–220, 2011.
Ciebiada M, Gorska-Ciebiada M, Barylski M, et al. Use of montelukast alone or in combination with desloratadine or levocetirizine in
patients with persistent allergic rhinitis. Am J Rhinol Allergy 25:e1–
e6, 2011.
Yamamoto H, Yamada T, Sakashita M, et al. Efficacy of prophylactic
treatment with montelukast and montelukast plus add-on loratadine
for seasonal allergic rhinitis. Allergy Asthma Proc 33:e17–e22, 2012.
Fairchild CJ, Durden E, Cao Z, and Smale P. Outcomes and cost
comparison of three therapeutic approaches to allergic rhinitis. Am J
Rhinol Allergy 25:257–262, 2011.
Carr WW, Ratner P, Munzel U, et al. Comparison of intranasal
azelastine to intranasal fluticasone propionate for symptom control
in moderate-to-severe seasonal allergic rhinitis. Allergy Asthma Proc
33:450–458, 2012.
Meier EJ, Torkildsen GL, Gow JA, et al. Bepotastine Besilate Ophthalmic Solutions Study Group. Integrated phase III trials of bepotastine besilate ophthalmic solution 1.5% for ocular itching associated
with allergic conjunctivitis. Allergy Asthma Proc 33:265–274, 2012.
40.
Baroody FM, Logothetis H, Vishwanath S, et al. Effect of intranasal
fluticasone furoate and intraocular olopatadine on nasal and ocular
allergen-induced symptoms. Am J Rhinol Allergy 27:48–53, 2013.
41. Gilbey P, McGruthers L, Morency AM, and Shrim A. Rhinosinusitisrelated quality of life during pregnancy. Am J Rhinol Allergy 26:283–
286, 2012.
42. Hermelingmeier KE, Weber RK, Hellmich M, et al. Nasal irrigation as
an adjunctive treatment in allergic rhinitis: A systematic review and
meta-analysis. Am J Rhinol Allergy 26:e119–e125, 2012.
43. Meltzer EO, Schatz M, Nathan R, et al. Reliability, validity, and
responsiveness of the Rhinitis Control Assessment Test in patients
with rhinitis. J Allergy Clin Immunol 131:379–386, 2013.
44. Schatz M, Zeiger RS, Chen W, et al. A comparison of the psychometric properties of the Mini-Rhinitis Quality of Life Questionnaire and
the Rhinitis Control Assessment Test. Am J Rhinol Allergy 26:127–
133, 2012.
45. Meltzer EO, LaForce C, Ratner P, et al. MP29–02 (a novel intranasal
formulation of azelastine hydrochloride and fluticasone propionate)
in the treatment of seasonal allergic rhinitis: A randomized, doubleblind, placebo-controlled trial of efficacy and safety. Allergy Asthma
Proc 33:324–332, 2012.
46. Carr W, Bernstein J, Lieberman P, et al. A novel intranasal therapy of
azelastine with fluticasone for the treatment of allergic rhinitis. J
Allergy Clin Immunol 129:1282–1289.e10, 2012.
47. Georgy MS, and Saltoun CA. Allergen immunotherapy: Definition, indication, and reactions. Allergy Asthma Proc 33(suppl 1):
S9–S11, 2012.
48. Uzzaman A, and Story R. Allergic rhinitis. Allergy Asthma Proc
33(suppl 1):S15–S18, 2012.
49. Settipane RA, Peters AT, and Borish L. Immunomodulation of allergic sinonasal disease. Am J Rhinol Allergy 27:S59–S62, 2013.
e
May–June 2014, Vol. 28, No. 3
Copyright © Oceanside Publications, Inc. All rights reserved. DO NOT COPY
For permission to copy go to https://www.oceansidepubl.com/permission.htm
Determining the role of allergy in sinonasal disease
Russell A. Settipane, M.D.,1 Larry Borish, M.D.,2 and Anju T. Peters, M.D.3
ABSTRACT
The contributing role of specific IgE sensitization in the pathophysiology of sinonasal diseases including rhinitis, chronic rhinosinusitis (CRS), and nasal
polyps is explored. Although it is estimated that sensitization to environmental allergens is present in 75% of patients with rhinitis, the role of allergy in CRS
and nasal polyps is less certain. However, when atopy is present in the setting of nasal polyps, it is associated with worse quality of life and a higher incidence
of asthma. Several theories have been put forth whereby inhalant aeroallergen exposure could drive the inflammatory response that occurs both in the nose and
in the sinuses. Tools available to determine the presence of allergic sensitization include skin tests for immediate hypersensitivity, in vitro testing for
allergen-specific IgE, and nasal allergen provocation testing. Whether by skin testing or in vitro testing, the identification of specific IgE requires clinical
correlation with the history and physical exam.
S
pecific IgE sensitization plays a major contributory role in the
pathophysiology of upper and lower respiratory diseases including rhinitis and asthma. Although it is estimated that allergic
sensitization to environmental allergens is present in 75% of patients
with rhinitis,1 the role of allergy in chronic rhinosinusitis (CRS) and
nasal polyps is less certain.2 In this article the role of allergy in
sinonasal disease is explored, followed by an overview of diagnostic
testing tools for determining immediate hypersensitivity.
EPIDEMIOLOGY OF ALLERGIC RHINITIS
IN CRS
There exists a high degree of overlap between CRS3 and allergic
rhinitis,4 but this relationship may be more coincidental than etiologic.5 Eighty-two percent of patients in an academic institution who
underwent sinus surgery were noted to have one or more positive
skin-prick test results to inhalant allergens. This was significantly
higher than that found in the National Health and Nutrition Examination Study III (54.3%) but comparable with the rhinitis control
group (72%) that did not have CRS.6 Although IgE-mediated hypersensitivity occurs in patients with CRS, the weight of the available
evidence suggests that allergic rhinitis contributes in a variable but
limited way to the mucosal inflammation of CRS.2 For example, in a
retrospective study of pediatric patients, allergy was shown to be a
risk factor for protracted symptoms despite endoscopic sinus surgery;
however, there is no evidence to support the theory that failure to
address allergy adversely affects the probability of success in sinus
surgery.7
With regard to nasal polyps, the prevalence in a population with
allergic rhinitis is comparable with that seen in the normal population8; and allergy does not appear to be a risk factor for the development of nasal polyps.9,10 Additionally, the presence of allergy does
From the 1Department of Medicine, Warren Alpert Medical School of Brown University, Providence, Rhode Island, 2Department of Medicine, Asthma and Allergic Disease
Center, Carter Immunology Center, University of Virginia Health System, Charlottesville, Virginia, and 3Division of Allergy-Immunology, Northwestern University,
Chicago, Illinois
RA Settipane received a research grant from Genentech (clinical trial). L Borish is
funded by NIH RO1 AI057483 and UO1 AI100799. AT Peters is a speaker for Baxter
Address correspondence and reprint requests to Russell A. Settipane, M.D., Allergy &
Asthma Center, 95 Pitman Street, Providence, RI 02906
E-mail address: [email protected]
Originally published in Am J Rhinol Allergy 27, S56 –S58, 2013
Copyright © 2014, OceanSide Publications, Inc., U.S.A.
American Journal of Rhinology & Allergy
not correlate with polyp size, symptom scores, grading of severity by
CT (Lund-Mackay), or rate of polyp recurrence.11 However, the presence of atopy in the setting of nasal polyps has been shown to be
associated with lower quality of life scores and a higher incidence of
asthma.12 Furthermore, there appears to be some subsets of CRS
where IgE sensitization may play a greater role. Among these are
patients with severe CRS with nasal polyps (CRSwNPs)13 who often
manifest chronic hyperplastic eosinophilic sinusitis in association
with multiple positive skin tests,5 allergic fungal rhinosinusitis,14 and
patients with local polyclonal IgE in the absence of systemic atopy,
the potential etiology of which is discussed in this article.15
THEORIES FOR ALLERGIC RHINITIS/CRS
PATHOPHYSIOLOGICAL LINKAGE
Discussion is warranted regarding several theories whereby inhalant aeroallergen exposure could drive the inflammatory response when it occurs concomitantly in the nose and sinuses (Table 1).5
A “direct aeroallergen reaction” is thought to be unlikely
because breathing alone does not drive aeroallergens into the
sinuses of patients who have not had their sinus ostia altered by
surgery.5 Except for the CRS subtype known as allergic fungal
rhinosinusitis,14 evidence for “sensitization to colonizing fungi” in
the pathogenesis of CRS appears to be limited to playing a role as
a disease modifier.2 There exists greater supporting evidence for a
“systemic allergic inflammatory process” involving the local nasal
airway, nasal-associated lymphatic tissue, the bone marrow, and
the sinuses. Supporting this concept, nasal allergen challenge has
been shown to result in inflammatory changes within both the
ipsilateral, but strikingly, also the contralateral maxillary sinus
cavity, marked by a significant increase in maxillary sinus eosinophils.16 Finally, the “sensitization to colonizing bacteria” hypothesis proposes that in CRSwNPs, exposure to Staphylococcus aureus
enterotoxins, induces an inflammatory mucosal response, resulting
in a skewing of T lymphocytes toward a Th2 phenotype, proinflammatory cytokine release, localized polyclonal IgE responses,
Treg inhibition, and accentuated eosinophil and mast cell activity.17–19 Superantigen-induced polyclonal IgE in airway disease has
been postulated to contribute to chronic inflammation by continuously activating mast cells. In studies where tissue fragments
from nasal polyp patients were stimulated with anti-IgE, mast cells
were activated, indicating that mucosal IgE antibodies in nasal
polyp tissue are functional and able to activate mast cells.20 The
polyclonal IgE response may be directed at bystander pathogens
Copyright © Oceanside Publications, Inc. All rights reserved. DO NOT COPY
For permission to copy go to https://www.oceansidepubl.com/permission.htm
S79
Table 1 Putative mechanisms linking allergic rhinitis to CRS
Direct aeroallergen reaction
Sensitization to colonizing fungi
Systemic allergic inflammation
Sensitization to colonizing bacteria
Source: Adapted from Ref. 5.
CRS ⫽ chronic rhinosinusitis.
such as the Staphylococcus enterotoxin. The role of IgE sensitization
to bystander pathogens is supported by anti-IgE clinical trial outcomes showing improvement in nasal polyps irrespective of the
presence of allergic sensitization to aeroallergens.21,22
Whether by skin testing or in vitro testing, the identification of
specific IgE requires clinical correlation with the history and physical
exam.24 In circumstances where the history strongly suggests allergen
sensitivity, but testing for specific IgE is negative, a nasal allergen
provocation test and/or detection of local nasal specific IgE may
confirm the diagnosis.35 In this condition, referred to as “entopy,”
local IgE production may be limited to the nasal mucosa in patients
who would otherwise be diagnosed as having nonallergic rhinitis.36,37
Finally, despite advances in evidence-based medicine, unproven
tests continue to be promoted for the determination of allergic sensitization. These include IgG4-specific antibody tests, cytotoxic tests,
provocation–neutralization, electrodermal testing, applied kinesiology, iridology, and hair analysis.38 Currently, there is neither evidence nor an immune/mechanistic basis to suggest that these tests
are useful.
SUMMARY
DIAGNOSTIC TESTING FOR IMMEDIATE
HYPERSENSITIVITY
Despite uncertainties regarding the contribution of allergy to CRS,
knowledge of specific allergic sensitizations may provide benefit to
patients who suffer from associated rhinitis symptoms. The primary
tools available to determine the presence of IgE-mediated hypersensitivity23 include skin tests and in vitro tests for allergen-specific IgE.
Although both techniques are able to identify specific IgE-mediated
hypersensitivity, skin testing by prick/puncture technique is the preferred diagnostic approach because of its overall sensitivity, specificity, and rapidity of performance.24–28 Skin testing detects the presence
of allergen-specific IgE bound to mast cells by eliciting allergeninduced mast cell degranulation and a resultant histamine wheal/
flare response. For a detailed description of the skin test procedure,
the reader is referred to diagnostic testing practice parameters.24
Factors that may affect interpretation and reliability of prick/puncture tests include the skill of the tester, the test instruments, extremes
of age, skin color, skin reactivity (including dermatographism), and
reagent potency. Additionally, concurrent medications affect the validity of skin testing; in particular, first-/second-generation antihistamines and tricyclic antidepressants should be held for 3–7 days
depending on the agent, and H2-antagonists for 1 day.24 Although
␤-adrenergic blocking agents do not interfere with the skin test response, caution with their concomitant use is warranted because of
the potential of ␤-blockers to impede epinephrine treatment of anaphylaxis, which is a rare but potential adverse reaction of skin testing.29 Because of this risk, a physician should always be available to
administer emergency epinephrine if necessary.30–32
Ideally, objective wheal-and-flare responses are recorded in millimeters, along with positive and negative controls. In the clinical
setting of strongly suspected hypersensitivity, intracutaneous tests (at
100- to 1000-fold more dilute) may be applied if prick/puncture tests
are negative. Intracutaneous tests are associated with increased risk of
inducing anaphylaxis, which can be fatal.33 This is more of a safety
concern when/if intracutaneous tests are performed without preceding prick/puncture tests. Interpretation of intracutaneous tests may
be confounded by false positive results because of irritant effects of
the testing reagent.
Immunoassays measuring serum-specific IgE concentrations
(kIU/L) have good sensitivity/specificity and can predict respiratory
responses after allergen exposure.24,28 These assays provide information that is not equivalent to skin testing but is considered complementary.34 The clinical efficacy of a total IgE measurement is limited;
it is much more valuable to measure serum-specific IgE. There are
circumstances when serum-specific IgE immunoassays may be preferable to skin testing such as in the setting of generalized dermatitis,
concomitant medications that may suppress the skin test, uncooperative patients, or when the history suggests an unusually greater risk
of anaphylaxis from skin testing.24
S80
Specific IgE sensitization plays a contributing role in sinonasal
disease, particularly in rhinitis, but also to an undetermined and
variable extent in CRS and nasal polyps. Although it is estimated that
allergic sensitization to environmental allergens is present in 75% of
patients with rhinitis, the role of allergic contribution to CRS is less
certain. However, when atopy is present in the setting of nasal polyps,
it is associated with worse quality of life and a higher incidence of
asthma. Several theories have been put forth whereby inhalant aeroallergen exposure could drive the inflammatory response that occurs
both in the nose and sinuses. In patients with CRS or nasal polyps,
when the history suggests the presence of allergic sensitization, skin
testing is the preferred test for the determination of immediate hypersensitivity. In certain clinical settings, in vitro testing for allergenspecific IgE provides an additional diagnostic tool. Finally, a nasal
allergen provocation test may confirm the presence of local allergy in
the setting of negative skin tests, but its efficacy is limited because of
labor intensity and because it is confounded by the potential for false
positive results. Whether by skin testing or in vitro testing, the identification of specific IgE sensitization, when clinically correlated with
the history and physical exam, allows the clinician to best determine
and treat the allergic contribution to sinonasal disease. Once an
allergic contribution has been determined, the clinician must weigh
the medical evidence as to what therapeutic interventions, including
immunotherapy,39 are appropriate.
CLINICAL PEARLS
• Although specific IgE sensitization can be determined by in vitro
assay, prick/puncture skin test technique provides more rapid
results, which, unlike in vitro tests, are not influenced by high total
IgE levels.
• Interpretation of skin testing may be confounded by extremes of
age, skin color, dermatographism, and antihistamine use.
• Interpretation of skin testing should be performed by an experienced practitioner, in the presence of positive and negative controls.
• Because skin testing may potentially result in anaphylaxis, a physician should always be available to administer epinephrine if necessary.
• Whether by skin testing or in vitro testing, the identification of
specific IgE requires clinical correlation with the history and physical exam to make the diagnosis of allergy.
• In a condition known as entopy, specific IgE sensitization can only
be identified in the nasal mucosa and can not be found systemically.
REFERENCES
1.
2.
Settipane RA. Rhinitis: A dose of epidemiological reality. Allergy
Asthma Proc 24:147–154, 2003.
Fokkens WJ, Lund VJ, Mullol J, et al. European position paper on
rhinosinusitis and nasal polyps 2012. Rhinol Suppl 23:1–298, 2012.
May–June 2014, Vol. 28, No. 3
Copyright © Oceanside Publications, Inc. All rights reserved. DO NOT COPY
For permission to copy go to https://www.oceansidepubl.com/permission.htm
3.
4.
5.
6.
7.
8.
9.
10.
11.
12.
13.
14.
15.
16.
17.
18.
19.
20.
Settipane RA, Peters AT, and Chandra R. Chronic rhinosinusitis.
Am J Rhinol Allergy 27:S11–S15, 2013.
Settipane RA, and Schwindt C. Allergic rhinitis. Am J Rhinol Allergy
27:S52–S55, 2013.
Kennedy JL, and Borish L. Chronic sinusitis pathophysiology: The
role of allergy. Am J Rhinol Allergy DOI: 10.2500/ajra.2013.27.3906
[Epub ahead of print date April 18, 2013].
Tan BK, Zirkle W, Chandra RK, et al. Atopic profile of patients failing
medical therapy for chronic rhinosinusitis. Int Forum Allergy Rhinol
1:88–94, 2011.
Lee TJ, Liang CW, Chang PH, and Huang CC. Risk factors for
protracted sinusitis in pediatrics after endoscopic sinus surgery. Auris Nasus Larynx 36:655–660, 2009.
Kern R. Allergy: A constant factor in the etiology of so-called mucous
nasal polyps. J Allergy 4:483, 1993.
Erbek SS, Erbek S, Topal O, et al. The role of allergy in the severity of
nasal polyposis. Am J Rhinol. 2007 21:686–690, 2007.
Settipane GA. Epidemiology of nasal polyps. In Nasal Polyps: Epidemiology, Pathogenesis and Treatment. Settipane GA, Lund VJ,
Bernstein JM, and Tos M (Eds). Providence, RI: OceanSide Publications, 17–24, 1997.
Pearlman AN, Chandra RK, Chang D, et al. Relationships between
severity of chronic rhinosinusitis and nasal polyposis, asthma, and
atopy. Am J Rhinol Allergy 23:145–148, 2009.
Dávila I, Rondón C, Navarro A, et al. Aeroallergen sensitization
influences quality of life and comorbidities in patients with nasal
polyposis. Am J Rhinol Allergy 26:e126–e131, 2012.
Settipane RA, Peters AT, and Chiu AG. Nasal polyps. Am J Rhinol
Allergy 27:S20–S25, 2013.
Laury AM, and Wise SK. Allergic fungal rhinosinusitis. Am J Rhinol
Allergy 27:S26–S27, 2013.
Bachert C, Gevaert P, Holtappels G, et al. Total and specific IgE in
nasal polyps is related to local eosinophilic inflammation. J Allergy
Clin Immunol 107:607–614,2001.
Baroody FM, Mucha SM, Detineo M, et al. Nasal challenge with
allergen leads to maxillary sinus inflammation. J Allergy Clin Immunol 121:1126–1132.e7, 2008.
Van Crombruggen K, Zhang N, Gevaert P, et al. Pathogenesis of
chronic rhinosinusitis: Inflammation. J Allergy Clin Immunol 128:
728–732, 2011.
Kim DW, Khalmuratova R, Hur DG, et al. Staphylococcus aureus
enterotoxin B contributes to induction of nasal polypoid lesions in an
allergic rhinosinusitis murine model. Am J Rhinol Allergy 25:e255–
e261, 2011.
Kim ST, Chung SW, Jung JH, et al. Association of T cells and eosinophils with Staphylococcus aureus exotoxin A and toxic shock syndrome toxin 1 in nasal polyps. Am J Rhinol Allergy 25:19–24, 2011.
Zhang N, Holtappels G, Gevaert P, et al. Mucosal tissue polyclonal IgE
is functional in response to allergen and SEB. Allergy 66:141–148, 2011.
American Journal of Rhinology & Allergy
21.
22.
23.
24.
25.
26.
27.
28.
29.
30.
31.
32.
33.
34.
35.
36.
37.
38.
39.
Pinto JM, Mehta N, DiTineo M, et al. A randomized, double-blind,
placebo-controlled trial of anti-IgE for chronic rhinosinusitis. Rhinology 48:318–324, 2010.
Gevaert P, Calus L, Van Zele T, et al. Omalizumab is effective in
allergic and nonallergic patients with nasal polyps and asthma. J
Allergy Clin Immunol 131:110–116.e1, 2013.
Uzzaman A, and Cho SH. Classification of hypersensitivity reactions.
Allergy Asthma Proc 33(suppl 1):S96–S99, 2012.
Bernstein IL, Li JT, Bernstein DI, et al. Allergy diagnostic testing: An
updated practice parameter. Ann Allergy Asthma Immunol 100
(suppl 3):S1–S148, 2008.
Tripathi A, and Kim JS. Diagnosis of immediate hypersensitivity. In
Patterson’s Allergic Diseases, 7th ed. Grammer LC, and Greenberger
PA (Eds). Philadelphia, PA: J.B. Lippincott, Williams & Wilkins,
123–135, 2009.
Tripathi A, and Patterson R. Clinical interpretations of skin test
results. Immunol Allergy Clin North Am 21:291–300, 2001.
Carr TF, and Saltoun CA. Skin testing in allergy. Allergy Asthma
Proc 33(suppl 1):S6–S8, 2012.
Makhija M, and O’Gorman MRG. Common in vitro tests for allergy
and immunology. Allergy Asthma Proc 33(suppl 1):S108–S111, 2012.
Greenberger PA, and Ditto AM. Anaphylaxis. Allergy Asthma Proc
33:S80–S83, 2012.
Lieberman P. The risk and management of anaphylaxis in the setting
of immunotherapy. Am J Rhinol Allergy 26:469–474, 2012.
Phillips JF, Lockey RF, Fox RW, et al. Systemic reactions to subcutaneous allergen immunotherapy and the response to epinephrine.
Allergy Asthma Proc 32:288–294, 2011.
Wallace DV. Anaphylaxis in the allergist’s office: Preparing your
office and staff for medical emergencies. Allergy Asthma Proc 34:
120–131, 2013.
Bernstein DI, Wanner M, Borish L, et al. Twelve-year survey of fatal
reactions to allergen injections and skin testing: 1990–2001. J Allergy
Clin Immunol 113:1129–1136, 2004.
Calabria CW, Dietrich J, and Hagan L. Comparison of serum-specific
IgE (ImmunoCAP) and skin-prick test results for 53 inhalant allergens in patients with chronic rhinitis. Allergy Asthma Proc 30:386–
396, 2009.
Rondón C, Campo P, Togias A, et al. Local allergic rhinitis: Concept,
pathophysiology, and management. J Allergy Clin Immunol 129:
1460–1467, 2012.
Khan DA. Allergic rhinitis with negative skin tests: Does it exist?
Allergy Asthma Proc 30:465–469, 2009.
Settipane RA, and Kaliner MA. Nonallergic rhinitis. Am J Rhinol
Allergy 27:S48–S51, 2013.
Shah R, and Greenberger PA. Unproved and controversial methods
and theories in allergy-immunology. Allergy Asthma Proc 33(suppl
1):S100–S10 2, 2012.
Settipane RA, Peters AT, and Borish L. Immunomodulation of allergic sinonasal disease. Am J Rhinol Allergy 27:S59–S62, 2013.
e
Copyright © Oceanside Publications, Inc. All rights reserved. DO NOT COPY
For permission to copy go to https://www.oceansidepubl.com/permission.htm
S81
The united allergic airway: Connections between allergic
rhinitis, asthma, and chronic sinusitis
Charles H. Feng, M.D.,1 Michaela D. Miller, M.D.,1 and Ronald A. Simon, M.D.2
ABSTRACT
Background: The united allergic airway is a theory that connects allergic rhinitis (AR), chronic rhinosinusitis, and asthma, in which seemingly disparate
diseases, instead of being thought of separately, are instead viewed as arising from a common atopic entity.
Objective: This article describes patients with such diseases; explores ideas suggesting a unified pathogenesis; elucidates the various treatment modalities
available, emphasizing nasal corticosteroids and antihistamines; and provides an update of the literature.
Methods: A literature review was conducted.
Conclusion: The aggregation of research suggests that AR, asthma, and chronic rhinosinusitis are linked by the united allergic airway, a notion that
encompasses commonalities in pathophysiology, epidemiology, and treatment.
A
llergic rhinitis (AR) occurs when the nasal passages become
inflamed; it is characterized by rhinorrhea, nasal congestion,
postnasal drip, and itchiness of the nose. The inflammatory cascade in
AR involves an immediate IgE-mediated mast cell response and a
late-phase response of basophils, eosinophils, and T cells driven by
the cytokines IL-4 and IL-5.1 In adults, risk factors for AR include
eczema and a family history of atopy.2 In children, risk factors for AR
include maternal cigarette smoking and higher blood IgE levels.3
Since the onset of the Industrial Revolution, AR has become the most
common atopic disorder in the United States, affecting 20–40 million
people annually, including up to 30% of adults and 40% of children.4,5
Asthma, on the other hand, involves inflammation of the bronchial
tree and can cause wheezing, shortness of breath, coughing, and chest
tightness. This condition, compared with AR, is far more prevalent at
a younger age and affects 10% of children and 8% of adults.6
Although AR, on the spectrum of medical afflictions, is considered
a relatively benign disease, patients with AR can have an impaired
quality of life, with difficulty sleeping, exhaustion during the day,
cognitive disturbances, and mood changes.7 Having AR also causes
socioeconomic consequences, because patients are forced to take time
off from school and work.8,9 Patients with asthma, by contrast, are
more likely to have physical limitations, impacting both their activities of daily living, such as going up stairs and performing chores, and
their ability to exercise. However, in patients with both asthma and
AR, there are more physical limitations when compared with ARonly patients, but no further impairment in quality of life.7
From the 1Department of Internal Medicine, 2Division of Allergy and Immunology,
Scripps Green Hospital, La Jolla, California
Presented at the North American Rhinology & Allergy Conference, February 4, 2012,
Puerto Rico
CH Feng and MD Miller contributed equally to this work
RA Simon is part of the Speaker Bureau for GlaxoSmithKline, Merck, Astra-Zeneca,
and Novartis. The remaining authors have no conflicts of interest to declare pertaining
to this article
Address correspondence and reprint requests to Charles Feng, M.D., 10666 Torrey
Pines Road MS:403C, La Jolla, CA 92037
E-mail address: [email protected]
Originally published in Am J Rhinol Allergy 26, 187–190, 2012
Copyright © 2014, OceanSide Publications, Inc., U.S.A.
S82
ALLERGIC RHINITIS AND ASTHMA
AR and asthma, rather than being considered two distinct diseases,
can be unified by the concept of a “united airway,” where allergic
symptoms of the upper and lower airways can be thought of as
manifestations of a common atopic entity.10 Epidemiological evidence
suggests a strong relationship between AR and asthma. AR can occur
in ⬎75% of patients with asthma, whereas asthma can affect up to
40% of patients with AR.11 Both diseases, which are IgE mediated, can
be triggered by similar allergens, including mold, animal dander, and
house-dust mites.12,13 Temporally, AR often occurs before the onset of
asthma. In a 10-year longitudinal study of children with AR, asthma
was eventually found in 19% of cases, and in 25% of the sample size,
asthma and AR developed simultaneously.12
Indeed, AR is a risk factor for asthma, and its presence is related to
asthma severity. For example, in a 23-year follow-up study of almost
2000 college students, patients with AR, when compared with controls without AR, were about three times more likely to develop
asthma.14 This idea was confirmed by a 15-year prospective study of
Finnish twins, which found that in patients with AR, male patients
were four times and female patients were six times as likely to get
asthma when compared with patients without AR.15 Taking this
notion one step further, Guerra et al.11 found that, after adjusting for
age, sex, atopic status, years of follow-up, smoking status, and the
presence of chronic obstructive pulmonary disease, AR was still an
independent risk factor for asthma. Of importance, AR and asthma
were linked, autonomous of the fact that they shared atopy as a
common causal agent.
In addition to the epidemiological evidence, several clinical reports
point to a common pathophysiological relationship between AR and
asthma. In the 1980s, allergists noted not only that AR patients
hyperresponsive to methacholine had a greater risk of developing
asthma, but also that the increase in bronchial reactivity was correlated with the pollen season.16,17 These findings were confirmed in
studies led by Ciprandi and colleagues, who showed that in a majority of patients with AR but no asthma, there is an increase in bronchial
hyperreactivity (BHR) after methacholine challenge.17,18 Moreover, in
the subset of patients with BHR, there is impairment in spirometry,
including forced vital capacity, forced expiratory volume in 1 second
(FEV1), and forced expiratory flow at 25–75%.17,18 More recently,
Ciprandi et al.19 showed that ⬃2⁄3 of patients with AR showed reversibility to bronchodilation testing (defined as an increase of ⬎12% in
May–June 2014, Vol. 28, No. 3
Copyright © Oceanside Publications, Inc. All rights reserved. DO NOT COPY
For permission to copy go to https://www.oceansidepubl.com/permission.htm
basal FEV1 values), despite having normal baseline FEV1 measurements. In fact, a forced expiratory flow at 25–75% value of ⱕ58.5%
predicts BHR and reversibility in patients with AR, and an FEV1
⬍83% is a good marker for early bronchial impairment in children
with AR.20,21
Four mechanisms have been postulated to account for the relationship between asthma and AR.22 First, the nose, by virtue of its anatomic location, warms, filters, and humidifies inhaled air. In fact,
exercise-induced bronchospasm is caused by cooling and drying in
the airways, which occurs with obligate mouth breathing during
vigorous activity. In addition, via the numerous submucosal glands
located in the nasal passages, the nose is able to sterilize air through
the release of antibacterial enzymes. With AR, nasal function may be
partially or completely lost as the congestion forces the patient to
become a mouth breather. Second, during an exacerbation of AR, the
inflammatory products from the upper airways may be aspirated
directly into the lower airways. Third, nasal inflammation may result
in local cytokine release into the bloodstream, which eventually
causes bronchoconstriction in the lower airways. Fourth, a nasal–
bronchial reflex may exist, where histamine and bradykinin stimulate
the afferent nasal sensory nerve. The neural signal then travels to the
central nervous system and activates the efferent vagus nerve, resulting in bronchial smooth muscle hyperreactivity.
The etiology for the connection between asthma and AR is likely
multifactorial. The data supporting a nasal–bronchial reflex is controversial. Although nasal blockage and aspiration of nasal contents
have long been accepted as contributing factors, there is a growing
body of evidence that suggests that a systemic response plays an
important role in the AR–asthma relationship. For example, in patients with seasonal AR but no asthma, nasal allergen testing not only
instigates bronchial airway responsiveness, but also increases eosinophil counts in the sputum samples of these patients.23
In another study, bronchial and nasal biopsy specimens were taken
before and 24 hours after nasal allergen testing in patients with AR.
At the 24-hour time point, there was an increase in eosinophils in both
the nasal and the bronchial epithelium.24 By the same token, segmental bronchial provocation in nonasthmatic AR patients resulted in
inflammation in the nose, as well as an increase in peripheral blood
eosinophilia.25 Supporting the idea from a different angle, a study
showed that eosinophil infiltration was present on nasal biopsy in
asthmatic patients who did not have AR.26 Ultimately, the eosinophilia, in both the upper and the lower airways results from an
increase in inflammatory cytokines, especially IL-5.27,28
If AR and asthma are linked, then it should not be surprising that
treating AR will also improve asthma symptoms. This was first
noticed in 1984, when intranasally administered beclomethasone and
flunisolide were, in asthmatic patients, found to markedly reduce
self-reports of shortness of breath and wheezing.29 Subsequent more
quantitative studies supported this notion. For instance, 4 weeks of
intranasal budesonide was found to reduce the severity of exerciseinduced asthma in children, as measured by FEV1,30 and 5 weeks of
intranasal beclomethasone led to a decrease in bronchial responsiveness.31
Furthermore, in a separate crossover study, patients with AR but
no asthma were found to have decreased bronchial hyperresponsiveness after 2 weeks of intranasal beclomethasone, but no change from
baseline after 2 weeks of bronchial beclomethasone.32Although these
studies, taken together, suggest that treating AR will help control
asthma, it is important to note that their sample sizes were small,
ranging from 11 to 26 patients. Indeed, a much larger study of 262
subjects randomized patients to either 6 weeks of intranasal fluticasone, inhaled fluticasone, their combination, or inhaled placebo, and
found that only inhaled fluticasone—and not intranasal fluticasone—
was effective in controlling bronchial reactivity.33 Thus, whether nasal
steroids are effective in treating asthma is still subject to debate.
Although nasal corticosteroids are of questionable efficacy, antihistamines, the first-line treatment for AR, have been shown to be highly
American Journal of Rhinology & Allergy
effective in treating asthma. Antihistamines, when compared with
nasal corticosteroids, are systemic, rather than local, medications that
directly target the histamine receptors on mast cells and T cells, in the
process stabilizing these cells and promoting anti-inflammatory activities. The presence of histamine receptors in both the nasal passages and the lungs, and the fact that AR and asthma are simultaneously improved with antihistamines, provides further support for the
united airway hypothesis.
One of the first large studies indicating that an antihistamine treats
AR as well as asthma randomized 186 patients with both conditions
to receive placebo or cetirizine, a second-generation H1-antagonist,
for 6 weeks.34 Cetirizine-treated patients reported a significant improvement in chest tightness, wheezing, shortness of breath, cough,
and nocturnal asthma when compared with controls. Similarly, a
study published by Spector et al.35 evaluated pulmonary function tests
in 12 asthmatic patients who were given varying doses of oral cetirizine (5,10, and 20 mg) as well as albuterol. All three cetirizine doses
were found to significantly improve pulmonary function measures
throughout the 8-hour testing period and provided a demonstrable
bronchodilatory effect. At the same time, the administration of both
albuterol and cetirizine appears to have an additive bronchodilatory
effect. And finally, Ubier et al.36 randomized asthmatic patients to
either cetirizine at 10 mg daily or placebo for 2 weeks, after which
there was a marked improvement in bronchial hyperresponsiveness,
as measured by methacholine challenge.
The use of antihistamines in combination with other medications
has also shown promise in asthma treatment. In a randomized trial
conducted by Corren et al.,37 193 patients with a history of seasonal
AR and asthma were administered a combination of loratadine and
pseudoephedrine, or placebo, for 6 weeks. Both groups were evaluated daily for nasal symptoms, chest symptoms, albuterol use, and
peak expiratory flow rates, as well as with weekly spirometry. By the
end of the study, the total nasal symptom score, total asthma symptom score, peak expiratory flow rates, weekly FEV1, and asthma
quality of life measures were all significantly improved when compared with placebo.
Ultimately, the treatment of AR not only reduces the physical
symptoms of asthma, but also has beneficial socioeconomic consequences. One retrospective cohort study examined, over the course of
a year, the rate of asthma-related emergency room (ER) visits and
hospitalizations in patients with asthma and AR after they were
treated with AR medications.38 Of 4944 subjects, 3587 patients were
treated for AR, and 1357 patients were untreated. Asthma-related
hospitalizations fell from 2.3 to 0.9 (61% decrease), and the incidence
of two or more asthma-related ER visits decreased from 1.3 to 0.6 per
patient (54% reduction). These findings were corroborated in a casecontrol study, which showed that patients with both AR and asthma,
who were treated with intranasal corticosteroids, had a significantly
lower risk of both asthma-related ER visits and hospitalizations.39
Moreover, treatment with both intranasal steroids and second-generation antihistamines was associated with an even lower risk of ER
visits or hospitalization.
ALLERGIC RHINITIS AND SINUSITIS
Patients afflicted with allergies have a predisposition for developing sinusitis. One study determined that both disorders exist in the
same patient 25–70% of the time,40 and another study found that 72 of
121 patients with chronic nasal symptoms and positive skin tests for
allergies had positive sinus computed tomography scans showing
sinusitis.41 By the same token, asthma severity is associated with a
more severe clinical presentation of rhinosinusitis.42 Moreover, Berrettini et al.43 found a statistically significant increase in sinusitis on
computed tomography scans in patients with perennial AR when
compared with a control group, and Baroody et al.44 determined that
nasal allergen challenge induced eosinophilic inflammation in the
maxillary sinus. Finally, in a cohort of patients with chronic sinusitis
Copyright © Oceanside Publications, Inc. All rights reserved. DO NOT COPY
For permission to copy go to https://www.oceansidepubl.com/permission.htm
S83
who were challenged with nasal allergen provocation tests, 41 positive nasal responses occurred in 29 patients. Of the 41 nasal responses,
31 were associated with radiographic changes on Water’s view sinus
radiographs, including increased mucosal edema and opacification of
the sinuses, suggesting that nasal allergens trigger changes in the
mucosal membranes of patients with sinusitis.45
The etiology of the link between AR and sinusitis is, akin to the
etiology between AR and asthma, likely multifactorial. Anatomically,
patients with AR have edematous nasal mucosa, damaged nasal cilia,
and overproduction of secretions, which could lead to a blockage of
ostial drainage from the sinuses. This blockage results in stagnant
debris that then becomes infected. From an immunologic perspective,
eosinophils, more prevalent during an AR flare, can cause chronic
inflammation in the mucosa, even when bacteria are not present.43
Notably, patients with both allergies and sinusitis, when compared
with patients with nonallergic sinusitis, have a distinct cytokine profile, with nasal polyp tissue that shows an increase in granulocyte
macrophage colony-stimulating factor, IL-3, IL-4, and IL-5, along with
an increased density of CD3⫹ T lymphocytes.46
Although chronic rhinosinusitis (CRS) can develop independently
of allergic pathways, there is a group of patients, diagnosed with
allergic fungal rhinosinusitis (AFRS), whose sinusitis and nasal polyposis are related to allergic inflammation. Hutcheson et al.47 compared the antibody responses in 64 patients with AFRS to 35 patients
with CRS and found no evidence of allergic disease. In the AFRS
cohort, serum total IgE, mean IgG anti–Alternaria-specific antibodies,
and the mean number of IgE antifungal antibody bands on immunoblotting, were all increased, showing that AFRS is a distinct entity
from CRS, with a unique allergic etiology. Although the existence of
AFRS does not necessarily support the notion of a united airway, the
fact that sinusitis can arise directly from allergic inflammation indicates the close relationship between allergies and rhinosinusitis.
Of course, AFRS is not the only clinical entity associated with nasal
polyposis—nasal polyps are found in a number of other diseases,
including cystic fibrosis, aspirin-exacerbated respiratory disease, and
Churg-Strauss syndrome.48 Furthermore, a retrospective study revealed that of 4986 hospitalized patients, 6.7% of asthmatic patients,
5% of CRS patients, and 2.2% of rhinitis patients had nasal polyps.49
Interestingly, IgE-mediated pathways are thought to play a role in the
pathogenesis of nasal polyposis, providing further support for the
connection between allergies and sinusitis. Specifically, Bernstein et
al.50 discovered increased serum levels of IgE antibodies to both
Staphylococcal enterotoxin B and toxic shock syndrome toxin in CRS
patients with nasal polyps, when compared with controls. Moreover,
there were high levels of IgE against Staphylococcal enterotoxin A and
B in the nares of these same patients. Thus, Staphylococcus aureus
exotoxins may act as superantigens in the nasal mucosa of CRS
patients. Subsequently, IgE antibodies directed against these exotoxins create a local allergic inflammatory reaction, resulting in the
growth of nasal polyps. Indeed, the presence of S. aureus actively
affects the clinical course of rhinosinusitis by augmenting the inflammatory response in nasal polyposis while also increasing local IgE
production in the nares.
attention should be given to the management of any concurrent AR as
well.
The information we have, to date, although promising, leaves a
number of questions that still require addressing. What is the exact
inflammatory cascade that leads a patient with AR to independently
have bronchial hyperactivity, and vice versa? Has the severity and
epidemiology of the diseases changed with the onset of a new generation of medications? What is the optimal treatment of patients with
both AR and asthma? And how can we better elucidate the relationship between sinusitis and asthma? We eagerly await the answers in
future studies.
REFERENCES
1.
2.
3.
4.
5.
6.
7.
8.
9.
10.
11.
12.
13.
14.
15.
16.
17.
CONCLUSION
The aggregation of research suggests that AR and asthma are, in
fact, one syndrome in two parts of the respiratory tract—this notion is
supported pathophysiologically, epidemiologically, and through numerous clinical studies. Being afflicted with AR is often the harbinger
of asthma at a future date. By the same token, allergies are associated
with an increased likelihood of having sinusitis, with a common
pathophysiology, and possibly treatment, tying the two disorders
together. Ultimately, we can posit that the united airway—where AR,
asthma, and sinusitis are inextricably linked—truly exists. Thus,
when entertaining a diagnosis of asthma, an evaluation of the upper
airways should be considered.51 In addition, when treating sinusitis,
S84
18.
19.
20.
21.
Broide DH. Allergic rhinitis: Pathophysiology. Allergy Asthma Proc
31:370–374, 2010.
Sibbald B, and Rink E. Epidemiology of seasonal and perennial
rhinitis: Clinical presentation and medical history. Thorax 46:895–
901, 1991.
Wright AL, Holberg CJ, Martinez FD, et al. Epidemiology of physician-diagnosed allergic rhinitis in childhood. Pediatrics 94:895–901,
1994.
Nathan RA, Meltzer EO, Selner JC, and Storms W. Prevalence of
allergic rhinitis in the United States. J Allergy Clin Immunol 99:S808–
S814, 1997.
Dykewicz MS, Fineman S, Skoner DP, et al. Diagnosis and management of rhinitis: Complete guidelines of the Joint Task Force on
Practice Parameters in Allergy, Asthma and Immunology. Am Acad
Allergy Asthma Immunol 81:478–518, 1997.
Centers for Disease Control. Asthma in the US: Growing every year.
Available online at www.cdc.gov/vitalsigns/asthma; accessed August 24, 2011.
Leynaert B, Neukirch C, Liard R, et al. Quality of life in allergic
rhinitis and asthma: A population-based study of young adults. Am J
Respir Crit Care Med 162:1391–1396, 2000.
Blaiss MS. Allergic rhinitis: Direct and indirect costs. Allergy Asthma
Proc 31:375–380, 2010.
Bernstein JA. Allergic and mixed rhinitis: Epidemiology and natural
history. Allergy Asthma Proc 31:365–369, 2010.
Marple BF. Allergic rhinitis and inflammatory airway disease: Interactions within the unified airspace. Am J Rhinol Allergy 24:249–254,
2010.
Guerra S, Sherrill DL, Martinez FD, and Barbee RA. Rhinitis is an
independent risk factor for adult-onset asthma. J Allergy Clin Immunol 109:419–425, 2002.
Bosquet J; and the ARIA Working Group. Allergic rhinitis and its
impact on asthma. J Allergy Clin Immunol 108:S147–S334, 2001.
Stevenson DD, Mathison DA, Tan EM, and Vaughan JH. Provoking
factors in bronchial asthma. Arch Intern Med 135:777–783, 1975.
Settipane RJ, Hagy GW, and Settipane GA. Long-term risk factors for
developing asthma and allergic rhinitis: A 23-year follow-up study of
college students. Allergy Proc 15:21–25, 1994.
Huovinen E, Kaprio J, Laitinen LA, and Koskenvuo M. Incidence and
prevalence of asthma among adult Finnish men and women of the
Finnish Twin Cohort from 1975 to 1990, and their relation to hay
fever and chronic bronchitis. Chest 115:928–936, 1999.
Braman SS, Barrows AA, DeCotiis BA, et al. Airway hyperresponsiveness in allergic rhinitis: A risk factor for asthma. Chest 91:671–
674, 1987.
Ciprandi G, Cirillo I, Tosca MA, and Vizzaccaro A. Bronchial hyperreactivity and spirometric impairment in patients with perennial
allergic rhinitis. Int Arch Allergy Immunol 133:14–18, 2004.
Ciprandi G, Cirillo I, Tosca MA, and Vizzaccaro A. Bronchial hyperreactivity and spirometric impairment in patients with seasonal allergic rhinitis. Respir Med 98:826–831, 2004.
Ciprandi G, Cirillo I, Pistorio A, et al. Impact of allergic rhinitis on
asthma: Effects on bronchodilation testing. Ann Allergy Asthma
Immunol 101:42–46, 2008.
Ciprandi G, Capasso M, and Tosca MA. Early bronchial involvement
in children with allergic rhinitis. Am J Rhinol Allergy 25:e30–e33,
2011.
Ciprandi G, Signori A, and Cirillo I. Relationship between bronchial
hyperreactivity and bronchodilation in patients with allergic rhinitis.
Ann Allergy Asthma Immunol 106:460–466, 2011.
May–June 2014, Vol. 28, No. 3
Copyright © Oceanside Publications, Inc. All rights reserved. DO NOT COPY
For permission to copy go to https://www.oceansidepubl.com/permission.htm
22.
23.
24.
25.
26.
27.
28.
29.
30.
31.
32.
33.
34.
35.
Togias A. Mechanisms of nose-lung interaction. Allergy 54:95–105,
1999.
Bonay M, Neukirch C, Grandsaigne M, et al. Changes in airway
inflammation following nasal allergic challenge in patients with seasonal rhinitis. Allergy 61:111–118, 2006.
Braunstahl GJ, Overbeek SE, Kleinjan A, et al. Nasal allergen provocation induces adhesion molecule expression and tissue eosinophilia in upper and lower airways. J Allergy Clin Immunol 107:469–
476, 2001.
Braunstahl GJ, Kleinjan A, Overbeek SE, et al. Segmental bronchial
provocation induces nasal inflammation in allergic rhinitis patients.
Am J Respir Crit Care Med 161:2051–2057, 2000.
Gaga M, Lambrou P, Papageorgiou N, et al. Eosinophils are a feature
of upper and lower airway pathology in non-atopic asthma, irrespective of the presence of rhinitis. Clin Exp Allergy 30:663–669, 2000.
Becky Kelly EA, Busse WW, and Jarjour NN. A comparison of the
airway response to segmental antigen bronchoprovocation in atopic
asthma and allergic rhinitis. J Allergy Clin Immunol 111:79–86, 2003.
Kurt E, Aktas A, Gulbas Z, et al. The effects of natural pollen
exposure on inflammatory cytokines and their relationship with nonspecific bronchial hyperresponsiveness in seasonal allergic rhinitis.
Allergy Asthma Proc 31:126–131, 2010.
Welsh PW, Stricker WE, Chu CP, et al. Efficacy of beclomethasone
nasal solution, flunisolide, and cromolyn in relieving symptoms of
ragweed allergy. Mayo Clin Proc 62:125–134, 1987.
Henriksen JM, and Wenzel A. Effect of an intranasally administered
corticosteroid (budesonide) on nasal obstruction, mouth breathing,
and asthma. Am Rev Respir Dis 140:1014–1018, 1984.
Corren J, Adinoff AD, Buchmeier AD, and Irvin CG. Nasal beclomethasone prevents the seasonal increase in bronchial responsiveness in patients with allergic rhinitis and asthma. J Allergy Clin
Immunol 90:250–256, 1992.
Aubier M, Levy J, Clerici C, et al. Different effects of nasal and
bronchial glucocorticosteroid administration on bronchial hyperresponsiveness in patients with allergic rhinitis. Am Rev Respir Dis
146:122–126, 1992.
Dahl R, Nielsen LP, Kips J, et al. Intranasal and inhaled fluticasone
propionate for pollen-induced rhinitis and asthma. Allergy 60:875–
881, 2005.
Grant JA, Nicodemus CF, Findlay SR, et al. Cetirizine in patients with
seasonal rhinitis and concomitant asthma: Prospective, randomized,
placebo-controlled trial. J Allergy Clin Immunol 95:923–932, 1995.
Spector SL, Nicodemus CF, Corren J, et al. Comparison of the bronchodilatory effects of cetirizine, albuterol, and both together versus
placebo in patients with mild-to-moderate asthma. J Allergy Clin
Immunol 96:174–181, 1995.
American Journal of Rhinology & Allergy
36.
Aubier M, Neukirch C, Peiffer C, and Melac M. Effect of cetirizine on
bronchial hyperresponsiveness in patients with seasonal allergic rhinitis and asthma. Allergy 56:35–42, 2001.
37. Corren J, Harris AG, Aaronson D, et al. Efficacy and safety of loratadine plus pseudoephedrine in patients with seasonal allergic rhinitis
and asthma. J Allergy Clin Immunol 100:781–788, 1997.
38. Crystal-Peters J, Neslusan C, Crown WH, and Torres A. Treating
allergic rhinitis in patients with comorbid asthma: The risk of asthma-related hospitalizations and emergency department visits. J Allergy Clin Immunol 109:57–62, 2002.
39. Corren J, Manning BE, Thompson SF, et al. Rhinitis therapy and the
prevention of hospital care for asthma: A case-control study. J Allergy Clin Immunol 113:415–419, 2004.
40. Furukawa CT. The role of allergy in sinusitis in children. J Allergy
Clin Immunol 90:515–517, 1992.
41. Iwens P, and Clement PA. Sinusitis in allergic patients. Rhinology
32:65–67, 1994.
42. Lin DC, Chandra RK, Tan BK, et al. Association between severity of
asthma and degree of chronic rhinosinusitis. Am J Rhinol Allergy
25:205–208, 2011.
43. Berrettini S, Carabelli A, Sellari-Franceshini S, et al. Perennial allergic
rhinitis and chronic sinusitis: Correlation with rhinologic risk factors.
Allergy 54:242–248, 1999.
44. Baroody FM, Mucha SM, Detineo M, and Naclerio RM. Nasal challenge with allergen leads to maxillary sinus inflammation. J Allergy
Clin Immunol 121:1126–1132, 2008.
45. Pelikan Z, and Pelikan-Filipek M. Role of nasal allergy in chronic
maxillary sinusitis—Diagnostic value of nasal challenge with allergen. J Allergy Clin Immunol 86:484–491, 1990.
46. Hamilos DL, Leung DY, Wood R, et al. Evidence for distinct cytokine
expression in allergic versus nonallergic chronic sinusitis. J Allergy
Clin Immunol 96:537–544, 1995.
47. Hutcheson PS, Schubert MS, and Slavin RG. Distinctions between
allergic fungal rhinosinusitis and chronic rhinosinusitis. Am J Rhinol
Allergy 24:405–408, 2010.
48. Settipane GA. Epidemiology of nasal polyps. Allergy Asthma Proc
17:231–236, 1996.
49. Settipane GA, and Chafee FH. Nasal polyps in asthma and rhinitis: A
review of 6,037 patients. J Allergy Clin Immunol 59:17–21, 1977.
50. Bernstein JM, Allen C, Rich G, et al. Further observations on the role
of Staphylococcus auerus exotoxins and IgE in the pathogenesis of
nasal polyposis. Laryngoscope 121:647–655, 2011.
51. Slavin RG. The upper and lower airways: The epidemiological and
pathophysiological connection. Allergy Asthma Proc 29:553–556,
2008.
e
Copyright © Oceanside Publications, Inc. All rights reserved. DO NOT COPY
For permission to copy go to https://www.oceansidepubl.com/permission.htm
S85
Immunomodulation of allergic sinonasal disease
Russell A. Settipane, M.D.,1 Anju T. Peters, M.D.,2 and Larry Borish, M.D.3
ABSTRACT
IgE hypersensitivity is important to the pathogenesis of allergic diseases and the development and persistence of airway inflammation. Allergic
immunomodulation encompasses various therapies that attempt to suppress or modify the immune mechanisms responsible for IgE-mediated disease. These
include allergy immunotherapy (AIT) in the forms of subcutaneous immunotherapy (SCIT) and sublingual immunotherapy (SLIT), as well as the emergence
of biological agents, such as anti-IgE, for allergic respiratory disease. Clinical evidence strongly supports the efficacy and safety of AIT for the treatment of
allergic rhinitis, allergic conjunctivitis, and allergic asthma, but for chronic rhinosinusitis evidence is lacking. In allergic rhinitis, the decision to initiate AIT
depends on the degree to which symptoms can be reduced by avoidance and medication, the amount and type of medication required to control symptoms, the
adverse effects of medication, the severity and duration of symptoms, and their effect on quality of life. AIT has the potential to produce sustained long-lasting
immune modulation and possibly avoid or reduce lifelong requirements for medical therapy. Although SLIT is currently being evaluated, SCIT remains the
preferred form of AIT in the United States because of robust efficacy data, availability of allergen extracts, and current Food and Drug Administration approval.
However, SLIT holds the potential for greater patient safety and convenience. Other immunomodulators such as anti-IgE also hold promise, but require further
investigation.
A
llergic immunomodulation encompasses various third-line therapies that attempt to suppress or modify the immune mechanisms responsible for IgE-mediated disease. These include allergy
immunotherapy (AIT) in the forms of subcutaneous immunotherapy
(SCIT) and sublingual immunotherapy (SLIT), as well as the emergence of biological agents for upper and lower airway disease such as
anti-IgE.1–3 AIT is defined as the repeated administration of relevant
allergens4 to patients with IgE-mediated conditions for the purpose of
providing protection against allergic symptoms and inflammatory
reactions associated with the natural exposure to the these allergens.1
In this article, SCIT, SLIT, and anti-IgE are briefly reviewed; however,
for a more in-depth discussion regarding AIT, the reader is referred to
recently published practice parameters relating to rhinitis5 and AIT1
as well as a recent AIT consensus report.6
SUBCUTANEOUS IMMUNOTHERAPY
SCIT: Description and Indications
SCIT, commonly referred to as “allergy shots,” is the oldest form of
AIT, dating back over 100 years, where allergen is administered
subcutaneously for the purpose of inducing allergen-specific immune
tolerance.7 In allergic rhinitis,8 SCIT may be considered if symptoms
are not controlled by allergen avoidance and pharmacotherapy or if
the patient prefers not to take medications or has medication-induced
adverse effects. In addition, SCIT may be considered if the patient
desires to avoid or reduce the need for long-term pharmacotherapy.1
However, before SCIT can be considered, it is imperative that the
From the 1Department of Medicine, Warren Alpert Medical School of Brown University, Providence, Rhode Island, 2Division of Allergy-Immunology, Northwestern University, Chicago, Illinois, and 3Department of Medicine, Asthma and Allergic Disease
Center, Carter Immunology Center, University of Virginia Health System, Charlottesville, Virginia
RA Settipane received a research grant from Genentech (clinical trial). AT Peters is a
speaker for Baxter. L Borish is funded by NIH RO1 AI057483 and UO1 AI100799
Address correspondence and reprint requests to Russell A. Settipane, M.D., Allergy &
Asthma Center, 95 Pitman Street, Providence, RI 02906
E-mail address: [email protected]
Originally published in Am J Rhinol Allergy 27, S59 –S62, 2013
Copyright © 2014, OceanSide Publications, Inc., U.S.A.
S86
patient has evidence of sensitization to relevant aeroallergens on
either skin9 or in vitro10 testing in the context of a compatible clinical
history.11 It is not appropriate to recommend AIT based solely on
results of skin testing or in vitro–specific IgE tests, without appropriate clinical correlation.1
With regard to chronic rhinosinusitis (CRS) and nasal polyps12,13
the role of an allergic contribution is uncertain.14 Guideline recommendations for the use of SCIT in CRS are assigned the lowest
strength, level D, because of limited evidence.15 There are no randomized prospective studies on SCIT in CRS; one retrospective study
indicated that immunotherapy may decrease the use of antibiotics
and improve sinonasal symptoms in patients with recurrent rhinosinusitis.16 Unfortunately, this study was limited not only by its retrospective nature, but also by the absence of objective assessment of
sinusitis severity. However, based on a recent report of SCIT efficacy
in mixed rhinitis,17 an argument can perhaps be made for a trial of
SCIT in select patients with coexisting CRS and allergic rhinitis.
SCIT: Mechanism
The immunologic response to SCIT is characterized by decreased
sensitivity of end organs and changes in the humoral and cellular
responses to the administered allergens. A number of possible mechanisms for the beneficial effects of immunotherapy have been suggested. Key to this process is the induction of a T-cell–tolerant state.6
Allergen-specific peripheral T-cell tolerance mediated by IL-10 and
transforming growth factor ␤ causes deviation toward a regulatory T
(Treg) cell response, which leads to a normal, healthy immune response to mucosal antigens. In addition to mediating T-cell tolerance,
IL-10 regulates specific antibody isotype formation and skews the
specific response from an IgE- to an IgG4-dominated phenotype. Treg
cells acting through their production of IL-10 suppress both total and
allergen-specific IgE and simultaneously increase IgG4 production.
Observed immunologic changes (Table 1)18–20 in response to SCIT
include the modulation of T- and B-cell responses by the generation of
allergen-specific Treg cells; increases in allergen-specific IgG4, and
IgA; decrease in IgE; and decreased tissue infiltration of mast cells
and eosinophils. Additionally, successful SCIT is associated with a
change toward a nonallergic Th1 cytokine profile (Th1 skewing),
which occurs within the constraints of a high IL-10 milieu, allowing
the associated interferon ␥ responses to ameliorate allergic inflamma-
May–June 2014, Vol. 28, No. 3
Copyright © Oceanside Publications, Inc. All rights reserved. DO NOT COPY
For permission to copy go to https://www.oceansidepubl.com/permission.htm
Table 1 Immunologic changes associated with SCIT
Antibody changes
Increase in allergen-specific IgG (specifically IgG4)
Early increase and late decrease in serum allergen-specific IgE
Decrease in seasonal rise of allergen-specific IgE
Cellular changes
Decreased mediator release from mast cells, basophils, and
eosinophils
Reduction of tissue mast cells and eosinophils
Induction of regulatory T cells and suppression of Th2 ⬎ Th1
cells
Increased secretion of IL-10 and TGF-␤
Decrease in histamine-releasing factors
Source: Adapted from Refs. 15–17.
SCIT ⫽ subcutaneous immunotherapy; TGF ⫽ transforming growth factor.
tion without producing the associated proinflammatory influences of
the interferon.21
SCIT: Efficacy and Safety in Allergic Rhinitis
Clinical research has been robust with multiple double-blind, placebo-controlled, randomized clinical trials and meta-analyses indicating
SCIT to be effective in a dose-dependent manner for the nasal and
ocular symptoms of allergic rhinitis.1,22 Additionally, SCIT acts as a
long-lasting disease modifier of allergic rhinitis altering the natural
history of the disease, having been shown to result in sustained
benefit after discontinuation. Double-blind, placebo-controlled trials
have shown that 3–4 years of grass pollen AIT remains effective for at
least 3 years and up to 12 years after the discontinuation of the
injections.6,23 In children SCIT has been shown to prevent the development of new sensitizations and possibly to prevent new onset
asthma.1
SCIT-associated anaphylactic fatalities are rare (1 in 2.5 million
injections) with ⬃3.4 fatal reactions annually in the United States.24
Large local reactions at the site of the injection are much more
common, occurring in ⬃9% of injection visits.25 An important area of
clinical research is the relationship between large local reactions and
systemic reactions. Unfortunately, evidence regarding the risk of
systemic anaphylaxis in patients with local reactions is limited to
retrospective studies, which are somewhat contradictory25,26; however, it appears that large local reactions do not predict the occurrence
of a systemic anaphylactic reaction in the subsequent dose and that
dose adjustments based on local reactions do not appear to prevent
systemic reactions.
Given the risk of anaphylaxis, SCIT should be administered only in
a setting where the prompt recognition and treatment of anaphylaxis
(with epinephrine) is available,27–30 the preferred location being in the
medical facility of the physician who prepared the patient’s AIT
extract. Patients should remain in the supervised medical facility and
be monitored for at least 30 minutes postinjection.1 Some experts
advocate for prescribing automatic injectable epinephrine to all SCIT
patients to address the rare occurrences of anaphylaxis beyond the
30-minute wait time.30
Risk factors for severe SCIT reactions include symptomatic asthma,
concomitant use of ␤-adrenergic blockers, and administration of injections during the height of the pollen season.1 Patients with asthma
must be assessed for degree of control before the administration of
each SCIT injection.
schedule interval is slowly increased to a range of every 2–4 weeks.
Based on studies that show long-lasting disease modification and
sustained benefit after SCIT is discontinued, AIT guidelines recommend 3–5 years duration of treatment. It is the persistence of benefit
and apparent alteration of the natural history of allergic rhinitis that
underlies SCIT’s cost savings in comparison with standard pharmacotherapies.31,32 Studies comparing cost-effectiveness between patients treated for 3 years with SCIT versus those treated with pharmacotherapy alone have indicated a potential cost savings as high as
80% with SCIT.32 Despite these benefits, adhering to a 3- to 6-month
long weekly build-up schedule can be challenging for patients. This
has led to investigations with ultrashort (4 dose) courses33 and accelerated schedules (referred to as “cluster” or “rush” schedules)1; however, the advantage of convenience associated with accelerated schedules is partially negated by higher rates of systemic allergic reactions
that range from 14.7 to 38% in premedicated subjects (corticosteroids/
antihistamines).1,34
SUBLINGUAL IMMUNOTHERAPY
SLIT: Description and Mechanism
SLIT refers to sublingual application of allergen for the purpose of
inducing allergen-specific tolerance. This form of AIT has the potential to be more convenient and safer than SCIT. Although considered
experimental in the United States, SLIT is widely used in Europe, and
variations of SLIT are currently being used by some U.S. practitioners.1 The precise mechanisms by which SLIT works remain unclear,
but similar to SCIT, it likely includes promoting modified Th1 and
Treg activity.35
SLIT: Efficacy and Safety
Clinical trials now underway in the United States have demonstrated efficacy in adults and children but are limited to the study of
single allergen preparations such as grass or ragweed pollen.36,37 A
2010 meta-analysis of SLIT revealed significant symptom improvement with SLIT for both seasonal and perennial allergic rhinitis.38
Similar to SCIT, SLIT appears to result in sustained long-lasting
therapeutic benefits. In the patients receiving SLIT for at least 3 years,
the clinical benefit persisted for at least 7 years and there were 75%
less new sensitizations in SLIT-treated patients compared with controls.39 A 2013 systemic review of 63 studies with 5131 participants
concluded that the available data provides a moderate grade level of
evidence to support the effectiveness of sublingual immunotherapy
for the treatment of allergic rhinitis, but high-quality studies are still
needed to answer questions regarding optimal dosing strategies.40 A
2012 meta-analysis of studies comparing SCIT versus SLIT in patients
with seasonal allergic rhinoconjunctivitis to grass pollen provides
solid evidence that although SCIT is more effective than SLIT in
controlling symptoms and in reducing the use of antiallergic medications, this higher efficacy occurs at the expense of a substantially
greater anaphylaxis rate with SCIT.41 Although SLIT appears to have
a more favorable safety profile, local reactions (oral pruritus and
edema) are common, occurring in 40% of patients; and anaphylaxis,
although rare, has been reported.1 Several barriers exist for the implementation of SLIT, the foremost being availability. No reagents are
currently available in the United States; and only a few single allergen
preparations are in development, making this an unrealistic option
for polysensitized (i.e. most) allergic rhinits patients.
MONOCLONAL ANTI-IgE
SCIT: Administration Schedules
Monoclonal Anti-IgE: Description
SCIT is usually initiated with injections administered one to three
times weekly starting at a low dose unlikely to cause anaphylaxis.
After 3–6 months the maintenance phase begins, and the injection
The sole Food and Drug Administration (FDA)–approved anti-IgE
therapy in the United States is omalizumab, which is a recombinant
humanized monoclonal anti-IgE antibody. Omalizumab is FDA ap-
American Journal of Rhinology & Allergy
Copyright © Oceanside Publications, Inc. All rights reserved. DO NOT COPY
For permission to copy go to https://www.oceansidepubl.com/permission.htm
S87
proved only for adults and adolescents (aged ⱖ12 years) with moderate-to-severe persistent asthma who have a positive skin test or in
vitro reactivity to a perennial aeroallergen, and whose symptoms are
inadequately controlled with inhaled corticosteroids.42
Monoclonal Anti-IgE: Mechanism
By selectively targeting and binding to circulating IgE, omalizumab
therapy results in a reduction of IgE binding to receptors on mast
cells, basophils, and dendritic cells43,44 and a down-regulation of their
expression of cell surface IgE receptors. This ultimately leads to a
decrease in the release of mediators in response to allergen exposure.
The end result is a reduction of both the acute (early phase) allergic
response and the subsequent (late-phase response) inflammatory and
physiological consequences.45
• SLIT, although not currently approved for use in the United States,
has been shown to be more effective than placebo; and available
data suggest that SLIT is safer than SCIT.
REFERENCES
1.
2.
3.
4.
5.
Monoclonal Anti-IgE: Efficacy and Safety
Clinical data for the use of omalizumab in asthma have been robust,
preliminary trials performed in patients with upper respiratory diseases have also demonstrated efficacy in patients with seasonal and
perennial allergic rhinitis.46–49 Omalizumab has been shown to be an
effective adjunct to SCIT.50 Adding omalizumab to SCIT improves its
safety and tolerability during build-up, the likelihood of the patient
reaching the maintenance phase, and the therapy’s overall effectiveness.6 Finally, anti-IgE for CRS with nasal polyps holds promise, but
the two trials reported thus far have not demonstrated the same
degree of benefit.3,51
Safety concerns with omalizumab include anaphylaxis, which has
resulted in the FDA issuing a black box warning. This anaphylaxis
can be associated with a protracted course and delayed onset of
symptoms even 12–24 hours after an injection.52,53 Retrospective evaluation of omalizumab-associated anaphylaxis cases has not identified
potential risk factors to identify patients at risk.53 In addition, initial
clinical trials suggested a higher rate of malignancy associated with
omalizumab; however, a recent pooled analysis of a larger number of
patients does not show a causal link between omalizumab and malignancy.54 Further investigation is necessary to define efficacy, safety,
and cost-effectiveness in upper respiratory disease states.
6.
7.
8.
9.
10.
11.
12.
13.
14.
15.
SUMMARY
16.
IgE hypersensitivity is important to the pathogenesis of allergic diseases and the development and persistence of airway inflammation.
Clinical evidence strongly supports the efficacy and safety of SCIT for
the treatment of allergic respiratory diseases including allergic rhinitis, allergic conjunctivitis, and allergic asthma, but for CRS evidence is
lacking. Although not quite as robust, efficacy and safety data for
SLIT in allergic rhinitis and asthma is growing. AIT should be considered in allergic rhinitis patients who experience poor symptom
control or adverse effects resulting from medications. SCIT remains
the preferred form of AIT in the United States because of robust
efficacy data, availability of allergen extracts, and current FDA approval. However, SLIT holds the potential for greater patient safety
and convenience. Other immunomodulators such as anti-IgE also
hold promise, but require further investigation.
17.
18.
19.
20.
21.
CLINICAL PEARLS
• With regard to respiratory allergy, specific AIT is indicated for the
treatment of seasonal and perennial allergic rhinitis, but not for
CRS.
• Most systemic reactions to SCIT usually occur within 30 minutes of
treatment. Therefore, patients should wait in a medical facility for a
full 30 minutes after a SCIT injection.
• Asthma control must be assessed before each SCIT injection in
patients who also have asthma. AIT injections are contraindicated
in poorly controlled asthma.
S88
22.
23.
24.
25.
Cox L, Nelson H, Lockey R, et al. Allergen immunotherapy: A
practice parameter third update. J Allergy Clin Immunol 127(suppl):
S1–S55, 2011.
Wise SK, and Schlosser RJ. Subcutaneous and sublingual immunotherapy for allergic rhinitis: What is the evidence? Am J Rhinol
Allergy 26:18–22, 2012.
Gevaert P, Calus L, Van Zele T, et al. Omalizumab is effective in
allergic and nonallergic patients with nasal polyps and asthma. J
Allergy Clin Immunol 131:110–116.e1, 2013.
Shah R, and Grammer LC. An overview of allergens. Allergy Asthma
Proc 33(suppl 1):S2–S5, 2012.
Wallace DV, Dykewicz MS, Bernstein DI, et al. The diagnosis and
management of rhinitis: An updated practice parameter. J Allergy
Clin Immunol 122:S1–S84, 2008.
Burks AW, Calderon MA, Casale T, et al. Update on allergy immunotherapy: American Academy of Allergy, Asthma & Immunology/
European Academy of Allergy and Clinical Immunology/PRACTALL
consensus report. J Allergy Clin Immunol 2013 Mar 13 DOI: 10.1016/
j.jaci.2013.01.049. [Epub ahead of print].
Larenas Linnemann DE. One hundred years of immunotherapy:
review of the first landmark studies. Allergy Asthma Proc 33:122–
128, 2012.
Settipane RA, and Schwindt C. Allergic rhinitis. Am J Rhinol Allergy
27:S52–S55, 2013.
Carr TF, and Saltoun CA. Skin testing in allergy. Allergy Asthma
Proc 33(Suppl 1):S6–8, 2012.
Makhija M, and O’Gorman MR. Common in vitro tests for allergy
and immunology. Allergy Asthma Proc 33(Suppl 1):S108–S111, 2012.
Settipane RA, Borish L, and Peters AT. Determining the role of
allergy in sinonasal disease. Am J Rhinol Allergy 27:S56–S58, 2013.
Settipane RA, Peters AT, and Chandra R. Chronic rhinosinusitis.
Am J Rhinol Allergy 27:S11–S15, 2013.
Settipane RA, Peters AT, and Chiu AG. Nasal polyps. Am J Rhinol
Allergy 27:S20–S25, 2013.
Kennedy JL, and Borish L. Chronic sinusitis pathophysiology: The
role of allergy. Am J Rhinol Allergy DOI: 10.2500/ajra.2013.27.3906
[Epub ahead of print date April 18, 2013].
Fokkens WJ, Lund VJ, Mullol J, et al. European position paper on
rhinosinusitis and nasal polyps 2012. Rhinol Suppl 23:1–298, 2012.
Nathan RA, Santilli J, Rockwell W, and Glassheim J. Effective of
immunotherapy for recurring sinusitis associated with allergic rhinitis as assessed by Sinusitis Outcomes Questionnaire. Ann Allergy
Asthma Immunol 92:668–672, 2004.
Smith AM, Rezvani M, and Bernstein JA. Is response to allergen
immunotherapy a good phenotypic marker for differentiating between allergic rhinitis and mixed rhinitis? Allergy Asthma Proc
32:49–54, 2011.
Frew A. Allergen immunotherapy. J Allergy Clin Immunol 125:S306–
S313, 2010.
Grammer LC, and Harris KE. Principles of immunologic management of allergic diseases due to extrinsic antigens. In Patterson’s
Allergic Diseases, 7th ed. Grammer LC, and Greenberger PA (Eds).
Philadelphia, PA: Lippincott, Williams & Wilkins, 187–196, 2009.
Georgy MS, and Saltoun CA. Allergen immunotherapy: Definition,
indication, and reactions. Allergy Asthma Proc 33(suppl 1):S9–S11,
2012.
Akdis M, and Akdis CA. Mechanisms of allergen-specific immunotherapy. J Allergy Clin Immunol 119:780–791, 2007.
Calderon MA, Penagos M, Sheikh A, et al. Sublingual immunotherapy for allergic conjunctivitis: Cochrane systematic review and metaanalysis. Clin Exp Allergy 41:1263–1272, 2011.
Durham SR, Walker SM, Varga EM, et al. Long-term clinical efficacy
of grass-pollen immunotherapy. N Engl J Med 341:468–475, 1999.
Bernstein DI, Wanner M, Borish L, et al. Immunotherapy Committee,
American Academy of Allergy, Asthma and Immunology. Twelveyear survey of fatal reactions to allergen injections and skin testing:
1990–2001. J Allergy Clin Immunol 113:1129–1136, 2004.
Calabria CW, Stolfi A, and Tankersley MS. The REPEAT study:
Recognizing and evaluation periodic local reactions in allergen im-
May–June 2014, Vol. 28, No. 3
Copyright © Oceanside Publications, Inc. All rights reserved. DO NOT COPY
For permission to copy go to https://www.oceansidepubl.com/permission.htm
26.
27.
28.
29.
30.
31.
32.
33.
34.
35.
36.
37.
38.
39.
40.
41.
munotherapy and associated systemic reactions. Ann Allergy
Asthma Immunol 106:49, 2011.
Roy SR, Sigmon JR, Olivier J, et al. Increased frequency of large local
reactions among systemic reactors during subcutaneous allergen immunotherapy. Ann Allergy Asthma Immunol 99:82–86, 2007.
Greenberger PA, and Ditto AM. Anaphylaxis. Allergy Asthma Proc
33(suppl 1):S80–S83, 2012.
Lieberman P. The risk and management of anaphylaxis in the setting
of immunotherapy. Am J Rhinol Allergy 26:469–474, 2012.
Phillips JF, Lockey RF, Fox RW, et al. Systemic reactions to subcutaneous allergen immunotherapy and the response to epinephrine.
Allergy Asthma Proc 32:288–294, 2011.
Wallace DV. Anaphylaxis in the allergist’s office: Preparing your
office and staff for medical emergencies. Allergy Asthma Proc 34:
120–131, 2013.
Hankin CS, Cox L, and Bronstone A. The health economics of allergen immunotherapy. Immunol Allergy Clin North Am 31:325–341,
2011.
Hankin CS, Cox L, Bronstone A, et al. AIT: Reduced health care costs
in adults and children with allergic rhinitis. J Allergy Clin Immunol
131:1084–1091, 2013.
DuBuske LM, Frew AJ, Horak F, et al. Ultrashort-specific immunotherapy successfully treats seasonal allergic rhinoconjunctivitis to
grass pollen. Allergy Asthma Proc 32:239–247, 2011.
Cox L. Accelerated immunotherapy schedules: Review of efficacy
and safety. Ann Allergy Asthma Immunol 97:126–137, 2006.
Guida G, Boita M, Scirelli T, et al. Innate and lymphocytic response
of birch-allergic patients before and after sublingual immunotherapy.
Allergy Asthma Proc 33:411–415, 2012.
Nayak AS, Atiee GJ, Dige E, et al. Safety of ragweed sublingual
allergy immunotherapy tablets in adults with allergic rhinoconjunctivitis. Allergy Asthma Proc 33:404–410, 2012.
Blaiss M, Maloney J, Nolte H, et al. Efficacy and safety of timothy
grass AIT tablets in North American children and adolescents. J
Allergy Clin Immunol 127:64–71, 71.e1–4, 2011.
Radulovic S, Wilson D, Calderon M, and Durham S. Systematic
reviews of sublingual immunotherapy (SLIT). Allergy 66:740–752,
2011.
Marogna M, Spadolini I, Massolo A, et al. Long-lasting effects of
sublingual immunotherapy according to its duration: A 15-year prospective study. J Allergy Clin Immunol 126:969–975, 2010.
Lin SY, Erekosima N, Kim JM, et al. Sublingual immunotherapy for
the treatment of allergic rhinoconjunctivitis and asthma: A systematic
review. JAMA 309:1278–1288, 2013.
Di Bona D, Plaia A, Leto-Barone MS, et al. Efficacy of subcutaneous
and sublingual immunotherapy with grass allergens for seasonal
American Journal of Rhinology & Allergy
allergic rhinitis: A meta-analysis-based comparison. J Allergy Clin
Immunol 130:1097–1107.e2, 2012.
42. Genentech, Inc. XOLAIR prescribing information. South San Francisco, CA; 2007. Last revision July, 2010.
43. Soresi S, and Togias A. Mechanisms of action of anti-immunoglobulin E therapy. Allergy Asthma Proc 27:S15–S23, 2006.
44. Prussin C, Griffith D, Boesel K, et al. Omalizumab treatment downregulates dendritic cell FcepsilonRI expression. J Allergy Clin Immunol 112:1147–1154, 2003.
45. Fahy JV, Fleming HE, Wong HH, et al. The effect of an anti-IgE
monoclonal antibody on the early and late-phase responses to allergen inhalation in asthmatic subjects. Am J Respir Crit Care Med
155:1828–1834, 1997.
46. Adelroth E, Rak S, Haahtela T, et al. Recombinant humanized mAbE25, an anti-IgE mAb, in birch pollen-induced seasonal allergic rhinitis. J Allergy Clin Immunol 106:253–259, 2000.
47. Nayak A, Casale T, Miller SD, et al. Tolerability of retreatment with
omalizumab, a recombinant humanized monoclonal anti-IgE antibody, during a second ragweed pollen season in patients with seasonal allergic rhinitis. Allergy Asthma Proc 24:323–329, 2003.
48. Chervinsky P, Casale T, Townley R, et al. Omalizumab, an anti-IgE
antibody, in the treatment of adults and adolescents with perennial allergic rhinitis. Ann Allergy Asthma Immunol 91:160–167,
2003.
49. Berger WE. Treatment of allergic rhinitis and other immunoglobulin
E-mediated diseases with anti-immunoglobulin E antibody. Allergy
Asthma Proc 27:S29–S32, 2006.
50. Kuehr J, Brauburger J, Zielen S, et al. Efficacy of combination treatment with anti-IgE plus specific immunotherapy in polysensitized
children and adolescents with seasonal allergic rhinitis. J Allergy Clin
Immunol 109:274–280, 2002.
51. Pinto JM, Mehta N, DiTineo M, et al. A randomized, double-blind,
placebo-controlled trial of anti-IgE for chronic rhinosinusitis. Rhinology 48:318–324, 2010.
52. Cox L, Lieberman P, Wallace D, et al. American Academy of Allergy
Asthma & Immunology/American College of Allergy, Asthma &
Immunology Omalizumab-Associated Anaphylaxis Joint Task Force
follow-up report. J Allergy Clin Immunol 128:210–212, 2011.
53. Limb SL, Starke PR, Lee CE and Chowdhury BA. Delayed onset and
protracted progression of anaphylaxis after omalizumab administration in patients with asthma. J Allergy Clin Immunol 120:1378–1381,
2007.
54. Busse W, Buhl R, Vidaurre CF, et al. Omalizumab and the risk of
malignancy: Results from a pooled analysis. J Allergy Clin Immunol
128:983–989, 2012.
e
Copyright © Oceanside Publications, Inc. All rights reserved. DO NOT COPY
For permission to copy go to https://www.oceansidepubl.com/permission.htm
S89
Subcutaneous and sublingual immunotherapy for allergic
rhinitis: What is the evidence?
Sarah K. Wise, M.D.,1 and Rodney J. Schlosser, M.D.2
ABSTRACT
Background: Increasing interest in sublingual immunotherapy (SLIT) among practitioners and patients has resulted in numerous publications and clinical
trials in recent years. With the clinical growth of SLIT, discussions of its efficacy, safety, and immunologic effects have intensified, as have comparisons to
subcutaneous immunotherapy (SCIT). In the United States, SCIT has been the traditional form of immunotherapy for inhalant allergy and is the only
immunotherapy method approved by the U.S. Food and Drug Administration at this time. The similarities and differences between SLIT and SCIT are often
discussed, yet clinical studies directly comparing these immunotherapy methods are scarce.
Methods: A literature review of specific issues and controversies between SLIT and SCIT for allergic rhinitis was conducted.
Results: Safety, efficacy, and immunologic effects of these two immunotherapy techniques are reviewed.
Conclusion: Unanswered questions relating to SLIT are examined.
A
ccording to the 2009 National Health Interview Survey, 7.8% of
adults and 9.8% of children in the United States had been
diagnosed with hay fever in the preceding 12 months.1,2 There were
13.1 ambulatory care visits for allergic rhinitis in 2006.3 Based on these
few figures, the public health impact of allergic rhinitis is evident.
Although many allergic rhinitis patients have been successfully
treated with environmental control measures, pharmacotherapy, and
subcutaneous immunotherapy (SCIT), interest in sublingual immunotherapy (SLIT) has grown considerably in recent years. A search of
published literature under the topic “sublingual immunotherapy”
revealed 21 citations in English in 1999; this has grown to 737 English
citations in a 2011 search of the PubMed database (www.ncbi.nlm.
nih.gov/pubmed). At the time of this article preparation, there were
57 clinical SLIT trials listed at www.clinicaltrials.gov. Twelve of these
trials are registered in the United States and 32 are registered in
Europe.
With the rapidly growing interest in SLIT for allergic rhinitis,
comparisons with traditional SCIT have markedly increased. In this
brief review of SLIT and SCIT, we will highlight specific similarities,
differences, and controversies of these two immunotherapy techniques. Recent findings relating to safety, efficacy, future research
needs, and unanswered questions regarding SLIT and SCIT will be
discussed (Table 1).
IMMUNOTHERAPY SAFETY
Systemic reactions and anaphylaxis are noted complications of
immunotherapy. In a 2007 Cochrane meta-analysis of SCIT for seaFrom the 1Department of Otolaryngology–Head and Neck Surgery, Emory University,
Atlanta, Georgia, and 2Ralph H. Johnson Veterans Affairs Medical Center and, Department of Otolaryngology–Head and Neck Surgery, Medical University of South
Carolina, Charleston, South Carolina
Presented at the North American Rhinology & Allergy Conference, February 5, 2011,
Puerto Rico
The authors have no conflicts of interest to declare pertaining to this article
Address correspondence and reprint requests to Rodney J. Schlosser, M.D., Department
of Otolaryngology–Head and Neck Surgery, Medical University of South Carolina, 135
Rutledge Avenue, MSC 550, Charleston, SC 29425
E-mail address: [email protected]
Originally published in Am J Rhinol Allergy 27, 18 –22, 2012
Copyright © 2014, OceanSide Publications, Inc., U.S.A.
S90
sonal allergic rhinitis, which included 51 randomized placebo-controlled trials, epinephrine was given for adverse reactions in 0.13% of
participants on active treatment (19 of 14,085 injections).4 No fatalities
were reported in this meta-analysis. Fatal reactions from SCIT in
clinical practice are reported at a rate of 1 in 2–2.5 million injections,
resulting in 3.4 deaths/year.5,6 Potential contributors to fatal SCIT
reactions include delay in epinephrine administration, previous immunotherapy reactions, suboptimal asthma control, administration of
injections during peak allergy season, and alterations in antigen extracts.6 In addition to fatal SCIT reactions, there is a systemic reaction
rate of 0.05–3.2% of injections (0.84–46.7% of patients) per year and a
near-fatal reaction rate of 23 per year (5.4 per 1 million injections).6–9
Serious systemic and fatal reactions due to SCIT are relatively rare.
However, the potential for a fatal systemic reaction caused by treatment for a nonfatal condition such as allergic rhinitis gives many
practitioners pause.
The potential for fatal systemic reactions from SCIT was highlighted in
the 1986 report of the British Committee on the Safety of Medicines.10
Based on 26 anaphylaxis-related deaths in this report, the safety of SCIT
was questioned and strict criteria for SCIT administration in the United
Kingdom were initiated. These new regulations included a postinjection
observation period of 2 hours and the requirement that injections be
given in a facility with full CPR capabilities. Subsequently, interest
increased in noninjection routes of immunotherapy administration, including oral (swallow), sublingual, bronchial, and intranasal. Of these,
sublingual administration was the most promising with regard to its
clinical efficacy, tolerability, and safety.
The safety profile of SLIT is one of the least controversial aspects in
its overall comparison with SCIT. Before 2006, there were no literature
reports of anaphylaxis due to SLIT. Between 2006 and 2009, there
were six published cases of anaphylaxis or possible anaphylaxis
related to SLIT.11–15 Certain factors have been hypothesized as contributors to SLIT anaphylaxis, including rush escalation or no escalation, use of latex antigen, multiple antigen therapy, treatment during
peak pollen season, previous intolerance to SCIT, and noncompliance
with treatment regimens.
In the 2010 Cochrane systematic review of SLIT for allergic rhinitis,
Radulovic et al. report that there were no cases of anaphylaxis and no
requirement for the use of adrenaline in 60 randomized, placebocontrolled trials.16 This Cochrane review did note mild-to-moderate
systemic reactions in both treatment and placebo groups. Treatment
May–June 2014, Vol. 28, No. 3
Copyright © Oceanside Publications, Inc. All rights reserved. DO NOT COPY
For permission to copy go to https://www.oceansidepubl.com/permission.htm
Table 1 Highlights of SCIT and SLIT for allergic rhinitis
Safety
Efficacy
Single vs multiantigen therapy
Dosing
Fatal reactions from SCIT occur at a rate of 1 in 2–2.5 million injections, resulting in 3.4 deaths/yr
Between 2006 and 2009, there were six published cases of possible or confirmed anaphylaxis
during SLIT; no fatalities have been reported with SLIT
Any patient undergoing immunotherapy should be educated on the potential risk of anaphylaxis
and the proper use of emergency epinephrine injectors
A 2007 Cochrane review of SCIT for seasonal allergic rhinitis showed significantly decreased
symptom scores and medication use
In 2010, a Cochrane review of SLIT for allergic rhinitis noted significant symptom reduction
overall, and for multiple subgroups (i.e., seasonal and perennial antigens, adults and children,
and short and long duration of therapy); medication scores were also significantly decreased
Large-scale controlled studies directly comparing SCIT and SLIT are lacking
The efficacy of multiantigen SCIT and SLIT remains controversial; well-designed, controlled
studies of multiantigen SCIT and SLIT are needed
The optimum SCIT maintenance dose for most antigens is 5–20 ␮g of major allergen
Optimum SLIT dosing has not been fully elucidated, although the median monthly SLIT
maintenance dose is 49 times the monthly SCIT maintenance dose
SCIT ⫽ subcutaneous immunotherapy; SLIT ⫽ sublingual immunotherapy.
discontinuation was attributed to adverse events in 41 of 824 SLIT
participants and 12 of 861 placebo participants; this included both
local and systemic reactions, although discontinuation for local reactions was more common. Because SLIT is a potentially attractive
option for the treatment of respiratory allergy in children, it is also
important to evaluate adverse events in this patient group. Two
meta-analyses have been dedicated to SLIT in the pediatric population. In 2006, Penagos et al. noted no severe systemic or lethal events
in a meta-analysis of 10 SLIT trials for allergic rhinitis in children,
although 1 of the included studies reported 3 patients with severe
asthma attributed to SLIT overdose.17 Similarly, a 2008 meta-analysis
of SLIT for allergic asthma in children reported no fatal or severe
systemic reactions in nine included studies.18 In a postmarketing
survey of single and multiple antigen SLIT in 433 children receiving
40,169 SLIT doses, 13 events were judged to be of moderate severity
and required medical advice.19 There was no emergency treatment
required, and no difference was seen in adverse events between
single and multiple antigen regimens. SLIT has also been reported
safe in children ⱕ5 years of age.20,21
Although the safety profile of SLIT is often quoted as being superior to SCIT, the practitioner must remain aware of the risks of
immunotherapy in general. Regardless of the route of immunotherapy selected, patients should be educated on expected side effects
versus worrisome systemic reactions. SCIT doses are routinely given
in the physician’s office, especially during escalation. It has also been
suggested that the first dose of SLIT be given in the physician’s office;
after this, SLIT doses are routinely administered at home. Because the
risk of anaphylaxis exists for both SCIT and SLIT, many also advocate
that any patient receiving immunotherapy should carry an emergency epinephrine injector and be fully educated on its appropriate
use.
IMMUNOTHERAPY EFFICACY
The efficacy of SLIT for allergic rhinitis, when compared with SCIT,
incites greater controversy. A Cochrane systematic review of injection
immunotherapy for seasonal allergic rhinitis was published in 2007
by Calderon et al.4 This meta-analysis included 51 double-blind, placebo-controlled, randomized trials of specific immunotherapy for
seasonal allergic rhinitis to tree, grass, or weed pollens. Fifteen trials
were assessed for standard mean difference (SMD) of symptom scores
and showed a significant reduction of symptoms in the immunotherapy group (SMD, ⫺0.73; 95% CI, ⫺0.97 to ⫺0.50; p ⬍ 0.00001). Data
from 13 trials showed significant medication reduction in the immunotherapy group (SMD, ⫺0.57; 95% CI, ⫺0.82 to ⫺0.33; p ⬍ 0.00001).
Demonstration of increased SCIT efficacy in symptom control has
American Journal of Rhinology & Allergy
also been shown for longer durations of maintenance therapy (up to
3 years), although the increased efficacy evidence from this study is
weak versus SCIT therapy duration of 1 year.22
Recent large meta-analyses of SCIT efficacy for perennial allergic
rhinitis have not been performed. In a 2011 Cochrane systematic
review of 88 randomized controlled SCIT trials for allergic asthma,
however, therapy with mite antigen was shown to have a marginal
benefit in asthma symptoms (SMD, ⫺0.48; 95% CI, ⫺0.96–0.00).23
SCIT for cat and dog allergens did not show improvement in asthma
symptoms in this meta-analysis. In contrast, objective measures of
bronchial hyperreactivity improved with SCIT in this Cochrane review for mite immunotherapy (SMD, ⫺0.98; 95% CI, ⫺1.39 to ⫺0.58),
pollen (SMD, ⫺0.55; 95% CI, ⫺0.84 to ⫺0.27), and animal dander
(SMD, ⫺0.61; 95% CI, ⫺0.95 to ⫺0.27).23 Bronchial hyperreactivity
was not significantly improved with SCIT for other allergens.
As clinical interest in SLIT has grown over the last 10 years, large
randomized controlled trials and meta-analyses have been reported
with increasing frequency. The first randomized clinical trial of lowdose SLIT with dust-mite antigen included 20 patients and was reported in 1986 by Scadding and Brostoff.24 Multiple large-scale randomized, double-blind, placebo-controlled SLIT efficacy trials have
been published in the last 5 years, beginning with the 2006 studies of
Durham et al.25 and Dahl et al.26 Both the Durham (855 patients
randomized) and Dahl (634 patients randomized) studies were multicenter multinational trials of pre- and coseasonal administration of
Timothy grass tablets in patients with symptomatic seasonal allergic
rhinitis, and both showed statistically significant reduction is allergic
rhinitis symptoms and medication use versus placebo groups.25,26
The most recent meta-analysis of SLIT for allergic rhinitis was
published in 2010 by Radulovic et al.16 For symptom assessment, 49
randomized placebo-controlled trials were included with 2333 total
participants receiving SLIT and 2256 receiving placebo. The SMD for
symptom scores favored SLIT at ⫺0.49 (95% CI, ⫺0.64 to ⫺0.34;
⬍0.00001). Subgroup analysis revealed significant symptom improvement with SLIT for seasonal and perennial allergens, adults and
children, treatment durations ranging from ⬍6 months to ⬎12
months, and major allergen content of 5–20 ␮g and ⬎20 ␮g. With
regard to individual antigens, there was significant symptom improvement for house-dust mites, grass pollen, ragweed, Parietaria,
and trees. Medication scores were assessed in 38 studies and revealed
an SMD of ⫺0.32 (95% CI, ⫺0.43 to ⫺0.21; p ⬍ 0.00001). A metaanalysis of SLIT for allergic rhinitis in pediatric patients was published in 2006 by Penagos et al.17 Ten pediatric studies, including 484
patients, were evaluated and revealed significant reduction in symptoms (SMD, ⫺0.56; 95% CI, 1.01–0.10; p ⫽ 0.02) and medication use
Copyright © Oceanside Publications, Inc. All rights reserved. DO NOT COPY
For permission to copy go to https://www.oceansidepubl.com/permission.htm
S91
Table 2 SLIT vs SCIT comparison studies
Author
Year
Allergen
Study Design
Quirino et al.35
1996
Grass pollen
Double-blind, double-dummy
Mungan et al.32
1999
Dust mite
Single-blind, placebo
controlled
No. of Patients
SCIT (n ⫽ 10)
SLIT (n ⫽ 10)
No placebo group
SCIT (n ⫽ 10)
SLIT (n ⫽ 15)
Placebo (n ⫽ 11)
Khinchi et al.34
2004
Birch pollen
Randomized, double-blind,
double-dummy,
placebo-controlled
Mauro et al.36
2007
Birch pollen
Randomized, double-blind,
double-dummy
Eifan et al.33
2010
Dust mite
Open label, randomized,
controlled
SCIT (n ⫽ 21)
SLIT (n ⫽ 18)
Placebo (n ⫽ 19)
SCIT (n ⫽ 19)
SLIT (n ⫽ 15)
SCIT (n ⫽ 16)
SLIT (n ⫽ 16)
Pharmacotherapy (n ⫽ 16)
Study Findings
Significant reduction in symptoms and
medications for SCIT and SLIT
groups
1 Total specific IgG, 1 specific IgG4,
and 2 skin reactivity for SCIT only
2 Rhinitis and asthma symptoms
with SCIT
2 Rhinitis symptoms with SLIT
2 Skin reactivity with SCIT
1 Specific IgG4 with SCIT
Significant reduction in symptoms and
medications for SCIT vs placebo
and SLIT vs placebo
No difference between SCIT and SLIT
groups
No difference in mean
symptom–medication score between
SCIT and SLIT
1 Specific IgG4 with SCIT
2 Rhinitis and asthma symptom
scores, total medication score, and
skin reactivity with SCIT and SLIT
2 Specific IgE with SCIT and SLIT
SCIT ⫽ subcutaneous immunotherapy; SLIT ⫽ sublingual immunotherapy.
(SMD, ⫺0.76; 95% CI, 1.46–0.06; p ⫽ 0.03) with SLIT. Subgroup
analyses indicated that treatment duration of ⬎18 months and SLIT
with pollen extracts were beneficial over shorter treatment durations
and dust-mite antigens in children.
Meta-analyses of SLIT have also been performed with regard to
specific antigens. A meta-analysis specific for seasonal grass pollen
SLIT treatment was also performed by Di Bona et al.27 This metaanalysis included 19 randomized, placebo-controlled SLIT trials, with
2971 total patients. It was found that grass allergen SLIT significantly
reduced symptoms (SMD, ⫺0.32; 95% CI, ⫺0.44 to ⫺0.21) and medication use (SMD, ⫺0.33; 95% CI, ⫺0.50 to ⫺0.16) versus placebo.
Similarly, a meta-analysis of SLIT for house-dust mite allergic rhinitis
showed significant symptom reduction in 194 active SLIT participants
versus 188 placebo participants (SMD, ⫺0.95; 95% CI, ⫺1.77 to ⫺0.14;
p ⫽ 0.02).28 Significant medication reduction was also seen with SLIT
for house-dust mite allergic rhinitis (SMD, ⫺1.88; 95% CI, ⫺3.65 to
⫺0.12; p ⫽ 0.04).
Often-discussed benefits of immunotherapy are the long-lasting
and preventative effects that can be seen after treatment. Recent
studies have shown such effects with SLIT. In an open, randomized
study of 216 children with allergic rhinitis with or without asthma,
Marogna et al. showed a decrease in new sensitizations in children
receiving SLIT (3.1%) versus controls (34.8%; odds ratio, 16.85; 95%
CI, 5.73–49.13).29 In addition, after 3 years, there was a decrease in
positive methacholine challenge results in the SLIT group. A largescale (257 patients) double-blind, placebo-controlled trial of SLIT in
patients with grass pollen allergy by Durham et al. showed sustained
reduction in rhinoconjunctivitis symptoms and medication scores in
the SLIT group at the 1-year time point after cessation of a 3-year SLIT
program.30 Finally, Marogna et al. have noted that clinical benefit
persists for 8 years after SLIT treatment is given for a 4- to 5-year
duration; new sensitizations were also reduced in SLIT groups.31
Controlled studies involving both SCIT and SLIT treatment groups
for direct comparison are relatively lacking.32–36 A summary of the
characteristics of five SCIT versus SLIT comparison studies is shown
in Table 2. In 2010, a prospective, randomized, open-label three-
S92
parallel-group trial was conducted in 48 children with allergic rhinitis
or allergic asthma who were monosensitized to house-dust mites.33
Both SLIT and SCIT showed significant reduction in rhinitis symptom
score and medication score, as well as significant reduction of serumspecific dust-mite IgE, compared with pharmacotherapy. A placebocontrolled double-blind double-dummy study in 71 adult patients (all
patients received both sublingual medication and subcutaneous injections) was reported by Khinchi et al. in 2004.34 In this study, both
SLIT and SCIT showed efficacy versus placebo. There was no statistically significant difference between SLIT and SCIT groups; however,
the study was not powered to detect a difference in the immunotherapy groups if one truly existed. Finally, a comparison of the magnitude of effects seen in SLIT and SCIT Cochrane reviews was performed by Cox in 2008, noting that the magnitude of effects seen with
SCIT may be larger than that with SLIT.7 Although large clinical
studies directly comparing the efficacy of SCIT and SLIT have not
been performed, certain patients and practitioners may be willing to
accept slightly reduced efficacy of SLIT in the face of a significantly
higher safety profile and convenience.
In 2009, the World Allergy Organization Position Paper on Sublingual Immunotherapy discussed a number of important points regarding the current status of SLIT efficacy.37 Among these points, although SLIT meta-analyses have shown benefit in the treatment of
allergic rhinitis in adults and allergic rhinitis and asthma in children,
there are limitations in the overall conclusions of these meta-analyses
imposed by the significant heterogeneity of the studies included in
them. Second, the efficacy and dose dependence of SLIT for grass pollen
allergy in adults and children has been well demonstrated in large, sufficiently powered, double-blind, randomized, controlled trials.
UNANSWERED QUESTIONS
Single Antigen Therapy versus Multiantigen Therapy
One of the biggest sources of discussion in the SCIT versus SCIT
debate is the clinical use of single antigens versus multiple antigens in
May–June 2014, Vol. 28, No. 3
Copyright © Oceanside Publications, Inc. All rights reserved. DO NOT COPY
For permission to copy go to https://www.oceansidepubl.com/permission.htm
immunotherapy prescriptions. Although most clinical trials of specific immunotherapy (SCIT and SLIT) have tested the effects of only
1 antigen, the average SCIT preparation in the United States includes
10 antigens.38 In the 2010 Cochrane systematic review of SCIT for
allergic asthma, only 6 of 88 trials tested multiple antigens.23 In the
2010 Cochrane review of SLIT for allergic rhinitis, only one trial
involved multiple antigens.16
Some difficulties in treating with multiple-antigen immunotherapy
in a controlled clinical trial setting include identifying potential subjects with a similar multiantigen allergy profile in the absence of other
positive reactions, as well as accurately assessing symptoms related to
specific allergy triggers when multiple environmental triggers may be
present. Although efficacy has been shown with multiantigen SLIT
(over single-antigen SLIT and placebo) in some studies,39 a recent
multiantigen SLIT study in the United States failed to establish significant differences in symptoms versus single-antigen SLIT or placebo.40 The belief that immunotherapy is more effective in patients
who are sensitized to only a single antigen tends to be more prevalent
outside the United States, whereas practitioners treating allergy in the
United States are more inclined to treat with multiple antigens in an
immunotherapy prescription.17,38 It is interesting to note, however,
that a recent open study in 51 Italian children with allergic polysensitization found that allergic sensitization to multiple allergens should
not be considered a barrier to treatment with SLIT. In this study by
Ciprandi et al., treatment groups included single-antigen, dual-antigen, and multiple-antigen therapies, with significant improvements
noted in symptoms, medication use, and number of sensitizations
after 12 months of therapy.41 The efficacy of multiantigen SLIT requires further clarification, especially in light of the suggestion that
multiantigen treatment may have contributed to the few cases of SLIT
anaphylaxis.11,13
CONCLUSIONS
SCIT has long been an accepted form of treatment for allergic
rhinitis, but interest in SLIT has grown considerably in recent years.
This has sparked debate regarding the benefits and shortcomings of
each of these immunotherapy methods. The safety of SLIT is not
routinely questioned, although a few cases of nonfatal anaphylaxis
have been reported. Recent meta-analyses of SLIT for allergic rhinitis
have shown overall efficacy, as well as efficacy in multiple subgroup
analyses. However, questions have been raised regarding the magnitude of SLIT efficacy versus SCIT, and few controlled studies have
been performed to directly compare SLIT and SCIT. Many unanswered questions remain regarding SLIT and its comparison with
SCIT, including the clinical practice of multiantigen therapy, which is
not routinely tested in randomized clinical trials.
REFERENCES
1.
2.
3.
4.
5.
6.
Optimum Immunotherapy Dosing
Although standardization of antigens and regulation of antigen
maintenance dose brings about some controversy with regard to
SCIT, the recommended optimal maintenance dose for most SCIT
published was published in a 1998 World Health Organization Position Paper.42,43 An optimal dose for SCIT has been defined as ‘‘the
dose of an allergen vaccine inducing a clinically relevant effect in the
majority of patients without causing unacceptable side effects” and is
typically 5–20 ␮g of major allergen per dose (50–250 of major allergen
per year).
A recommended treatment dose for SLIT is less clear. Because of
different antigen production and standardization techniques worldwide, translation of clinical trial antigen doses to daily clinical practice may be difficult. At this time, there is no universally accepted
SLIT dosing schedule. However, published SLIT doses are notably
higher than SCIT doses. Furthermore, maintenance schedules differ
between SLIT (typically given daily) and SCIT (typically given
monthly). An individual SLIT dose may range from 0.0006 to 21 times
an individual SCIT dose, but the median monthly SLIT dose is ⬃49
times higher than the median SCIT dose (range, 0.017 to ⬎500 times
higher).44 Many studies have indicated that improvement in clinical
response occurs more frequently with moderate-to-high SLIT doses,
but the optimal SLIT dose still has not been fully elucidated for most
antigens.45–47 A notable exception is the optimal SLIT maintenance
dose for grass pollen antigen. Dose-finding studies by Durham et al.
and Didier et al. have identified that the most advantageous maintenance dose for SLIT with grass pollen is between 15 and 25 ␮g of
major allergen daily.25,45 Finally, because of the safety and tolerability
of SLIT, maintenance treatment has often been given preseasonally or
coseasonally in clinical trials treating for a single seasonal antigen.25,26
This is in contrast to year-round monthly SCIT maintenance injections. The effect of these seasonal SLIT dosing schedules has not been
extensively studied with regard to the potential for recurrence of
symptoms long term.
American Journal of Rhinology & Allergy
7.
8.
9.
10.
11.
12.
13.
14.
15.
16.
17.
18.
19.
Pleis J, Ward B, and Lucas J. Summary health statistics for U.S.
Adults: National Health Interview Survey 2009. Vital Health Statistics Series 10, No. 249, 2010.
Bloom B, Cohen R, and Freeman G. Summary health statistics for U.S.
Children: National Health Interview Survey 2009. Vital Health Statistics Series 10, No. 244, 2010.
Schappert S, and Rechtsteiner E. Ambulatory medical care utilization
estimates for 2006. National Health Statistics Rep 8:1–32, 2008.
Calderon M, Alves B, Jacobson M, et al. Allergen injection immunotherapy for seasonal allergic rhinitis. Cochrane Database of Syst Rev
1:CD001936, 2007.
Cox L, Larenas-Linneman D, Lockey R, and Passalacqua G. Speaking
the same language: The World Allergy Organization Subcutaneous
Immunotherapy Systemic Reaction Grading System. J Allergy Clin
Immunol 125:569–574, 2010.
Bernstein D, Wanner M, Borish L, and Liss G. Twelve-year survey of
fatal reactions to allergen injections and skin testing: 1990–2001. J
Allergy Clin Immunol 113:1129–1136, 2004.
Cox L. Sublingual immunotherapy and allergic rhinitis. Curr Allergy
Asthma Rep 8:102–110, 2008.
Amin H, Liss G, and Bernstein D. Evaluation of near-fatal reactions to
allergen immunotherapy injections. J Allergy Clin Immunol 117:169–
175, 2006.
Stewart G, and Lockey R. Systemic reactions from allergen immunotherapy. J Allergy Clin Immunol 90:567–578, 1992.
Committee on the Safety of Medicines update: Desensitizing vaccines. BMJ 293:948, 1986.
Dunsky E, Goldstein M, Dvorin D, and Belecanech G. Anaphylaxis to
sublingual immunotherapy. Allergy 61:1235, 2006.
Antico A, Pagani M, and Crema A. Anaphylaxis by latex sublingual
immunotherapy. Allergy 61:1236–1237, 2006.
Eifan A, Keles S, Bahceciler N, and Barlan I. Anaphylaxis to multiple
pollen allergen sublingual immunotherapy. Allergy 62:567–568, 2007.
Blazowski L. Anaphylactic shock because of sublingual immunotherapy overdose during third year of maintenance dose. Allergy 63:374,
2008.
de Groot H, and Bijl A. Anaphylactic reaction after the first dose of
sublingual immunotherapy with grass pollen tablet. Allergy 64:963–
964, 2009.
Radulovic S, Calderon M, Wilson D, and Durham S. Sublingual
immunotherapy for allergic rhinitis. Cochrane Database Syst Rev
12:CD002893, 2010.
Penagos M, Compalati E, Tarantini F, et al. Efficacy of sublingual
immunotherapy in the treatment of allergic rhinitis in pediatric patients 3 to 18 years of age: A meta-analysis of randomized, placebocontrolled, double-blind trials. Ann Allergy Asthma Immunol 97:
141–148, 2006.
Penagos M, Passalacqua G, Compalati E, et al. Metaanalysis of the
efficacy of sublingual immunotherapy in the treatment of allergic
asthma in pediatric patients, 3 to 18 years of age. Chest 133:599–609,
2008.
Agostinis F, Foglia C, Landi M, et al. The safety of sublingual immunotherapy with one or multiple pollen allergens in children. Allergy
63:1637–1639, 2008.
Copyright © Oceanside Publications, Inc. All rights reserved. DO NOT COPY
For permission to copy go to https://www.oceansidepubl.com/permission.htm
S93
20.
21.
22.
23.
24.
25.
26.
27.
28.
29.
30.
31.
32.
33.
S94
di Rienzo V, Minelli M, Musarra A, et al. Post-marketing survey on
the safety of sublingual immunotherapy in children below the age of
5 years. Clin Exp Allergy 35:560–564, 2005.
Fiocchi A, Paino G, La Grutta S, et al. Safety of sublingual-swallow
immunotherapy in children aged 3 to 7 years. Ann Allergy Asthma
Immunol 95:254–258, 2005.
Giovannini M, Braccioni F, Sella G, et al. Comparison of allergen
immunotherapy and drug treatment in seasonal rhinoconjunctivitis:
A 3-years study. Eur Ann Allergy Clin Immunol 37:69–71, 2005.
Abramson M, Puy R, and Weiner J. Injection allergen immunotherapy for asthma. Cochrane Database Syst Rev 8:CD00118, 20106.
Scadding G, and Brostoff J. Low dose sublingual therapy in patients
with allergic rhinitis due to dust mite. Clin Allergy 16:483–491, 1986.
Durham S, Yang W, Pedersen M, et al. Sublingual immunotherapy
with once-daily grass allergen tablets: A randomized controlled trial
in seasonal allergic rhinoconjunctivitis. J Allergy Clin Immunol 117:
802–809, 2006.
Dahl R, Kapp A, Colombo G, et al. Efficacy and safety of sublingual
immunotherapy with grass allergen tablets for seasonal allergic rhinoconjunctivitis. J Allergy Clin Immunol 118:434–440, 2006.
Di Bona D, Plaia A, Scafidi V, et al. Efficacy of sublingual immunotherapy with grass allergens for seasonal allergic rhinitis: A systematic review and meta-analysis. J Allergy Clin Immunol 126:558–566,
2010.
Compalati E, Passalacqua G, Bonini M, and Canonica G. The efficacy
of sublingual immunotherapy for house dust mites respiratory allergy: Results of a GA2LEN meta-analysis. Allergy 64:1570–1579,
2009.
Marogna M, Thomassetti G, Bernasconi A, et al. Preventative effects
of sublingual immunotherapy in childhood: An open randomized
controlled study. Ann Allergy Asthma Immunol 101:206–211, 2008.
Durham S, Emminger W, Capp A, et al. Long-term clinical efficacy in
grass pollen-induced rhinoconjunctivitis after treatment with SQstandardized grass allergy immunotherapy tablet. J Allergy Clin
Immunol 125:121–128, 2010.
Marogna M, Spadolini I, Massolo A, et al. Long-lasting effects of
sublingual immunotherapy according to its duration: A 15-year prospective study. J Allergy Clin Immunol 126:969–975, 2010.
Mungan D, Misirligil Z, and Gurbuz L. Comparison of the efficacy of
subcutaneous and sublingual immunotherapy in mite-sensitive patients with rhinitis and asthma—A placebo controlled study. Ann
Allergy Asthma Immunol 82:485–490, 1999.
Eifan A, Akkoc T, Yildiz A, et al. Clinical efficacy and immunological
mechanisms of sublingual and subcutaneous immunotherapy in
asthmatic/rhinitis children sensitized to house dust mite: An open
randomized controlled trial. Clin Exp Allergy 40:922–932, 2010.
34.
35.
36.
37.
38.
39.
40.
41.
42.
43.
44.
45.
46.
47.
Khinchi M, Poulsen L, Carat F, et al. Clinical efficacy of sublingual
and subcutaneous birch pollen allergen-specific immunotherapy:
A randomized, placebo-controlled, double-blind, double-dummy
study. Allergy 59:45–53, 2004.
Quirino T, Iemoli E, and Siciliani E. Sublingual versus injective
immunotherapy in grass pollen allergic patients: A double blind
(double dummy) study. Clin Exp Allergy 26:1253–1261, 1996.
Mauro M, Russelo M, Incorvaia C, et al. Comparison of efficacy,
safety and immunologic effects of subcutaneous and sublingual immunotherapy in birch pollinosis: A randomized study. Eur Ann
Allergy Clin Immunol 39:119–122, 2007.
Canonica G, Bosquet J, Casale T, et al. Sub-lingual Immunotherapy:
World Allergy Organization Position Paper 2009. Allergy 64(suppl
91):1–59, 2009.
Nelson H. Multiantigen immunotherapy for allergic rhinitis and
asthma. J Allergy Clin Immunol 123:763–769, 2009.
Marogna M, Spadolini I, Massolo A, et al. Effects of sublingual
immunotherapy for multiple or single allergens in polysensitized
patients. Ann Allergy Asthma Immunol 98:274–280, 2007.
Amar S, Harbeck R, Sills M, et al. Response to sublingual immunotherapy with grass pollen extract: Monotherapy versus combination
in a multiallergen extract. J Allergy Clin Immunol 124:150–156, 2009.
Ciprandi G, Cadario G, Di Gioacchino G, et al. Sublingual immunotherapy in children with allergic polysensitization. Allergy Asthma
Proc 31:227–231, 2010.
Bosquet J, Lockey R, and Malling H. Allergen immunotherapy: Therapeutic vaccines for allergic diseases. A WHO position paper. J
Allergy Clin Immunol 102:558–562, 1998.
van Rhee R. Indoor allergens: Relevance of major allergen measurements and standardization. J Allergy Clin Immunol 119:270–277,
2007.
Cox L, Linnemann D, Nolte H, et al. Sublingual immunotherapy: A
comprehensive review. J Allergy Clin Immunol 117:1021–1035, 2006.
Didier A, Malling H, Worm M, et al. Optimal dose, efficacy, and
safety of once-daily sublingual immunotherapy with a 5-grass pollen
tablet for seasonal allergic rhinitis. J Allergy Clin Immunol 120:1338–
1345, 2007.
Horak F, Jaeger S, Worm M, et al. Implementation of pre-seasonal
sublingual immunotherapy with a five-grass pollen tablet during
optimal dosage assessment. Clin Exp Allergy 39:394–400, 2008.
Malling H, Montagut A, Melac M, et al. Efficacy and safety of 5-grass
pollen sublingual immunotherapy tablets in patients with different
clinical profiles of allergic rhinoconjunctivitis. Clin Exp Allergy 39:
387–393, 2009.
e
May–June 2014, Vol. 28, No. 3
Copyright © Oceanside Publications, Inc. All rights reserved. DO NOT COPY
For permission to copy go to https://www.oceansidepubl.com/permission.htm
The risk and management of anaphylaxis in the setting
of immunotherapy
Phil Lieberman, M.D.
ABSTRACT
Background: Anaphylactic events due to immunotherapy are probably not completely preventable. There is always an inherent risk surrounding the
administration of an allergen to an individual who is sensitized to the substance administered.
Methods: There are, however, effective measures to reduce the risk of these events, and to optimize the assurance of a good outcome in the face of such an
event.
Results: Of prime importance in preventing these episodes is the regular assessment of the patient’s health status, especially in regard to asthma, and the
careful attention to the prevention of dosing errors.
Conclusion: Of equal importance, in regard to assuring a good outcome should such an event occur, are the rapid recognition of symptoms and the
immediate injection of epinephrine, the drug of choice for the treatment of any episode of anaphylaxis.
B
ecause allergen immunotherapy introduces an allergen into an
allergic individual, hypersensitivity reactions are probably unavoidable. There are, however, measures to minimize the risk and
effective therapy to treat any such reactions. This is a review of
procedures that have been suggested to minimize these risks and
protocols designed to treat such reactions if they do occur.
It draws heavily on consensus statements and evidence-based guidelines. The three references used extensively are the most recent allergen
immunotherapy parameter,1 the most recent update of the anaphylaxis
parameter,2 and a consensus publication on systemic reactions to immunotherapy sponsored by the World Allergy Organization.3
The most recent immunotherapy practice parameter1 states, “Although there is a low risk of severe systemic reactions with appropriately administered allergen immunotherapy, life-threatening and
fatal reactions do occur.”
Because such reactions are life-threatening, although they are extremely rare, it is imperative that actions be taken to minimize them and
protocols designed to treat them rapidly and efficiently are in place.
INCIDENCE
Allergic disease exerts a significant toll on the health care system4
and allergen immunotherapy is an effective and cost-effective therapy
in the treatment of allergic respiratory tract disease.5 With this therapy, however, as noted, anaphylactic reactions are probably inevitable. Unfortunately, the exact incidence of these events is unknown. In
addition, although we have some data, the exact incidence of near
fatal or fatal reactions is also imprecisely established. The reasons for this
are numerous. For example, reaction rates differ with the dose and
technique used, the allergen used, and the definition applied to define a
reaction. For example, severe systemic reactions occur at markedly different rates depending on the frequency of administration of allergy
injections. With conventional immunotherapy, the rates of severe systemic reactions are probably ⬍1%, whereas with rush immunotherapy
reported reaction rates have been in some instances ⬎30%.6–9
In addition, as with any adverse reaction to a therapeutic agent,
From Allergy and Asthma Care, Germantown, Tennessee
Presented at the North American Rhinology & Allergy Conference, Puerto Rico,
February 4, 2011
The author has no conflicts of interest to declare pertaining to this article
Address correspondence and reprint requests to Phil Lieberman, M.D., Allergy and
Asthma Care, 7205 Wolf River Boulevard, Germantown, TN 38138
E-mail address: [email protected]
Originally published in Am J Rhinol Allergy 27, 469 – 474, 2013
Copyright © 2014, OceanSide Publications, Inc., U.S.A.
American Journal of Rhinology & Allergy
reporting rates are probably not completely trustworthy. Also, response rates to surveys designed to assess incidence are usually, low,
⬍30%.10
Another difficulty innate to the determination of the incidence of
such reactions is that data gathering techniques are limited for the
most part to retrospective analyses or surveys taken of allergists
practicing immunotherapy. In addition, there are reviews of such
studies. For example, in the previously mentioned World Allergy
Organization document,3 it was concluded that by analyzing reaction
rates reported from studies between 1995 and 2010, the percentage of
systemic reactions per injection with conventional immunotherapy
protocols was ⬃0.2%.
One example of survey collected data was published by Amin et
al.10 in 2006. This survey was sent to members of the American
Academy of Allergy, Asthma, and Immunology seeking information
about reactions encountered in their practice. The desire was to
evaluate the incidence of fatal and near fatal reactions. There were 646
respondents. Two hundred seventy-three reported near fatal reactions between 1990 and 2001. This gave an incidence of 23 per year, or
5.4 events per million injections. The authors performed the study
because they noted that in previous evaluations, there were very few
if any descriptions of serious or near fatal systemic reactions. In these
previous studies, they noted that it was reported that 5–7% of patients
receiving immunotherapy experienced reactions, but there was no
mention of the number that were fatal or near fatal or a detailed
description of these events.11–13
Before this survey of fatal and near fatal episodes there were other
studies in North America that were performed to characterize and
estimate the incidence of reactions to immunotherapy. Lockey et al.14
reported 24 fatal reactions that occurred between 1973 and 1984. They
estimated that there was one fatal reaction per 2.8 million injections.
Reid et al.15 recorded 15 immunotherapy-related deaths between 1985
and 1989. They estimated one fatality in every 2 million injections.
Bernstein and colleagues performed a survey that documented 41
fatal reactions between 1990 and 2001.16 Their estimate was that there
was one fatal reaction per every 2.5 million injections.
FACTORS THAT MAY PREDISPOSE OR
INCREASE THE SEVERITY OF SYSTEMIC
REACTIONS DURING IMMUNOTHERAPY
Many factors have been identified that may enhance the risk of a
systemic reaction during immunotherapy or make such a reaction
more severe (Table 1). Very few of these, however, have been definitively established as a predisposing factor. Data collected regarding
many such factors show conflicting results.
Copyright © Oceanside Publications, Inc. All rights reserved. DO NOT COPY
For permission to copy go to https://www.oceansidepubl.com/permission.htm
S95
Table 1 Factors that may increase the frequency or enhance the
severity of a reaction during immunotherapy
Asthma
Dosing errors
Concomitant medication
Administration of injections during the pollen season
First injection from a new vial
A high level of sensitivity to the allergen administered
A history of a previous systemic reaction to allergen injections
Preceding large local reactions
ASTHMA
The presence of asthma may not increase the risk of a reaction, but
asthma is a risk factor for a severe reaction, and if the asthma is
unstable, it enhances this risk.17 In addition, it increases the risk of
fatal reactions.10
CONCOMITANT MEDICATION
Although ␤-adrenergic blocking agents do not seem to affect the
frequency of the occurrence of systemic reactions to immunotherapy,
they are a risk factor for a more serious event and can complicate
therapy.1,17 The issue of patients treated with immunotherapy and
simultaneously receiving a ␤-adrenergic blocker is one that is commonly encountered and one that has generated intense interest as
well as some controversy.18,19 This controversy has been generated in
part because of the difficulties that are presented to the clinician when
substitutions for ␤-blockers need to be made before the initiation of
immunotherapy and because in some instances immunotherapy can
be carefully performed even in patients who are receiving venom
while on ␤-adrenergic blockers.20 In addition, it has been shown that
␤-adrenergic blockers may not increase the risk of anaphylactic events
to radiocontrast material.21 However, there clearly are data that support the fact that ␤-adrenergic blockers may increase the risk of
anaphylaxis after the administration of a known allergen, complicate
its therapy, and worsen the severity of an event.14,22–46 Taken together,
overall, it appears quite clear that ␤-adrenergic blocking agents can
have an adverse effect on the outcome of an anaphylactic episode and
perhaps can increase the predisposition toward these episodes. They
may do so in several ways. When a patient is taking a ␤-adrenergic
blocker, there is a diminished response to the ␤-adrenergic effects of
epinephrine. This may make a patient less responsive to the endogenous compensatory response produced by the patient’s own production of epinephrine as well as exogenously administered epinephrine given for therapy. In addition, it should be clarified that in a case
of anaphylaxis, the relative contraindication extends not only to unselective ␤-adrenergic agents but also to relatively selective ␤-adrenergic blockers. This is because, in contrast to asthma, one is concerned
not only with the ␤-adrenergic effect on smooth muscle in the lungs
but also with the ␤-adrenergic effect on the cardiovascular system.3
Thus, it is desirable, in patients receiving immunotherapy, to, when
possible, discontinue the use of ␤-adrenergic agents. Angiotensinconverting enzyme inhibitors clearly increase the risk of an anaphylactic event during immunotherapy to venoms, but no such risk has
been noted, to date, regarding systemic reactions to inhalants.
PRECEDING LARGE LOCAL REACTIONS
Data regarding the occurrence of large local reactions are difficult
to interpret in that results have been somewhat conflicting. Originally, studies failed to find that preceding large local reactions were
a risk factor for a systemic event.46,48 At least, in these studies, there
was no difference in the incidence of systemic reactions in a group of
patients where dose adjustments were made based on the occurrence
of large local reactions versus a group in which the large local reac-
S96
Table 2 Unusual clinical manifestations of fatal and near fatal
anaphylactic reactions due to the administration of
immunotherapy
Upper airway obstruction is more frequent
Severe cardiovascular manifestations are more frequent
Gastrointestinal symptoms occur only rarely
Cutaneous manifestations are less common
Bronchospasm occurs more frequently
Table 3 Actions designed to diminish the risk of an anaphylactic
event during immunotherapy
A general health assessment and, specifically, an assessment of the
state of a patient’s asthma at the time of the injection should be
made
A peak expiratory flow might be performed to assist in this
evaluation, and if asthma is active, consideration of withholding
the injection should be made
Dosage adjustments should be made in patients having any
manifestation of a systemic reaction and continuing
immunotherapy
Consideration should be given to making dosage adjustments in
those patients who are highly sensitive
A minimum of 30 min wait time after an injection for all patients,
and if patients are at increased risk, consideration of extending
this wait time should be made
The patient should be educated regarding manifestations of
anaphylaxis and told to report any symptoms immediately
Careful attention to dosing errors and proper identification of the
patient should be done prior to administration of injection
The dosage should be lowered when a freshly prepared extract is
administered and when there has been a significant amount of
time between injections (patients late for injections)
Source: Adopted and modified from Ref. 1.
tions were not used to alter the immunotherapy dose. It was concluded that large local reactions were not accurate predictors of a
subsequent systemic event. However, another investigation performed as a retrospective review designed to compare the frequency
of preceding large local reactions in patients who had a systemic
reaction versus a matched “control group” of subjects not experiencing a systemic reaction found that there was a significant increase in
the frequency of large local reactions in patients who had experienced
a systemic reaction.49
These data are difficult to interpret as far as their clinical significance; however, overall, it appears as if large local reactions are not
adequate predictors of a future systemic event in that dosage adjustments based on local reactions fail to alter the frequency of systemic
events. Nonetheless, individuals who have experienced a systemic
event have a higher incidence of large local reactions than those who
have never had a systemic reaction.
ADMINISTRATION OF INJECTIONS DURING
THE POLLEN SEASON
As with large local reactions, data are conflicting on whether or not
immunotherapy injections given during the pollen season is a risk
factor compared with injections administered outside the pollen season. Some studies have shown that there is no difference in the
incidence of reactions when injections are given “in season” versus
when they are given “out of season.”50,51
However, in the previously mentioned study by Amin and colleagues,10 the administration of injections during the pollen season
was reported by 46% of respondents. In addition, it was hypothesized
May–June 2014, Vol. 28, No. 3
Copyright © Oceanside Publications, Inc. All rights reserved. DO NOT COPY
For permission to copy go to https://www.oceansidepubl.com/permission.htm
Table 4 Equipment suggested for treatment of an in-office anaphylactic event comparing two recent parameters published by the joint
task force
Allergen Immunotherapy: A Practice Parameter,
Third Update 1
Stethoscope and sphygmomanometer
Tourniquet, syringes, hypodermic needles, and
i.v. catheters (e.g., 14–18G)
Aqueous epinephrine 1:1000 w/v
Equipment to administer oxygen by mask
Intravenous fluid setup
Antihistamine for injection
Corticosteroids for intramuscular or i.v.
injection
Equipment to maintain airway
Glucagon (patients taking ␤-blockers)
The Diagnosis and Management of Anaphylaxis Practice Parameter: 2010 Update2
Universal Equipment
Stethoscope and sphygmomanometer
Injectable aqueous epinephrine 1:1000
Oxygen and equipment for administering
Intravenous fluids and equipment for administering them
Tourniquets, syringes, hypodermic needles, and large bore needles (e.g., 14G or 16G)
The following equipment and supplies should be considered depending on the
availability of emergency support services:
One-way valve face mask with oxygen inlet port
Diphenhydramine or similar injectable antihistamine
Corticosteroids for i.v. use
Vasopressor for i.v. use
Some clinicians may strongly consider the following:
Glucagon
Automatic defibrillator
Oral airway
Source: Refs. 1 and 2.
that “priming” during the season could be a predisposing factor in
regard to near fatal reactions. Thus, as with local reactions, it is
difficult to make a definitive statement on whether administration of
injections during the pollen season is a risk factor, but data, to date,
seem to imply that administration during the season may increase the
risk of systemic reactions.
DOSING ERRORS
Dosing errors account for a significant number of anaphylactic
reactions to immunotherapy. In the Amin et al.10 study they were the
second most common factor reported to be associated with events,
accounting for 26% of episodes.
FIRST INJECTION FROM A NEW VIAL
In two studies,15,16 the first injection from a new vial of extract was
a risk factor for a systemic event. Because of this it has been suggested
that the dose be lowered when a new vial is started.1 There is no
accepted consensus as to the amount the dose should be lowered.
Table 5 Practices and procedures to be in place for the
management of an anaphylactic event
Office facilities administering allergy injections should have an
established action plan to treat anaphylaxis
It is advisable to rehearse such a plan periodically
It is advisable to maintain a review of the treatment cart to make
sure all medications are up to date and all equipment is present
Physicians and office staff should maintain clinical proficiency
regarding therapy of anaphylaxis
All telephone numbers for paramedical rescue squads and hospital
emergency rooms should be available
Immunotherapy injections should be administered by healthcare
professionals trained in the treatment of anaphylaxis
The drugs that patients take should be reviewed on a regular basis
to make sure they are not taking a medication that might affect
the treatment of an event
A flow sheet for treatment of anaphylactic events should be
available, and treatment measures and dosages recorded on this
flow sheet should an event occur
Source: Adopted from Ref. 3.
A HIGH LEVEL OF SENSITIVITY TO THE
ALLERGEN ADMINISTERED
A high level of sensitivity to the allergen being administered has
been found to be a risk factor for systemic events.1
A HISTORY OF A PREVIOUS SYSTEMIC
REACTION TO ALLERGEN INJECTIONS
It is interesting to note that some patients experiencing a severe
reaction on occasion report previous milder events occurring earlier
in the course of immunotherapy.10
TIMING OF SYSTEMIC REACTIONS RELATED
TO THE ADMINISTRATION OF THE INJECTION
It is clear that most systemic reactions occur within 30 minutes
after an injection. In addition, almost all severe systemic reactions
start within this period of time.1,3 However, fatal reactions can
begin later than 30 minutes postinjection, and systemic reactions
can occur in rare instances more than 2 hours after the shot is
given.1,17 Of note, while speaking of timing, it is also necessary to
American Journal of Rhinology & Allergy
recognize that, although rare, biphasic reactions to immunotherapy can occur. Thus, patients can be treated successfully, discharged from the office, and then experience a recurrence of symptoms.52,53 Based on an overall assessment of this information, a
30-minute waiting period for all patients receiving allergen immunotherapy has been suggested.1
Because anaphylactic episodes to immunotherapy can occur after patients have left the physician’s office after a 30-minute wait,17
consideration has been given to supplying patients receiving allergen immunotherapy a prescription for an automatic epinephrine
injector, and that the patient be required to have this injector with
them on days when they receive their injections. To the author’s
knowledge, however, there is no consensus recommendation regarding this issue. Therefore, at least at this time, it appears that
whether or not to issue epinephrine injectors to patients who are
treated with immunotherapy remains at the discretion of the physician caring for the patient.
Copyright © Oceanside Publications, Inc. All rights reserved. DO NOT COPY
For permission to copy go to https://www.oceansidepubl.com/permission.htm
S97
Anaphylaxis preparedness
1
Patient presents with possible/probable acute anaphylaxis
2
NO
Initial assessment supports potential anaphylaxis?
e.g.: nonlocalized urticaria after immunotherapy
3
Consider other diagnosis
4
YES
Immediate intervention:
Assess airway, breathing, circulation, mentation
Inject epinephrine and reevaluate for repeat injection if
necessary
Supine position (if cardiovascular involvement suspected)
5
NO
Good clinical
response?
Subsequent emergency care that may be necessary
depending on response to epinephrine:
Consider:
Call 911 and request assistance
Recumbent position with elevation
lower extremity
Establish airway
O2
Repeat epinephrine injection if indicated
IV fluids if hypotensive; Rapid
volume expansion
Consider inhaled bronchodilators if wheezing
H1 and H2 Antihistamines
Corticosteroids
6
NO
YES
Observation
Length and setting of observation
c must be individualized
Autoinjectable epinephrine
9
YES
Make sure patient has telephone number
of physician on call, and take patient's
telephone number to consider calling later
to assess her/his condition and answer any
questions
References:
10
Good
clinical
response?
Call 911 if not already done
Consider:
Epinephrine intravenous infusion
Other intravenous vasopressors
Consider Glucagon
7
Cardiopulmonary arrest during anaphylaxis:
CPR and ACLS measures
Prolonged resuscitation efforts encouraged
(if necessary)
Consider:
High-dose epinephrine
Rapid volume expansion
Atropine for asystole or pulseless electric
activity
Transport to emergency dept or ICU
8
21
Figure 1. Algorithm for the treatment of an anaphylactic event in the outpatient setting (i.v.). (Adopted from Ref. 2.)
CLINICAL MANIFESTATIONS
The clinical manifestations of anaphylaxis during immunotherapy
are similar to those occurring in anaphylactic reactions to any injected
allergen. However, there are salient features of fatal and near fatal
events that are of note10 (Table 2).
For example, although cutaneous features are the most common
clinical manifestations in anaphylactic reactions taken as a whole,54 in
near fatal and fatal immunotherapy reactions they do not predominate.10 Respiratory failure and hypotension or shock are the most
frequently recorded events. Over 90% of patients with fatal reactions
experience respiratory failure, and hypotension occurs in 88% of near
fatal events and 81% of fatal reactions.
Cutaneous signs appeared in 70% of near fatal reactions and in only
29% of those that were fatal. This may be because of the fact that the
hypotensive state of these patients prevents blood flow from reaching
the skin.54
S98
A striking finding was that when patients exhibited a history of
poorly controlled or labile asthma, there was a prominently increased
risk of fatal events, and most of these who did have reactions experienced fatal rather than near fatal episodes.10
PREVENTION
In light of these findings, there have been several suggestions to
reduce the incidence and severity of systemic reactions due to immunotherapy1,3,10 (Table 3). Because asthma is clearly one of the most
important risk factors, it has been suggested that patients not receive
allergy injections when their asthma is unstable or when their peak
expiratory flow is “considered low for that patient” or “is substantially reduced compared with the patient’s baseline value.”1 It has
also been suggested that the absolute value of the forced expiratory
volume at 1 second be used as a measure to exclude patients from
receiving immunotherapy. In this regard, it has been proposed that an
May–June 2014, Vol. 28, No. 3
Copyright © Oceanside Publications, Inc. All rights reserved. DO NOT COPY
For permission to copy go to https://www.oceansidepubl.com/permission.htm
forced expiratory volume at 1 second below 70% of their predicted
value should eliminate an asthmatic patient for consideration of the
institution of aeroallergen immunotherapy.17
Patients who have experienced systemic reactions should have
dosage adjustments made. The amount the dose should be adjusted is
dependent on the physician’s judgment in regard to that particular
patient. Obviously, this decision can be based on the severity of the
event in question. In some instances, it may be decided that immunotherapy should be discontinued.
The degree of allergen sensitivity has been considered a risk factor
for anaphylactic events occurring during immunotherapy.3 Thus,
consideration of dose adjustments can be made in patients who show
a high degree of sensitivity as manifested by skin test reactivity.
Any risk factor would lead the physician administering immunotherapy to consider a wait of ⬎30 minutes. This includes a previous
reaction, a high degree of skin test sensitivity, a patient with asthma,
etc.
It is important that the patient recognize the early manifestations of
an anaphylactic episode and be told to report any manifestation
immediately. Episodes can begin insidiously, and patients may ignore
the early clinical expressions of a harbinger of a more severe reaction.
Thus, any patient receiving immunotherapy should be acquainted
with all of the manifestations of anaphylaxis and be told to report any
of these manifestations, as noted, promptly.
As mentioned earlier, dosing areas are one of the most common
causes of anaphylaxis to immunotherapy injections. Therefore, measures should be in place to minimize the chance of error. Efforts
should be made to enhance the distinction between different dilutions
of extract. Color coding systems can be used to accomplish this. The
person administering the injection should clearly identify the patient
by name and assure that the vial from which the injection is drawn is
for that patient. Careful record keeping as to dates and doses for each
injection should be used. The patient’s medication regimen should be
frequently monitored to see if there have been changes in medication
(e.g., the addition of a ␤-blocker), which might signify an increased
risk for a reaction. In addition, as noted previously, consideration
should be given to lowering the dose when a freshly prepared extract
is administered, and a schedule for reduction of dosing should be
available to delineate dose reductions due to an inordinate lapse of
time between injections.
SUGGESTED EQUIPMENT IN THE OFFICE FOR
TREATMENT OF A SYSTEMIC REACTION
There have been a number of articles written that have mentioned
what equipment should be available for the treatment of an in-office
anaphylactic reaction.54 Two recent documents1,2 list such equipment,
and their suggestions are compared in Table 4.
In addition to the equipment noted in Table 4, any facility in which
allergy injections are administered should have certain procedures in
place to facilitate a rapid response to an event (Table 5).3
MANAGEMENT OF ANAPHYLAXIS
The management of an anaphylactic event occurring to immunotherapy is identical to the management of an episode due to
exposure to any other injected allergen. Epinephrine is the drug of
choice and should be given at the first sign of an anaphylactic
episode.1–3 A delay in the administration of epinephrine has been
found to be a risk factor for poor outcomes and, in some studies,
for a biphasic reaction.55
Epinephrine can be administered every 5–10 minutes as necessary,
and this can be liberalized based on clinical judgment. Intravenous
administration can be considered if needed because of a poor response to intramuscular or subcutaneous injection, but it is preferably
administered where cardiovascular monitoring is available.
Immediate assessment of vital signs and the airway should be
performed, and the patient should be placed in a supine position with
American Journal of Rhinology & Allergy
legs elevated. Oxygen should be started simultaneously with the
initial evaluation. Patients should stay in this recumbent position
until the cardiovascular system is stable. Fatalities have been associated with prematurely assuming the upright position.56
If there is a good and rapid response to these early measures
consisting of oxygen, epinephrine, and positioning, the patient can be
observed (the length of time must be individualized) and then discharged from the facility. It is suggested that they be supplied with a
prescription for an automatic epinephrine injector at that time because symptoms can recur. They should be given the phone number
where the physician on call can be reached should symptoms reappear.
One could also consider, at the time of administration of epinephrine, calling for emergency services, but that is usually done if there
is no quick and adequate response to the initial therapy. Of course,
this decision is dependent on the severity of the symptoms at the time
of the initial evaluation.
In addition, should the blood pressure remain low, i.v. fluids
should be administered, for wheezing an inhaled bronchodilator
given, and consideration should be given to the i.v. administration of
an H1/H2-antihistamine and corticosteroids.
An algorithm outlining the treatment of an office event is shown in
Fig. 1.
In conclusion, anaphylactic episodes due to allergen immunotherapy probably are unavoidable, but there are strategies available to
minimize the frequency of their occurrence and to enhance the outcome of these events. Of primary importance is a level of awareness
and the institution of treatment immediately should any manifestation of an anaphylactic event occur.
REFERENCES
1.
Cox L, Nelson H, Lockey R, et al. Allergen immunotherapy: A
practice parameter. J Allergy Clin Immunol 127:S1–S55, 2011.
2. Lieberman P, Nicklas R, Oppenheimer J, et al. The diagnosis and
management of anaphylaxis practice parameter: 2010 Update. J Allergy Clin Immunol 126:477–480, 2010.
3. Cox L, Larenas-Linnemann D, and Lockey R. Speaking the same
language: The World Allergy Organization subcutaneous immunotherapy systemic reaction grading system. J Allergy Clin Immunol
125:569–574, 2010.
4. Blaiss MS. Allergic rhinitis: Direct and indirect costs. Allergy Asthma
Proc 31:375–380, 2010.
5. Alzakar RH, and Alsamarai AM. Efficacy of immunotherapy for
treatment of allergic asthma in children. Allergy Asthma Proc 31:324–
330, 2010.
6. Larenas-Linnemann D. Subcutaneous and sublingual immunotherapy in children: Complete update on controversies, dosing, and
efficacy. Curr Allergy Asthma Rep 8:465–474, 2008.
7. Bernstein DI, Epstein T, Murphy-Berendts K, and Liss GM. Surveillance of systemic reactions to subcutaneous immunotherapy injections: Year 1 outcomes of the ACAAI and AAAAI collaborative
study. Ann Allergy Asthma Immunol 104:530–535, 2010.
8. Windom H, and Lockey R. An update on the safety of specific
immunotherapy. Curr Opin Allergy Clin Immunol 8:571–576,
2008.
9. Portnoy J, Bagstad K, Kanarek H, et al. Premedication reduces the
incidence of systemic reactions during inhalant rush immunotherapy with mixtures of allergenic extracts. Ann Allergy 73:409–418,
1994.
10. Amin HS, Liss GM, and Bernstein DI. Evaluation of near-fatal reactions to allergen immunotherapy injections. J Allergy Clin Immunol
117:169–175, 2006.
11. Ragusa FV, Passalacqua G, Gambardella R, et al. nonfatal systemic
reactions to subcutaneous immunotherapy: A 10-year experience.
J Investig Allergol Clin Immunol 7:151–154, 1997.
12. Greenberg MA, Kaufman CR, Gonzalez GE, et al. Late and immediate systemic-allergic reactions to inhalant allergen immunotherapy. J
Allergy Clin Immunol 77:865–870, 1986.
13. Ragusa VF, and Massolo A. Non-fatal systemic reactions to subcutaneous immunotherapy: A 20-year experience comparison of two
10-year periods. Allergy Immunol (Paris) 36:52–55, 2004.
Copyright © Oceanside Publications, Inc. All rights reserved. DO NOT COPY
For permission to copy go to https://www.oceansidepubl.com/permission.htm
S99
14.
15.
16.
17.
18.
19.
20.
21.
22.
23.
24.
25.
26.
27.
28.
29.
30.
31.
32.
33.
34.
35.
Lockey RF, Benedict LM, Turkeltaub PC, and Bukantz SC. Fatalities
from immunotherapy (IT) and skin testing (ST). J Allergy Clin Immunol 79:660–677, 1987.
Reid MJ, Lockey RF, Turkeltaub PC, and Platts-Mills TA. Survey of
fatalities from skin testing and immunotherapy 1985–1989. J Allergy
Clin Immunol 92:6–15, 1993.
Bernstein DI, Wanner M, Borish L, and Liss GM. Twelve-year survey
of fatal reactions to allergen injections and skin testing. J Allergy Clin
Immunol 113:1129–1136, 2004.
Nelson HS (Ed). Allergen Immunotherapy. In: Immunotherapy for
Inhalant Allergens. Philadelphia, PA: Mosby, an affiliate of Elsevier,
Inc., 1657–1678, 2009.
Lieberman P, Kemp S, Oppenheimer J, et al. Letter to the editor. J
Allergy Clin Immunol 116:3933–3936, 2005.
Miller MM, and Miller MM. Beta-blockers and anaphylaxis: Are the
risks overstated? J Allergy Clin Immunol 116:931–933, 2005.
Muller UR, and Haeberli G. Use of beta-blockers during immunotherapy for hymenoptera venom allergy. J Allergy Clin Immunol
115:606–610, 2005.
Greenberger PA, Meyers SN, and Kramer BL. Effects of beta-adrenergic and calcium channel antagonists on the development of anaphylactoid reactions from radiographic contrast media during cardiac angiography. J Allergy Clin Immunol 80:698–672, 1987.
Lang DM, Alpern MB, Visintainer PF, and Smith ST. Increased risk
for anaphylactoid reactions from contrast media in patients on betaadrenergic blockers or with asthma. Ann Intern Med 115:270–276,
1991.
Lang DM, Alpern MB, Visintainer PF, and Smith ST. Elevated risk of
anaphylactoid reactions from radiographic contrast media is associated with both beta-blocker exposure and cardiovascular disorders.
Arch Intern Med 153:2033–2040, 1993.
Hepner MJ, Ownby DR, Anderson JA, et al. Risk of systemic reactions
in patients taking beta-blocker drugs receiving allergen immunotherapy injections. J Allergy Clin Immunol 86:407–411, 1990.
Alarn MM, Alvarez del Real G, and Hsieh FH. Cardiopulmonary
resuscitation (CPR) in patients with acute anaphylaxis taking betablockers. J Allergy Clin Immunol 115:S38, 2005 (Abs).
Jacobs RL, Rake GW, Fournie DC, et al. Potentiated anaphylaxis in
patients with drug-induced beta-adrenergic blockade. J Allergy Clin
Immunol 68:125–127, 1981.
Newman BR, and Schultz LK. Epinephrine-resistant anaphylaxis in a
patient taking propranolol hydrochloride. Ann Allergy 47:35–37,
1981.
Awai LW, and Makori YA. Insect sting anaphylaxis and beta-adrenergic blockade: A relative contraindication. Ann Allergy 53:43–49,
1984.
Laxenaire MC, Torrens J, and Moneret-Vautrin DA. Fatal anaphylactic shock in a patient treated with beta-blockers (French). Ann Fr
Anesth Reanim 3:453–455, 1984.
Benitah E, Nataf P, and Herman D. Anaphylactic complications in
patients treated with beta-blockers: Apropos of 14 cases (French).
Therapie 41:139–142, 1986.
Comaille G, Leynadier F, Modiano D, and Dry J. Severity of anaphylactic shock in patients treated with beta-blockers (French). Presse
Med 14:790–791, 1985.
Toogood JH. Beta-blocker therapy and the risk of anaphylaxis. Can
Med Assoc J 136:929–932, 1987.
Berkelman RI, Finton RJ, and Elsea WR. Beta-adrenergic antagonists
and fatal anaphylactic reactions to oral penicillin (letter). Ann Intern
Med 104:134, 1986.
Capellier G, Boillot A, and Cordier A. Anaphylactic shock in patients
treated with beta-blockades (French). Presse Med 18:181, 1989.
Stark BJ, and Sullivan TJ. Biphasic and protracted anaphylaxis. J
Allergy Clin Immunol 78:76–83, 1986.
S100
36.
37.
38.
39.
40.
41.
42.
43.
44.
45.
46.
47.
48.
49.
50.
51.
52.
53.
54.
55.
56.
Hamilton G. Severe adverse reactions to urography in patients taking
beta-adrenergic blocking agents. Can Med Assoc J 133:122–126, 1985.
Zaloga GP, DeLacey W, Holmboe B, and Chernow B. Glucagon
reversal of hypotension in a case of anaphylactic shock. Ann Intern
Med 105:65–66, 1986.
Kim Y. Interaction between beta-blockers and epinephrine on hemodynamics of spontaneously hypertensive rats. Res Commun Chem
Pathol Pharmacol 80:3–19, 1993.
Matsummura Y, Tan EN, and Vaughn JH. Hypersensitivity to histamine and systemic anaphylaxis in mice with pharmacologic betaadrenergic blockade: Protection by nucleotides. J Allergy Clin Immunol 58:387–394, 1976.
Forfang K, and Simonsen S. Effects of atenolol and pindolol on the
hypokalemia and cardiovascular responses to adrenaline. Eur J Clin
Pharmacol 37:23–26, 1989.
Raptis S, Rosenthal J, Wetzel D, and Moulopoulos S. Effects of
cardioselective and non-cardioselective beta-blockade on adrenalineinduced metabolic and cardiovascular response in man. Eur J Clin
Pharmacol 20:17–22, 1981.
Madowitz JS, and Schweiger MJ. Severe anaphylactoid reaction to
radiographic contrast media. J Am Med Assoc 241:2813–2815, 1979.
Brummett RE. Warning to otolaryngologists using local anesthetics
containing epinephrine: Potential serious reaction occurring in patients treated with beta-adrenergic receptor blockers. Arch Otolaryngol 110:561, 1984.
Anonymous. Physician’s Desk Reference. Montvale, NJ: Thomson
PDR, 3337, 2005.
Bousquet J, Lockey RF, Malling HJ, et al. WHO position paper:
Allergen immunotherapy: Therapeutic vaccines for allergic diseases.
Allergy 53(suppl 44):1–42, 1998.
Lang DM. Anaphylactoid and anaphylactic reactions, hazards of
betablockers. Drug Saf J 12:299–304, 1996.
Tankersley MS, Butler KK, Butler WK, and Goetz DW. Local reactions during allergen immunotherapy do not require dose adjustment. J Allergy Clin Immunol 106:840–843, 2000.
Kelso JM. The rate of systemic reactions to immunotherapy injections
is the same whether or not the dose is reduced after a local reaction.
Ann Allergy Asthma Immunol 92:225–227, 2004.
Roy SR, Sigmon JR, Olivier J, et al. Increased frequency of large local
reactions among systemic reactors during subcutaneous allergen immunotherapy. Ann Allergy Asthma Immunol 99:82–86, 2007.
Lin MS, Tanner E, Lynn J, et al. Nonfatal systemic allergic reactions
induced by skin testing and immunotherapy. Ann Allergy 71:557–
562, 1993.
Tikelman DG, Cole WQ, and Tunno J. Immunotherapy: A one year
prospective study to evaluate risk factors of systemic reactions. J
Allergy Clin Immunol 95:8–14, 1995.
Confino-Cohen R and Goldberg A. Allergen immunotherapy-induced biphasic systemic reactions: incidence, characteristics, and outcome: a prospective study. Ann Allergy Asthma Immunol 104:73–78,
2010.
Scranton SE, Gonzalez EG, and Waibel KH. Incidence and characteristics of biphasic reactions after allergen immunotherapy. J Allergy
Clin Immunol 123:493–498, 2009.
Lieberman P. Anaphylaxis. In Middleton’s Allergy: Principles and
Practice. Atkinson F, Bochner B, Busse W, Holgate S, Lemanske R,
and Simons FER (Eds). Philadelphia, PA: Mosby, an affiliate of
Elsevier, Inc., 1027–1051, 2009.
Lieberman P. Biphasic anaphylactic reactions. Ann Allergy Asthma
Immunol 95:217–228, 2005.
Pumphrey R. Fatal posture in anaphylactic shock. J Allergy Clin
Immunol 112:451–452, 2003.
e
May–June 2014, Vol. 28, No. 3
Copyright © Oceanside Publications, Inc. All rights reserved. DO NOT COPY
For permission to copy go to https://www.oceansidepubl.com/permission.htm
Pathophysiology of hereditary angioedema
Bruce L. Zuraw, M.D.,1 and Sandra C. Christiansen, M.D.2
ABSTRACT
Background: Laryngeal angioedema may be associated with significant morbidity and even mortality. Because of the potential severity of attacks, both
allergists and otolaryngologists must be knowledgeable about the recognition and treatment of laryngeal angioedema. This study describes the clinical
characteristics and pathophysiology of bradykinin-mediated angioedema.
Methods: A literature review was conducted concerning the clinical characteristics and pathophysiology of types I and II hereditary angioedema (HAE),
type III HAE, acquired C1 inhibitor (C1INH) deficiency, and angiotensin-converting enzyme (ACE) inhibitor-associated angioedema.
Results: The diagnosis of type I/II HAE is relatively straightforward as long as the clinician maintains a high index of suspicion. Mutations in the
SERPING1 gene result in decreased secretion of functional C1INH and episodic activation of plasma kallikrein and Hageman factor (FXII) of the plasma
contact system with cleavage of high molecular weight kininogen and generation of bradykinin. In contrast, there are no unequivocal criteria for making a
diagnosis of type III HAE, although a minority of these patients may have a mutation in the factor XII gene. Angioedema attacks and mediator of swelling
in acquired C1INH deficiency are similar to those in type I or II HAE; however, it occurs on a sporadic basis because of excessive consumption of C1INH in
patients who are middle aged or older. ACE inhibitor-associated angioedema should always be considered in any patient taking an ACE inhibitor who
experiences angioedema. ACE is a kininase, which when inhibited is thought to result in increased bradykinin levels. Bradykinin acts on vascular endothelial
cells to enhance vascular permeability.
Conclusion: Laryngeal swelling is not infrequently encountered in bradykinin-mediated angioedema. Novel therapies are becoming available that for the
first time provide effective treatment for bradykinin-mediated angioedema. Because the characteristics and treatment of these angioedemas are quite distinct
from each other and from histamine-mediated angioedema, it is crucial that the physician be able to recognize and distinguish these swelling disorders.
L
aryngeal angioedema can be associated with significant morbidity or even mortality.1–3 Treatment of laryngeal angioedema is
therefore of critical importance to many physicians, most notably
allergists, otolaryngologists, primary care physicians, and emergency
medicine physicians. Effective treatment of laryngeal angioedema
requires both an accurate diagnosis of the cause of the swelling as
well as an appreciation of the underlying pathophysiology of the
process. This review will summarize both the clinical characteristics
and the pathophysiology of several of the most important causes of
laryngeal angioedema.
Laryngeal angioedema, like all angioedema and urticaria, results from
increased vascular permeability with movement of fluid from the vascular space to the interstitial space.4–7 This may occur from either a
histamine-mediated process or a bradykinin-mediated process. The current review will focus on the pathophysiology of recurrent angioedema
that is believed to be caused by bradykinin. The specific forms of recurrent angioedema covered will be hereditary angioedema (HAE), acquired C1 inhibitor (C1INH) deficiency, and angiotensin-converting enzyme (ACE) inhibitor-associated angioedema (Fig. 1). Most of the
discussion will focus on HAE because it is the most completely understood of these conditions. The accompanying article by Christiansen will
then review the treatment of laryngeal angioedema.8
From the 1Department of Medicine, University of California, San Diego, and San Diego
Veteran’s Affairs Medical Center, La Jolla, California, and 2Department of Allergy,
Kaiser Permanente and University of California, San Diego, California
Presented at the First North American Rhinology and Allergy Conference, Puerto Rico,
February 3, 2011
B. Zuraw received grant support from ViroPharma, Pharming, and Shire and is a
consultant for ViroPharma, Shire, Santarus, Dyax, and CSL Behring; S. Christiansen
has no conflicts to declare pertaining to this article
Address correspondence and reprint requests to Bruce L. Zuraw, M.D., 9500 Gilman
Drive, Mailcode 0732, La Jolla, CA 92093-0732
E-mail address: [email protected]
Originally published in Am J Rhinol Allergy 25, 373–378, 2011
Copyright © 2014, OceanSide Publications, Inc., U.S.A.
American Journal of Rhinology & Allergy
HAE WITH LOW C1INH ACTIVITY
HAE was first accurately described by Dr. William Osler in 1888.1
Dr. Osler recognized the strong heritable nature of the disease and
provided a very comprehensive description of the attacks in multiple
generations of a single family. HAE is inherited in an autosomal
dominant manner. It affects both male and female gender equally.
Approximately 50% of the children of an affected parent will inherit
the disease. In addition, the disease does not skip generations. Surveys of HAE patients and their families have revealed that only ⬃75%
of patients with HAE have a positive family history of angioedema,
with the other 25% of HAE patients having de novo mutations.9
HAE is a rare disease, with an estimated prevalence of 1 per 50,000
in the general population.10–12 There is no known ethnic difference in
the prevalence of HAE. Because it is so uncommon, an accurate
diagnosis of HAE must begin with the physician having a high index
of suspicion based on the clinical characteristics (Table 1).11,13,14 Patients with HAE typically have recurrent angioedema without urticaria. The most commonly affected locations of swelling in HAE are
the extremities, the gastrointestinal tract, the external genitourinary
tract, the face, and the oropharynx/larynx. The probability that a
given attack will involve the skin or abdomen is nearly 50% each. All
other attack locations, including genitourinary and laryngeal attacks,
account for only 3.6% of attacks.15
Attacks of swelling in HAE patients are usually prolonged, with the
swelling typically slowly increasing over ⬃24 hours and then resolving even more slowly over the subsequent 2–4 days. A clinical observation that is useful for suggesting that a patient might have HAE is
the lack of a clear response to the standard medicines used to treat
allergic swelling (antihistamines, corticosteroids, and epinephrine).11,13 Attacks are frequently but not always preceded by prodromal symptoms, most classically a serpiginous nonpruritic rash called
erythema marginatum.13,16
The swelling in HAE is often quite severe and may be associated
with considerable morbidity and even mortality.11,13,14 The most
Copyright © Oceanside Publications, Inc. All rights reserved. DO NOT COPY
For permission to copy go to https://www.oceansidepubl.com/permission.htm
S101
Figure 1. Bradykinin-mediated angioedema. Hereditary angioedema (HAE), acquired C1 inhibitor (C1INH) deficiency,
and angiotensin-converting enzyme (ACE)
inhibitor-associated angioedema are all
thought to result from bradykinin. Types I
and II HAE as well as acquired C1INH
deficiency are characterized by low C1INH
functional activity, based on decreased synthesis or increased catabolism, respectively.
The decreased C1INH activity prevents effective regulation of the contact system and
results in enhanced bradykinin generation.
Type III HAE may involve enhanced factor
XII activity in at least some patients. The
enhanced factor XII activity may lead to
increased plasma kallikrein activation and
enhanced bradykinin generation. ACE is an
endopeptidase that degrades bradykinin,
among other substrates. ACE inhibitors,
therefore, may decrease the normal catabolism of bradykinin and lead to elevated bradykinin levels.
feared swelling in HAE is laryngeal attacks, which can occlude the
airway and result in asphyxiation.17–22 Intubation may be lifesaving in
an HAE patient with severe laryngeal angioedema; however, difficulty due to the distorted upper airway occasionally requires tracheotomy to preserve the airway. It is critically important to consider all
patients with HAE at risk for asphyxiation because of laryngeal
attacks, irrespective of whether they have ever had a laryngeal attack
in the past or how severe their disease is. Over 50% of patients report
having experienced at least one laryngeal attack.15
Abdominal attacks frequently cause significant morbidity, including severe abdominal pain, nausea and vomiting, and orthostatic
hypotension due to third spacing of fluid. The severe nature of these
symptoms can mimic a surgical abdomen, and many HAE patients
have undergone unnecessary abdominal surgery for HAE attacks.
Patients frequently require treatment in the emergency department or
even hospitalization for abdominal attacks. Narcotic addiction has
been a problem in some HAE patients, because of the need for
repeated treatments with potent opiate painkillers. Although often
considered benign by physicians, even extremity attacks can prevent
patients from going to work or attending school when they involve
the dominant hand or the feet.
Approximately 50% of the patients began swelling before the age of
10 years, and almost all patients reported onset of symptoms before
the age of 20 years.13 Despite the importance of making the diagnosis,
a delay of ⬃10–20 years between symptom onset and proper diagnosis has been observed.13,23
Attacks occur unpredictably, with varying frequency and severity.
The average frequency of attacks in untreated HAE patients is unclear
but variable. Disease severity is highly variable both between and
sometimes within kindreds. Furthermore, no simple relationship has
been observed between disease severity and plasma C1INH levels.
Although most attacks occur without a clear precipitating factor,
stress and minor trauma are each well recognized to be capable of
provoking HAE attacks. Additionally, many women report that exogenous estrogens (from oral contraceptives or hormonal replace-
S102
ment therapy) significantly worsen attack frequency and severity.
Pregnancy is associated with increased disease severity in about
one-third of women; however, another third of women report lessened angioedema during pregnancy. A striking finding, however, is
the lack of swelling that occurs at the time of parturition.13 Once a
diagnosis of HAE is suspected, confirming the diagnosis is usually
straightforward and is based on laboratory measurement of the complement C4 level as well as the C1INH level or activity (Table 2).11,24
PATHOPHYSIOLOGY OF HAE WITH LOW C1INH
ACTIVITY
In 1963, Dr. Virginia Donaldson found that patients with HAE were
deficient in C1INH activity while their unaffected relatives as well as
patients with other forms of angioedema and normal controls all had
normal C1INH activity.25 Two years later, Dr. Fred Rosen discovered
that ⬃15% of HAE patients had normal C1INH levels but low C1INH
activity (type II HAE) as opposed to the more common pattern of low
C1INH levels and activity.26
C1INH is a member of the serine protease inhibitor (serpin) superfamily.27 Like other serpin inhibitors (such as ␣-1-antitrypsin and
antithrombin), C1INH functions like a “molecular mousetrap.”28
Most of the structure is very rigid and under considerable stress;
however, the reactive mobile loop, located at the top of the protein is
mobile. The reactive mobile loop contains the active site where
C1INH is attacked by its target proteases. Once a protease cleaves the
peptide bond at the active site, there is a large-scale rearrangement of
C1INH in which an arm of the reactive mobile loop inserts into the
central ␤-sheet, trapping the target protease. There is a 1:1 stoichiometric relationship between the protease and C1INH, and each molecule of protease inhibited consumes one molecule of C1INH. The
mechanism of inhibition thus involves a suicide inactivation.29
The C1INH gene, SERPING1, is organized into eight exons with
intervening introns. Disease causing mutations are scattered throughout the gene, in fact ⬎200 different mutations associated with types I
May–June 2014, Vol. 28, No. 3
Copyright © Oceanside Publications, Inc. All rights reserved. DO NOT COPY
For permission to copy go to https://www.oceansidepubl.com/permission.htm
Table 1 Distinguishing bradykinin-mediated angioedemas based on clinical characteristics
Characteristic
Type I HAE
Type II HAE
Type III HAE
Acquired C1INH
Deficiency
ACE Inhibitor
Associated
Age of onset
Childhood or teenage
Childhood or teenage
Middle age or older
Adult
Family history
Predilection for specific
swelling sites
Prodrome
Predominantly affects
women
Associated with using
an ACE inhibitor
Usually ⫹
None
Usually ⫹
None
Negative
None
Negative
Especially face
Often
No
Often
No
Teenage or young
adult
Essential
Especially face and
extremities
Probably not
Yes
Probably not
No
No
No
Noⴱ
Noⴱ
Noⴱ
Noⴱ
Yes
ⴱMay be made worse by an ACE inhibitor.
ACE ⫽ angiotensin-converting enzyme; C1INH ⫽ C1 inhibitor; HAE ⫽ hereditary angioedema.
Table 2 Distinguishing bradykinin-mediated angioedemas based on laboratory profile
Test
Type I HAE
Type II HAE
Type III HAE
Acquired C1INH
Deficiency
ACE Inhibitor Associated
C1INH antigenic level
C1INH functional level
C4
C1q
Factor XII mutation
Low
Low
Low
Normal
Absent
Normal
Low
Low
Normal
Absent
Normal
Normal
Normal
Normal
Occas found
Low
Low
Low
Low
Absent
Normal
Normal
Normal
Normal
Absent
ACE ⫽ angiotensin-converting enzyme; C1INH ⫽ C1 inhibitor; HAE ⫽ hereditary angioedema.
and II HAE have been described. Interestingly, mutations in the
reactive mobile loop near or at the active site result in a dysfunctional
protein that characterizes type II HAE. There is evidence that type I
HAE often involves failure of the nascent protein to fold properly
within the endoplasmic reticulum.
The mechanism by which a deficiency of C1INH causes increased
vascular permeability and angioedema has been the subject of intense
investigation over the past 30 years. Incubation of HAE plasma ex vivo
at 37°C generates a factor that caused smooth muscle contraction and
increased vascular permeability.30 This “vascular permeability-enhancing factor” was correctly assumed to be the mediator of swelling
in HAE; however, the final characterization of the factor remained
elusive and controversial for many years.
C1INH is the principle inhibitor of several complement and contact
system proteases as well as a minor inhibitor of coagulation factor XIa
and plasmin.27 During HAE attacks, each of these plasma proteolytic
cascades is activated with the potential to generate several vasoactive
compounds. Two potential mediators of swelling in HAE were identified as likely candidates to mediate enhanced vascular permeability
in HAE: C2 kinin, generated through activation of the classic complement and fibrinolytic pathways,31 and bradykinin, generated
through activation of the contact system.32 Despite initial suggestions that C2 kinin represented the vascular permeability-enhancing activity, compelling laboratory and clinical data have conclusively shown that bradykinin is the primary mediator of swelling
in HAE.32–42
The nanopeptide bradykinin is generated when active plasma kallikrein cleaves high molecular weight kininogen.43 Plasma kallikrein
is activated from its inactive zymogen by the protease factor XII, and
both plasma kallikrein and factor XII are normally inhibited by
C1INH (Fig. 1). The released bradykinin moiety potently increases
vascular permeability by binding to its cognate receptor (the bradykinin B2-receptor) on vascular endothelial cells. Given the plethora of
evidence supporting bradykinin as the mediator of swelling in HAE,
it was of little surprise that drugs targeting bradykinin generation or
action have shown efficacy during angioedema attacks.44,45
American Journal of Rhinology & Allergy
TYPE III HAE
In 2000, two separate groups described a familial form of angioedema in which the C1INH was absolutely normal.46,47 Clinically, this
new type of HAE (which is often called type III HAE) resembles types I and
II HAE, although type III HAE presents at a somewhat older age and
appears to have fewer abdominal and more facial attacks (Table 1).48 Type
III HAE was initially thought to occur exclusively in women, particularly during times of increased estrogen exposure. Subsequently,
affected men have been found49,50; and the strength of the relationship
between estrogen exposure and angioedema is shown to be modest.48
At the current time, there are no firm criteria for making a diagnosis
of type III HAE; however, it should be considered in patients with
recurrent angioedema who have a strong family history of angioedema as well as normal C1INH antigenic and functional levels and
a normal C4 level (Table 2).
A mutation in the factor XII gene (Thr328Lys or Thr328Arg) that
cosegregated with disease presence was described in some families
with type III.51 This mutation was reported to cause a gain-of-function
in factor XII activity, an observation that was particularly exciting
because a gain-of-function in factor XII would be expected to result in
enhanced generation of bradykinin and thus explain the pathogenesis
of this disorder (Fig. 1). Since then it has become clear that only a
minority of families with type III HAE have a mutation in factor XII.48
Furthermore, a recent study failed to confirm the gain-of-function in
the mutant factor XII.52
The genetic heterogeneity of type III HAE suggests that this diagnosis may encompass a heterogeneous group of disorders. Because
type III HAE can clearly cause severe laryngeal angioedema, the lack
of clear diagnostic or pathophysiological understanding of this disease is of significant concern and requires additional research.
ACQUIRED C1INH DEFICIENCY
In addition to its deficiency on a hereditary basis, C1INH deficiency
also occurs in a sporadic acquired form (acquired C1INH deficiency).53 These patients, who typically present in middle age or
Copyright © Oceanside Publications, Inc. All rights reserved. DO NOT COPY
For permission to copy go to https://www.oceansidepubl.com/permission.htm
S103
older, experience recurrent angioedema that is similar if not identical
to HAE attacks.54 Patients with acquired C1INH deficiency are also at
significant risk for laryngeal attacks. Hereditary and acquired C1INH
deficiencies are, however, relatively simple to differentiate based on
the lack of family history and much later age of onset in the acquired
form (Table 1). Laboratory evaluation of acquired C1INH deficiency
typically shows low C4 levels as well as low C1INH levels and activity
similar to type I HAE; however, the C1q level is also frequently reduced
in acquired C1INH deficiency but not in HAE (Table 2).55
The primary basis for the C1INH deficiency in acquired C1INH
deficiency is increased catabolism of C1INH rather than decreased
secretion of functional C1INH as found in HAE (Fig. 1).56,57 The
increased C1INH catabolism in acquired C1INH deficiency often
related to underlying conditions. Many of these patients have tumors,
particularly lymphoreticular malignancies, or other diseases that may
consume C1INH.58,59 Importantly, successful treatment of the underlying disease may resolve the acquired C1INH deficiency.60 Patients
with acquired C1INH deficiency also frequently have autoantibodies
directed to C1INH, not infrequently associated with a monoclonal
gammopathy of unknown significance.61,62 The autoantibody has
been shown to interfere with normal C1INH-protease interactions,
favoring the degradation of C1INH into a smaller cleaved dysfunctional protein.39,63
CONCLUSIONS
ACE INHIBITOR-ASSOCIATED ANGIOEDEMA
REFERENCES
ACE inhibitors are a class of commonly used antihypertensive
medications that are well recognized to be associated with angioedema in rare patients.64 The angioedema associated with ACE inhibitor tends to show a predilection for involving the face, lips,
tongue, and throat (Table 1).65 The overall prevalence of ACE inhibitor-induced angioedema is estimated to range from 1 per 1000 patients using these drugs.66 A recent large study among patients in the
United States Veterans Affairs system revealed an overall incidence of
1.97 cases per 1000 patients initiating ACE inhibitor therapy (compared with 0.51 cases per 1000 in patients initiating therapy with an
antihypertensive drug other than an ACE inhibitor.67 Substantial
variation in the risk of developing angioedema are seen among subsets of patients with black patients having a nearly fourfold increase
in risk and women having a 1.5-fold increase in risk. The risk of
developing angioedema while using an ACE inhibitor is highest
during the 1st month of treatment but does not disappear even in
patients who have been taking an ACE inhibitor for years.67,68
The angioedema resulting from use of an ACE inhibitor occurs on
a class-specific rather than a drug-specific basis. All patients who
develop angioedema without urticaria while taking an ACE inhibitor
should be suspected of potentially having ACE inhibitor-associated
angioedema. Because this angioedema tends to be recurrent and
potentially life-threatening when involving the larynx,69,70 patients
who develop ACE inhibitor-associated angioedema must discontinue
the use of all ACE-I drugs. Although there has been concern about
switching patients who experience angioedema on an ACE inhibitor
to an angiotensin receptor blocker, several studies have shown that
there is no evidence of increased risk when such a patient is switched
to an angiotensin receptor blocker.71–73
The pathophysiology of ACE inhibitor-associated angioedema is
thought to relate to decreased catabolism of bradykinin. ACE, also
known as kininase 2, is an endopeptidase that degrades a variety of
peptides including bradykinin (Fig. 1). A drug (omapatrilat) that
inhibited both ACE and neutral endopeptidase (which is also involved in the degradation of bradykinin) had a substantially increased risk of angioedema.74 Furthermore, patients with a history of
ACE inhibitor-associated angioedema were more likely than patients
who tolerated ACE inhibitors without angioedema to have decreased
plasma aminopeptidase P (another endopeptidase involved in the
degradation of bradykinin) activity.75 Other studies suggest that in
addition to bradykinin, substance P (a peptide also degraded by ACE)
may be involved in ACE inhibitor-associated angioedema.76
S104
Bradykinin is a pluripotent peptide mediator, exerting different
effects depending on the tissue in which it is generated. When generated in the vascular space, bradykinin can mediate enhanced vascular permeability—leading to movement of fluid from the vasculature space into the interstitial fluid (angioedema). Mechanistic studies
have suggested (and in some cases proven) that increased levels of
bradykinin are responsible for the angioedema associated with HAE,
acquired C1INH deficiency, and ACE inhibitor-associated angioedema. Recognition of this fact is critically important because bradykinin-mediated angioedema is different in many key respects than the
more common histamine-mediated angioedema. Severe laryngeal
swelling is not infrequently encountered in bradykinin-mediated angioedema, and bradykinin-mediated angioedema does not respond to
the drugs typically used to treat histamine-mediated angioedema.
Furthermore, novel therapies are becoming available that for the first
time provide effective treatment for bradykinin-mediated angioedema. Because the characteristics and treatment of these angioedemas are quite distinct from each other and histamine-mediated angioedema, it is crucial that the physician be able to recognize and
distinguish these swelling disorders.
1.
2.
3.
4.
5.
6.
7.
8.
9.
10.
11.
12.
13.
14.
15.
16.
Osler W. Hereditary angio-neurotic oedema. Am J Med Sci 95:362–
367, 1888.
Zilberberg M, Jacobsen T and Tillotson G. The burden of hospitalizations and emergency department visits with hereditary angioedema and angioedema in the United States, 2007. Allergy Asthma
Proc 31:511–519, 2010.
Lumry WR, Castaldo AJ, Vernon MK, et al. The humanistic burden of
hereditary angioedema: Impact on health-related quality of life, productivity, and depression. Allergy Asthma Proc 31:407–414, 2010.
Asero R, Riboldi P, Tedeschi A, et al. Chronic urticaria: A disease at
a crossroad between autoimmunity and coagulation. Autoimmun
Rev 7:71–76, 2007.
Davis AE III. The pathogenesis of hereditary angioedema. Transfus
Apher Sci 29:195–203, 2003.
Agostoni A, Cicardi M and Porreca W. Peripheral edema due to
increased vascular permeability: A clinical appraisal. Int J Clin Lab
Res 21:241–246, 1992.
Donaldson VH, Ratnoff OD, Dias Da Silva W, and Rosen FS. Permeability-increasing activity in hereditary angioneurotic edema plasma.
II. Mechanism of formation and partial characterization. J Clin Invest
48:642–653, 1969. (PMCID: 322269.)
Christiansen SC and Zuraw BL. Hereditary angioedema: Management of laryngeal attacks. Am J Rhinology & Allergy 25:379–382,
2011.
Pappalardo E, Cicardi M, Duponchel C, et al. Frequent de novo
mutations and exon deletions in the C1inhibitor gene of patients with
angioedema. J Allergy Clin Immunol 106:1147–1154, 2000.
Bowen T, Cicardi M, Farkas H, et al. 2010 International consensus
algorithm for the diagnosis, therapy and management of hereditary
angioedema. Allergy Asthma Clin Immunol 6:24, 2010. (PMCID:
2921362.)
Zuraw BL. Clinical practice. Hereditary angioedema. N Engl J Med
359:1027–1036, 2008.
Bygum A. Hereditary angio-oedema in Denmark: A nationwide survey. Br J Dermatol 161:1153–1158, 2009.
Frank MM, Gelfand JA and Atkinson JP. Hereditary angioedema:
The clinical syndrome and its management. Ann Intern Med 84:586–
593, 1976.
Agostoni A and Cicardi M. Hereditary and acquired C1-inhibitor
deficiency: Biological and clinical characteristics in 235 patients. Medicine (Baltimore) 71:206–215, 1992.
Bork K, Meng G, Staubach P and Hardt J. Hereditary angioedema:
New findings concerning symptoms, affected organs, and course.
Am J Med 119:267–274, 2006.
Prematta MJ, Kemp JG, Gibbs JG, et al. Frequency, timing, and type
of prodromal symptoms associated with hereditary angioedema attacks. Allergy Asthma Proc 30:506–511, 2009.
May–June 2014, Vol. 28, No. 3
Copyright © Oceanside Publications, Inc. All rights reserved. DO NOT COPY
For permission to copy go to https://www.oceansidepubl.com/permission.htm
17.
18.
19.
20.
21.
22.
23.
24.
25.
26.
27.
28.
29.
30.
31.
32.
33.
34.
35.
36.
37.
38.
39.
40.
41.
Papadopoulou-Alataki E. Upper airway considerations in hereditary
angioedema. Curr Opin Allergy Clin Immunol 10:20–25, 2010.
Bork K, Hardt J, Schicketanz KH and Ressel N. Clinical studies of
sudden upper airway obstruction in patients with hereditary angioedema due to C1 esterase inhibitor deficiency. Arch Intern Med
163:1229–1235, 2003.
Bork K and Ressel N. Sudden upper airway obstruction in patients
with hereditary angioedema. Transfus Aphers Sci 29:235–238, 2003.
Bork K and Barnstedt SE. Laryngeal edema and death from asphyxiation after tooth extraction in four patients with hereditary angioedema. J Am Dent Assoc 134:1088–1094, 2003.
Bork K, Siedlecki K, Bosch S, et al. Asphyxiation by laryngeal edema
in patients with hereditary angioedema. Mayo Clin Proc 75:349–354,
2000.
Agostoni A, Cicardi M, Cugno M and Storti E. Clinical problems in
the C1-inhibitor deficient patient. Behring Inst Mitt 93:306–312, 1993.
Roche O, Blanch A, Caballero T, et al. Hereditary angioedema due to
C1 inhibitor deficiency: Patient registry and approach to the prevalence in Spain. Ann Allergy Asthma Immunol 94:498–503, 2005.
Gompels MM, Lock RJ, Abinun M, et al. C1 inhibitor deficiency:
Consensus document. Clin Exp Immunol 139:379–394, 2005.
Donaldson VH and Evans RR. A biochemical abnormality in hereditary angioneurotic edema: Absence of serum inhibitor of C’1-esterase. Am J Med 35:37–44, 1963.
Rosen FS, Pensky J, Donaldson V and Charache P. Hereditary angioneurotic edema: Two genetic variants. Science 148:957–958, 1965.
Davis AE III. C1 inhibitor and hereditary angioneurotic edema. Annu
Rev Immunol 6:595–628, 1988.
Lomas DA, Belorgey D, Mallya M, et al. Molecular mousetraps and
the serpinopathies. Biochem Soc Trans 33:321–330, 2005.
Patston PA, Gettins P, Beechem J and Schapira M. Mechanism of
serpin action: Evidence that Cl inhibitor functions as a suicide substrate. Biochemistry 30:8876–8882, 1991.
Donaldson VH, Ratnoff OD, Da Silva WD and Rosen FS. Permeability-increasing activity in hereditary angioneurotic edema plasma. II.
Mechanism of formation and partial characterization. J Clin Invest
48:642–653, 1969.
Donaldson VH, Rosen FS and Bing DH. Role of the second component of complement (C2) and plasmin in kinin release in hereditary
angioneurotic edema (H.A.N.E.) plasma. Trans Assoc Am Physicians
40:174–183, 1977.
Curd JG, Prograis L Jr., and Cochrane CG. Detection of active kallikein in induced blister fluids of hereditary angioedema patients. J
Exp Med 152:742–747, 1980.
Curd JG, Yelvington M, Burridge N, et al. Generation of bradykinin
during incubation of hereditary angioedema plasma. Mol Immunol
19:1365, 1982.
Fields T, Ghebrehiwet B and Kaplan AP. Kinin formation in hereditary angioedema plasma: Evidence against kinin derivation from C2
and in support of “spontaneous” formation of bradykinin. J Allergy
Clin Immunol 72:54–60, 1983.
Lammle B, Zuraw BL, Heeb MJ, et al. Detection and quantitation of
cleaved and uncleaved high molecular weight kininogen in plasma
by ligand blotting with radiolabeled plasma prekallikrein or factor
XI. Thromb Haemost 59:151–161, 1988.
Berrettini M, Lammle B, White T, et al. Detection of in vitro and in
vivo cleavage of high molecular weight kininogen in human plasma
by immunoblotting with monoclonal antibodies. Blood 68:455–462,
1986.
Schapira M, Silver LD, Scott CF, et al. Prekallikrein activation and
high- molecular-weight kininogen consumption in hereditary angioedema. N Engl J Med 308:1050–1054, 1983.
Nussberger J, Cugno M, Amstutz C, et al. Plasma bradykinin in
angio-oedema. Lancet 351:1693–1697, 1998.
Zuraw BL and Curd JG. Demonstration of modified inactive first
component of complement (C1) inhibitor in the plasmas of C1 inhibitor-deficient patients. J Clin Invest 78:567–575, 1986.
Zuraw BL, Lammle B, Sugimoto S, et al. Cleavage of high molecular
weight kininogen in plasma during attacks of angioedema in hereditary angioedema. J Allergy Clin Immunol 79:177, 1987.
Shoemaker LR, Schurman SJ, Donaldson VH and Davis AE. Hereditary angioneurotic oedema: Characterization of plasma kinin and
vascular permeability-enhancing activities. Clin Exp Immunol 95:22–
28, 1994.
American Journal of Rhinology & Allergy
42.
43.
44.
45.
46.
47.
48.
49.
50.
51.
52.
53.
54.
55.
56.
57.
58.
59.
60.
61.
62.
63.
64.
65.
66.
Han ED, MacFarlane RC, Mulligan AN, et al. Increased vascular
permeability in C1 inhibitor-deficient mice mediated by the bradykinin type 2 receptor. J Clin Invest 109:1057–1063, 2002.
Cochrane CG and Griffin JH. The biochemistry and pathophysiology
of the contact system of plasma. Adv Immunol 33:241–306, 1982.
Zuraw BL and Christiansen SC. New promise and hope for treating
hereditary angioedema. Expert Opin Investig Drugs 17:697–706,
2008.
Bork K, Frank J, Grundt B, et al. Treatment of acute edema attacks in
hereditary angioedema with a bradykinin receptor-2 antagonist
(Icatibant). J Allergy Clin Immunol 119:1497–1503, 2007.
Binkley KE and Davis A III. Clinical, biochemical, and genetic characterization of a novel estrogen-dependent inherited form of angioedema. J Allergy Clin Immunol 106:546–550, 2000.
Bork K, Barnstedt SE, Koch P and Traupe H. Hereditary angioedema
with normal C1-inhibitor activity in women. Lancet 356:213–217,
2000.
Bork K, Wulff K, Hardt J, et al. Hereditary angioedema caused by
missense mutations in the factor XII gene: Clinical features, trigger
factors, and therapy. J Allergy Clin Immunol 124:129–134, 2009.
Bork K, Gul D and Dewald G. Hereditary angio-oedema with normal
C1 inhibitor in a family with affected women and men. Br J Dermatol
154:542–545, 2006.
Martin L, Raison-Peyron N, Nothen MM, et al. Hereditary angioedema with normal C1 inhibitor gene in a family with affected
women and men is associated with the p.Thr328Lys mutation in the
F12 gene. J Allergy Clin Immunol 120:975–977, 2007.
Cichon S, Martin L, Hennies HC, et al. Increased activity of coagulation factor XII (Hageman factor) causes hereditary angioedema
type III. Am J Hum Genet 79:1098–1104, 2006.
Bork K, Kleist R, Hardt J and Witzke G. Kallikrein-kinin system and
fibrinolysis in hereditary angioedema due to factor XII gene mutation
Thr309Lys. Blood Coagul Fibrinolysis 20:325–332, 2009.
Caldwell JR, Ruddy S, Schur PH and Austen KF. Acquired C1 inhibitor deficiency in lymphosarcoma. Clin Immunol Immunopathol
1:39–52, 1972.
Zingale LC, Castelli R, Zanichelli A and Cicardi M. Acquired deficiency of the inhibitor of the first complement component: Presentation, diagnosis, course, and conventional management. Immunol
Allergy Clin North Am 26:669–690, 2006.
Breitbart SI and Bielory L. Acquired angioedema: Autoantibody associations and C1q utility as a diagnostic tool. Allergy Asthma Proc
31:428–434, 2010.
Melamed J, Alper CA, Cicardi M and Rosen FS. The metabolism of C1
inhibitor and C1q in patients with acquired C1-inhibitor deficiency. J
Allergy Clin Immunol 77:322–326, 1986.
Zuraw BL and Altman LC. Acute consumption of C1 inhibitor in a
patient with acquired C1-inhibitor deficiency syndrome. J Allergy
Clin Immunol 88:908–918, 1991.
Schreiber AD, Zweiman B, Atkins P, et al. Acquired angioedema with
lymphoproliferative disorder: Association of C1 inhibitor deficiency
with cellular abnormality. Blood 48:567–580, 1976.
Cicardi M, Beretta A, Colombo M, et al. Relevance of lymphoproliferative disorders and of anti-C1 inhibitor autoantibodies in acquired
angio-oedema. Clin Exp Immunol 106:475–480, 1996.
Gelfand JA, Boss GR, Conley CL, et al. Acquired C1 esterase inhibitor
deficiency and angioedema: A review. Medicine 58:321–328, 1979.
Jackson J and Feighery C. Autoantibody-mediated acquired deficiency of C1 inhibitor. N Engl J Med 318:122–123, 1988.
Jackson J, Sim RB, Whelan A and Feighery C. An IgG autoantibody
which inactivates C1-inhibitor. Nature 323:722–724, 1986.
Malbran A, Hammer CH, Frank MM and Fries LF. Acquired angioedema: Observations on the mechanism of action of autoantibodies
directed against C1 esterase inhibitor. J Allergy Clin Immunol 81:
1199–1204, 1988.
Brown NJ and Vaughan DE. Angiotensin-converting enzyme inhibitors. Circulation 97:1411–1420, 1998.
Byrd JB, Adam A and Brown NJ. Angiotensin-converting enzyme
inhibitor-associated angioedema. Immunol Allergy Clin North Am
26:725–737, 2006.
Slater EE, Merrill DD, Guess HA, et al. Clinical profile of angioedema
associated with angiotensin converting-enzyme inhibition. JAMA
260:967–970, 1988.
Copyright © Oceanside Publications, Inc. All rights reserved. DO NOT COPY
For permission to copy go to https://www.oceansidepubl.com/permission.htm
S105
67.
68.
69.
70.
71.
Miller DR, Oliveria SA, Berlowitz DR, et al. Angioedema incidence in
US veterans initiating angiotensin-converting enzyme inhibitors. Hypertension 51:1624–1630, 2008.
Agostoni A, Cicardi M, Cugno M, et al. Angioedema due to angiotensin-converting enzyme inhibitors. Immunopharmacology 44:21–
25, 1999.
Brown NJ, Snowden M and Griffin MR. Recurrent angiotensin-converting enzyme inhibitor-associated angioedema. JAMA 278:232–233,
1997.
Roberts DS, Mahoney EJ, Hutchinson CT, et al. Analysis of recurrent
angiotensin converting enzyme inhibitor-induced angioedema. Laryngoscope 118:2115–2120, 2008.
Cicardi M, Zingale LC, Bergamaschini L and Agostoni A. Angioedema associated with angiotensin-converting enzyme inhibitor use:
Outcome after switching to a different treatment. Arch Intern Med
164:910–913, 2004.
S106
72.
73.
74.
75.
76.
Yusuf S, Teo K, Anderson C, et al. Effects of the angiotensin-receptor
blocker telmisartan on cardiovascular events in high-risk patients
intolerant to angiotensin-converting enzyme inhibitors: A randomised controlled trial. Lancet 372:1174–1183, 2008.
Johnsen SP, Jacobsen J, Monster TB, et al. Risk of first-time hospitalization for angioedema among users of ACE inhibitors and angiotensin receptor antagonists. Am J Med 118:1428–1429, 2005.
Kostis JB, Packer M, Black HR, et al. Omapatrilat and enalapril in
patients with hypertension: The Omapatrilat Cardiovascular Treatment vs. Enalapril (OCTAVE) trial. Am J Hypertens 17:103–111, 2004.
Adam A, Cugno M, Molinaro G, et al. Aminopeptidase P in individuals with a history of angio-oedema on ACE inhibitors. Lancet 359:
2088–2089, 2002.
Byrd JB, Touzin K, Sile S, et al. Dipeptidyl peptidase IV in angiotensin-converting enzyme inhibitor associated angioedema. Hypertension 51:141–147, 2008. (PMCID: 2749928.)
e
May–June 2014, Vol. 28, No. 3
Copyright © Oceanside Publications, Inc. All rights reserved. DO NOT COPY
For permission to copy go to https://www.oceansidepubl.com/permission.htm
Rhinology MOC Review - Questions
1. Which of the following is an effect of intranasal steroid therapy in
a patient with nasal congestion and sleep disordered breathing
A. Increases daytime somnolence
B. Improves sleep quality
C. Improves Apnea-Hypopnea Index when used with nasal dilators
D. Reduces Apnea-Hypopnea Index to near physiologic levels
2. Which is a cause of anaphylaxis associated with immunotherapy?
A. Use of an ACE inhibitor on patient receiving injections for
inhalant allergen
B. Prior history of immunotherapy
C. History of pneumonia
D. Dosing errors
3. A patient presents with recurrent unilateral epistaxis, nasal obstruction, and numbness along the V2 distribution. Nasal endoscopy reveals a frond –like mass in the posterior nasal cavity. The
most appropriate next step in management is:
A. Biopsy in the office setting
B. Biopsy in the operating room
C. Treatment with antibiotics
D. CT and MRI scan
4. Which of the following is the most effective single agent medication for allergic rhinitis?
A. Oral antihistamine
B. Montelukast
C. Nasal steroid spray
D. Ipratroprium bromide
5. Which of the following surgical maneuvers most likely cause
empty nose syndrome?
A. Septoplasty
B. Internal nasal valve augmentation
C. Inferior turbinate resection
D. Reduction of concha bullosa
6. Which pathogen is not typically associated with invasive fungal
sinusitis?
A. Candida
B. Aspergillus
C. Rizomucor
D. Rizopus
7. What is the most common location of sinonasal inverted papillomas?
A. Sphenoid sinus
B. Maxillary sinus
C. Nasal septum
D. Ethmoid sinus
8. The edematous mucosa observed in allergic rhinitis results from
an inflammatory cascade characterized by:
A. IgE-mediated cell response
B. Mucopurulence
C. Overexpression of MUC5
D. IL-2 overexpression
9. What type of pathologic response characterizes staphylococcal
superantigen stimulation?
A. Th2 lymphocytic response
B. Predominant neutrophilic response
C. Dysfunction of membrane-bound pattern recognition receptors
D. Stimulation of regulatory T cells
American Journal of Rhinology & Allergy
10. Which of the following is a Bent and Kuhn criteria for allergic
fungal rhinosinusitis?
A. Atopic history
B. Thickened mucus
C. Neutrophils
D. Hyphae invading the tissues
11. All of the following are appropriate methods to improve the
internal nasal valve except:
A. Submucosal resection of the caudal inferior turbinate
B. Spreader grafts
C. Alar batten grafts
D. Butterfly grafts
12. Which of the following is true regarding the modified endoscopic
medial maxillectomy procedure for cystic fibrosis patients?
A. Physical debridement of mucus from the maxillary sinus is
less challenging
B. There is no improved access over regular maxillary antrostomy
C. The maxillary sinus can be cured of chronic mucus transport
dysfunction
D. There is a high likelihood further endoscopic sinus surgery
will be required for the maxillary sinus
13. Which of the following is the primary mediator that increases
vascular permeability in both hereditary and ACE inhibitor-associated angioedema?
A. Angiotensin
B. Histamine
C. Bradykinin
D. Alpha-1-antitrypsin
14. Which symptom(s) is/are most characteristic of gustatory rhinitis?
A. Fetid odor
B. Watery rhinorrhea with coffee
C. Nasal puritus, sneezing, and conjunctivitis with spicy foods
D. Anosmia
15. A heterogenous group of patients with chronic nasal symptoms
that are not immunologic or infectious in origin and are usually
not associated with eosinophilia best describes:
A. Gustatory rhinitis
B. Non-allergic rhinitis with eosinophilia syndrome
C. Rhinitis medicamentosa
D. Non-allergic rhinitis
16. Challenging an individual with an aeroallergen and demonstrating a significant increase in mucosal eosinophils in the mucosa on
the challenged side as well as the contralateral nasal cavity supports which theory for a pathophysiologic relationship between
allergy rhinitis and CRS?
A. Sensitization to colonizing fungi
B. Systemic allergic inflammatory process
C. Sensitization to colonizing bacteria
D. Sensitization to IgE
17. The recombinant monoclonal anti-IgE antibody omalizumab is
FDA approved for which indication?
A. Inadequately controlled moderate to severe resistant, IgE mediated asthma
B. IgE mediated urticaria
C. Allergic rhinitis
D. Severe bee venom allergies
18. Which is one of the four diagnostic criteria for Wegener’s granulomatousis?
Copyright © Oceanside Publications, Inc. All rights reserved. DO NOT COPY
For permission to copy go to https://www.oceansidepubl.com/permission.htm
S107
A.
B.
C.
D.
Elevated liver enzymes
Presence of extravascular eosinophils on biopsy
Oral ulcers or nasal discharge
Positive c-ANCA
19. What is the most common risk factor for acute fulminant invasive
fungal sinusitis?
A. Elevated blood sugars
B. Neutropenia
C. Fungal colonization
D. Allergic fungal sinusitis
20. First line surgical therapy for pediatric chronic rhinosinusitis is
A. Balloon sinuplasty
B. Bilateral maxillary antrostomies
C. Adenoidectomy
D. Ethmoidectomy
21. All of the following interventions are useful for acute epistaxis
except:
A. Administration of topical vasoconstrictive agent
B. Pinching the bridge of the nose
C. Silver nitrate cautery
D. Sphenopalatine artery ligation
22. Which of the following medical interventions has the highest
level of evidence and grade of recommendations as a treatment
for chronic rhinosinusitis with nasal polyposis?
A. Guaifenesin
B. Topical anti-mycotics
C. Topical nasal steroids
D. Short term oral antibiotics
23. The next most appropriate step in management for an elderly
male with gradual onset of anosmia with a negative sinus CT
scan and normal nasal endoscopy is:
A. High dose oral steroids and topical steroids
B. A three week course of broad spectrum antibiotic
C. Neurologic evaluation
D. Endoscopic sinus surgery
24. When hereditary angioedema affects the head and neck, the most
common area of involvement is:
A. Larynx
B. Face
C. Auricle
D. Soft palate
25. A patient from sub-Sahara Africa comes in with atrophy of the
nasal mucosa, anosmia, and a foul odor. Culture of the nasal
crusts would likely grow:
A. Klebsiella ozaenae
B. Streptococcus pneuoniae
C. Varicella zoster
D. Pseudomonas aeruginosa
26. Which of the following medications can impact the validity of
skin testing for allergen-specific IgE?
A. First-, but not second-generation antihistamines
B. Beta-adrenergic blocking agents
C. Second-, but not first-generation antihistamines
D. Tricyclic antidepressants
27. When performing an endoscopic ethmoidectomy, violation of the
lateral lamella of the cribriform plate may occur. This is usually
due to a dissection carried too far
A. Posteriolaterally
B. Inferiolaterally
C. Superiolaterally
D. Superiomedially
S108
28. CT imaging of the paranasal sinuses has which of the following
advantages over MRI?
A. Improved definition of soft tissues
B. Optimal discrimination of bone
C. Negligible radiation exposure
D. Optimal differentiation of soft tissue masses
29. Which of the following is a potential complication of failure to
communicate the enlarged natural ostium of the maxillary sinus
with an accessory ostium during endoscopic sinus surgery?
A. Mucus re-entry into the antrum
B. Intraoperative optic nerve injury
C. Obstruction of the ethmoid infundibulum due residual tissue
D. Stenosis of the natural ostium
30. Which is not an anatomic cause of nasal obstruction?
A. Deviated septum
B. Allergic rhinitis
C. Turbinate hypertrophy
D. Adenoid hypertrophy
31. Which of the following is not a potential contributor to fatal SCIT
reactions?
A. Prior immunotherapy reactions
B. Alteration in immunotherapy allergen extracts
C. Multiple antigen therapy
D. Suboptimal asthma control during immunotherapy
32. What is the most common clinical sign in all anaphylactic reactions?
A. Diffuse urticaria
B. Respiratory failure
C. Hypotension
D. Gastrointestinal discomfort
33. Fibrous dysplasia and ossifying fibromas can be most easily
differentiated on the basis of which factor?
A. CT appearance
B. Histology
C. Location
D. Clinical behavior
34. Which is a possible complication of sphenopalatine artery ligation for posterior epistaxis:
A. Palatal anesthesia
B. It carries a risk of cerebrovascular accident
C. It carries a risk of toxic shock syndrome
D. Jaw claudication
35. The most common intraoperative complication during functional
endoscopic sinus surgery (FESS) on patient’s with nasal polyps is:
A. Cerebrospinal fluid leak
B. Intraoperative hemorrhage
C. Orbital injury
D. Loss of smell
36. Which type of chronic anosmia is most likely to improve, at least
transiently, after a course of oral and topical steroids?
A. Post-traumatic anosmia
B. Post-viral induced anosmia
C. Anosmia following anterior skull base surgery
D. Anosmia associated with chronic rhinosinusitis with nasal
polyps
37. What is the most appropriate indication for endoscopic sinus
surgery in a cystic fibrosis patient?
A. Nasal polyposis with significant symptoms refractory to extended culture-directed antibiotic treatment.
B. Chronic symptoms with brittle lung function awaiting a lung
transplant
May–June 2014, Vol. 28, No. 3
Copyright © Oceanside Publications, Inc. All rights reserved. DO NOT COPY
For permission to copy go to https://www.oceansidepubl.com/permission.htm
C. Extensive mucosal thickening on CT scan with minimal
symptoms
D. Purulence seen on endoscopy despite medical management
38. Nasal surgery has which of the following effects on sleep disordered breathing?
A. Decreased CPAP tolerance
B. Reduction of day-time somnolence
C. Reduction in Apnea-Hypopnea Index
D. Objective improvement in snoring
39. The internal nasal valve includes which of the following anatomic
sites as one of its boundaries?
A. Alar rim laterally
B. Nasal sill inferiorly
C. Septum medially
D. Pyriform aperture laterally
40. Which is the most common radiographic findings on CT scan of
allergic fungal rhinosinusitis?
A. Lytic bony lesions
B. Symmetric, homogenous opacification of the paranasal sinuses
C. Foci of near metallic density
D. Evidence of telecanthus
41. Which of the following is true of SLIT?
A. Meta-analysis shows that multi-antigen therapy is effective
B. The optimum SLIT dose is well defined.
C. SLIT is as effective as SCIT
D. No deaths due to anaphylaxis have been reported
42. Which describes a type 4 orbital complication based on Chandler’s classification?
A. Preseptal cellulitis: eyelid swelling, erythema, no limitation of
extraocular eye motion
B. Subperiosteal abcess: pus between periorbita and lamina papyracea
C. Orbital abcess: collection of pus within orbital tissues
D. Orbital cellulitis: diffuse, postseptal edema of orbital contents
without discrete abscess
43. Allergen specific peripheral T-cell tolerance is predominately
mediated by
A. IL-4
B. IL-5
C. IL-7
D. IL 10
American Journal of Rhinology & Allergy
44. Diagnostic criteria for Churg Strauss include all of the following
except:
A. Asthma
B. The presence of nasal polyps
C. Peripheral eosinophilia 10%
D. Polyneuropathy
45. What are the most common benign sinonasal tumors?
A. Juvenile Nasopharyngeal Angiofibroma
B. Inverted papilloma
C. Osteomas
D. Fibrous dyplasia
46. Which of the following sinonasal malignancies harbors the poorest 5 year survival?
A. Squamous cell carcinoma
B. Adenocarcinoma
C. Mucosal melanoma
D. Olfactory neuroblastoma
47. The primary symptom that is significantly worse in the late phase
response of Type I hypersensitivity is:
A. Nasal congestion
B. Pruritis
C. Sneezing
D. Rhinorrhea
48. What is the relationship between the drainage pathway of the
supraorbital ethmoid air cell and that of the frontal sinus? The
supraorbital ethmoid drains:
A. Anteromedial
B. Anterolateral
C. Posteromedial
D. Posterlateral
49. A patient presents with a history consistent with acute sinusitis.
Which of the following is an indication for CT scan?
A. Symptoms persistent for 2 weeks
B. Failure of a 5-day course of antibiotics
C. Periorbital erythema
D. Culture positive for pneumococcus
50. Where can the sphenoid ostium be located?
A. Superior to the most posterior ethmoid air cell
B. Lateral to the most posterior ethmoid air cell
C. Anterior to the most posterior ethmoid air cell
D. Medial to the most posterior ethmoid air cell
Copyright © Oceanside Publications, Inc. All rights reserved. DO NOT COPY
For permission to copy go to https://www.oceansidepubl.com/permission.htm
S109
Rhinology MOC Review - Answers
Question
number
Answer
Article title
Article starting
page number
1.
2.
3.
4.
5.
6.
7.
8.
9.
10.
11.
12.
13.
14.
15.
16.
17.
18.
19.
20.
21.
22.
23.
24.
25.
26.
27.
28.
29.
30.
31.
32.
33.
34.
35.
36.
37.
38.
39.
40.
41.
42.
43.
44.
45.
46.
47.
48.
49.
50.
B
D
D
C
C
A
B
A
A
A
C
A
C
B
D
B
A
C
B
C
B
C
C
B
A
D
D
B
A
B
C
A
D
A
B
D
A
B
C
C
D
C
D
B
C
C
A
D
C
D
The role of the nose in sleep-disordered breathing
The risk and management of anaphylaxis in the setting of immunotherapy
Sinonasal malignancies
Allergic rhinitis
Nasal obstruction
Invasive fungal rhinosinusitis
Benign sinonasal neoplasms
The united allergic airway: Connections between allergic rhinitis, asthma, and chronic sinusitis
Chronic rhinosinusitis
Allergic fungal rhinosinusitis
Augmenting the nasal airway: Beyond septoplasty
Cystic fibrosis chronic rhinosinusitis: A comprehensive review
Pathophysiology of hereditary angioedema
Nonallergic rhinitis
Nonallergic rhinitis
Determining the role of allergy in sinonasal disease
Immunomodulation of allergic sinonasal disease
Granulomatous diseases and chronic sinusitis
Invasive fungal rhinosinusitis
Pediatric rhinosinusitis: Definitions, diagnosis and management—An overview
Epistaxis
Nasal polyps
Olfactory disorders
Pathophysiology of hereditary angioedema
Nonallergic rhinitis
Determining the role of allergy in sinonasal disease
Surgery for sinonasal disease
Chronic rhinosinusitis
Sinus anatomy and function
Nasal obstruction
Subcutaneous and sublingual immunotherapy for allergic rhinitis: What is the evidence?
The risk and management of anaphylaxis in the setting of immunotherapy
Benign sinonasal neoplasms
Epistaxis
Nasal polyps
Olfactory disorders
Cystic fibrosis chronic rhinosinusitis: A comprehensive review
The role of the nose in sleep-disordered breathing
Sinus anatomy and function
Allergic fungal rhinosinusitis
Subcutaneous and sublingual immunotherapy for allergic rhinitis: What is the evidence?
Pediatric rhinosinusitis: Definitions, diagnosis and management—An overview
Immunomodulation of allergic sinonasal disease
Granulomatous diseases and chronic sinusitis
Benign sinonasal neoplasms
Sinonasal malignancies
Allergic rhinitis
Sinus anatomy and function
Chronic rhinosinusitis
Surgery for sinonasal disease
S60
S95
S31
S75
S7
S24
S27
S82
S11
S22
S54
S38
S101
S71
S71
S79
S86
S35
S24
S47
S9
S16
S68
S101
S71
S79
S51
S11
S3
S7
S90
S95
S27
S9
S16
S68
S38
S60
S3
S22
S90
S47
S86
S35
S27
S31
S75
S3
S11
S51
S110
May–June 2014, Vol. 28, No. 3
Copyright © Oceanside Publications, Inc. All rights reserved. DO NOT COPY
For permission to copy go to https://www.oceansidepubl.com/permission.htm
Copyright © Oceanside Publications, Inc. All rights reserved. DO NOT COPY
For permission to copy go to https://www.oceansidepubl.com/permission.htm
Save the Date!
ginny
North American
Rhinology & Allergy Conference
February 5-8, 2015
Boca Raton ~ Florida
Jointly Sponsored by the American
College of Allergy, Asthma and
Immunology (ACAAI)
and North American Rhinology &
Allergy Conference (NARAC)
NARAC dually represents
ENTs and allergists, with the
goal of improving the accuracy
of diagnosis and effectiveness
of treatments for rhinologic
and allergic diseases. NARAC
features lectures, panel
discussions and PBL breakout
sessions.
~ NARAC Poster Session ~
Call for Abstracts*
*Abstracts publishable in
American Journal of
Rhinology & Allergy
www.NARAConference.org
Copyright © Oceanside Publications, Inc. All rights reserved. DO NOT COPY
For permission to copy go to https://www.oceansidepubl.com/permission.htm